「JAMA」の記事一覧

アンジオテンシン変換酵素阻害薬またはアンジオテンシン受容体拮抗薬とCOVID-19診断および死亡率の関連
アンジオテンシン変換酵素阻害薬またはアンジオテンシン受容体拮抗薬とCOVID-19診断および死亡率の関連
Association of Angiotensin-Converting Enzyme Inhibitor or Angiotensin Receptor Blocker Use With COVID-19 Diagnosis and Mortality JAMA. 2020 Jul 14;324(2):168-177. doi: 10.1001/jama.2020.11301. 原文をBibgraph(ビブグラフ)で読む 上記論文の日本語要約 【重要性】アンジオテンシン変換酵素阻害薬(ACEI)/アンジオテンシン受容体拮抗薬(ARB)によってコロナウイルス感染症(COVID-19)に感染しやすくなり、ウイルスの機能性受容体、アンジオテンシン変換酵素2の発現が亢進することによって転帰が悪化すると思われる。 【目的】ACEI/ARBの使用とCOVID-19の診断およびCOVID-19患者の転帰との関連を調べること。 【デザイン、設定および参加者】COVID-19患者の転帰を調べるため、デンマークの全国レジストリのデータを用いて後ろ向きコホート研究を実施した。ICD-10コードを用いて2020年2月22日から5月4日までのCOVID-19患者を特定し、診断日から転帰または試験期間終了日(2020年5月4日)まで追跡した。COVID-19の感受性を調べるため、コホート内症例対照デザインのCox回帰モデルを用いて、2020年2月1日から同年5月4日まで他の降圧薬を比較したACEI/ARB使用とCOVID-19診断発生率との関連を検討した。 【曝露】発症日前6カ月間の処方と定義したACEI/ARBの使用。 【主要評価項目】この後ろ向きコホート研究で、主要評価項目は死亡とし、死亡または重症COVID-19の複合転帰を副次評価項目とした。コホート内症例対照の感受性解析では、転帰はCOVID-19診断とした。 【結果】この後ろ向きコホート研究には、COVID-19患者4480例を組み入れた(年齢中央値54.7歳[四分位範囲40.9-72.0歳]、47.9%が男性)。ACEI/ARB使用者895例(20.0%)および非使用者3585例(80.0%)であった。ACEI/ARB者の18.1%、非使用者の7.3%が30日以内に死亡したが、この関連は、年齢、性別および既往例で調整した後、有意差はなくなった(調整後ハザード比[HR]、0.83、95%CI 0.67-1.03)。ACEI/ARB使用者の31.9%、非使用者の14.2%に、30日以内に死亡または重症COVID-19が発生した(調整HR 1.04、95%CI 0.89-1.23)。COVID-19感受性を検討するコホート内症例対照研究では、高血圧の既往歴があるCOVID-19患者571例(年齢中央値73.9歳、54.3%が男性)を年齢および性別でマッチさせたCOVID-19がない対照5710例と比較した。COVID-19患者の86.5%がACEI/ARBを使用していたのに対し、対照は85.4%で、他の降圧薬と比較するとACEI/ARBにCOVID-19高発症率との有意な関連は認められなかった(調整HR 1.05、95%CI 0.80-1.36)。 【結論および意義】高血圧患者のACEI/ARB使用歴にCOVID-19診断またはCOVID-19患者の死亡または重症化との有意な関連は認められなかった。この結果から、COVID-19大流行下で臨床的に示唆されるACEI/ARB投与中止は支持されるものではない。 第一人者の医師による解説 COVID-19の流行に際し ACEI/ARBの服薬変更は不要の可能性を示唆 岩部 真人、二木 寛之、岩部 美紀、山内 敏正 東京大学大学院医学系研究科糖尿病・代謝内科 MMJ. February 2021;17(1):9 現在世界中でパンデミックを引き起こしている新型コロナウイルス感染症(COVID-19)は、一度は感染拡大の速度が緩んだようにみえた。しかし各種規制緩和などの影響でいわゆる第2波とも呼ぶべき再拡大が各国で起きており、日本でも感染者数は下げ止まっているなど、いまだ予断を許さない状況が続いている。 COVID-19を引き起こす病原体SARS-CoV-2はアンジオテンシン変換酵素(ACE)2を介して細胞内に侵入することが示されている(1)。一方、降圧薬のACE阻害薬(ACEI)やアンジオテンシン受容体拮抗薬(ARB)はACE2の発現を上昇させることが知られており(2)、COVID-19の罹患率、重症化率および死亡率への影響が懸念されてきた。ACEIやARBは降圧薬として第1選択となることも多く、投薬対象人口も非常に大きいことから、この関係を検証することは急務とされ、さまざまなコホートで研究が行われている。 本研究はデンマークでのデータを基に、重症化率・死亡率についてはACEI/ARB使用中の患者を含むCOVID-19患者4,480人を対象とした後ろ向きコホート研究により、罹患率についてはコホート内症例対照研究により検討したものである。まず重症化率・死亡率について、ACEI/ARB使用中の895人を含むCOVID-19患者4,480人のコホートで年齢、性別、既往歴について補正すると、ACEI/ARB使用者と非使用者の間で死亡率に有意な差はなかった(ハザード比[HR], 0.83)。同様に死亡率または重症化率についても両群間に有意差はなかった(HR, 1.04)。次に罹患率について、高血圧の既往のあるCOVID-19患者571人と年齢、性別がマッチした高血圧の既往のある非COVID-19患者5,710人との間で比較検討された。その結果、両群間のACEI/ARB使用率に有意な差は なく、ACEI/ARBの使用はCOVID-19の罹患率を上昇させていなかった(HR, 1.05)。 2019年12月以降COVID-19は世界的に拡大し、それから1年が経過しようとしている。COVID-19の病態メカニズムについて、いまだ明らかになっていないことは多いが、疫学的にさまざまな情報が集約されつつある。ACEI/ARBについては、特にSARS-CoV-2はACE2を介して細胞内に侵入することが明らかになり、COVID-19流行初期は服薬に伴うリスクが懸念されていた。しかしながらその後、本研究結果を含めACEI/ARBはCOVID-19の罹患率、重症化率、死亡率を上昇させないことが明らかになり、各学会からもACEI/ARBの使用継続を支持する声明が世界的に出されている。一方、本研究を含めて現在のエビデンスは、ほぼすべて後ろ向き研究の結果であり、各国で進行中の前向き研究に対して、今後も注視し続ける必要がある。 1. Wang Q, et al. Cell. 2020;181(4):894-904. 2. Vaduganathan M, et al. N Engl J Med. 2020;382(17):1653-1659.
SARS-CoV-2による一時的な小児多臓器系炎症性症候群患児58例の臨床的特徴
SARS-CoV-2による一時的な小児多臓器系炎症性症候群患児58例の臨床的特徴
Clinical Characteristics of 58 Children With a Pediatric Inflammatory Multisystem Syndrome Temporally Associated With SARS-CoV-2 JAMA. 2020 Jul 21;324(3):259-269. doi: 10.1001/jama.2020.10369. 原文をBibgraph(ビブグラフ)で読む 上記論文の日本語要約 【重要性】新型コロナウイルス感染症2019の発症率が高い地域で、発熱および炎症を伴うまれな症候群を呈する患児の報告が浮上している。 【目的】SARSコロナウイルス2(SARS-CoV-2)による一時的な小児多臓器系炎症性症候群(PIMS-TS)の基準を満たした入院患児の臨床所見および検査値の特徴を明らかし、他の小児炎症性疾患の特徴と比較すること。 【デザイン、設定および参加者】2020年3月23日から5月16日の間に、発熱の持続と公開されているPIMS-TSの診断基準を満たした入院患児58例(中央値9歳、33例が女児)の症例集積。最終追跡日は、2020年5月22日であった。診療記録の臨床症状および検査結果を要約し、2002年から2019年に欧米で入院した川崎病(KD)患児(1132例)、川崎病ショック症候群患児(45例)および中毒性ショック症候群患児(37例)の特徴と比較した。 【曝露】英国、米国および世界保健機関(WHO)が定義するPIMS-TSの基準を満たした患児の兆候、症状、検査および画像所見。 【主要評価項目】PIMS-TSの基準を満たした患児の臨床、検査および画像所見、他の小児炎症性疾患の特徴との比較。 【結果】PIMS-TSの基準を満たす患児58例(平均年齢9歳[IQR 5.7-14]、女児20例[34%])を特定した。58例中15例(26%)がSARS-CoV-2ポリメラーゼ連鎖反応検査の結果が陽性、46例中40例(87%)がSARS-CoV-2 IgG検査陽性であった。合わせて、58例中45例(78%)がSARS-CoV-2感染しているか、感染歴があった。全例に発熱および嘔吐(58例中26例、45%)、腹痛(58例中31例、53%)、下痢(58例中30例、52%)などの非特異的症状が見られた。58例中30例(52%)が発疹、58例中26例(45%)が結膜充血を呈した。検査所見の評価で、C反応性蛋白(229 mg/L[IQR 156-338]、全58例で評価)、フェリチン(610 μg/L[IQR 359-128]、58例中53例で評価)などで著名な炎症が認められ、58例中29例がショック(心筋機能障害の生化学的根拠あり)を来たし、強心薬および蘇生輸液投与を要した(機械的換気を実施した29例中23例[79%]を含む)。13例が米国心臓協会が定義するKDの基準を満たし、23例にKDやショックの特徴がない発熱および炎症があった。8例(14%)が冠動脈拡張または冠動脈瘤を来した。PIMS-TSとKDまたはKDショック症候群を比較すると、年齢が高く(年齢中央値9歳[IQR 5.7-14] vs. 2.7歳[IQR 1.4-4.7]および3.8歳[IQR 0.2-18])、C反応性蛋白(中央値229mg/L vs. 67mg/L[IQR 40-150 mg/L]および193mg/L[IQR 83-237])炎症マーカー高値などの臨床所見や検査所見に差が見られた。 【結論および意義】このPIMS-TSの基準を満たした患児の症例集積では、発熱や炎症から心筋障害、ショック、冠動脈瘤までさまざまな徴候、症状および疾患重症度が見られた。KDおよびKDショック症候群の患児と比較からは、この疾患に関する知見が得られ、他の小児炎症性疾患とは異なるものであることが示唆される。 第一人者の医師による解説 COVID-19重症化抑制の治療法につながる 臨床的意義の大きな研究 永田 智 東京女子医科大学小児科学講座主任教授 MMJ. February 2021;17(1):12 新型コロナウイルス感染症(COVID-19)の高感染率地域で重症急性呼吸器症候群コロナウイルス2型(SARS-CoV-2)に関連した小児炎症性多系統症候群(PIMS-TS)が報告されており、川崎病との類似性が話題になった。 本論文では、2020年3月?5月に英国の8病院に入院した、英国、米国疾病対策予防センター(CDC)または世界保健機関(WHO)のPIMS-TS基準を満たした小児58人について、臨床的特徴をカルテレビューで抽出し、2002~19年に欧米の病院に入院した川崎病患者(1,132人)、川崎病ショック症候群患者(45人)、トキシックショック症候群患者(37人)と比較検討した。 これらPIMS-TS症例はSARS-CoV-2 PCR検査で58人中15人(26%)が陽性、SARS-CoV-2 IgG抗体検査で46人中40人(87%)が陽性であった。全例で発熱、約半数で嘔吐、腹痛、下痢の消化器症状を認めていた。同様に約半数は発疹や結膜充血といった川崎病類似の症候を呈していた。血清CRPは中央値22.9mg/dL(四分位範囲[IQR],15.6~33.8)、フェリチンも中央値610μg/L(IQR,359~1,280)と全般に高値であった。58人中29人(50%)が心筋機能障害を高頻度に伴うショックをきたし、そのうち23人(79%)は人工呼吸器管理を要していた。13人は米国心臓学会(AHA)の川崎病基準を満たし、興味深いことに8人は冠動脈拡張または冠動脈瘤を合併していた。 本研究では、PIMS-TS、川崎病、川崎病ショック症候群の三者が比較された。PIMS-TSでは罹患年齢の中央値が9歳(IQR,5.7~14)と一般の川崎病(中央値2.7歳)や川崎病ショック症候群(中央値3.8歳)より高かった。また、PIMS-TSの血清CRPは、川崎病(中央値6.7mg/dL)よりかなり高値の傾向にあったが、川崎病ショック症候群とは僅差であった(中央値19.3mg/dL)。 これらのPIMS-TS症例は、発熱・心筋障害・ショック・冠動脈瘤形成といった多臓器にまたがる多彩な症候を示していたが、全般に重症であった。これまで、川崎病に類似した症候をもつ他の疾患は数少なく、しかも冠動脈病変を有する疾患はほとんど報告されていない。PIMS-TSを丁寧に調べることにより、川崎病の病態発症、特に冠動脈病変の発生機序を考える上で、重要なヒントが得られる可能性がある。さらに、川崎病の標準治療である免疫グロブリン大量療法がCOVID-19の重症型であるPIMS-TSの治療に有効であることが証明されれば、COVID-19の重症化抑制に安全性の高い有効な治療選択肢が加わることになり、当検討の臨床的な意義は大きいことになる。
ビタミンD長期的補給がうつまたはうつ症状リスクおよび気分スコアの変化にもたらす効果 無作為化試験
ビタミンD長期的補給がうつまたはうつ症状リスクおよび気分スコアの変化にもたらす効果 無作為化試験
Effect of Long-term Vitamin D3 Supplementation vs Placebo on Risk of Depression or Clinically Relevant Depressive Symptoms and on Change in Mood Scores: A Randomized Clinical Trial JAMA. 2020 Aug 4;324(5):471-480. doi: 10.1001/jama.2020.10224. 原文をBibgraph(ビブグラフ)で読む 上記論文の日本語要約 【重要性】25-ヒドロキシビタミンD値が低いと後にうつ病リスクが高くなるが、長期にわたる高用量摂取を検討した大規模試験がほとんどない。 【目的】ビタミンD3補給が後のうつ病リスクおよび気分スコアにもたらす効果を検証すること。 【デザイン、設定および参加者】米国で成人2万5871例を対照に心血管疾患とがんの予防を検討した無作為化試験VITALの補助的試験、VITAL-DEP(Vitamin D and Omega-3 Trial-Depression Endpoint Prevention)に50歳以上の男女1万8353例が参加した。1万6657例にうつ病発症リスク(うつ病既往歴なし)、1696例にうつ病再発リスク(うつ病既往歴があるが、過去2年間うつ病の治療を受けていない)があった。2011年11月から2014年3月にかけて無作為化し、2017年12月31日に割り付けた治療が終了し、この日が最終追跡日となった。 【介入】2×2要因デザインを用いて、ビタミンD3(コレカルシフェロール1日当たり2000IU)、魚油またはプラセボに無作為化。9181例をビタミンD3、9172例をマッチさせたプラセボに割り付けた。 【主要評価項目】主要評価項目は、うつ病または治療を要するうつ症状のリスク(総発症例数および再発例数)および気分スコア(8項目から成る患者健康質問票うつ尺度[PHQ-8]、0[症状なし]から24点[症状が多い]、臨床的意義のある最小変化量は0.5点)の平均差。 【結果】無作為化した1万8353例(平均年齢67.5[SD 7.1]歳、女性49.2%)の治療期間中央値は5.3年間で、90.5%が試験を完了した(試験終了時生存していた参加者の93.5%)。ビタミンD3群(うつ病または治療を要するうつ症状609例、1000人年当たり12.9)とプラセボ群(同625例、13.3)のうつ病または治療を要するうつ症状のリスクに有意差は見られず(ハザード比0.97、95%CI 0.87-1.09、P=0.62)、うつ病発症または再発に群間差はなかった。時間の経過に伴う気分スコアの変化に治療群間の有意差は見られず、PHQ-8スコアの平均変化量も有意差はなかった(気分スコアの平均変化量0.01点、95%CI -0.04-0.05)。 【結論および意義】試験開始時に治療を要するうつ症状がない50歳以上の男女を中央値5.3年間追跡した結果、プラセボを比較して、ビタミンD3治療によってうつ病の発症や再発、治療を要するうつ症状、気分スコアの変化に有意な差は認められなかった。この結果は、うつ病予防のための成人へのビタミンD3投与は支持するものではない。 第一人者の医師による解説 大規模 RCTによる長期介入でも効果は確認できず 山口 智史/佐々木 司(教授) 東京大学大学院教育学研究科健康教育学分野 MMJ. February 2021;17(1):15 うつ病は疾病負荷の主要因の1つとなっている(1)。特に、高齢者のうつ病は十分に治療されていないケースが多いため(2)、医療において、高齢者のうつ病予防は重要な領域である。高齢者では、血中ビタミンD濃度が低い場合に抑うつリスクが高いことが、横断研究のメタ解析によって示されている(3)。しかし、ビタミンD濃度を上昇させる介入がうつ病予防に有効であるかどうかについては十分に検討されていない。今回報告されたVITAL-DEP試験は、ビタミン Dのサプリメントを毎日服用することで、うつ病の予防が可能であるかを5年にわたる介入により検証した、ランダム化二重盲検プラセボ対照試験である。 試験対象者は、50歳以上の男女で、研究参加時点までにうつ病の診断を受けたことがない人か、うつ病既往があっても寛解後2年以上経過している人である。主要評価項目は2つあり、1つ目は、うつ病の発症および再発の有無である(うつ病の発症および再発は、医師によるうつ病の診断を受けたかどうか、うつ病の治療を受けているかどうか、うつ症状を測る質問票[PHQ-8]で点数がカットオフ以上となったかどうかのいずれかを指す)。2つ目は、PHQ-8で測ったうつ症状の長期的変化である。割り付けられた参加者は18,353人、このうち57.5%が5年間研究に参加し続けた。追跡調査は年1回の郵送による質問票で行われ、うつ病の発症と再発の確認、うつ症状の測定が行われた。また、割り付けられた錠剤の3分の2以上を服用したと回答し、プロトコール遵守とみなされた参加者は毎年の調査で90%を超えていた。ビタミンD服用群9,181人中、うつ病が発症または再発した人は609人であった(12.9/1,000人・年)。一方、プラセボ群では9,172人中625人であった(13.3/1,000人・年)。うつ病の発症・再発に関して、ビタミンD服用群とプラセボ群の間で、ハザード比は有意とならなかった。また、質問票で測ったうつ症状の点数も、2群間に有意差は認められなかった。これは、介入期間中のどの時点でも同様であった。 今回の結果は、10,000人を超える非常に多くの一般住民に対して、5年という長期の介入を続けたランダム化試験によるものである。著者らはこれらの点をVITAL-DEP試験の強みとして挙げており、得られた結果は、成人のうつ病予防のためにビタミンDを服用させることを支持するものではなかったと結論している。 1. Mathers CD, et al. PLoS Med. 2006;3(11):e442. 2. Unutzer J, et al. J Am Geriatr Soc. 2000;48(8):871-878. 3. Li H, et al. Am J Geriatr Psychiatry. 2019;27(11):1192-1202.
ビタミンD低値喘息患児の重度喘息増悪にもたらすビタミンD3補給の効果 VDKA無作為化臨床試験
ビタミンD低値喘息患児の重度喘息増悪にもたらすビタミンD3補給の効果 VDKA無作為化臨床試験
Effect of Vitamin D3 Supplementation on Severe Asthma Exacerbations in Children With Asthma and Low Vitamin D Levels: The VDKA Randomized Clinical Trial JAMA. 2020 Aug 25;324(8):752-760. 原文をBibgraph(ビブグラフ)で読む 上記論文の日本語要約 【重要性】重度の喘息増悪は死に至ることがあり、医療費もかかる。ビタミンD3補給によって小児期の重度喘息増悪が抑制できるかは明らかになっていない。 【目的】ビタミンD3補給によって、ビタミンD低値喘息患児の重度増悪までの時間が改善するかを明らかにすること。 【デザイン、設定および参加者】Vitamin D to Prevent Severe Asthma Exacerbations(VDKA)試験は、血清25-ヒドロキシビタミンD値が30ng/mL未満で低用量吸入ステロイドを投与している6~16歳の高リスク喘息患児で、ビタミンD3補給によって重度増悪までの時間が改善するかを検討した二重盲検プラセボ対照無作為化臨床試験である。米国7施設から被験者を組み入れた。2016年2月に、400例を目標に登録を開始した。試験は無益性のため早期に(2019年3月)中止され、2019年9月に追跡を終了した。 【介入】被験者をビタミンD3群(1日当たり4000IU、96例)とプラセボ群(96例)に割り付け、48週間にわたって投与し、フルチカゾン176μg/日(6~11歳)または220μg/日(12~16歳)投与を継続した。 【主要評価項目】主要評価項目は、重度喘息増悪までの時間とした。ウイルス誘発重度増悪までの時間、試験期間の中間時点で吸入ステロイド用量が減少した被験者の割合および期間中のフルチカゾン累積投与量を副次評価項目とした。 【結果】無作為化した192例(平均年齢9.8歳、女児88例[40%])のうち180(93.8%)が試験を完遂した。ビタミンD3群の36例(37.5%)およびプラセボ群の33例(34.4%)が1回以上の重度増悪を来した。プラセボと比べると、ビタミンD3補給による重度増悪までの時間の有意な短縮は見られず、増悪までの平均期間はビタミンD3群240日、プラセボ群253日だった。(平均群間差-13.1日、95%CI -42.6~16.4、調整ハザード比1.13、95%CI 0.69~1.85、P=0.63)。同様に、プラセボと比較したビタミンD3補給によるウイルス誘発重度増悪、試験期間の中間時点で吸入ステロイド用量が減少した被験者の割合および期間中のフルチカゾン累積投与量の改善度に有意差はなかった。両群の重度有害事象発現率はほぼ同じだった(ビタミンD3群11例、プラセボ群9例)。 【結論および意義】喘息が持続する低ビタミンD値の小児で、プラセボと比べてビタミンD3補給によって重度喘息増悪まで時間の有意な改善は認められなかった。この結果からは、この患者群に重度喘息増悪予防のためビタミンD3を補給することは支持されない。 第一人者の医師による解説 小児に対するプラクティスとしてのビタミンD投与は中止すべき 横山 彰仁 高知大学医学部呼吸器 ・アレルギー内科学教授 MMJ. February 2021;17(1):17 ビタミンDは肺の重要な成長因子であり、また免疫系において制御性T細胞の誘導、Th2やTh17反応の抑制、IL-10産生などを引き起こすことが知られている。さらに、気道のマイクロバイオームに影響し、平滑筋肥大を抑制しコラーゲン沈着を抑制することで気道リモデリングに抑制的に働くことも報告されている。こうした研究に一致するように、血中ビタミン D濃度が低下した患者では、重症の喘息増悪、肺機能低下、ステロイド反応性の低下などが生じることも知られている。 以上から、ビタミンDには喘息の1次予防効果が期待されるが、残念ながらその有用性は不明である。妊婦や幼児へのビタミンD補充は後年の家ダニへの感作抑制につながるとの報告もあるが、喘息発症を抑制するかは不明である。ただし、ビタミンDには、ライノウイルスの増殖を抑制し、インターフェロンによる抗ウイルス作用を促進するなど、ウイルス感染による発作を抑制する可能性はある。実際にメタ解析ではビタミンD補充は、喘息増悪のリスクを有意に低下させることが示されている。ただ、16歳以下に関しては有意な結果は得られていない。以上から、小児へのビタミンD投与は推奨されるに至っていないが、これまでの研究では、血中濃度が低い、重症増悪リスクが高い患児を対象としていないなどの問題点が指摘されていた。 本研究の利点として以下の3点が挙げられる:①参加者の血中ビタミンD濃度を測定し、濃度が低いことを確認した上で試験に登録している、②補充により実際に血中濃度が上昇したことを確認している、③前年に重症増悪歴のあるリスクが高い患児を対象とし、重症増悪発症までの期間を主要評価項目としている。 当初、本試験では重症増悪発症率で16%の絶対差を検出できるサンプルサイズの400人を目標として設定したが、予定されていた中間解析で有効性が認められず早期中止となった。最終的には目標の半分以下の192人を、48週間のプラセボ群またはビタミンD群に1:1に割り付けた。結果として、重症増悪はビタミン群で36人(37.5%)、プラセボ群で33人(34.4%)に認められ、主要評価項目である発症までの期間はもとより、ウイルス感染による重症増悪、吸入ステロイド薬の減量や累積使用量に関しても有意差は認められず、ビタミンD投与の有効性は認められなかった。 今回の結果を踏まえると、既報から小児に対しプラクティスとしてビタミン D濃度を測定し、投与する施設があるならば、中止すべきであろう。
基礎インスリン療法を実施している2型糖尿病患者の血糖制御に用いる持続血糖モニタリングの効果:無作為化比較試験
基礎インスリン療法を実施している2型糖尿病患者の血糖制御に用いる持続血糖モニタリングの効果:無作為化比較試験
Effect of Continuous Glucose Monitoring on Glycemic Control in Patients With Type 2 Diabetes Treated With Basal Insulin: A Randomized Clinical Trial JAMA. 2021 Jun 8;325(22):2262-2272. doi: 10.1001/jama.2021.7444. 原文をBibgraph(ビブグラフ)で読む 上記論文の日本語要約【重要性】持続血糖モニタリング(CGM)は、強化インスリン療法中の2型糖尿病成人患者にとって便益があることが示されているが、食前インスリン療法を実施していない基礎インスリン療法中の2型糖尿病患者でのCGM使用は検討されていない。【目的】プライマリケア診療で、食前インスリン療法を実施していない基礎インスリン療法中の2型糖尿病成人患者でCGMの有効性を検討すること。【デザイン、設定および参加者】この無作為化臨床試験は、米15施設(登録期間2018年7月30日から2019年10月30日;最終追跡調査日2020年7月7日)で実施し、プライマリケア医から糖尿病治療を受けており、食前インスリン療法を実施せず持効型または中間型の基礎インスリン製剤を1日1~2回投与している2型糖尿病成人患者を登録した。インスリン以外の血糖降下薬による薬物療法の有無は問わなかった。【介入】CGM群(116例)と従来の血糖測定器によるモニタリング(BGM)群(59例)に2対1の割合で無作為化により割り付けた。【主要評価項目】主要評価項目は、8カ月時点のヘモグロビンA1c(HbA1c)とした。CGMで測定した血糖目標値が70~180mg/dLの範囲内にある時間(の割合)(TIR)、血糖値が250mg/dLを超える時間、8カ月時の平均血糖値を主な副次評価項目とした。【結果】無作為化により割り付けた175例(平均年齢[SD]57歳[9];女性88例[50%];人種・民族的マイノリティ92例[53%]、治療前のHbA1c平均値[SD]9.1%[0.9%])のうち、165例(94%)が試験を完了した。CGM群のHbA1c平均値が治療前の9.1%から8カ月後の8.0%に、BGM群では9.0%から8.4%に低下した(調整後群間差、-0.4%[95%CI、-0.8~-0.1];P=0.02)。CGM群をBGM群と比較すると、CGMで測定した平均TIRは59%に対して43%(調整後群間差、15%[95%CI、8~23];P<0.001)、血糖値が250mg/dLを超える時間の割合は11%に対して27%(調整後群間差、-16%[95%CI、-21~-11];P<0.001)、平均血糖値の平均は179mg/dLに対して206mg/dL(調整後群間差、-26mg/dL[95%CI、-41~-12];P<0.001)であった。CGM群1例(1%)、BGM群1例(2%)に重度の低血糖が発現した。【結論および意義】食事前インスリン投与を実施しておらず、基礎インスリン療法のみでは血糖制御不良の2型糖尿病成人患者で、従来の血糖モニタリングよりも持続血糖モニタリングの方が8カ月後のHbA1c値が有意に低下した。 第一人者の医師による解説 日本ではこの数年にFGMが普及 新たなデバイスで糖尿病診療が進化 石原 寿光 日本大学医学部糖尿病代謝内科教授 MMJ. December 2021;17(6):179 持続的血糖モニタリング(CGM)は、1型糖尿病患者はもちろん、強化インスリン治療中の2型糖尿病患者に用いた場合、血糖コントロールに有効であることが示されているが、ボーラスインスリン注射を行わない、基礎インスリン補充のみで治療されている2型糖尿病患者における有効性は検証されていない。そこで、本論文の著者らは、持効型あるいは中間型インスリンの1日1回または2回注射で治療中の2型糖尿病患者(インスリン以外の血糖降下薬併用の有無は問わない)におけるCGMの有効性を、従来の指先などで採血して行う自己血糖測定(SMBG)を対照として、ランダム化対照試験(MOBILE試験)により検討した。米国の15施設で、2018年7月30日~19年10月30日に175人の患者の組み入れが行われ、主要評価項目として8カ月後のHbA1cが評価された。CGM群の患者にはDexcomG6CGMシステムが装着され、適宜SMBGも併用された。その結果、8カ月後のHbA1cは、CGM群ではベースラインの9.1%から8.0%、SMBG群で9.0%から8.4%に低下し、低下の度合いは有意にCGM群で大きかった(群間差のP=0.02)。また、グルコース値が70~180mg/dLに入っている1日のうちの時間の割合は、CGM群では59%、SMBG群では43%とCGM群の方が有意に高く(P<0.001)、250mg/dL超の時間の割合はそれぞれ11%と27%とCGM群の方が有意に低かった(P<0.001)。重症な低血糖の発生率はそれぞれ1%と2%のみであった。したがって、持効型あるいは中間型インスリンの1日1回または2回注射で治療中の2型糖尿病患者においても、CGMは有効であると考えられた。今後、数年単位でこの効果が持続するかなどを検証していく必要があると思われる。日本では、この数年にFreestyleリブレTMによるFlashGlucoseMonitoring(FGM)が普及してきている。現在の保険適用は、強化インスリン療法施行中の患者が主体であるが、基礎インスリンのみの患者への適用拡大も検討されている。また、インスリンを使っていない経口糖尿病薬のみの患者でのFGMの有効性も報告されており(1)、新たなデバイスが糖尿病診療を進化させつつある。 1. Wada E, et al. BMJ Open Diabetes Res Care. 2020;8(1):e001115.
虚血性脳卒中患者の心房細動検出に用いる植込み型ループレコーダーと体外式ループレコーダーの比較:PER DIEM無作為化臨床試験
虚血性脳卒中患者の心房細動検出に用いる植込み型ループレコーダーと体外式ループレコーダーの比較:PER DIEM無作為化臨床試験
Effect of Implantable vs Prolonged External Electrocardiographic Monitoring on Atrial Fibrillation Detection in Patients With Ischemic Stroke: The PER DIEM Randomized Clinical Trial JAMA. 2021 Jun 1;325(21):2160-2168. doi: 10.1001/jama.2021.6128. 原文をBibgraph(ビブグラフ)で読む 上記論文の日本語要約【重要性】体外式ループレコーダーまたは植込み型ループレコーダーを用いた心電図モニタリングによる虚血性脳卒中後の心房細動(AF)または心房粗動の相対検出率は明らかになっていない。【目的】虚血性脳卒中を発症して間もない患者で、植込み型ループレコーダーを用いた12カ月間のモニタリングが従来の体外式ループレコーダーを用いた30日間のモニタリングよりもAF発生を多く検出できるかを明らかにすること。【デザイン、設定および参加者】カナダ・アルバータ州の大学病院2施設および市中病院1施設で、2015年5月から2017年11月にかけて虚血性脳卒中発症6カ月以内でAFの既往歴がない患者300例を登録し、医師主導型非盲検無作為化臨床試験を実施した。最終追跡が2018年10月であった。【介入】参加者を植込み型ループレコーダー(150例)と体外式ループレコーダー(150例)による持続的心電図モニタリングに1対1の割合で無作為化により割り付けた。参加者は、経過観察のため30日後、6カ月後および12カ月後に受診した。【主要評価項目】主要評価項目は、AF確定(definite AF)またはほぼ確実なAF(highly probable AF)(無作為化後12カ月以内に2分以上持続する新たなAFの判定)とした。新規AFのtime to event解析、虚血性脳卒中の再発、頭蓋内出血、死亡および12カ月以内に発生した機器関連の重篤な有害事象などの8項目を事前に定めた副次評価項目とした。【結果】無作為化により割り付けた300例(年齢中央値64.1歳[四分位範囲、56.1~73.7];女性121例[40.3%];CHA2DS2-VASc[うっ血性心不全、高血圧、75歳以上、糖尿病、脳卒中または一過性虚血発作、血管疾患、65~74歳、性別]スコア中央値4点[四分位範囲、3~5]の原因不明の脳卒中66.3%)のうち273例(91.0%)が24時間以上の心臓モニタリングを完了し、259例(86.3%)が割り付けたモニタリングおよび12カ月間の経過観察を完遂した。主要評価項目は、植込み型ループレコーダー群の15.3%(150例中23例)、体外式ループレコーダー群の4.7%(150例中7例)に発現した(群間差、10.7%[95%CI、4.0~17.3%];リスク比、3.29[95%CI、1.45~7.42];P=0.003)。8項目の副次評価項目のうち、6項目に有意差が認められなかった。植込み型ループレコーダー群の5例(3.3%)に虚血性脳卒中が再発したのに対して、体外式ループレコーダー群では8例(5.3%)に再発を認めた(群間差、-2.0%[95%CI、-6.6~2.6%])。各群1例(0.7%)に頭蓋内出血が発生し(群間差、0%[95%CI、-1.8~1.8%])、各群3例(2.0%)が死亡し(群間差、0%[95%CI、-3.2~3.2%])、それぞれ1例(0.7%)と0例(0%)に機器関連の重篤な有害事象が発現した。【結論および意義】AFの既往歴がない虚血性脳卒中患者で、植込み型ループレコーダーによる12カ月間の心電図モニタリングのAF検出率が、30日間の体外式ループレコーダーによるモニタリングよりも有意に高かった。このモニタリング法による臨床成績および相対的な費用効果を比較する詳細な研究が必要である。 第一人者の医師による解説 社会的経済的状態と疾病の発症について 生活習慣との相互作用を含めた研究が必要 門脇 孝 虎の門病院院長 MMJ. December 2021;17(6):188 心原性脳塞栓症は最重症のノックアウト型脳梗塞をきたす。原因の大部分は高齢化で増加が予想される心房細動(AF)だが、幸い抗凝固療法によりリスクを約3分の1に低減することができる。したがって、AF診断の強化が要介護削減、健康寿命延伸のカギを握る。虚血性脳卒中(IS)は心原性に限らずAF有病率が高い。再発予防の観点からAF診断が重要だが、ホルター心電図などでは検出困難な発作性AFが多数潜在している。このような潜在性AFの検出には植込み型ループレコーダー(ILR)が極めて有用だが、その保険適用は発症原因が特定できない潜因性脳梗塞(CS)に限られる。本研究は、既知のAFがない発症半年以内のISを対象とし、4週間可能な限り体外式ループレコーダー(ELR)を装着する群と、ILRで1年間監視する群とに無作為に割り付け、2分間以上持続するAFの検出率を比較検討したものである(各群150人)。その結果、AF検出率はELR群の4.7%に対し、ILR群では15.3%と10.7%高く、リスク比は3.29であった(P=0.003)。ILRはELRより有意にAF検出率を高められるという結果が示されたことから、ILRの適応をすべてのISに拡大すべきかどうかが議論となるが、本研究の対象患者は66%がCSである上に、病型別の検出率も示されていないため、CS以外のISでの有用性は明らかでない。長時間の心電図モニタで病型別のAF検出率を検討した報告はほとんどない。参考までに、当院では入院時心電図でAFが認められないすべてのISに対し、原則1週間デュランタR(ZAIKEN)による非侵襲的心電図モニタを行っており、Stroke2021で筆者が当院の臨床データを発表した時点で7日目まで監視しえた患者は1,066人に達する(1)。病型別のAF検出率は、TIA0%、ラクナ梗塞3.9%、アテローム血栓性脳梗塞5.4%、CS10.5%であり、CS以外の病型におけるAF検出率はCSのおよそ半分以下となっている。したがって、CS以外のISではILRによるAF検出率の上乗せ効果はさほど大きくなく、侵襲的で高額なILRの適応をすべてのISに拡大するほどのメリットは得られないと予想される。潜在性AFの検出は「Thelonger,thebetter」であり、検出力の観点からはILRが最も優れるものの検査数は限られる。重要なのは検出数を増やすことであり、まずは広く実施可能で、検出力も比較的高い非侵襲的長時間心電図モニタによるAF監視の強化を優先すべきであろう。 1. 第 46 回日本脳卒中学会学術集会:虚血性脳卒中急性期の非侵襲的長時間心電図モニタによる悉皆的心房細動スクリーニングの有用性(抄録番号:卒中 O-061-2)
中等症ないし重症の喘息に用いる2剤併用または3剤併用の吸入療法と喘息の転帰:系統的レビューおよびメタ解析
中等症ないし重症の喘息に用いる2剤併用または3剤併用の吸入療法と喘息の転帰:系統的レビューおよびメタ解析
Triple vs Dual Inhaler Therapy and Asthma Outcomes in Moderate to Severe Asthma: A Systematic Review and Meta-analysis JAMA. 2021 Jun 22;325(24):2466-2479. doi: 10.1001/jama.2021.7872. 原文をBibgraph(ビブグラフ)で読む 上記論文の日本語要約【重要性】中等症ないし重症の喘息に対して、吸入コルチコステロイド(ICS)および長時間作用型β2刺激薬(LABA)への長時間作用型抗コリン薬(LAMA)追加による便益と害は明らかになっていない。【目的】制御不良の喘息が持続する小児および成人を対象に、3剤併用療法(ICS、LABA、LAMA)と2剤併用療法(ICS、LABA)の転帰と有害事象を系統的に統合し比較すること。【データ入手元】2017年11月から2020年12月8日までのMEDLINE、Embase、CENTRAL、ICTRP、FDA、EMAのデータベース。言語による制約を設けなかった。【試験の選択】独立した2名の研究者が、中等症ないし重症の喘息に用いる3剤併用療法と2剤併用療法を比較した無作為化臨床試験(RCT)を選択した。【データの抽出および統合】レビュアー2名が独立してデータの抽出とバイアスのリスクを評価した。個々の患者の増悪データを含めランダム効果メタ解析を用いた。GRADE(Grading of Recommendations, Assessment, Development and Evaluation)アプローチを用いて科学的根拠の確実性(質)を評価した。【主要評価項目】重度の増悪、喘息制御(7項目の喘息制御に関する質問票[ACQ-7]で測定、各項目のスコア0~6[完全に制御~重度の制御不良]、最小重要差0.5)、生活の質(喘息関連の生活の質[AQLQ]ツールで測定、1~7点[重度障害~障害なし]、最小重要差0.5)、死亡率および有害事象。【結果】3種類のLAMAを用いて小児と成人計11,894例(平均年齢52歳[範囲、9~71];女性57.7%)を登録したRCT 20件を対象とした。確実性の高い根拠で、3剤併用療法を2剤併用療法と比較すると、重度増悪リスクの低下(試験9件[9,932例];22.7% vs 27.4%;リスク比、0.83[95%CI、0.77~0.90])および喘息制御の改善(14試験[1万1,230例];標準化平均差[SMD]、-0.06[95% CI、-0.10~-0.02])、ACQ-7尺度の平均差-0.04[95% CI -0.07~-0.01])に有意な関連を認めた。2剤併用療法と3剤併用療法の間に、喘息関連QOL(7試験[5,247例)];SMD、0.05[95%CI -0.03~0.13];AQLQスコアの平均差、0.05[95%CI、-0.03~0.13];確実性中等度の根拠)または死亡率(試験17件[11,595例];0.12% vs 0.12%;リスク比、0.96[95%CI 0.33~2.75];確実性の高い根拠)の有意差は認められなかった。3剤併用療法に口腔乾燥および発声障害との関連を認めた(10試験[7,395例];3.0% vs 1.8%;リスク比、1.65[95% CI 1.14~2.38]、確実性の高い根拠]。しかし、治療関連の有害事象と重篤な有害事象に群間差は認められなかった(確実性中等度の根拠)。【結論および意義】中等症ないし重症の喘息の小児(6~18歳)および成人で、3剤併用療法は2剤併用療法と比較して、重度増悪の減少および喘息制御改善の中等度改善との有意な関連を認め、QOLや死亡に有意差はなかった。 第一人者の医師による解説 LAMAの効果が高い集団や副作用の少ない集団が判明すれば 有用なエビデンス 入江 美聡(助教)/福永 興壱(教授) 慶應義塾大学医学部呼吸器内科 MMJ. December 2021;17(6):174 喘息はどの年齢においても有病率の高い慢性呼吸器疾患である。国際的なガイドラインでは6歳以上の中等症・重症喘息患者に適した長期管理薬として吸入ステロイド薬(ICS)と長時間作用型β2刺激薬(LABA)の配合剤による治療(2剤併用療法)が推奨されている。長時間作用型抗コリン薬(LAMA)はLABAと異なる機序での気管支拡張作用を有するが、2剤併用療法でもコントロールが不良な場合におけるLAMA追加投与の効果や有害事象については不明確である。そのため、ICS/LABA/LAMAの3剤併用療法は弱い推奨とされている。これまで、3剤併用療法と2剤併用療法を比較した系統的レビューは2017年までの検索に限られていた。対象研究はチオトロピウムを用いた少数の研究のみに限られており、有害事象についての言及がなされず、喘息増悪における3剤併用療法の有効性は明らかではなかった。2017年以降、米食品医薬品局(FDA)による小児におけるチオトロピウムの使用が認可され、3剤併用療法について多くの試験がなされてきた。今回の論文はコントロール不良な小児および成人の中等症・重症喘息における3剤併用療法と2剤併用療法の効果と有害事象についての系統的レビューおよびメタ解析である。3種類のLAMAを対象とした20件の無作為化試験(小児・成人患者計11,894人)について解析が行われた。主要評価項目は重症増悪、喘息コントロール、生活の質(QOL)、死亡率、有害事象である。結果は重症増悪のリスクが3剤併用療法で有意に低下し(リスク比[RR],0.83)、AsthmaControlQuestionnaire(ACQ)やAsthmaControlTest(ACT)で評価した喘息コントロールが改善した(標準化平均差,-0.06)。一方、喘息関連のQOLや死亡率に差はなかった。また有害事象に関しては3剤併用療法で口渇と発声障害が有意に増加したが、重篤な有害事象について有意差はなかった。GlobalInitiativeforAsthma(GINA)のガイドラインや日本の「喘息予防・管理ガイドライン2018」でもICS/LABAの2剤併用療法における追加療法としてロイコトリエン拮抗薬(LTRA)、LAMA、各種抗体製剤、経口ステロイド薬が挙げられているが、優先順位は示されていない。現実的には各種抗体製剤は薬剤費用の観点から、経口ステロイド薬は長期使用に伴う副作用の観点からそれぞれ第1選択とはなりにくく、LTRAとLAMAのどちらかを(あるいは両薬同時を)追加治療として優先的に選択することが多い。今後、LTRAと比較しLAMAの効果が高い集団や、LAMAの副作用が出現しにくい集団が判明すれば、喘息増悪予防や症状コントロールにおいて有用なエビデンスとなると考える。
成人の反復性片頭痛に用いる急性期治療:系統的レビューとメタ解析
成人の反復性片頭痛に用いる急性期治療:系統的レビューとメタ解析
Acute Treatments for Episodic Migraine in Adults: A Systematic Review and Meta-analysis JAMA. 2021 Jun 15;325(23):2357-2369. doi: 10.1001/jama.2021.7939. 原文をBibgraph(ビブグラフ)で読む 上記論文の日本語要約【重要性】片頭痛は頻度が高く著しい病的状態を引き起こすことがあり、急性期治療に幾つか選択肢がある。【目的】成人の反復性片頭痛に用いる急性期治療の便益と害を評価すること。【データ入手元】開始時から2021年2月24日までの多数のデータベース。【試験の選択】片頭痛発作に用いる急性期治療の効果または害を評価した無作為化臨床試験と系統的レビュー。【データ抽出および統合】独立したレビュアーが試験を選択し、データを抽出した。Hartung-Knapp-Sidik-Jonkman法による分散補正およびDerSimonian-Lairdのランダム効果モデルを用いてメタ解析を実施し、試験数が少ない場合はMantel-Haenszel法に基づく固定効果モデルを用いた。【主要評価項目】主要評価項目は、疼痛消失、疼痛緩和、疼痛消失の持続、疼痛緩和の持続、有害事象とした。科学的根拠の強さ(SOE)をAgency for Healthcare Research and Quality Methods Guide for Effectiveness and Comparative Effectiveness Reviewsを用いて各等級に分類した。【結果】系統的レビュー15報からトリプタンと非ステロイド抗炎症薬に関する科学的根拠を要約した。他の介入について、患者2万8,803例を対象とした無作為化臨床試験115件を対象とした。プラセボと比較すると、トリプタンと非ステロイド抗炎症薬を別々に使用した場合に、2時間後および翌日の疼痛軽減(中程度ないし高度のSOE)および軽度で一過性の有害事象のリスク上昇との有意な関連を認めた。プラセボと比較すると、カルシトニン遺伝子関連ペプチド受容体拮抗薬(低度ないし高度のSOE)、lasmiditan(5-HT1F受容体作動薬;高度のSOE)、dihydroergotamine(中等度ないし高度のSOE)、ergotamine+カフェイン(中等度のSOE)、アセトアミノフェン(中等度のSOE)、制吐薬(低度のSOE)、butorphanol(低度のSOE)、トラマドールとアセトアミノフェンの併用(低度のSOE)に疼痛軽減および軽度の有害事象増加との有意な関連があった。オピオイドに関する結果は、低度または不十分なSOEに基づくものであった。遠隔電気神経調節(中等度のSOE)、経頭蓋磁気刺激(低度のSOE)、外部三叉神経刺激(低度のSOE)、非侵襲的迷走神経刺激(低度のSOE)などの非薬物療法に疼痛改善との有意な関連があった。非薬物療法群とシャム群との間に有害事象の有意差は認められなかった。【結論および意義】片頭痛の急性期治療が幾つかあるが、それぞれの治療を裏付ける科学的根拠の強さにばらつきがある。トリプタン、非ステロイド抗炎症薬、アセトアミノフェン、dihydroergotamine、カルシトニン遺伝子関連ペプチド拮抗薬、lasmiditanおよび一部の非薬物療法に疼痛および機能の改善との有意な関連を認めた。オピオイドを始めとするその他多くの介入に関する科学的根拠は少なかった。 第一人者の医師による解説 ゲパント系薬剤やディタン系薬剤の国内承認で 片頭痛の急性期治療は大きな変革 北村 英二 北里大学医学部脳神経内科学講師 MMJ. December 2021;17(6):173 2016年の世界の疾病負荷研究(GlobalBurdenofDiseaseStudy)によると、世界人口のおよそ14.4%が片頭痛に罹患しており、片頭痛は障害生存年数の第2位に位置する疾患である。片頭痛診療では生活習慣や環境要因に対する生活指導に加え、急性期治療と予防療法が行われる。日本では2021年に抗カルシトニン遺伝子関連ペプチド(CGRP)抗体、抗CGRP受容体抗体による予防療法が承認され、片頭痛診療の大きな変革期が到来している。本論文の目的は成人の反復性片頭痛(国際頭痛分類第3版[ICHD-3]片頭痛診断基準に準じ、片頭痛日数が月に15日未満で慢性片頭痛に該当しないもの)に対する急性期治療の有益性と有害性を評価することである。トリプタンと非ステロイド系抗炎症薬(NSAIDs)に関するエビデンスについて15の系統的レビューを、その他の介入研究については115の無作為化臨床試験(患者数28,803人)を評価した。プラセボと比較して、トリプタンとNSAIDsを個別に使用した場合、それぞれ2時間後と1日後の痛みが有意に減少し(エビデンスの強さ[SOE]:中~高)、軽度・一過性有害事象のリスクが上昇した。またプラセボと比較して、CGRP受容体拮抗薬(SOE:低~高)、ラスミディタン(5-HT1F受容体作動薬)(SOE:高)、ジヒドロエルゴタミン(SOE:中~高)、エルゴタミン+カフェイン(SOE:中)、アセトアミノフェン(SOE:中)、制吐薬(SOE:低)、ブトルファノール(SOE:低)、およびトラマドールとアセトアミノフェンの併用(SOE:低)は有意に痛みを軽減し、軽度有害事象の増加を認めた。オピオイドに関してはSOEが低いか不十分であった。一方、片頭痛の非薬物療法については、ニューロモデュレーション(REN)(SOE:中)、経頭蓋磁気刺激装置(TMS)(SOE:低)、経皮的三叉神経刺激装置(e-TNS)(SOE:低)、非侵襲的迷走神経刺激装置(nVNS)(SOE:中)が有意に痛みを改善した。有害事象については、非薬物療法と偽(sham)治療で有意差はなかった。片頭痛に対していくつかの急性期治療があるが、今回の結果から、そのエビデンスの強さはさまざまであることが示された。日本でもゲパント系薬剤(CGRP受容体拮抗薬)(1),(2)、ディタン系薬剤(選択的5-HT1F受容体作動薬)(3)が承認されれば、片頭痛の予防療法のみならず急性期治療も大きな変革期を迎えることが予想される。 1. Croop R, et al. Lancet. 2021;397(10268):51-60.2. Lipton RB, et al. N Engl J Med. 2019;381(2):142-149.3. Goadsby PJ, et al. Brain. 2019;142(7):1894-1904.
妊娠中のインフルエンザワクチン接種と小児期早期の健康転帰との関連
妊娠中のインフルエンザワクチン接種と小児期早期の健康転帰との関連
Association of Maternal Influenza Vaccination During Pregnancy With Early Childhood Health Outcomes JAMA. 2021 Jun 8;325(22):2285-2293. doi: 10.1001/jama.2021.6778. 原文をBibgraph(ビブグラフ)で読む 上記論文の日本語要約【重要性】妊娠中の季節性インフルエンザワクチン接種によって、妊婦と新生児のインフルエンザによる疾患が減少する。妊娠中の季節性インフルエンザワクチン接種と小児期の有害な健康転帰との関連性については科学的根拠が少ない。【目的】妊娠中のインフルエンザワクチン接種と小児期早期の健康転帰との関連を評価すること。【デザイン、設定および参加者】健康管理データと紐付けた出生登録を用いた後ろ向きコホート研究。2010年10月1日から2014年3月31日までの間に記録されたカナダ・ノバスコシア州の全生児出生を2016年3月31日まで追跡した。逆確率による重み付けを用いて母体の病歴や可能性のあるその他の交絡因子を調整し、調整ハザード比(HR)と発生率比(IRR)を推定し95%CIを添えた。【曝露】妊娠中の季節性インフルエンザワクチン接種。【主要評価項目】免疫関連転帰(喘息、感染症)、非免疫関連転帰(悪性新生物、感覚障害)および非特異的転帰(緊急医療または入院医療の利用)を小児期の転帰とし、救急診療部門と入院のデータベースから測定した。【結果】小児28,255例(女児49%、在胎37週以降の出生92%)のうち10,227例(36.2%)が妊娠中に季節性インフルエンザワクチン接種を受けた母親から出生した。平均3.6年の追跡期間中、母親のインフルエンザワクチン接種と小児期の喘息(発生率、1,000人年当たり3.0 vs. 2.5;差0.53、95%CI ?0.15-1.21];調整済みハザード比1.22[95%CI 0.94~1.59])、悪性新生物(1,000人年当たり0.32 vs. 0.26、差0.06/人年[95%CI -0.16~0.28]、調整済みハザード比1.26[95%CI 0.57~2.78])および感覚障害(1000人年当たり0.80 vs. 0.97;差-0.17[95%CI -0.54~0.21];調整済みハザード比0.82[95%CI 0.49~1.37])との間に有意な関連は認められなかった。妊娠中の母親のインフルエンザワクチン接種と小児期早期の感染症(発生率、1,000人年当たり184.6 vs. 179.1、差5.44[95%CI 0.01~10.9];調整済み発生率比1.07[95%CI 0.99~1.15])、緊急医療または入院医療の利用(1,000人年当たり511.7 vs. 477.8/人年;差33.9/人年[95%CI 24.9~42.9];調整済み発生率比1.05[95%CI 0.99~1.16])との間に有意な関連は認められなかった。【結論および意義】平均追跡期間3.6年の住民を対象としたコホート研究で、妊娠中のインフルエンザワクチン接種に小児期早期の有害健康転帰との有意な関連は認められなかった。 第一人者の医師による解説 妊婦へのインフルエンザワクチン接種推奨 参考となる報告 三鴨 廣繁 愛知医科大学大学院医学研究科臨床感染症学教授 MMJ. December 2021;17(6):186 妊婦が特にインフルエンザに罹患しやすいわけではないが、感染すると、重症化や合併症を起こすリスクが高く、2009年新型インフルエンザは胎児死亡や早産のリスク上昇をもたらした。妊娠中の季節性インフルエンザワクチン接種は母体および新生児のインフルエンザ罹患抑制に有効であるが、妊婦のインフルエンザワクチン接種が小児期の健康への悪影響と関連しているかどうかについての証拠は限られている。著者らは、妊娠中の母親のインフルエンザワクチン接種と幼児期の健康転帰との関連を評価した。2010年10月1日~14年3月31日にカナダ・ノバスコシアで出生したすべての子どもが登録され、16年3月31日まで追跡調査された。本研究は、健康管理データにリンクされた出生登録を使用した後ろ向きコホート研究である。調整ハザード率(HR)と発生率比(IRR)は母親の病歴およびその他の潜在的な交絡因子に対して補正しながら推定された。解析対象となった28,255人の子ども(49%が女性、92%が妊娠37週以上で出生)のうち、10,227人(36.2%)が妊娠中に季節性インフルエンザワクチン接種を受けた女性が出産した。平均3.6年間の追跡期間中、妊婦のインフルエンザワクチン接種と小児喘息の間に有意な関連は認められなかった(1,000人・年あたり3.0対2.5;差0.53;調整済みHR,1.22)、悪性新生物(0.32対0.26;差0.06;調整済みHR,1.26)、感覚障害(1,000人・年あたり0.80対0.97;差?0.17;調整済みHR,0.82)。妊婦のインフルエンザワクチン接種は幼児期の感染症(1,000人・年あたり184.6対179.1;差5.44;調整済みIRR,1.07)、救急や入院患者の医療サービスの利用(1,000人・年あたり511.7対477.8;差33.9;調整済みIRR,1.05)と有意に関連していなかった。したがって、著者らは、妊婦のインフルエンザワクチン接種は、幼児期の健康への悪影響のリスク上昇と有意に関連していなかった、と結論している。妊娠中の季節性インフルエンザワクチン接種により、母体および新生児のインフルエンザ感染を減らすことが可能である。妊娠中の季節性インフルエンザワクチン接種が小児期の健康へ与える影響に関する証拠は限られているが、今回のカナダでのデータベースコホート研究では、妊娠中の母体インフルエンザワクチン接種は、幼児期の健康への影響は認められなかった。新型コロナウイルス感染症(COVID-19)流行期においてもインフルエンザワクチン接種の重要性が叫ばれているが、妊婦へのインフルエンザワクチン接種推奨にあたって参考となる報告と考える。
寛解期の関節リウマチに用いる従来型合成疾患修飾性抗リウマチ薬の半用量と一定用量が再燃に及ぼす作用の比較:ARCTIC REWIND無作為化臨床試験
寛解期の関節リウマチに用いる従来型合成疾患修飾性抗リウマチ薬の半用量と一定用量が再燃に及ぼす作用の比較:ARCTIC REWIND無作為化臨床試験
Effect of Half-Dose vs Stable-Dose Conventional Synthetic Disease-Modifying Antirheumatic Drugs on Disease Flares in Patients With Rheumatoid Arthritis in Remission: The ARCTIC REWIND Randomized Clinical Trial JAMA. 2021 May 4;325(17):1755-1764. doi: 10.1001/jama.2021.4542. 原文をBibgraph(ビブグラフ)で読む 上記論文の日本語要約【重要性】従来型合成疾患修飾性抗リウマチ薬(csDMARD)を投与している関節リウマチ(RA)患者で寛解維持が達成可能な治療目標となっているが、臨床的寛解患者をどう治療するのが最適であるかは明らかになっていない。【目的】寛解を維持しているRA患者を対象に、csDMARD漸減が再燃リスクに及ぼす影響をcsDMARD一定量継続と比較すること。【デザイン、設定および参加者】ARCTIC REWINDは、ノルウェーの病院内リウマチ診療所10施設で実施した多施設共同並行群間非盲検無作為化非劣性試験であった。2013年6月から2018年6月までの間に、csDMARD一定用量を投与し12カ月間寛解を維持しているRA患者160例を登録した。最終来院は2019年6月であった。【介入】患者をcsDMARD半用量群(80例)とcsDMARD一定用量群(80例)に無作為化により割り付けた。【主要評価項目】主要評価項目は、試験開始から12カ月後の追跡調査までに再燃を認めた患者の割合とし、Disease Activity Score(DAS)スコア1.6超(RA寛解の閾値)、DASスコア0.6以上増加、腫脹関節数2カ所以上を再燃と定義した。このほか、患者および医師がともに臨床的に重大な再燃が生じたことを合意した場合も疾患の再燃とした。リスク差20%を非劣性マージンと定義した。【結果】登録した患者160例(平均年齢55.1歳[SD 11.9];女性66%)のうち、156例に割り付けた治療を実施し、このうち重大なプロトコールの逸脱が認められなかった155例を主要解析集団とした(半用量群77例、一定用量群78例)。csDMARD半用量群の19例(25%)が再燃したのに対して、一定用量群では5例(6%)に再燃を認めた(リスク差18%、95%CI 7-29)。半用量群の34例(44%)および一定用量群の42例(54%)に有害事象が発現したが、試験中止に至る患者はいなかった。死亡は認められなかった。【結論および意義】csDMARDを投与している寛解期RA患者で、12カ月間で再燃した患者の割合について、半用量による治療の一定用量に対する非劣性が示されなかった。一定用量群の方が再燃した患者数が有意に少なかった。以上の結果から、半用量投与は支持されない。 第一人者の医師による解説 寛解維持でリウマチ薬減量を希望する患者は多く 医師の丁寧な説明が必要 伊藤 聡 新潟県立リウマチセンター副院長 MMJ. December 2021;17(6):181 欧州リウマチ学会(EULAR)の推奨では、関節リウマチ(RA)患者が従来型抗リウマチ薬(csDMARDs)を使用し寛解を維持していた場合、csDMARDsの減量を考慮することが示唆されている(1)。しかしその根拠となる確固としたエビデンスはない。本論文は、ノルウェーの病院リウマチ科10施設で行われた、36カ月間の多施設共同、無作為化、非盲検、並行群間比較、非劣性試験(ARCTICREWIND試験)の報告である。寛解を維持しているRA患者を、csDMARDsを半量に減量する群と、減量しない群に分けて、再燃のリスクについて検討した。主要評価項目は、ベースラインから12カ月後までの再燃である。再燃の定義は、(1)DiseaseActivityScore(DAS)が1.6を超える(2)DASが0.6以上増加する(3)腫脹関節が2カ所以上ある(4)患者と主治医の双方が臨床的に再燃したと判断した──の組み合わせとし、非劣性マージンは20%とした。減量群77人、非減量群78人が主要評価項目の解析対象とされた。両群ともに、メトトレキサート(MTX)の単剤療法が多く(減量群:経口52人、皮下注14人、非減量群:経口51人、皮下注10人)、その他スルファサラゾピリジンやヒドロキシクロロキンの併用、他のcsDMARDsの単剤あるいは併用療法が行われていた。MTX使用量は平均で19mg/週程度であった。結果、再燃は減量群の25%、非減量群の6%に認められた(リスク差18%)。有害事象は減量群で44%、非減量群で54%に発現し、主に上気道感染などの軽度の感染症であった。重篤な有害事象の発現率は減量群5%、非減量群3%で、治療中止例や死亡例はなかった。本試験は、寛解維持患者においてcsDMARDsの半量減量は、12カ月間の再燃率に関して非減量に対する非劣性を示すことができず、再燃は非減量群で有意に少なかった。このことは、寛解を維持していても、csDMARDsを半量に減量する戦略を支持しない結果となった。RA患者は臨床的寛解を導入し維持できると、治療費、副作用の懸念などから抗リウマチ薬の減量や中止を希望することが多い。当院では生物学的製剤を中止する、いわゆるバイオフリーを実践し維持しているが(2)、本研究の結果からは、csDMARDsの減量は行わない方がよいだろう。筆者は、患者には「再燃すると再び寛解に導入するのは難しいので、寝た子は起こさないようにしましょう」と説明している。 1. Smolen JS,et al.Ann Rheum Dis.2020;79(6):685-699.2. Ito S,et al.Mod Rheumatol.2021;31(4):919-923.
ポルトガルの小児で検討したB群髄膜炎菌ワクチン接種とB群侵襲性髄膜炎菌感染症の関連
ポルトガルの小児で検討したB群髄膜炎菌ワクチン接種とB群侵襲性髄膜炎菌感染症の関連
Association of Use of a Meningococcus Group B Vaccine With Group B Invasive Meningococcal Disease Among Children in Portugal JAMA. 2020 Dec 1;324(21):2187-2194. doi: 10.1001/jama.2020.20449. 原文をBibgraph(ビブグラフ)で読む 上記論文の日本語要約【重要性】小児のB群侵襲性髄膜炎菌感染症を予防するワクチンには多成分B群髄膜炎菌ワクチン(4CMenB)以外にないが、マッチさせた対照とワクチンの効果を比較した試験はない。 【目的】4CMenB接種とB群侵襲性髄膜炎菌感染症の関連を明らかにすること。 【デザイン、設定および参加者】発生密度対症例対照研究。2014年10月から2019年3月までの間にポルトガルの小児病院31施設を受診した患者を特定し、死亡または退院まで追跡した(最終追跡2019年6月)。検査で侵襲性髄膜炎菌感染症が確定したポルトガルに居住する小児および思春期小児を対象とした。同時期に同じ病院に無関係の病態で入院した対照(通常1例につき2例)を性別、年齢および居住地でマッチさせた。 【曝露】全国データベースから取得した4CMenBによる予防接種(年齢により2~4用量を推奨)。 【主要評価項目】主要評価項目は、対照と比較した予防接種完了者のB群侵襲性髄膜炎菌感染症とした。副次評価項目は、対照と比較した予防接種完了者の全血清型侵襲性髄膜炎菌感染症および1回以上接種した対照と比較した症例の侵襲性髄膜炎菌感染症とした。 【結果】侵襲性髄膜炎菌感染症患児117例のうち、98例が組み入れ基準を満たし、82例がB型侵襲性髄膜炎菌感染症であった。69例がワクチン接種を完了する年齢に達しており、保護されていたとみなした。この69例の月齢中央値は24(四分位範囲4.5~196)カ月、42例が男児であり、入院期間中央値は8(四分位範囲0~86)日間であった。症例69例中5例(7.2%)と対照142例中33例(23.1%)がワクチン接種を完了していた(差-16.0%[95%CI -26.3%~-5.7%]、オッズ比[OR]0.21[95%CI 0.08~0.55])。全血清型の侵襲性髄膜炎菌感染症でみると、症例85例中6例(7.1%)と対照175例中39例(22.3%)がワクチン接種を完了していた(差-15.2%[95%CI -24.3%~-6.1%]、OR 0.22[95%CI 0.09~0.53])。B群感染症では、症例82例中8例(9.8%)と対照168例中50例(29.8%)が1回以上ワクチンを接種していた(差-20.0%[95%CI -30.3%~-9.7%]、OR 0.18[95%CI 0.08~0.44])。全血清型の侵襲性髄膜炎菌感染症では、症例98例中11例(11.2%)と対照201例中61例(30.3%)が1回以上ワクチンを接種して受けていた(差-19.1%[95%CI -28.8%~-9.5%]、OR 0.23[95%CI 0.11~0.49])。 【結論および意義】ポルトガルでのワクチン接種開始から最初の5年間で、侵襲性髄膜炎菌感染症を発症した小児の方が発症しなかった対照の小児よりも4CMenBワクチンを接種した割合が低かった。この結果は、臨床現場での4CMenBワクチン使用を周知するのに有用である。 第一人者の医師による解説 国内未承認のB群髄膜炎菌ワクチン 今後の承認を期待 神谷 元 国立感染症研究所実地疫学研究センター主任研究官 MMJ. December 2021;17(6):185 本論文は、ポルトガルの小児科医療機関31施設が参加し、B群髄膜炎菌(MenB)ワクチンの有効性を年齢、性別、居住地区、受診医療機関についてマッチングした症例対照研究により検討した結果の報告である。調査期間(2014年10月~19年3月)、ポルトガルではMenBワクチンは定期接種化されておらず、国内の1歳児のMenBワクチン接種率(2回)は56.7%(2018年)であった。299人の小児が参加し、MenBによる侵襲性髄膜炎菌感染症(IMD)の予防効果をエンドポイントとした解析では、オッズ比が0.21(95%信頼区間[CI], 0.08?0.55)、他の血清群を含めたIMDの予防効果はオッズ比が0.22(95% CI, 0.09?0.53)となり、ワクチン効果(VE)は78~79%と一定の効果を認めた。また、調査期間におけるIMDの原因菌の内訳はB群が84%を占めていたが、MenBワクチン接種者でIMDを発症した11人のうち、8人はMenB、3人はそれ以外の血清群の菌による感染であった。11人の転帰は良好で合併症も認められなかった(未接種者では26%に合併症が認められた)。髄膜炎菌ワクチンは4つの血清群(A、C、W、Y)の莢膜多糖体を用いた4価ワクチン(MCV4ワクチン)が実用化されているが、B群がこのワクチンに含まれていない理由は、B群の莢膜多糖体がヒトの脳の糖鎖と構造が似ているため、ほかの血清群のようにワクチン成分として莢膜多糖体を利用できないことにある。しかし、近年の技術と研究の進歩により、外膜の表層蛋白を用いたMenBワクチンが開発され、米国、カナダ、オーストラリア、欧州では承認されている。このワクチンは、MenBに対する予防効果はもちろんのこと、髄膜炎菌に共通する外膜の表層蛋白を用いているため、ほかの血清群による髄膜炎菌感染症への予防効果も期待されている。日本では2021年7月時点でMenBワクチンは未承認であるが、国内のIMDサーベイランスの結果によると、一定の割合でMenBによるIMDが報告されている(1)。また、東京2020大会のような国際的なマスギャザリングが開催されると国内でそれまで検出されることが少ない髄膜炎菌が認められ、IMD発症事例も起こるため(2)、MenBワクチンの国内での承認が今後期待される。なお、ポルトガルではその後2020年に2カ月、4カ月、12カ月齢児にMenBワクチンを定期接種化している(3)。 1. 国立感染症研究所. IASR.2018;39:1-2. https://bit.ly/2W5FwwO 2. Kanai M, et al. Western Pac Surveill Response J. 2017;8(2):25-30. 3. ECD C. Vaccine S cheduler Pneumo co ccal Dis eas e:Recommended vaccinations https://bit.ly/39xLERD
敗血症ショックの臓器障害にもたらすアスコルビン酸、副腎皮質およびチアミンの効果 ACTS無作為化比較試験
敗血症ショックの臓器障害にもたらすアスコルビン酸、副腎皮質およびチアミンの効果 ACTS無作為化比較試験
Effect of Ascorbic Acid, Corticosteroids, and Thiamine on Organ Injury in Septic Shock: The ACTS Randomized Clinical Trial JAMA. 2020 Aug 18;324(7):642-650. 原文をBibgraph(ビブグラフ)で読む 上記論文の日本語要約 【重要性】アスコルビン酸、副腎皮質ステロイドおよびチアミンの併用は、敗血症ショックの有望な治療法として考えられている。 【目的】アスコルビン酸、副腎皮質ステロイドおよびチアミンの併用によって敗血症ショックの臓器障害を改善するかを明らかにするため。 【デザイン、設定および参加者】成人敗血症ショック患者に用いるアスコルビン酸、副腎皮質ステロイドおよびチアミンの併用をプラセボと比較した多施設共同無作為化盲検比較試験。2018年2月9日から2019年10月27日にかけて、米国の14施設で205例を組み入れた。2019年11月29日まで追跡した。 【介入】被験者を非経口アスコルビン酸(1500mg)、ヒドロコルチゾン(50mg)およびチアミン(100mg)6時間に1回、4日間投与するグループ(103例)と同じタイミングでマッチさせた用量のプラセボを投与するグループ(102例)に無作為に割り付けた。 【主要評価項目】主要評価項目は、登録時と72時間後のSOFAスコア(範囲0~24点、0点が最も良好)の変化量とした。腎不全および30日死亡率を重要な副次評価項目とした。試験薬を1回以上投与した患者を解析対象とした。 【結果】無作為化した205例(平均年齢68[SD 15]歳、女性90例[44%])のうち200例(98%)に試験薬を1回以上投与し、全例が試験を完遂し、解析対象とした(介入群101例、プラセボ群99例)。全体で、登録後72時間にわたるSOFAスコアの変化を見ると、時間と治療群の間に有意差は見られなかった(平均SOFAスコア変化:介入群9.1点から4.4点[-4.7点] vs プラセボ群9.2点から5.1点[-4.1点]、調整平均差-0.8、95%CI -1.7~0.2点、交互作用のP=0.12)。腎不全発生率(介入群31.7%vs プラセボ群27.3%、調整リスク差0.03、95%CI -0.1~0.2点、P=0.58)や30日死亡率(34.7%vs 29.3%、ハザード比1.3、95%CI 0.8-2.2、P=0.26)にも有意差は見られなかった。よく見られた銃独な有害事象は、高血糖(介入群12例、プラセボ群7例)、高ナトリウム血症(それぞれ11例と7例)、新規院内感染症(それぞれ13例と12例)であった。 【結論および意義】敗血症ショックで、アスコルビン酸、副腎皮質ステロイドおよびチアミンの併用による登録後72時間のSOFAスコア低下量はプラセボと有意差が見られなかった。このデータからは、敗血症ショック患者にこの併用療法のルーチンの使用は支持されない。 第一人者の医師による解説 ACTS試験:注目のMetabolic resuscitation療法 またも期待外れ 西田 修 藤田医科大学医学部麻酔・侵襲制御医学講座主任教授 MMJ. February 2021;17(1):25 2016年、敗血症の定義と診断基準が変更された(1)。「感染症に対する制御不能な宿主反応に起因する生命を脅かす臓器障害」と定義され、「全身性炎症」として評価する従来の診断基準から、「臓器障害そのものの進展」に重きを置いた診断基準に変更されている。 救命率は向上してきているものの、依然として致死率は高く、最近の全世界的な調査によると、すべての死亡原因の約20%は敗血症関連といわれている。敗血症は一刻を争う治療が必要とされるが、感染巣のコントロール・過不足のない輸液・昇圧薬の適正使用と人工呼吸管理などのライフサポートが主体であり劇的な改善をもたらす治療法はない。このような中で、細胞の機能を改善し組織障害を防ぐ手立てとして、“metabolic resuscitation”の考えが近年注目され、ステロイド(Hydrocortisone)、ビタミンC(Ascorbic acid)、ビタミンB1(Thiamine)の併用療法(HAT療法)が試みられるようになった。また、新型コロナウイルス感染症(COVID-19)でもアスコルビン酸は補助療法として提唱されている。2017年に発表された後ろ向き前後比較研究(2)では、敗血症・敗血症ショック患者の院内死亡率が31.9%(40.4→8.5%)低下、昇圧薬使?期間が約3分の1に短縮という治療成績を示した。これを検証するための無作為化対照試験(RCT)が複数実施され、その結果が最近報告されてきているが、いずれも期待したほどの効果はみられていない。本論文で報告された大規模なACTS試験もその1つで、成人の敗血症性ショック患者200人をアスコルビン酸(1,500mg)、ヒドロコルチゾン(50mg)およびチアミン(100mg)を6時間ごとに4日間静注する群もしくはプラセボ群に割り付け、治療効果を比較した。主要評価項目は、各臓器の障害の程度を示すSequential Organ Failure Assessment(SOFA)スコアの変化(入室時と72時間後の比較)としている。両群の患者背景に差はなかった。SOFAスコアの変化において両群間に有意差はなく、副次評価項目の30日死亡率、腎機能障害などでも有意差はなかった。循環改善効果として、shock free daysに有意差を認めているが、差は1日である。ヒドロコルチゾン単独群を対照としたRCT(3)では循環改善効果が示されなかったことから、ACTS試験における差はヒドロコルチゾンの効果であると推定できる。最頻度の有害事象として高血糖と高ナトリウム血症を認めている。 今回のACTS試験ならびに他のRCTの結果を総合的に考えると、HAT療法はルーチンで用いるべきものではなく、効果は限定的であると考えられる。 1. Singer M, et al. JAMA. 2016;315(8):801-810. 2. Marik PE, et al. Chest. 2017;151(6):1229-1238. 3. Fujii T, et al. JAMA. 2020;323(5):423-431.
急性冠症候群に用いるチカグレロル単剤療法とチカグレロル+アスピリン併用療法が大出血および心血管イベントにもたらす効果 TICO無作為化臨床試験
急性冠症候群に用いるチカグレロル単剤療法とチカグレロル+アスピリン併用療法が大出血および心血管イベントにもたらす効果 TICO無作為化臨床試験
Effect of Ticagrelor Monotherapy vs Ticagrelor With Aspirin on Major Bleeding and Cardiovascular Events in Patients With Acute Coronary Syndrome: The TICO Randomized Clinical Trial JAMA. 2020 Jun 16;323(23):2407-2416. doi: 10.1001/jama.2020.7580. 原文をBibgraph(ビブグラフ)で読む 上記論文の日本語要約 【重要性】短期間の抗血小板薬2剤併用療法(DAPT)後のアスピリン投与中止が、出血抑制戦略として評価されている。しかし、チカグレロル単剤療法の戦略は、急性冠症候群(ACS)患者に対象を限定して評価されていない。 【目的】薬剤溶出性ステントで治療したACS患者で、3カ月間のDAPT後にチカグレロル単剤療法に切り替えることによってチカグレロル主体の12カ月間のDAPTより純有害臨床事象が減少するかと明らかにすること。 【デザイン、設定および参加者】韓国の38施設で、2015年8月から2018年10月にかけて、薬剤溶出性ステントで治療したACS患者3056例を対象に、多施設共同無作為化試験を実施した。2019年10月に追跡が終了した。 【介入】患者を3カ月間のチカグレロルとアスピリンを用いたDAPT後にチカグレロル単剤療法(1日2回90mg)へ切り替えるグループ(1527例)とチカグレロル主体の12カ月間のDAPTを実施するグループ(1529例)に割り付けた。 【主要評価項目】主要評価項目は、1年後の純臨床有害事象とし、大出血および有害心脳血管イベントの複合(死亡、心筋梗塞、ステント血栓症、脳卒中、標的病変の血行再建術のいずれか)と定義した。重大な有害心脳血管イベントを事前に副次評価項目に規定した。 【結果】無作為化した3056例[平均年齢61歳、女性628例(20%)、ST上昇型心筋梗塞36%]のうち2978例(97.4%)が試験を完遂した。主要評価項目は、3カ月間のDAPT後チカグレロル単剤療法への切り替え群の59例(3.9%)、チカグレロル主体の12カ月間のDAPT群の89例(5.9%)に発生した(絶対差-1.98%[95%CI 3.50~-0.45%]、ハザード比[HR]0.66[95%CI 0.48~0.92]、P=0.01)。事前に副次評価項目に規定した10項目中8項目に有意差が見られなかった。3カ月間のDAPT後チカグレロル単剤療法への切り替え群の1.7%、チカグレロル主体の12カ月間のDAPT群の3.0%に大出血が発生した(HR 0.56[0.34~0.91]、P=0.02)。重大な有害心脳血管イベントの発生率には、3カ月間のDAPT後チカグレロル単剤療法への切り替え群(2.3%)とチカグレロル主体の12カ月間のDAPT群(3.4%)に有意な差が見られなかった(HR 0.69[95%CI 0.45~1.06]、P=0.09)。 【結論および意義】薬剤溶出性ステントで治療した急性冠症候群で、3カ月間のDAPT後のチカグレロル単剤療法への切り替えによって、チカグレロル主体の12カ月間のDAPTよりも、1年時の大出血および有害心脳血管イベントの複合転帰が控えめだが統計的に有意に減少した。この試験で検討した患者集団および予想されるイベント発生率がこれより低い患者には、この試験で検討した治療を検討すべきである。 第一人者の医師による解説 ACS患者に新世代 DESを留置した後のDAPT期間は3カ月で良い 前村 浩二 長崎大学大学院医歯薬学総合研究科循環器内科学教授 MMJ. February 2021;17(1):20 冠動脈にステントを留置した後には、ステント内血栓症を防ぐために一定期間、抗血小板薬2剤併用療法(DAPT)を行う必要があり、通常アスピリンとP2Y12受容体拮抗薬を使用する。第1世代の薬剤溶出性ステント(DES)では留置後長期間経ってもステントが内膜に覆われず血栓を形成することがあったため、DES留置後は1年間、可能ならさらに長期間DAPTを継続することが推奨された。その後DESは改良され、第2、3世代のDESではステント血栓症は少なくなったため、DAPT期間を短縮できるとする報告が相次いでいる。 本研究は、DES留置を受けた急性冠症候群(ACS)患者に、チカグレロルとアスピリンによるDAPTを3カ月行った後に、チカグレロル単剤群とDAPT12カ月群で全臨床的有害事象を比較した試験である。その結果、1年以内の大出血と心血管イベントの複合ではチカグレロル単剤群が3.9%、12カ月DAPT群が5.9%であり単剤群の方が優れていた。この試験では新世代の極薄型ストラット生体吸収性ポリマーDESを用いたことがDAPT期間の短縮に寄与したと考えられる。また、DAPT後にアスピリン単剤でなくチカグレロル単剤にしたことも、DAPT期間短縮に寄与した可能性が高い。チカグレロルはP2Y12受容体を直接阻害するため、効果発現までの時間が短く、欧米ではACS患者に多く使用されている。しかし日本人を多く含む研究であるPHILO試験において、チカグレロルはクロピドグレルに比べ、統計学的有意差はないものの、大出血や心血管イベントが多い傾向にあった(1)。そのため日本ではクロピドグレルまたはプラスグレルが多く使用され、チカグレロルはこれらが使用できない場合のみ適応とされている。クロピドグレルを用いた試験としては、日本でDAPT1カ月+クロピドグレル単剤投与とDAPT12カ月を比較したSTOPDAPT-2試験が行われ、DAPT1カ月群の優越性が示された(2)。現在ACS患者を対象としたSTOPDAPT-2ACS試験が進行中である。 このようにDAPT期間を短縮できるという報告が相次いでいるため、日本のガイドラインが最近更新された。2020年の日本循環器学会「冠動脈疾患患者における抗血栓療法ガイドライン」フォーカスアップデート版では、ACS患者は血栓リスクが高いと考えられるため、出血リスクが低い場合のDAPT期間は3~12カ月を推奨しているが、高出血リスク患者では1~3カ月を推奨している。このようにDESの改良によりDAPT期間は以前より短くなり、個々の患者の出血リスクと血栓リスクを勘案して決定することになる。 1. Goto S, et al. Circ J. 2015;79(11):2452-2460. 2. Watanabe H, et al. JAMA. 2019;321(24):2414-2427.
血漿P-tau217によるアルツハイマー病と他の神経変性疾患の識別
血漿P-tau217によるアルツハイマー病と他の神経変性疾患の識別
Discriminative Accuracy of Plasma Phospho-tau217 for Alzheimer Disease vs Other Neurodegenerative Disorders JAMA. 2020 Aug 25;324(8):772-781. doi: 10.1001/jama.2020.12134. 原文をBibgraph(ビブグラフ)で読む 上記論文の日本語要約 【重要性】現在のアルツハイマー病(AD)を診断する検査法には限界がある。 【目的】ADを診断するバイオマーカーに用いるスレオニン部位でリン酸化された血漿タウ(P-tau217)の有用性を調べること。 【デザイン、設定および参加者】横断的研究3件――AD患者34例およびADがない47例を対象とした米アリゾナ州の神経病理学コホート(コホート1、登録期間2007年5月から2019年1月)、認知機能障害がない参加者(301例)と臨床的に診断を受けた軽度認知機能(MCI)がある参加者(178例)、アルツハイマー型認知症(121例)および他の神経変性疾患(99例)を対象としたスウェーデンのBioFINDER-2コホート(コホート2、2017年4月から2019年9月)、PSEN1 E280A遺伝子変異保有者365例および非変異保有者257例を対象としたコロンビア常染色体優性ADがある親族コホート(コホート3、2013年12月から2017年2月)。 【曝露】血漿P-tau217。 【主要評価項目】血漿P-tau217のAD(臨床的または神経病理学的に診断)識別精度を主要評価項目とした。タウ病理像との関連を副次評価項目とした(神経病理学的にまたはPETで確認)。 【結果】コホート1は平均年齢83.5(SD 8.5)歳、女性38%、コホート2は平均年齢69.1(SD 10.3)歳、女性51%、コホート3は平均年齢35.8(SD 10.7)歳、女性57%だった。コホート1は、生前の血漿P-tau217からADと非ADを神経病理学的に識別でき[曲線下面積(AUC)0.89、95%CI 0.81-0.97]、血漿中P-tau181やニューロフィラメント軽鎖(NfL)よりも識別精度が有意に高かった(AUC範囲0.50-0.72、P<0.05)。コホート2での血漿P-tau217のアルツハイマー型認知症とその他の神経変性疾患の識別精度は、血漿中P-tau181やNfL、MRI検査よりも有意に高かった(AUCの範囲0.50-0.81、P<0.001)が、脳脊髄液(CSF)P-tau217、CSF P-tau181、タウPET検査との有意差はなかった(AUC範囲0.90-0.99、P>0.15)。コホート3では、約25歳以上のPSEN1変異保有者の血漿中P-tau217値が非保有者よりも高く、変異保有者がMCIを発症したと推定された時期の約20年前に増加が始まっていた。コホート1では、血漿中P-tau217値にβアミロイドプラークがある参加者のタウ変化との有意な関連がみられたが(Spearmanのρ=0.64、P<0.001)、βアミロイドプラークがない参加者ではこの関連はみられなかった(Spearmanのρ=0.15、P=0.33)。コホート2では、血漿中P-tau217でタウPET検査の異常を正常と見分けることができ(AUC 0.93、95%CI 0.91-0.96)、血漿中P-tau181や血漿NfL、CSF P-tau181、CSFのAβ42/Aβ40比およびMRI検査よりも有意に精度が高かったが(AUC範囲0.67-0.90、P<0.05)、精度にCSF P-tau217との有意差はなかった(AUC 0.96、P=0.22)。 【結論および意義】コホート3件の参加者1402例で、血漿中P-tau217によってADとその他の神経変性疾患を見分けることができ、血漿およびMRI検査のバイオマーカーより精度が有意に高かったが、主要なCSFやPET検査の測定法との有意差はなかった。この方法を最適化し、多様な集団を対象に結果を検証し、実臨床に用いる潜在的な役割を明らかにするため詳細な研究が必要である。 第一人者の医師による解説 実臨床や多様な集団から対象者を十分確保した 縦断的研究による検証が必要 石井 一弘 筑波大学医学医療系神経内科学准教授 MMJ. April 2021;17(2):46 2050年にアルツハイマー病(AD)の患者数が全世界で1億人に達するとの試算もある。ADの疾患修飾薬が利用可能になれば、低侵襲の採血で測定でき、しかも疾患早期から正確な診断が可能な診断マーカーの開発が望まれる。ADの原因蛋白であるAβ蛋白分子種(Aβ40、Aβ42)、各種リン酸化タウ蛋白を血漿、髄液で測定し、さらに生体内のこれら蛋白をPETで可視化し、その分布や脳部位で定量をし、診断バイオマーカーとする試みが行われている。しかしながら、これらバイオマーカーを用いてのAD早期診断には限界がある。最近、217番目のスレオニンがリン酸化したタウ蛋白(P-tau217)は、181番目のスレオニンがリン酸化したタウ蛋白(P-tau181)に比べ、より正確にしかも、より早期にADを診断できることが報告された(1)。 本研究では3つのコホート研究から得られた1,402人分の血漿試料を用いて、P-tau217濃度を測定し、ADに対する診断精度(感度、特異度)を他の血漿、髄液バイオマーカーと比較し、有用性を検討した。その結果、血漿P-tau217は臨床的に診断されたADを他の神経変性疾患と正確に鑑別することができ、病理学的にADと診断された患者と病理学的にADではない患者を判別することができた。さらに血漿P-tau217は血漿P-tau181、血漿ニューロフィラメント軽鎖(NfL)や大脳皮質厚や海馬容積などの脳 MRI測定値と比較し、臨床的ADをより正確に診断した。一方、髄液P-tau181、髄液P-tau217やTau-PETとの比較では、鑑別精度に有意差はなかった。加えて、血漿P-tau217濃度は神経原線維変化などのTau病理と相関し、TauPETでの正常と異常を他の髄液、血漿のバイオマーカーより正確に判別可能であった。 本研究の限界として、選択された集団を用いた横断的コホート研究であることが挙げられる。そのため、実臨床や多様な集団から十分な対象者数を確保した縦断的研究による検証を行わなければならない。また、測定法についてもP-tau217測定の感度向上と最適化、実用化に向けての測定自動化やカットオフ値の設定が必要である。他の神経変性疾患の鑑別への応用でも、十分な疾患数を確保し、鑑別精度を上げる必要がある。これらの限界を考慮しても、血漿P-tau217測定はADの早期診断やTau関連疾患との鑑別においては、今後、十分に注目される診断バイオマーカーになるであろう。 1. Janelidze S, et al. Nat Commun. 2020;11(1):1683.
電子たばこ+カウンセリングとカウンセリング単独の禁煙効果を比較 無作為化比較試験
電子たばこ+カウンセリングとカウンセリング単独の禁煙効果を比較 無作為化比較試験
Effect of e-Cigarettes Plus Counseling vs Counseling Alone on Smoking Cessation: A Randomized Clinical Trial JAMA. 2020 Nov 10;324(18):1844-1854. doi: 10.1001/jama.2020.18889. 原文をBibgraph(ビブグラフ)で読む 上記論文の日本語要約 【重要性】禁煙に電子たばこを用いることはいまだに議論の余地がある。 【目的】禁煙対策として個別カウンセリングと併用する電子たばこを評価すること。 【デザイン、設定および参加者】カナダの17施設で、2016年11月から2019年9月の間に禁煙の意欲がある成人を組み入れた無作為化比較試験(801例をスクリーニング、274例が適格、151例が辞退)。製造の遅延によって早期中止となった(486例中376例、目標の77%)。24週間の結果(2020年3月)を報告する。 【介入】12週間のニコチン含有電子たばこ使用(128例)、ニコチン非含有電子たばこ使用(127例)、電子たばこ不使用(121例)に無作為に割り付けた。全群に個別カウンセリングを実施した。 【主要転帰および評価項目】主要評価項目は、12週時の禁煙点有病率(7日間想起、呼気一酸化炭素を用いて生化学的に検証)とした。参加者の欠損データを喫煙とみなした。何回かの経過観察で調べた副次評価項目7項目は、他の経過観察時の禁煙点有病率、禁煙の継続、1日の禁煙本数の変化、重度有害事象、有害事象、有害事象による脱落および治療のアドヒアランスだった。 【結果】無作為化した参加者376例[平均年齢52歳、女性178例(47%)]の自己報告の喫煙率は12週時で299例(80%)、24週時で278例(74%)だった。禁煙点有病率は、12週時はニコチン含有電子たばこ+カウンセリング群の方がカウンセリング単独群よりも有意に高かった[21.9% vs. 9.1%、リスク差(RD)12.8、95%CI 4.0-21.6]が、24週時は有意差がなかった(17.2%vs. 9.9%、RD 7.3、95%CI -1.2-15.7)。ニコチン非含有電子たばこ+カウンセリングの禁煙点有病率は、12週時はカウンセリング単独と有意差がなかった(17.3%vs. 9.1%、RD 8.2、95%CI -0.1-16.6)が、24週時はカウンセリング単独より有意に高かった(20.5%vs. 9.9%、RD 10.6、95%CI 1.8-19.4)。有害事象がよく見られ[ニコチン含有電子たばこ+カウンセリング120例(94%)、ニコチン非含有電子たばこ+カウンセリング118例(93%)、カウンセリング単独88例(73%)]、多かったのが咳嗽(64%)と口腔内乾燥(53%)だった。 【結論および意義】禁煙意欲がある成人を対象にしたニコチン含有電子たばこ+カウンセリングで、カウンセリング単独よりも12週時の禁煙点有病率が高かった。しかし、この差は24週時に有意性が消失し、早期中止とニコチン含有および非含有電子たばこの結果の矛盾があったため、試験の解釈が限定されることから、詳細な研究が必要である。 第一人者の医師による解説 個別集中的なカウンセリングは禁煙成功に寄与 さらなる禁煙支援の検討が必要 小川 史洋 横浜市立大学医学部救急医学教室助教 MMJ. April 2021;17(2):39 紙巻きたばこを吸うことによる長期的な健康への悪影響は十分に証明されており、紙巻きたばこ使用者の多くが禁煙を試みている。2010~2011年に実施された米国の調査研究では、少なくとも5週間の禁煙補助薬および/または行動療法を行ったにもかかわらず、過去1年間に禁煙を試みたと報告した8,263人の70%以上が再喫煙していた(1)。 現在、電子たばこ(e-cigarette)を多くの喫煙者が禁煙補助具として使用しているが、禁煙における電子たばこの有効性はいまだに物議を醸している。禁煙に関する電子たばこの効果臨床試験が報告されており、電子たばこを使用することでさまざまな喫煙アウトカムがわずかに改善することを示唆する報告もあるが、その効果は十分に証明されておらず、さらなる検討が必要である。 本論文は、電子たばこの禁煙に関する効果について、電子たばことカウンセリングの併用をカウンセリング単独と比較・検討したランダム化比較試験(E3 Trial)の報告である。本試験では、登録した801人のうち最終的に376人(平均年齢52歳;女性47%)を個別カウンセリングに加えて、ニコチンを含有する電子たばこ、もしくはニコチンを含有しない電子たばこを使用する群、または電子たばこを使用しない群(計3群)にランダムに割り付け、12週間の介入を行い、12、24週目の喫煙状況を比較した。その結果、禁煙を試みている喫煙者に対して、電子たばことカウンセリングを組み合わせることにより、短期間での禁煙効果は期待できるが、長期間での効果は期待できず、さらに、非ニコチン電子たばことニコチン電子たばこ間の差も明らかでないため、さらなる検討が必要である、と著者らは結論づけている。 過去の報告からも、電子たばこ単独では禁煙の成功に大きく貢献する可能性は低く、禁煙の手段として推奨または促進すべきではないと考えられている。多くの電子たばこが、「たばこをやめたい人のための電子たばこ」「電子たばこで禁煙」などと宣伝されているが、製品説明の中に禁煙成功の効果的な用法・用量が示されていないものも多く、電子たばこを医学的な禁煙支援ツールとして支持する科学的証拠はなく、薬物療法など禁煙効果が科学的に証明された禁煙方法と横並びに電子たばこを扱うことはできない。一方、禁煙支援に関する個別集中的なカウンセリングは禁煙成功に寄与するとされており(2)、今後さらなる禁煙支援についての検討が待たれるところである。 1. Siahpush M, et al. BMJ Open 2015;5(1):e006229. 2. Lancaster T, et al. Cochrane Database Syst Rev 2017;3:CD001292.
COVID-19重症患者に用いる全身性コルチコステロイド投与と死亡率の関連メタ解析
COVID-19重症患者に用いる全身性コルチコステロイド投与と死亡率の関連メタ解析
Association Between Administration of Systemic Corticosteroids and Mortality Among Critically Ill Patients With COVID-19: A Meta-analysis JAMA. 2020 Oct 6;324(13):1330-1341. doi: 10.1001/jama.2020.17023. 原文をBibgraph(ビブグラフ)で読む 上記論文の日本語要約 【重要性】新型コロナウイルス感染症(COVID-19)の有効な治療法が必要であり、臨床試験データから、低用量デキサメタゾンで呼吸管理を要するCOVID-19入院患者の死亡率が低下することが示されている。 【目的】通常治療またはプラセボと比較したコルチコステロイド投与と28日全死因死亡率との関連を推定すること。 【デザイン、設定および参加者】COVID-19重症患者1703例を対象にコルチコステロイドの有効性を評価した無作為化臨床試験7件のデータを統合した前向きメタ解析。試験は、2020年2月26日から6月9日にかけて12カ国で実施され、最終追跡日は2020年7月6日であった。個々の試験、全体および事前に定義した下位集団別に統合したデータを集計した。Cochrane Risk of Bias Assessment Tool用いてバイアスリスクを評価した。I2統計量を用いて試験間の結果の非一貫性を評価した。主要解析は、全死因死亡率の分散逆数重み付け固定効果メタ解析とし、介入と死亡率との関連はオッズ比(OR)で定量化した。このほか、(異質性のPaule-Mandel推定値とHartung-Knapp調整を用いて)変量効果メタ解析、リスク比を用いて分散逆数重み付け固定効果解析を実施した。 【曝露】患者をデキサメタゾン、ヒドロコルチゾン、メチルプレドニゾロンいずれかの全身投与群(678例)と通常治療またはプラセボ投与群(1025例)に無作為に割り付けた。 【主な転帰評価項目】主要転帰評価項目は、無作為化後28日時の全死因死亡率とした。治験責任医師が定義した重篤な有害事象を副次評価項目とした。 【結果】計1703例(年齢中央値60歳[四分位範囲52~68歳]、女性488例[29%])を解析の対象とした。試験7件中6件で死亡率の結果が「低度」、1件は無作為化法が「懸念あり」とバイアスリスクを評価した。5試験が28日時の死亡率、1試験が21日死亡率、1試験が30日死亡率を報告していた。コルチコステロイドに割り付けた678例中222例、通常治療またはプラセボに割り付けた1025例中425例が死亡した(OR要約値0.66[95%CI 0.53~0.82]、固定効果メタ解析に基づくP<0.001)。試験間の結果は不一致性がほとんどなく(I2=15.6%、異質性のP=0.31)、変量効果メタ解析に基づくOR要約値は0.70(95%CI 0.48~1.01、P=0.053)であった。 死亡の関連を示す固定効果OR要約値は、通常治療またはプラセボと比較して、デキサメタゾンで0.64(95%CI 0.50~0.82、P<0.001、3試験計1282例中527例死亡)、ヒドロコルチゾで0.69(同0.43~1.12、P=0.13、3試験計374例中94例死亡)、メチルプレドニゾロンで0.91(同0.29~2.87、P=0.87、1試験47例中26例死亡)であった。重篤な有害事象が報告された6試験のうち、64件がコルチコステロイド投与群354例、80件が通常治療またはプラセボ群342例に発生した。 【結論および意義】COVID-19重症患者を対象とした臨床試験の前向きメタ解析では、通常治療またはプラセボと比較すると、全身性副腎皮質ステロイド投与によって28日全死因死亡率が低下した。 第一人者の医師による解説 COVID-19に対するステロイド療法 当面デキサメタゾン 6mg投与が望ましい 藤島 清太郎 慶應義塾大学医学部総合診療教育センター・センター長 MMJ. April 2021;17(2):42 新型コロナウイルス感染症(COVID-19)パンデミックの中、世界でさまざまな臨床試験が同時進行的に行われ、その成果が日々オンライン上で公表されている。治療薬としては抗ウイルス薬に加えて、副腎皮質ステロイドやトシリズマブなどの抗炎症薬も有力な候補となっている。 本論文は、2020年2~6月に12カ国で実施された重症COVID-19患者を対象とした7件のランダム化対照試験(RCT)の世界保健機関(WHO)による前向きメタ解析である。本解析には1,703人の患者が組み入れられ、ステロイド投与群で有意な28日死亡率の低下を認め、重篤な有害事象には差がなかった。サブ解析では、投与薬別でデキサメタゾン(DEX)(3試験、1,282人)でのみ有意差があり、ヒドロコルチゾン(3試験、374人)とメチルプレドニゾロン(1試験、47人)では差を認めなかった。以上の結果は、解析対象患者の57%を英国のRECOVERY試験参加者が占めていることを踏まえて解釈する必要がある。同 RCTではDEX 6mg/日の最長10日間投与による有意な28日死亡率改善が示された。サブ解析では人工呼吸管理群、酸素投与群での有効性が示唆されている。一方、本論文のメタ解析では、より高用量や他のステロイドの有用性は明らかになっていない。 また、COVID-19重症例は敗血症、急性呼吸促迫症候群(ARDS)を高率に合併するが、抗炎症薬の適応や投与量の判断に当たっては、両病態に対する抗炎症薬開発の歴史も踏まえる必要がある。すなわち、過去半世紀以上にわたり膨大な数の抗炎症薬 RCTが実施された中で、最近の一部ステロイド試験を除き生存率改善効果が認められず、サイトカインストームに代表される過剰炎症反応が病態形成や不良転帰に関与しているという単純仮説は両病態に必ずしも当てはまらない可能性がある。COVID-19病態下では、炎症性サイトカイン値が他の原因によるARDSや敗血症よりもむしろ低く、DEXが免疫異常の修復により効果を発揮している可能性も最近示されている(1),(2)。以上を勘案すると、現状ではステロイド療法の適応と判断したCOVID-19患者に対し、DEX 6mg/日の投与が妥当と思われる。今後は、患者要因、重症度、診断マーカーなどに基づく治療の個別化に向けたさらなる研究の遂行が望まれる。 本論文以降も新たなメタ解析が逐次公表されており、これらの最新版を活用し(3)、不応性の病態に探索的な治療を行った場合は、その結果にかかわらずエビデンスとして集積・発信していくことが、将来的な治療成績の向上につながることを申し添えたい。 1. Sarma A, et al. Res Sq. 2021:rs.3.rs-141578. 2. Leisman DE, et al. Lancet Respir Med. 2020;8(12):1233-1244. 3. 日本版敗血症診療ガイドライン 2020 特別委員会 . COVID-19 薬物療法に関する Rapid/Living recommendations 第三版 . 2021.
小児期の鉛暴露とMRIで測定した中年期の脳構造の統合性の関連
小児期の鉛暴露とMRIで測定した中年期の脳構造の統合性の関連
Association of Childhood Lead Exposure With MRI Measurements of Structural Brain Integrity in Midlife JAMA. 2020 Nov 17;324(19):1970-1979. doi: 10.1001/jama.2020.19998. 原文をBibgraph(ビブグラフ)で読む 上記論文の日本語要約 【重要性】小児期の鉛曝露に脳の発達の阻害と関連があるが、脳構造の統合性にもたらす長期的な影響が未だ明らかになっていない。 【目的】小児期の鉛暴露によってMRIで測定した中年期の脳構造の統合性が低下するという仮説を検証すること。 【デザイン、設定および参加者】ダニーデン研究では、ニュージーランドで1972~1973年に出生した集団代表コホート(解析対象564例)を45歳まで(2019年4月まで)追跡した。 【曝露】11歳時に測定した小児期の鉛曝露。 【主要評価項目】45歳時のMRIで評価した脳構造の統合性(主要評価項目):灰白質(皮質厚、表面積、海馬体積)、白質(大脳白質病変、拡散異方性[理論的範囲0{完全に等方性拡散}~100{完全に異方性拡散}]およびBrain Age Gap Estimation[BrainAGE、実年齢と機械学習アルゴリズムで推定した脳年齢の差の複合指標{0:脳年齢と実年齢が同じ;正数は脳年齢が高く、負数は脳年齢が若い}])。45歳時の認知機能をウェクスラー成人知能検査第4版(WAIS-Ⅳ、IQ範囲40~160点、平均100点に標準化)を用いて客観的に、情報提供者および自己報告(zスコア単位;尺度平均0[SD 1])で主観的に評価した。 【結果】最初の参加者1,037例中997例が45歳時点で生存しており、そのうち564例(男性302例、女性262例)が11歳時に鉛検査を受けていた(追跡期間中央値34[四分位範囲33.7~34.7]年)。11歳時点の血中鉛濃度が平均10.99(SD 4.63)μg/dLであった。共変量で調整後、小児期の血中鉛濃度が5μg/dL増加するたびに、皮質表面積の1.19cm2減少(95%CI -2.35~-0.02cm2、P=0.05)、海馬体積の0.10cm3減少(95%CI -0.17~-0.03cm3、P=0.006)、拡散異方性の低下(b=-0.12、95%CI -0.24~-0.01、P=0.04)、45歳時のBrainAGEの0.77歳上昇(95%CI 0.02~1.51、P=0.05)が認められた。血中鉛濃度と対数変換した白質病変体積(b=0.05 log mm3、95%CI -0.02~0.13 log mm3、p=0.17)や平均皮質厚(b=-0.004mm、95%CI -0.012~0.004mm、p=0.39)との間に統計的な有意差は認められなかった。小児期の血中鉛濃度が5μg/dL増加するたびに、45歳時のIQスコア2.07低下(95%CI -3.39~-0.74、P=0.02)、情報提供者が評価した認知機能障害スコア0.12増加(95%CI 0.01~0.23、P=0.03)との有意な関連が認められた。小児期の血中鉛濃度と自己報告による認知的問題との間に統計学的有意な関連は認められなかった(b=-0.02ポイント、95%CI -0.10~0.07、P=0.68)。 【結論および意義】中央値で34年追跡したこの縦断的コホート研究では、小児期の血中鉛濃度高値にMRIを用いた脳構造の測定項目との関連が見られ、中年期の脳構造の統合性が低下することが示唆された。多重比較のため、第1種の過誤が生じた結果があると考えられる。 第一人者の医師による解説 脳表面積や海馬容積を減少させ 成人期脳機能と負の相関があることを示唆 高橋 孝雄(教授)/三橋 隆行(専任講師) 慶應義塾大学医学部小児科学教室 MMJ. April 2021;17(2):56 小児期の化学物質などへの曝露が知能に悪影響を与えることがこれまで指摘されてきた。具体的には、水銀、鉛、多環式芳香族炭化水素やダイオキシン類の低濃度曝露が小児の知能に悪影響を与える可能性が報告されている。鉛については、1980年代まで使用された有鉛ガソリンによる大気汚染の影響や、現在禁止されている鉛を含有した白色塗料の経口摂取があり、小児の血中鉛濃度と知能発達との関連性が報告されてきた。 本論文は、ニュージーランド・ダニーデンで1972~73年に出生し、11歳時に血中鉛濃度を測定された出生コホートを対象とした縦断的前向きコホート研究(Dunedin Study)の報告である。先行解析では鉛曝露量が増えると38歳時の知能指数が低下する相関性が示されていたことから(1)、生後小児期の血中鉛濃度と45歳時の脳構造異常との関連性を検討した。脳MRI画像をもとに各脳構造を計測した結果、血中鉛濃度の上昇に伴い脳表面積と海馬容積が減少することが明らかとなった。さらに、脳機能の参考指標となる拡散強調画像により得られる異方性比率(global fractional anisotropy)の低下や、機械学習を用いた人工知能による推定脳年齢が悪化する点も判明した。 本研究の評価できる点としては、他のコホート研究に比べ社会経済的背景による鉛曝露量の偏りがない点が挙げられる。他の先行研究では、高収入の家庭の子どもはそうでない子どもに比べ鉛曝露量が多くても知能指数が下がりにくいといった報告(2)があるが、今回、家庭の経済状況や教育レベルといったバイアスを排除し、純粋な鉛曝露の影響を明らかにできた点が評価できる。 一方、本研究の限界として、今回検出された脳構造の異常が小児期にすでに存在したのか、あるいは成人に至る過程で生じたのか不明な点が挙げられる。また、仮に生後の鉛曝露のみの影響を検出しているとしても、脳の成熟化の異常なのか、完成された脳構造の変性による表面積の減少などなのかについては不明な点が残されている。 以上の限界はあるものの、本成果は小児期の鉛曝露が小児期のみならず成人期の認知機能に悪影響を与える可能性を解剖学的な脳の構造異常により裏付けたものと評価できる。被験者の主観的な認知機能には変化がなく、また偽陽性の可能性は残されてはいるものの、鉛曝露が中年期の認知機能を悪化させている可能性が危惧される。今後は、他の化学物質についても同様の検討が行われることが必要であろう。 1. Reuben A, et al. JAMA. 2017;317 (12):1244-1251. 2. Marshall AT, et al. Nat Med. 2020;26(1):91-97.
院外亜硝酸ナトリウム投与が心停止後病院到着までの生存率にもたらす効果 無作為化臨床試験
院外亜硝酸ナトリウム投与が心停止後病院到着までの生存率にもたらす効果 無作為化臨床試験
Effect of Out-of-Hospital Sodium Nitrite on Survival to Hospital Admission After Cardiac Arrest: A Randomized Clinical Trial JAMA. 2021 Jan 12;325(2):138-145. doi: 10.1001/jama.2020.24326. 原文をBibgraph(ビブグラフ)で読む 上記論文の日本語要約 【重要性】心停止モデル動物で、蘇生時に亜硝酸ナトリウムを投与することによって生存率が改善することが認められているが、ヒトを対象とした臨床試験で有効性が評価されていない。 【目的】院外心停止の蘇生時に救急医療隊員が亜硝酸ナトリウムを非経口投与することによって病院到着までの生存率が改善するかを明らかにすること。 【デザイン、設定および参加者】米ワシントン州キング郡で、心室細動の有無を問わず院外心停止を来した成人1502例を対象とした第II相二重盲検プラセボ対照無作為化臨床試験。2018年2月8日から2019年8月19日の間に救急医療隊員が蘇生処置を実施した患者を登録した。2019年12月31日までに追跡調査とデータ抽出を終えた。 【介入】適格な院外心停止患者を亜硝酸ナトリウム45mg(500例)、亜硝酸ナトリウム60mg(498例)、プラセボ(499例)を投与する群に(1対1対1の割合で)無作為に割り付け、蘇生処置実施中にできるだけ早くボーラス投与した。 【主要評価項目】主要評価項目は病院到着時の生存率とし、片側仮説検定で評価した。副次評価項目は、院外変数(自己心拍再開率、再心停止率、血圧維持を目的としたノルエピネフリン使用)と院内変数(退院時の生存率、退院時の神経学的転帰、24、48、72時間までの累積生存率、集中治療室在室日数)とした。 【結果】無作為化した院外心停止患者1502例(平均年齢64歳[SD 17]、女性34%)のうち99%が試験を完了した。全体で、亜硝酸ナトリウム45mg群の205例(41%)、同60mg群の212例(43%)、プラセボ群の218例(44%)が病院到着まで生存していた。45mg群とプラセボ群の平均差は-2.9%(片側95%CI -8.0%~∞、P=0.82)、60mg投与群とプラセボ群の平均差は-1.3%(片側95%CI -6.5%~∞、P=0.66)であった。事前に規定した副次評価項目7項目には有意差は認められず、退院時の生存者数が亜硝酸ナトリウム45mg群66例(13.2%)、同60mg群72例(14.5%)、プラセボ群74例(14.9%)で、亜硝酸ナトリウム45mg群とプラセボ群の平均差は-1.7%(両側検定の95%CI -6.0~2.6%、P=0.44)、同60mg群とプラセボ群の平均差は-0.4%(同-4.9~4.0%、P=0.85)であった。 【結論および意義】院外心停止を来した患者で、亜硝酸ナトリウムの投与は、プラセボと比較して病院到着時の生存率が有意に改善することはなかった。この結果から、院外心停止の蘇生時に亜硝酸ナトリウムの使用は支持されない。 第一人者の医師による解説 心肺停止蘇生後の神経障害抑制 他の薬剤も含めさらなる研究の進展を期待 今井 寛 三重大学医学部附属病院救命救急・総合集中治療センター センター長・教授 MMJ. April 2021;17(2):58 心停止患者において脳神経障害は主な死因であり、蘇生された患者のほとんどは意識を取り戻すことはない。心肺蘇生法の進歩にもかかわらず、米国で2005~15年に収集されたデータによると、院外心停止後に自己心拍再開した患者の80%以上が退院前に死亡している。亜硝酸投与療法は虚血と再灌流後の細胞障害とアポトーシスを抑制し、また多数の動物モデルにおいて細胞保護効果を認めている。げっ歯類の心停止モデルでは、蘇生中に低用量亜硝酸塩を単回静脈内投与すると生存率が48%向上したと報告されている。他の動物モデルでは、心停止後の再灌流初期の亜硝酸塩濃度が10~20μMの間であれば生存率の改善と関連していることが示唆された。院外心停止患者125人を対象とした第1相非盲検試験の結果では、心停止の場合、蘇生中に亜硝酸ナトリウム45mgまたは60mgを投与すると投与後10~15分以内に血清中亜硝酸濃度が10~20μMに到達した(1)。 本研究はこれらの知見に基づき、院外心肺停止の傷病者に対して蘇生中に亜硝酸ナトリウムを急速静注することによって生存入院率が上がるかどうかについて第2相無作為化二重盲検プラセボ対照試験として検討された。ワシントン州キング郡で2018年2月8日~19年8月19日に登録された院外心停止患者(すべての初期波形を対象、外傷を除く)は1,502人で、亜硝酸ナトリウム45mg群(500人)、60mg群(498人)、プラセボ群(生食、499人)に無作為に割り付けられ、救急隊員が蘇生中にできる限り早く静注した。その結果、生存入院した患者は亜硝酸ナトリウム45mg群205人(41%)、60mg群212例(43%)、プラセボ群218人(44%)であり、プラセボ群との平均差は45mg群で-2.9%(片側95% CI, -8.0%~∞;P=0.82)、60mg群で-1.3%(片側95% CI, -6.5%~∞;P=0.66)といずれも有意差を認めなかった。事前に設定した7つの副次評価項目(再心停止率、救急隊員によるノルアドレナリン使用、自己心拍再開率、集中治療室[ICU]滞在日数、24・48・72時間までの累積生存率、退院までの生存率、および退院時の神経学的状態)についても有意差を認めなかった。したがって、著者らは院外心肺停止に対する蘇生中の亜硝酸ナトリウム静注は支持されないと結論付けている。 心肺停止蘇生後の神経障害抑制は重要な課題であり、亜硝酸ナトリウムだけでなく他の薬剤も含めてさらなる研究が進むことを期待する。 1. Kim F, et al. Circulation. 2007;115(24):3064-3070.
深層学習と標準法を用いた生殖細胞系列遺伝子検査による前立腺がんと悪性黒色腫患者の病原性変異検出の比較
深層学習と標準法を用いた生殖細胞系列遺伝子検査による前立腺がんと悪性黒色腫患者の病原性変異検出の比較
Detection of Pathogenic Variants With Germline Genetic Testing Using Deep Learning vs Standard Methods in Patients With Prostate Cancer and Melanoma JAMA. 2020 Nov 17;324(19):1957-1969. doi: 10.1001/jama.2020.20457. 原文をBibgraph(ビブグラフ)で読む 上記論文の日本語要約 【重要性】検出可能な生殖細胞系列変異があるがん患者が10%に満たず、これは病原性変異の検出が不完全であることが原因の一つになっていると思われる。 【目的】深層学習によってがん患者の病原性生殖細胞系列変異がさらに多く特定できるかを評価すること。 【デザイン、設定および参加者】2010年から2017年の間に米国と欧州で組み入れた前立腺がんと悪性黒色腫の2つのコホートの便宜的標本で、標準生殖細胞検出法と深層学習法を検討する横断的研究。 【曝露】標準法または深層学習法を用いた生殖細胞系列変異の検出。 【主要評価項目】主要評価項目は、がん素因遺伝子118個の病原性変異の検出能とし、感度、特異度、陽性適中率(PPV)および陰性適中率(NPV)で推定した。副次評価項目は、米国臨床遺伝・ゲノム学会(ACMG)が指定している治療可能な遺伝子59個および臨床的に重要なメンデル遺伝子5197個の検出能とした。感度および真の特異度は、標準基準がないため算出できなかったが、真陽性変異と真陰性変異の割合を推定することとし、いずれかの方法で有効と判断した全変異で構成された参照変異一式の中から各方法を用いて特定した。 【結果】前立腺がんコホートは1072例(診断時の平均[SD]年齢63.7[7.9]歳、欧州系857例[79.9%])、悪性黒色腫コホートは1295例(診断時の平均[SD]年齢59.8[15.6]歳、女性488例[37.7%]、欧州系1060例[81.9%])を組み入れた。深層学習法の法が標準法よりも、がん素因遺伝子の病原性変異が多く検出された(前立腺がん:198個 vs. 182個、悪性黒色腫:93個 vs. 74個)。感度は、前立腺がん94.7% vs. 87.1%(差7.6%、95%CI 2.2~13.1%)、悪性黒色腫74.4% vs. 59.2%(同15.2%、3.7~26.7%)、特異度は、前立腺がん64.0% vs. 36.0%(同28.0%、1.4~54.6%)、悪性黒色腫63.4% vs. 36.6%(同26.8%、95%CI 17.6~35.9%)、PPVは前立腺がん95.7% vs. 91.9%(同3.8%、-1.0~8.4%)、悪性黒色腫54.4% vs. 35.4%(同19.0%、9.1~28.9%)、NPVは前立腺がん59.3% vs. 25.0%(差34.3%、10.9~57.6%)、悪性黒色腫80.8% vs. 60.5%(同20.3%、10.0~30.7%)であった。ACMG遺伝子をみると、前立腺がんコホートでは両方法の感度に有意差がなかったが(94.9 vs. 90.6%[差4.3%、95%CI -2.3~10.9%])、悪性黒色腫コホートでは、深層学習法の方が感度が高かった(71.6% vs. 53.7%[同17.9%、1.82~34.0%])。深層学習法はメンデリアン遺伝子でも感度が高かった(前立腺がん99.7% vs. 95.1%[同4.6%、3.0~6.3%]、悪性黒色腫91.7% vs. 86.2%[同5.5%、2.2~8.8%])。 【結論および意義】前立腺がん患者と悪性黒色腫患者を組み入れた2つの独立のコホートから成る便宜的標本で、深層学習を用いた生殖細胞系列遺伝子検査による病原性変異検出の感度および特異度が、現行の標準遺伝子検査法よりも高かった。臨床転帰の観点からこの結果の意義を理解するには、さらに詳細な研究が必要である。 第一人者の医師による解説 バリアントの検出手法は 新たな手法によって改善する余地あり 水上 圭二郎(研究員)/桃沢 幸秀(チームリーダー) 理化学研究所生命医科学研究センター基盤技術開発研究チーム MMJ. June 2021;17(3):92 現在行われている遺伝学的検査の多くは、患者のDNAを次世代シークエンサーと呼ばれる機械を用いて解読し、そのデータをコンピュータを用いて解析することによって、患者の遺伝子における塩 基配列の違い(バリアント)を検出する。このようにして検出されたバリアント情報は、疾患との関連性などの臨床的な解釈を付与された後、検査結 果として報告される。医師はこの検査結果に基づき疾患の発症予測や予後判定、治療方針の決定などを行う。遺伝学的検査の過程をバリアント検出と臨床的な解釈付けの2つに分けた場合、一般的に前者は高い正確性があると認識されているため、多くの研究は後者に焦点を当てたものになっているのが現状である。 しかしながら、前者についても重要な研究が行われており、2018年にGoogle Brainチームという人工知能の研究チームより、DeepVariantというバリアント検出に深層学習を用いたソフトウエア が報告された1 。このソフトウエアは、次世代シークエンサーの生データからバリアントを検出するまでの途中過程で生じる画像を大量に学習し、未知のバリアント検出に利用するという、とてもユニークな手法を用いている。本研究では、前立腺がんとメラノーマ患者由来の大規模データを用いて、このソフトウエアとヒトゲノム解析において世界の中心的な役割を果たしてきているBroad Instituteが開発したGenome Analysis Toolkit(GATK)のHaplotypeCallerという現在最も汎用されているソフトウエアを、臨床的に重要な遺伝子のバリアントに着目し、バリアント保有者数 、感度 、特異度 、陽性・陰性的中率について比較した。 これら2つの手法を比較した結果、特にBRCA1/2などの遺伝性腫瘍関連遺伝子群において、DeepVariantは全評価項目において従来法のHaplotypeCallerに比べ性能が良いことが示された。例えば、前立腺がん患者データにおいて検出されたバリアント保有者は、DeepVariantで198人、従来法で182人だった。この要因の1つとして、DeepVariantでは集団において保有者が1人しかいないような極めて頻度が低いバリアントも高感度に検出できたことが挙げられる。一方、 DeepVariantだけ検出できないバリアントも存在したことから、検出感度を最大にするためには両者の併用も考慮する必要があるとしている。 以上のように、バリアントの検出手法はすでに確立されたものと一般的には考えられているが、深層学習など新たな手法を用いることによってま だまだ改善する余地が残されていることが、本論文では示されていた。 1. Poplin R, et al. Nat Biotechnol. 2018;36(10):983-987.
高心血管リスク患者の主要有害心血管イベントに対する高用量オメガ3脂肪酸とコーン油の比較 STRENGTH無作為化臨床試験
高心血管リスク患者の主要有害心血管イベントに対する高用量オメガ3脂肪酸とコーン油の比較 STRENGTH無作為化臨床試験
Effect of High-Dose Omega-3 Fatty Acids vs Corn Oil on Major Adverse Cardiovascular Events in Patients at High Cardiovascular Risk: The STRENGTH Randomized Clinical Trial JAMA. 2020 Dec 8;324(22):2268-2280. doi: 10.1001/jama.2020.22258. 原文をBibgraph(ビブグラフ)で読む 上記論文の日本語要約 【重要性】オメガ3脂肪酸エイコサペンタエン酸(EPA)およびドコサヘキサエン酸(DHA)によって心血管リスクが低下するかはいまだに明らかになっていない。 【目的】EPAとDHA(オメガ3 CA)のカルボン酸製剤が心血管転帰にもたらす効果、および心血管リスクが高いアテローム性異常脂質血症患者の脂質および炎症マーカーにもたらす文書化された良好な作用を明らかにすること。 【デザイン、設定および参加者】心血管リスクが高く、高トリグリセリド血症があり、高比重リポ蛋白コレステロール(HDL-C)が低いスタチン治療中の患者でオメガ3 CAとコーン油を比較した多施設共同二重盲検無作為化試験(2014年10月30日から2017年6月14日の間に登録、2020年1月8日に試験終了、最終患者診療日2020年5月14日)。北米、欧州、南米、アジア、オーストラリア、ニュージーランドおよび南アフリカの22カ国の大学病院および市中病院675施設で計1万3078例を無作為化した。 【介入】スタチンを含む通常治療に加えてオメガ3 CA 4g/日(6539例)と不活性対照のコーン油(6539例)を投与するグループに患者を無作為に割り付けた。 【主要評価項目】主要有効性評価項目は、心血管死、非致命的心筋梗塞、非致命的脳卒中、冠動脈血行再建、入院を要する不安定狭心症の複合とした。 【結果】(予定していたイベント1600件のうち)1384例に主要評価項目のイベントが発生した時点で、オメガ3 CAの臨床的便益の可能性が対照のコーン油よりも低いことが示唆された中間解析に基づき、試験は早期に中止された。1万3078例(平均年齢[SD]62.5[9.0]歳、女性35%、糖尿病70%、低比重リポタンパク質[LDL]コレステロール中央値75.0mg/dL、トリグリセリド中央値240mg/dL、HDL-C中央値36mg/dL、高感度CRP中央値2.1mg/L)のうち1万2633例(96.6%)が試験を完了し、主要評価項目の発生状況を確認した。主要評価項目は、オメガ3 CAで治療した患者の785例(12.0%)、コーン油で治療した患者の795例(12.2%)に発生した(ハザード比0.99[95%CI 0.90~1.09]、P =0.84)。オメガ3 CA群(24.7%)の方がコーン油群(14.7%)よりも、消化器系有害事象の発現率が高かった。 【結論および意義】スタチンで治療している心血管リスクが高い患者で、通常治療へのオメガ3 CA追加は、コーン油と比較した主要有害心血管イベントの複合転帰の有意差がなかった。この結果は、高リスク患者の主要有害心血管イベント減少を目的としたこのオメガ3脂肪酸製剤の使用を支持するものではない。 第一人者の医師による解説 評価が分かれるω -3脂肪酸製剤 さらなる検証と代理エンドポイントの再検討が必要 原 眞純 帝京大学医学部附属溝口病院・病院長、第四内科学講座主任教授 MMJ. June 2021;17(3):79 大規模介入試験の結果から、高用量のスタチンで低比重リポ蛋白コレステロール(LDL-C)を十分に低下させても、心血管疾患(CVD)発症は30%程度しか減少せず、LDL-C低下だけでは解決しない残余リスクが指摘されてきた。高トリグリセライド(TG)血症は残余リスクの1つとされフィブラートやω-3脂肪酸製剤などを用いてTGを低下させる介入が試みられてきた。 本研究では、CVD既往のある2次予防患者を50%以上、糖尿病患者も約70%含む高リスク群に、最大限のスタチン投与に加えてエイコサペンタエン酸(EPA)とドコサヘキサエン酸(DHA)の両方を含むω-3脂肪酸のカルボン酸製剤を追加投与し、心血管イベント抑制効果を検証している。その結果、心血管複合エンドポイント減少効果を示すことができない見通しとなり、研究は早期終了となった。 ω-3脂肪酸製剤に関する先行大規模試験の結果には相違がある。効果を示した研究としては、EPAのみを投与した日本のJELIS研究(1)と海外のREDUCE-IT研究(2)があるが、その他多くの研究では心血管イベント抑制効果は証明されなかった。本研究では、従来のエステル化製剤と異なり膵リパーゼによる加水分解を経ずに吸収されるカルボン酸製剤が用いられたが、この製剤が効果に影響を及ぼしたかどうかについては明らかでない。また、DHAが動脈硬化を促進するという知見は報告されていないが、DHAを含むω-3脂肪酸製剤では心血管イベント減少は証明されていない。なお、REDUCE-IT研究では、対照群においてC反応性蛋白(CRP)の上昇がみられることから、対照薬として用いた鉱物油の投与により心血管イベントが増加した結果、EPA群との有意差が得られたとの見方もある。このため、本研究では対照薬としてコーン油が選択されている。 本研究では、脂質改善効果やC反応性蛋白の低下など、ω-3脂肪酸製剤に期待される代理エンドポイントの改善は得られている。にもかかわらず心血管イベントが減少しなかったことは、これらが代理エンドポイントとして妥当でない可能性が示唆される。メンデルランダム化研究の結果などから、TGを低下させる介入がCVD発症を抑制することが示唆されているものの、TG上昇がどのような機序で動脈硬化に関わっているかには諸説あり、TG上昇に伴って増加するレムナントやsmall dense LDLなどが真の動脈硬化促進因子であることも示唆されている。評価が分かれるω-3脂肪酸製剤については、今後さらなる検証が必要であると同時に、ペマフィブラートなど、残余リスクに対する他の介入でも作用機序や有効性の指標とすべき代理エンドポイントの再検討が必要であろう。 1. Saito Y, et al. Atherosclerosis. 2008;200(1):135-140. 2. Bhatt DL, et al. N Engl J Med. 2019;380(1):11-22.
急性虚血性脳卒中に用いる機械的血栓除去術単独と機械的血栓除去術+静脈内血栓溶解療法併用による機能的転帰に対する効果の比較 SKIP無作為化臨床試験
急性虚血性脳卒中に用いる機械的血栓除去術単独と機械的血栓除去術+静脈内血栓溶解療法併用による機能的転帰に対する効果の比較 SKIP無作為化臨床試験
Effect of Mechanical Thrombectomy Without vs With Intravenous Thrombolysis on Functional Outcome Among Patients With Acute Ischemic Stroke: The SKIP Randomized Clinical Trial JAMA. 2021 Jan 19;325(3):244-253. doi: 10.1001/jama.2020.23522. 原文をBibgraph(ビブグラフ)で読む 上記論文の日本語要約 【重要性】急性大血管閉塞性脳卒中で、機械的血栓除去術に静脈内血栓溶解療法を併用する必要があるかどうか明らかになっていない。 【目的】機械的血栓除去術単独が、脳梗塞後の良好な転帰で静脈内血栓溶解療法+機械的血栓除去術に対して非劣性を示すかを明らかにすること。 【デザイン、設定および参加者】2017年1月1日から2019年7月31日の間に日本の23の病院ネットワークで組み入れた大血管閉塞に伴う急性期脳梗塞患者204例を対象とした医師主導型多施設共同無作為化非盲検非劣性臨床試験であり、最終経過観察日は2019年10月31日であった。 【介入】患者を機械的血栓除去術単独群(101例)と静脈内血栓溶解療法(アルテプラーゼ0.6mg/kg)+機械的血栓除去術併用群(103例)に無作為に割り付けた。 【主要評価項目】有効性の主要評価項目は、90日時の修正ランキン尺度スコア0~2点(0点[無症状]~6点[死亡])と定義した転帰良好とし、非劣性マージンオッズ比0.74、片側有意閾値0.025(97.5%CI)で評価した。副次評価項目に90日死亡率などの7項目を規定した。あらゆる脳出血、36時間以内の症候性脳出血などの4項目を安全性の評価項目とした。 【結果】204例(年齢中央値74歳、男性62.7%、National Institutes of Health Stroke Scaleスコア中央値18点)のうち全例が試験を完遂した。機械的血栓除去術単独群の60例(59.4%)、静脈内血栓溶解療法+機械的血栓除去術併用群の59例(57.3%)の転帰が良好であり、群間で有意差はなかった(差2.1%[片側の97.5%CI -11.4%~∞]、オッズ比1.09[同0.63~∞]、非劣性のP=0.18)。7項目の有効性評価項目と4項目の安全性評価項目のうち、90日死亡率(8例[7.9%] vs. 9[8.7%]、差-0.8%[95%CI -9.5%~7.8%]、オッズ比0.90[同0.33~2.43]、P>0.99)などの10項目に有意差がなかった。機械的血栓除去術単独群の方が併用群よりもあらゆる脳内出血の発症率が低かった(34例[33.7%] vs. 52例[50.5%]、差-16.8%[同-32.1%~1.6%]、オッズ比0.50[同0.28~0.88]、P=0.02)。両群間で症候性脳内出血の頻度に有意差はなかった(6例[5.9%] vs. 8[7.7%]、差-1.8%[同-9.7%~6.1%]、オッズ比0.75[同0.25~2.24]、P=0.78)。 【結論および意義】急性大血管閉塞に伴う脳梗塞に用いる機械的血栓除去術単独は、機能的転帰に関して、静脈内血栓溶解療法と機械的血栓除去術の併用に対する非劣性が示されなかった。しかし、効果推定の信頼区間が広かったため、劣性であるとの結論を示すこともできなかった。 第一人者の医師による解説 機械的血栓回収療法の施行前の t-PA投与が不要になる可能性を示唆 木村 和美 日本医科大学大学院医学研究科神経内科分野大学院教授 MMJ. June 2021;17(3):78 ガイドラインには、「機械的血栓回収療法を行うときは、t-PA静注療法の適応例に対してはt-PA静注療法を優先すること(グレード A)」と記載されている。t-PA静注療法は、脳主幹動脈閉塞の早期再開通率が高くない上に、薬剤による出血合併症のリスクがあり、また、治療に要する時間、複数の医療スタッフの必要性など、コスト・ベネフィットが高くない。以上の理由から、この数年来「機械的血栓回収療法の施行前に、t-PA投与が必要か否か」が急性期脳梗塞の治療上解決すべき大きな命題であった。 本論文で報告されたSKIP研究の目的は、脳主幹動脈閉塞を伴う急性期脳梗塞患者を対象としたラ ンダム化比較試験(RCT)により機械的血栓回収療法単独と併用療法(機械的血栓回収療法+t-PA静注療法)の間で患者転帰良好に差があるか否かを明らかにすることである。目標症例数は、過去の文献より算出し200人が適切と判断した。適格基準は(1)年齢18〜85歳(2)急性期脳梗塞(3)発症前mRS(Rankin Scale)スコア2以下(4)閉塞血管は内頸動脈と中大脳動脈(5)初診時NIHSS(National Institutes of Health Stroke Scale)は6以上(6)ベースラインASPECTS(Alberta Stroke Program Early CT Score)6以上(7)発症から4時間以内に穿刺が見込まれる患者である。主要評価項目は発症後90日の転帰良好(m RS 0〜2)の割合とし、機械的血栓回収療法単独が併用療法に対して非劣性であるか否かを検証した(非劣性マージンのオッズ比0.74)。有害事象評価項目は、発症後36時間の頭蓋内出血の割合とした。 結果は、患者204人の登録があり、機械的血栓回収療法単独群が101人、併用群が103人であった。患者背景は2群間で差はなく、均等に割り付けされていた。主要評価項目の発症後90日のm RS 0〜2の割合は、単独群59.4%と併用群57.3%であり、補正なしのロジスティック回帰モデルにおけるオッズ比は1.09(97.5%CI,0.63〜∞;P=0.17)で、機械的血栓回収療法単独群の方が転帰良好例は多いが、非劣性は証明できなかった。発症後36時間の頭蓋内出血の割合は、単独群が34人(33.7%)、併用群が50人(50.5%)と、併用群の方が有意に多かった(P=0.02)。以上より、脳主幹動脈閉塞例には、t-PA投与なしに可及的速やかに機械的血栓回収療法を行う方が、患者の転帰が良好になる可能性が示されたが、非劣性は証明できなかった。中国から同様な研究が2件(DIRECT-MT(1)、DEVT(2))報告されており、非劣性を証明している。そのほか、世界では3件のRCTがon goingであり、結果が楽しみである。SKIP研究は、機械的血栓回収療法の施行前に、t-PA投与が不要になる可能性を示唆した研究で、今後、脳梗塞急性期治療にパラダイムシフトが起こるかもしれない。 1. Yang P, et al. N Engl J Med. 2020;382(21):1981-1993. 2. Zi W,et al.JAMA.2021;325(3):234-243.
脳性麻痺患者のエクソーム解析の分子診断率
脳性麻痺患者のエクソーム解析の分子診断率
Molecular Diagnostic Yield of Exome Sequencing in Patients With Cerebral Palsy JAMA. 2021 Feb 2;325(5):467-475. doi: 10.1001/jama.2020.26148. 原文をBibgraph(ビブグラフ)で読む 上記論文の日本語要約 【重要性】脳性麻痺は、運動や姿勢に影響を及ぼすよく見られる神経発達障害であり、他の神経発達障害と併発することが多い。脳性麻痺は出生時仮死に起因することが多いが、最近の研究から、仮死が脳性麻痺症例に占める割合が10%未満であることが示唆されている。 【目的】脳性麻痺患者でエクソーム解析の分子診断率(病原性変異および病原性の可能性が高い変異の検出率)を明らかにすること。 【デザイン、設定および参加者】2012~18年のデータを用いた臨床検査紹介コホートと2007~17年のデータを用いた医療機関主体コホートから組み入れた脳性麻痺患者の後ろ向きコホート研究。 【曝露】コピー数変異を検出するエクソーム解析。 【主要評価項目】主要評価項目は、エクソーム解析の分子診断率とした。 【結果】臨床検査紹介コホート1345例の年齢中央値は8.8歳(四分位範囲4.4~14.7歳、範囲0.1~66歳)で、601例(45%)が女性であった。医療機関主体コホート181例の年齢中央値は41.9歳(同28.0~59.6歳、範囲4.8~89歳)で、96例(53%)が女性であった。エクソーム解析の分子診断率は、臨床検査紹介コホートで32.7%(95%CI 30.2~35.2%)、医療機関主体のコホートでは10.5%(同6.0~15.0%)であった。分子診断率は、知的障害、てんかん、自閉症スペクトラム障害がない患者の11.2%(同6.4~16.2%)から、上記の3疾患がある患者の32.9%(同25.7~40.1%)まで幅があった。遺伝子229個(1526例のうち29.5%)から病原性変異および病原性の可能性が高い変異が同定され、そのうち2例以上(1526例のうち20.1%)に遺伝子86個の変異があり、両コホートから変異がある遺伝子10個が独立して同定された。 【結論および意義】エクソーム解析を実施した脳性麻痺患者の2つのコホートで、病原性変異および病原性の可能性が高い変異の有病率は、主に小児患者から成るコホートで32.7%、主に成人患者から成るコホートで10.5%であった。一連の結果の臨床的意義を理解するために、さらに詳細な研究が必要である。 第一人者の医師による解説 脳性麻痺児にエクソーム解析が行われれば、3分の1で病的バリアントを検出 武内 俊樹 慶應義塾大学医学部小児科専任講師 MMJ. June 2021;17(3):86 脳性麻痺は、脳を原因とする運動や姿勢の異常であるが、実際には、運動障害に限らず、知的障害や発達障害を合併することも多い。これまで脳性麻痺は、胎児期や分娩時の低酸素虚血が主な原因と考えられてきたが、近年では、明らかな分娩時低酸素虚血に起因するものは、脳性麻痺の10%程度を占めるに過ぎないことがわかってきた。また、知的障害や自閉症の多くについて、コピー数変異、遺伝子変異・多型と関連していることがわかってきている。 本論文は、脳性麻痺の小児・成人患者におけるエクソーム解析による分子遺伝学的診断率(病的バリアントおよび病的と思われるバリアントの有病率)を明らかにすることを目的とした後方視的研究の報告である。2012~18年にエクソーム解析を受けた「臨床検査室紹介コホート」(1,345人、年齢中央値8.8歳)、および主に民間医療機関(Geisinger)共同ゲノム解析プロジェクト(DiscovEHR)において2007~17年にエクソーム解析を受けた「医療機関登録コホート(181人、年齢中央値41.9歳)の2群に分けて検討した。病的および病的と思われるバリアントの有病率は、「臨床検査室紹介コホート」では32.7%(95%信頼区間[CI], 30.2 ~ 35.2%)、「医療機関登録コホート」では10.5%(95% CI, 6.0 ~ 15.0%)であった。 本研究の意義は、大規模な脳性麻痺の集団に対してエクソーム解析を行った場合に、かなりの頻度で病的バリアントが検出されることを明らかにした点である。特に、小児を中心とする「臨床検査室紹介コホート」における病的バリアントの有病率は、一般的な、未診断疾患患者に対するエクソーム解析の診断率に近い。すなわち、「脳性麻痺」という臨床診断の有無にかかわらず、原因不明の知的障害、運動障害の小児においては、両親も含めたトリオのエクソーム解析を行うことで、3割ほどで分子遺伝学的診断を得ることができる。本研究の限界点として、保険病名をもとに抽出した後方視的研究であり脳性麻痺の定義が厳密ではないこと、また「医療機関登録コホート」の患者は成人であり、両親が高齢のため、両親と本人のデータの比較ができていない点が挙げられる。診断法が進歩した今日でも、運動障害、知的障害・発達障害の原因を特定することは容易ではないが、日本における脳性麻痺児においても、エクソーム解析が行われれば、本研究と同程度の頻度で、症状を説明しうる遺伝子異常が検出されると類推される。
標準治療を受けた早期トリプルネガティブ乳がんで検討した低用量かつ高頻度のカペシタビン維持療法と経過観察が無病生存率にもたらす効果の比較 SYSUCC-001無作為化臨床試験
標準治療を受けた早期トリプルネガティブ乳がんで検討した低用量かつ高頻度のカペシタビン維持療法と経過観察が無病生存率にもたらす効果の比較 SYSUCC-001無作為化臨床試験
Effect of Capecitabine Maintenance Therapy Using Lower Dosage and Higher Frequency vs Observation on Disease-Free Survival Among Patients With Early-Stage Triple-Negative Breast Cancer Who Had Received Standard Treatment: The SYSUCC-001 Randomized Clinical Trial JAMA. 2021 Jan 5;325(1):50-58. doi: 10.1001/jama.2020.23370. 原文をBibgraph(ビブグラフ)で読む 上記論文の日本語要約 【重要性】乳がんのサブタイプのうち、トリプルネガティブ乳がんは標準治療後の再発率がいくぶん高く、予後が不良である。再発と死亡リスクを下げる効果的な戦略が求められている。 【目的】早期トリプルネガティブ乳がんで、標準的な術後化学療法後に用いる低用量カペシタビン維持療法の有効性と有害事象を評価すること。 【デザイン、設定および参加者】2010年4月から2016年12月の間に、中国の大学病院と臨床施設計13施設で実施した無作為化臨床試験。最終追跡調査日は2020年4月30日であった。参加者(443例)は早期のトリプルネガティブ乳がん患者であり、標準的な術後化学療法を終了していた。 【介入】標準的な術後化学療法終了後、適格患者をカペシタビン650mg/m^2を1年間にわたって1日2回投与するグループ(222例)と経過観察するグループ(221例)に1対1の割合で無作為に割り付けた。 【主要評価項目】主要評価項目は無病生存率であった。無遠隔転移生存率、全生存率、局所無再発生存率、有害事象を副次評価項目とした。 【結果】無作為化した443例のうち、34例を最大の解析対象集団[平均年齢(SD)46(9.9)歳、T1/T2期93.1%、リンパ節転移陰性61.8%]とした(98.0%が試験を完遂)。追跡調査期間中央値61カ月(四分位範囲44~82)の後、イベント94件が発生し、内訳はカペシタビン群38件(再発37例、死亡32例)、経過観察群56件(再発56例、死亡40例)であった。推定5年無病生存率は、カペシタビン群82.8%、観察群73.0%であった(再発または死亡リスクのハザード比[HR]0.64[95%CI 0.42~0.95]、P=0.03)。追跡調査期間中央値61カ月(四分位範囲44-82)の後、イベントが94件発生し、内訳はカペシタビン群38件(再発37件、死亡32件)、観察群56件(再発56件、死亡40件)であった(再発または死亡リスクのHR 0.64、95%CI 0.42~0.95、P=0.03)。カペシタビン群と観察群を比較すると、推定5年無遠隔転移生存率は85.8% vs 75.8%(遠隔転移または死亡リスクのHR 0.60、95%CI 0.38~0.92、P=0.02)、推定5年全生存率は85.5% vs 81.3%(死亡リスクのHR 0.75、95%CI 0.47~1.19、P=0.22)、推定5年局所無再発生存率は85.0% vs 80.8%(局所再発または死亡リスクのHR 0.72、95%CI 0.46~1.13、P =0.15)であった。最も発現頻度が高かったカペシタビン関連の有害事象は手足症候群(45.2%)であり、7.7%からグレード3の有害事象が報告された。 【結論および意義】標準的な術後治療を受けた早期トリプルネガティブ乳がんで、1年間の低用量カペシタビン維持療法によって、経過観察と比べて5年無病生存率が有意に改善した。 第一人者の医師による解説 忍容性高く5年無病生存率を10%上昇 患者選択と至適投与法のさらなる検討必要 三階 貴史 北里大学医学部乳腺・甲状腺外科学主任教授 MMJ. June 2021;17(3):88 近年、乳がん薬物療法の進歩は著しく、ホルモン受容体陽性、またはHER2陽性タイプの転移性乳 がんの予後は年単位で改善した。一方、トリプルネガティブ乳がん(TNBC)に関しては最近、PARP阻害薬や免疫チェックポイント阻害薬が日本でも保険診療で使用されているが、いまだ予後不良である。 転移性乳がんに対する新たな分子標的治療の開発が進む一方で、経口5-FU製剤はその効果と副作用の少なさから、こと日本では術後療法への応用が1980年代から進められていた。その有効性を示す日本発のエビデンスは2000年代に入って示されたが、高リスク患者に対してはアントラサイクリン系、タキサン系薬剤が国際的な標準治療となるにつれ、高齢者など一部の患者に対する選択肢としての位置づけに留まっていた。しかし、2017年に術前化学療法後に病理学的に腫瘍の残存を認めた患者に対するカペシタビン投与が、特にTNBCで有効であることが日韓国際共同試験の結果で示され(1)、現在NCCNガイドラインでは標準治療として推奨されている(2)。 本論文はTNBC患者に標準的な手術、術前/術後化学療法、放射線療法を行った後、1,300mg/m2/日という低用量(通常2,500mg/m2/日)でカペシタビンを2週内服、1週休薬で1年間投与することの有効性と副作用を明らかにすることを目的として中国で行われた多施設共同試験の結果である。解析対象はカペシタビン群221人、経過観察 群213人、観察期間中央値は61カ月であった。その結果、主要評価項目である5年無病生存率はカペシタビン群で経過観察群よりも有意に高いことが示された(82.8%対73.0%;ハザード比[HR],0.64)。また、副次評価項目である5年無遠隔転移生存率もカペシタビン群の方が有意に高かったが (85.5%対75.8%;HR,0.60)、5年全生存率、5年無局所再発生存率の統計学的有意差は認められなかった。アジア人の経口5-FU製剤に対する忍容性は高いと考えられているが、低用量で行われた本試験でもカペシタビンの相対用量強度(予定投与量に対する実際の投与量の割合)の中央値は85%であり、主な副作用である手足症候群は全グレードで45%、グレード3で8%の発現率であった。 これまでにも乳がん術後カペシタビン投与の有効性を検討する臨床試験はいくつか行われているものの、結果はcontroversialである。いまだ日本では術後療法としての投与は保険適用外であるが、TNBCの予後を改善するためにもカペシタビン投与が必要な患者選択と至適投与法の確立が待たれる。 1. Masuda N, et al. N Engl J Med. 2017;376(22):2147-2159. 2. NCCN Clinical Practice Guidelines in Oncology Breast Cancer (Version 4.2021) (https://www.nccn.org/professionals/physician_gls/pdf/breast.pdf)
一過性脳虚血発作の発症と長期的な脳卒中リスクとの関連
一過性脳虚血発作の発症と長期的な脳卒中リスクとの関連
Incidence of Transient Ischemic Attack and Association With Long-term Risk of Stroke JAMA. 2021 Jan 26;325(4):373-381. doi: 10.1001/jama.2020.25071. 原文をBibgraph(ビブグラフ)で読む 上記論文の日本語要約 【重要性】一過性脳虚血発作(TIA)とその後の脳卒中リスクの関連を正確に推定することによって、TIAを発症した患者の予防策を向上させ、脳卒中の負担を抑制させることができる。 【目的】集団のTIA発症率、TIA後の脳卒中リスクの時期と長期的傾向を明らかにすること。 【デザイン、設定、参加者】ベースラインでTIA、脳卒中の既往のない参加者1万4059例から後ろ向きに収集したデータの後ろ向きコホート研究(Framingham Heart Study)。1948年から2017年12月31日まで追跡した。TIA未発症例の標本とTIA初発例を年齢と性別で(5対1の比率で)マッチさせた。 【曝露】時間(TIA発症率の算出、時間傾向分析)、TIA(マッチさせた縦断コホート)。 【主要評価項目】主要評価項目は、TIA発症率、短期(7日、30日、90日)と長期(1~10年)で比較したTIA後の脳卒中発症率、マッチさせたTIA未発症対照例と比較したTIA後の脳卒中、3分類の期間(1954~1985年、1986~1999年、2000~2017年)別に評価したTIA後90日時脳卒中リスクの経時的な傾向。 【結果】追跡調査期間66年の参加者1万4059例(36万6209人・年)のうち435例がTIAを発症し(女性;229例、平均年齢73.47[SD 11.48]歳、男性;206例、平均年齢70.10[SD 10.64]歳)、TIA未発症の対照例2175例とマッチさせた。TIAの推定発症率は1000人・年当たり1.19であった。TIA後の追跡調査期間中央値8.86年の間に、130例(29.5%)が脳卒中を発症した。そのうち28例(21.5%)が初回TIA発症後7日以内、40例(30.8%)が30日以内、51例(39.2%)が90日以内、63例(48.5%)が1年以上経過後に脳卒中を発症した。脳卒中発症までの期間中央値は1.64年(四分位範囲0.07~6.6年)であった。年齢と性別で調整した脳卒中発症の10年累積ハザードは、TIA発症例(435例中130例が脳卒中発症)が0.46(95%CI 0.39-0.55)、マッチさせた対照のTIA未発症例(2175例中165例が脳卒中発症)が0.09(95%CI 0.08-0.11)であり、完全調整後ハザード比(HR)は4.37(95%CI 3.30-5.71、P<0.001)であった。1948~1985年(16.7%、TIA発症155例中26例が脳卒中発症)と比較したTIA後90日脳卒中リスクは、1986~1999年では11.1%(162例中18例が脳卒中発症)、2000~2017年では5.9%(118例中7例が脳卒中発症)であった。第1期(1948~1985年)と比較すると、90日間脳卒中リスクのHRは、第2期(1986~1999年)で0.60(95%CI 0.33-1.12)、第3期(2000~2017年)で0.32(95%CI 0.14-0.75)であった(傾向のP=0.005)。 【結論および意義】今回の1948~2017年を対象とした集団コホート研究で、推定粗TIA発症率は1.19/1000人・年であり、脳卒中リスクは、TIA発症後の方がマッチさせたTIA未発症の対照よりも有意に高かった。TIA発症後の脳卒中リスクは、最も近い2000~2017年の方がそれ以前の1948~1985年よりも有意に低かった。 第一人者の医師による解説 TIAは脳卒中の強い危険因子 長期間にわたる血管リスク管理を徹底すべき 犬塚 諒子/藥師寺 祐介(主任教授) 関西医科大学神経内科学講座 MMJ. August 2021;17(4):109 一過性脳虚血発作(TIA)は切迫(脳)卒中の主要な先駆症状である。近年の2次予防的介入の進歩は、TIA発症後の短期的のみならず、長期的な脳卒中発症リスクの低下をもたらしていると思われるが、既報はない。そのことを明らかにするために、本研究ではFramingham Heart Study(FHS)のデータを用い、後ろ向き検証がなされた。対象として、1948年から2017年までのFHS参加者のうち、登録時にTIAや脳卒中の既往のない14,059人が抽出された(発症率コホート)。その中で、TIAを発症した症例(TIA群)と、それらに年齢・性別をマッチさせた対照(非 TIA群)を、1:5の比で抽出した縦断的解析用の集団も用意された(調整済み縦断的コホート:それぞれn=435、n=2,175)。これら2種のコホートを用いて、① TIA発症率② TIA後の脳卒中発症率(時代的変遷も含む)が検証された。 TIAの推定発症率は1.19/1000人・年であった。また、TIA後の脳卒中発症率は中央値8.9年の追跡期間中で29.5%であり、発症までの期間中央値は1.64年であった。TIA群の脳卒中発症リスクは、非 TIA群に比べ4.4倍と有意に高かった。TIA後90日間での脳卒中発症率の時代別変遷は、1948~85年で16.7%、1986~99年で11.1%、2000~17年で5.9%であり、1948~85年と比較し、2000年以降で有意に低かった。 本研究で示された時代ごとのTIA後の脳卒中発症率の低下は、薬物療法や外科的介入(頸動脈内膜剥離術、頸動脈ステント留置術)などのエビデンス構築、および普及を反映しているものであろう。しかし、TIA後の脳卒中発症率は非 TIA群の約4倍といまだ高い結果は見逃せない。さらに、TIA後の脳卒中発症率に関しては、従来、比較的短期的なイベントとして警鐘を鳴らされてきた感が強いが、本研究結果をみると、脳卒中の発症は短期間内にプラトーに達するわけではなく、追跡期間全体にわたって増加し、かつ半数(49%)は初回 TIAから1年後以降に発症することが示されたことは特筆すべきであろう。本研究は、後ろ向き研究であることなどから、結果解釈に制限はあるものの、TIAに限った均一な集団での、これまでにない長期間の追跡研究結果としての価値がある。事実、近年における5年を超える追跡期間を有した代表的研究であるTIAregistry.org projectはTIAに加え、軽症虚血性脳卒中も含んでいた側面を持つ(1)。本研究は、TIAは完成型脳卒中の強い危険因子であり、長期間にわたる血管リスク管理を徹底すべき疾患であることを示唆している。すなわち、TIA患者を診た場合は、「脳卒中になる一歩手前の崖っぷちにいる患者」と認識し、長期間にわたる生活習慣改善や内服調整を要することを説明しなければならない。 1. Amarenco P, et al. N Engl J Med. 2018;378(23):2182-2190.
初回再発を認めた高リスクB細胞性急性リンパ性白血病患児の無事象生存期間にもたらすブリナツモマブと化学療法の作用の比較:無作為化臨床試験
初回再発を認めた高リスクB細胞性急性リンパ性白血病患児の無事象生存期間にもたらすブリナツモマブと化学療法の作用の比較:無作為化臨床試験
Effect of Blinatumomab vs Chemotherapy on Event-Free Survival Among Children With High-risk First-Relapse B-Cell Acute Lymphoblastic Leukemia: A Randomized Clinical Trial JAMA. 2021 Mar 2;325(9):843-854. doi: 10.1001/jama.2021.0987. 原文をBibgraph(ビブグラフ)で読む 上記論文の日本語要約 【重要性】ブリナツモマブは、CD3/CD19を標的とした二重特異性T細胞誘導作用を有する抗体製剤であり、再発または難治性B細胞性急性リンパ性白血病(B-ALL)患児に有効である。 【目的】初回再発を認めた高リスクB-ALL患児で、同種造血幹細胞移植前のブリナツモマブによる3回目地固め療法後の無事象生存期間を地固め化学療法と比較すること。 【デザイン、設定および参加者】この第III相無作為化試験では、2015年11月から2019年7月までの間に患者を登録した(データ打ち切り日、2019年7月17日)。13カ国47施設で、無作為化時に形態学的完全寛解(M1 marrow、骨髄中芽球細胞5%未満)またはM2 marrow(骨髄中芽球細胞5%以上25%未満)で、28日齢を超える18歳未満の初回再発高リスクB-ALL患児を登録した。 【介入】患者をブリナツモマブ1サイクル(54例、15μg/m2/日、4週間、持続点滴静注)と3コース目地固め化学療法(54例)に割り付けた。 【主要評価項目】主要評価項目は無事象生存率とした(事象:再発、死亡、二次がんまたは完全寛解未達成)。有効性に関する主な副次評価項目は全生存率とした。微小残存病変陰性化および有害事象発現率をその他の副次評価項目とした。 【結果】計108例を無作為化により割り付け(年齢中央値5.0歳[四分位範囲{IQR}4.0~10.5]、女児51.9%、M1 marrow 97.2%)、全例を解析対象とした。本試験への登録は、予め定めた中止基準に従って、早期有効中止となった。追跡期間中央値22.4カ月(IQR 8.1~34.2)での事象発生率は、ブリナツモマブ群31%、地固め化学療法群57%であった(log-rank検定のP<0.001、ハザード比0.33、95%CI 0.18~0.61)。ブリナツモマブ群の8例(14.8%)、地固め化学療法群の16例(29.6%)が死亡した。全生存のハザード比は0.43(95%CI 0.18~1.01)だった。ブリナツモマブ群の微小残存病変陰性化が地固め化学療法群よりも多かった(90%[49例中44例] vs. 54%[48例中26例]、差35.6%[95%CI 15.6~52.5])。致命的な有害事象は報告されなかった。ブリナツモマブ群と地固め化学療法群を比較すると、重篤な有害事象発現率はそれぞれ24.1% vs 43.1%、グレード3以上の有害事象発現率は57.4% vs 82.4%であった。ブリナツモマブ群の2例に治療中止に至る有害事象が報告された。 【結論および意義】初回再発を認めた高リスクB-ALL患児で、同種造血幹細胞移植前のブリナツモマブ1サイクルによる治療によって、多剤強化標準化学療法に比べ、追跡調査期間中央値22.4カ月で無事象生存率が改善した。 第一人者の医師による解説 安全に深い寛解を達成し 同種造血幹細胞移植の成績向上に寄与することを示唆 森 毅彦 東京医科歯科大学大学院医歯学総合研究科血液内科学教授 MMJ. October 2021;17(5):150 小児急性リンパ性白血病(ALL)は成人のそれとは異なり、標準的な多剤併用化学療法により高い治癒率を得ることができる。しかし、再発した場合の予後は不良であり、その根治のためには同種造血幹細胞移植(HSCT)が実施される。同種HSCTは移植後の合併症による死亡と移植後のALL再発が、その成績に大きく影響する。移植後再発のリスクは残存腫瘍が少ないほど低いため、深い寛解を達成して移植に臨むのが理想的である。そのために毒性の強い化学療法を行ってきたが、近年、新規治療法が導入されてきている。その1つがブリナツモマブであり、bispecifi c T-cell engager(BiTE)抗体と呼ばれ、異なる抗原結合部位をもつ2重特異性抗体である。B細胞性腫瘍が発現するCD19と抗腫瘍効果を発揮するT細胞表面上のCD3を標的としている。小児再発・治療抵抗性 ALLを対象とした試験において39%の寛解率、そのうちの約半数が微少残存腫瘍の消失を達成した(1)。 本論文は再発後の治療で寛解を達成した小児高リスクALL患者を対象に、同種HSCT前の3回目地固め療法(1コース)をブリナツモマブ単剤と多剤併用化学療法に無作為に割り付けた臨床試験の結果を示したものである。この治療後に同種HSCTを実施する患者が対象であり、年齢中央値は5歳であった。本試験は中間評価にてブリナツモマブ群の成績が優れていたことから、早期に中止となった。24カ月無イベント生存率はブリナツモマブ群66.2%、化学療法群27.1%と有意差がみられた。24カ月再発率も24.9%と70.8%、微少残存腫瘍陰性化率も90%と54%と有意差がみられた。重篤な有害事象はブリナツモマブ群で少なかった。ブリナツモマブにより安全に深い寛解を達成し、同種HSCTの成績向上に寄与することが示唆された。 本研究の限界としては、小児を対象としていること、化学療法により寛解を達成した患者を対象としていること、1コースのブリナツモマブと化学療法を比較している点などが挙げられる。実診療では若年・成人のALL患者も多く、ブリナツモマブを非寛解例に使用することや複数コース使用するケースも多い。またブリナツモマブ以外にもCD19を標的としたchimeric antigen receptor T-cell (CAR-T)療法やCD22を標的とした抗体薬物複合体のイノツズマブ オゾガマイシンも実診療で使用可能となっており、これらの薬剤との比較や併用療法などの有効性・安全性を評価する試験が実施されることで、再発ALLの最適な治療法の発展につながっていくと考えられる。 1.von Stackelberg A, et al. J Clin Oncol. 2016;34(36):4381-4389.
重篤患者に用いる人工呼吸器のウィーニングおよび離脱の実践
重篤患者に用いる人工呼吸器のウィーニングおよび離脱の実践
Ventilator Weaning and Discontinuation Practices for Critically Ill Patients JAMA. 2021 Mar 23;325(12):1173-1184. doi: 10.1001/jama.2021.2384. 原文をBibgraph(ビブグラフ)で読む 上記論文の日本語要約 【重要性】重篤な患者のほとんどが侵襲的人工呼吸療法(invasive mechanical ventilation:IMV)を受けるが、実臨床でどのようにIMVから離脱しているかを明らかにした研究はほとんどない。 【目的】地域によるIMV離脱法のばらつき、初回離脱と予後の関連性、離脱方法の選定と初回自発呼吸トライアル(SBT)不成功の関連因子を明らかにすること。 【デザイン、設定および参加者】世界6地域19カ国の142の集中治療室(ICU)(カナダ27施設、インド23施設、英国22施設、欧州26施設、オーストラリア・ニュージーランド21施設、米国23施設)で、24時間以上IMVを受ける重篤患者を検討した国際共同前向き観察研究。 【曝露】IMV。 【主要評価項目】主解析で、初回IMV離脱方法(抜管、自発呼吸トライアル[SBT]または気管切開)と臨床転帰(人工呼吸期間、ICUおよび院内死亡率、ICU入室および入院日数)の関連性を明らかにした。副次解析で、SBTの結果とSBTのタイミングおよび臨床転帰の関連性を検討した。 【結果】1,868例(年齢中央値[四分位範囲]、61.8(48.9~73.1)歳;男性1,173例[62.8%])のうち、424例(22.7%)に直接抜管、930例(49.8%)に初回SBT実施(761例[81.8%]が成功)、150例(8.0%)に気管切開を実施し、364例(19.5%)が離脱前に死亡した。各地域で、治療、毎日のスクリーニング、SBTの手法、換気モードに関する指示書の使用および離脱に携わる臨床医が果たす役割に差があった。直接抜管と比べると、初回SBTの方がICU死亡率が高く(20例[4.7%] vs 96例[10.3%]、絶対差5.6%[95%CI、2.6~8.6])、人工呼吸器装着期間が長く(中央値2.9日 vs 4.1日;絶対差1.2日、[95%CI、0.7~1.6])、ICU在室期間が長かった(中央値6.7日 vs 8.1日;絶対差1.4日[95%CI、0.8~2.4])。初回SBTが不成功の患者は、成功した患者よりもICU死亡率が高く(29例[17.2%] vs 67例[8.8%]、絶対差8.4%[95%CI、2.0~14.7])、人工呼吸器装着期間(中央値6.1日 vs 3.5日;絶対差2.6日[95%CI、1.6~3.6])およびICU在室期間が長かった(中央値10.6日 vs 7.7日;絶対差2.8日[95%CI、1.1~5.2])。早期に初回SBTを実施した患者に比べると、後期(挿管から2.3日以降)に実施した患者の方が人工呼吸器装着期間(中央値2.1日 vs 6.1日;絶対差4.0日[95%CI、3.7~4.5])およびICU在室期間が長く(中央値5.9日 vs 10.8日;絶対差4.9日[95%CI、4.0~6.3])、入院期間が長かった(中央値14.3日 vs 22.8日;絶対差8.5日[95%CI、6.0~11.0])。 【結論および意義】2013~2016年にカナダ、インド、英国、欧州、オーストラリア・ニュージーランド、米国のICU 142施設で侵襲的人工呼吸療法の離脱を検討した観察研究では、地域間で離脱方法にばらつきがあることが示された。 第一人者の医師による解説 直接抜管群が転帰良好 しかし優位性を示すのではなく地域間でウィーニング手技実践にバラツキ 佐々木 勝教 医療法人 横浜未来ヘルスケアシステム 戸塚共立第2病院救急科部長 MMJ. October 2021;17(5):156 本研究は世界6地域19カ国にある142の集中治療室(ICU)で24時間以上人工呼吸管理を受けた患者を対象に人工呼吸器からの離脱法、転帰を前向き観察研究で以下の項目を検討した:主解析では①人工呼吸器からの離脱法(直接抜管[自発呼吸トライアルせずに抜管]、自発呼吸トライアル= SBT、気管切開)②臨床転帰(人工呼吸管理期間、ICUおよび院内死亡率、ICU滞在および入院日数)、副次解析ではSBTのアウトカム、施行のタイミングと臨床転帰の関連。 結果、対象患者1,868人中、22.7%に直接抜管、49.8%に初回 SBT、8.0%に気管切開が実施された。離脱前に死亡した患者は19.5%であった。ただし、地域間で、離脱に関する手順書の使用、毎日のスクリーニング、SBTの手法、換気モード、離脱において臨床医の果たす役割に関して差異がみられた。直接抜管群と比較し、初回SBT群の方が、ICU死亡率が高く(4.7 対 10.3%)、人工呼吸器装着期間(中央値2.9 対 4.1日)、ICU滞在日数(中央値6.7 対 8.1日)も長かった。また、初回 SBTが不成功だった群は、成功した群と比較し、ICU死亡率が高く(17.2 対 8.8%)、人工呼吸器装着期間(中央値6.1 対 3.5日)、ICU滞在日数もより長い傾向だった(中央値10.6 対 7.7日)。早期にSBTが成功した群と、挿管から3.3日以降の時期(後期)に成功した群との比較でも同様の傾向がみられ、人工呼吸器装着期間(中央値2.1 対 6.1日)、ICU滞在期間(中央値5.9 対 10.8日)、入院日数(中央値14.3 対 22.8日)は長かった。 結果を表層的に解釈すると、SBTより直接抜管(しかも8.5%が計画外抜管)を選択した方が、転帰が良好な印象を受ける。この点は併載された論説1でも指摘しており、背景としてSBT施行群の方が、より高齢で、悪性腫瘍、高血圧の合併率が高い可能性があることが問題とされている。同様に早期 vs 後期のSBTの比較でも、早期 SBTを実施できない理由が明確になっていない。また、各地域間でウィーニング手技のバラツキが結果に影響した可能性がある。例えば、直接抜管は米国で同国全体の約4%であったが、オーストラリア/ニュージーランドでは両国全体のおよそ6割であった。このように、本論文の結語においては、バラツキの補正が十分ではないため、直接抜管の優位性を示すのではなく、各地域、施設でのウィーニング手技はさまざまであったと結論づけている。
カリフォルニア州の病院の質改善介入、州の政策イニシアティブおよび初産正期産単胎頭位分娩の帝王切開率
カリフォルニア州の病院の質改善介入、州の政策イニシアティブおよび初産正期産単胎頭位分娩の帝王切開率
Hospital Quality Improvement Interventions, Statewide Policy Initiatives, and Rates of Cesarean Delivery for Nulliparous, Term, Singleton, Vertex Births in California JAMA. 2021 Apr 27;325(16):1631-1639. doi: 10.1001/jama.2021.3816. 原文をBibgraph(ビブグラフ)で読む 上記論文の日本語要約 【重要性】帝王切開率の安全な低下が国家の優先事項である。 【目的】帝王切開率低下を目的とした多角的介入策を実施するカリフォルニア州の初産正期産単胎頭位(NTSV)分娩の帝王切開率 【デザイン、設定および参加者】2014~2019年の米国およびカリフォルニア州産院施設238箇所のNTSV分娩757万4,889件の帝王切開率を検討した観察研究。2016~2019年にかけて、California Maternal Quality Care CollaborativeがSmart Care Californiaと協同して、帝王切開率を下げるため多数の対策を導入した。NTSV分娩の帝王切開率が23.9%を超える病院に対して、2016年7月から2019年6月までの18カ月間にわたる質改善プログラムへの参加を呼びかけ、3つのコホートに振り分けた。 【曝露】この共同研究では、集学的チームがメンター制度、知識の共有および迅速なデータのフィードバックなどで支援する多数の戦略を導入した。非営利団体、州政府機関、購買者および医療制度間の協力関係によって、透明性、報奨プログラムおよび報酬を通じて外部環境に対処した。 【主要評価項目】主要評価項目は、カリフォルニア州のNTSV分娩の帝王切開率に見られる変化とし、差の差分析でカリフォルニア州の帝王切開率を米国のその他の州と比較した。このほか、患者別、病院別の交絡因子で調節したmixed multivariable logistic regression modelを用いて、共同研究および外部の州全体の取り組みを評価した。共同研究参加病院のNTSV分娩の帝王切開率は、非参加病院の帝王切開率および参加病院の共同研究参加前の帝王切開率と比較した。 【結果】2014年から2019年までの間に、米国でNTSV分娩757万4,889件が発生し、そのうち91万4,283件がカリフォルニア州の病院238施設で発生したものであった。カリフォルニア州の全病院は、NTSV分娩の帝王切開率が23.9%を超える149施設を含め、帝王切開率低下を目標とする州の取り組みの影響下にあり、そのうち91施設(61%)が質改善共同研究に参加した。カリフォルニア州のNTSV分娩の帝王切開率は、2014年の26.0%(95%CI、25.8%~26.2%)から2019には22.8%(95%CI、22.6%~23.1%)に低下した(相対リスク、0.88;95%CI、0.87~0.89)。(カリフォルニア州を除く)米国のNTSV分娩の帝王切開率は、2014年、2019年ともに26.0%であった(相対リスク、1.00、95%CI、0.996~1.005)。差の差分析からは、カリフォルニア州のNTSV分娩の帝王切開率低下度は(カリフォルニア州を除く)米国より3.2%(95%CI、1.7~3.5%)高いことが明らかになった。病院間や共同研究参加前の期間と比較すると、modified stepped-wedg解析を用いて患者データや期間で補正後、共同研究活動への曝露にNTSV分娩の帝王切開率オッズ低下との関連が認められた(24.4% vs 24.6%;調整オッズ比、0.87[95%CI、0.85~0.89])。 【結論および意義】2014~2019年のカリフォルニア州のNTSV分娩を検討したこの観察研究では、病院全体で取り組む共同研究の導入および経腟分娩を支援する州のイニシアティブによって、時間の経過と共に帝王切開率が低下した。 第一人者の医師による解説 初産低リスクの帝王切開率データのない日本 同様の取り組みの是非は不明 板橋 家頭夫 愛正会記念茨城福祉医療センターセンター長・昭和大学名誉教授 MMJ. October 2021;17(5):154 帝王切開(CS)は、リスクの高い分娩において母子の救命に寄与してきた。一方で、安易なCSの導入が母子にリスクを負わせていることも事実である。CSは経腟分娩に比べ母体の死亡率や合併症発症率が高く、さらに回数に応じて以後の分娩で子宮破裂、胎盤異常、子宮外妊娠、死産、早産などのリスクを高める(1)。CSによって娩出された児は、ホルモン環境や細菌学的環境、物理的環境などが経腟分娩の児とは異なっており、これが新生児の生理機能を変化させる可能性が高いと考えられている(1)。短期的な影響として、免疫系の発達の変化によるアレルギー疾患(アトピー、気管支喘息、食物アレルギーなど)、肥満のリスク、腸内細菌叢の多様性の低下などが挙げられている(1)。加えてエピゲノムの変化による将来的な健康への影響も懸念されている(1)。したがって、いかに不必要なCSを回避するかが世界的な課題となっており、世界保健機関(WHO)はCS率の目標を10~15%としている。 本論文は、米国カリフォルニア州においてステークホルダー組織 California Maternal Quality Care Collaborative(CMQCC)主導による介入がCS率に与えた影響に関する観察研究の報告である。本研究では、2016~19年に、初産、正期産、単胎、頭位(nulliparous、term、singleton、vertex;NTSV)の4条件を満たす分娩(NTSV分娩)のCS率が23.9%以上の施設に対しコホート研究の参加を呼びかけ、メンターシップや学習の共有、迅速なデータフィードバックなど複数の戦略を州政府の政策のもとで実施し、NTSV分娩におけるCS率低下の有無を評価した。その結果、同州におけるNTSV分娩のCS率は、2014年の26.0%から19年には22.8%に低下した(相対リスク,0.88;95%信頼区間[CI], 0.87~0.89)。一方、カリフォルニア州を除いた米国におけるNTSV分娩のCS率は、2014年、19年ともに26.0%で、同州よりも絶対差で3.2%(95% CI, 1.7~3.5%)高かった。また、CMQCCの取り組みに参加した施設は、参加しなかった施設に比べCS率が有意に低下した。以上より、著者らは、カリフォルニア州の政策の下に実施されたこのような取り組みがNTSV分娩のCS率低下に寄与したと結論付けている。日本では2013年の特定健診や保険レセプトのデータからは、国内全体のCS率が18.5%と推測されており(2)、経済協力開発機構(OECD)加盟国の平均28%に比べ明らかに低い。しかしながら、NTSV分娩のCS率のデータはなく、カリフォルニア州のような取り組みの是非については明らかでない。 1. Sandall J, et al. Lancet. 2018 ;392(10155):1349-1357. 2. Maeda E, et al. J Obstet Gynaecol Res. 2018;44(2):208-216.
III期大腸がんで標準補助療法と併用するセレコキシブとプラセボが無病生存率にもたらす効果の比較:CALGB/SWOG 80702(Alliance)無作為化比較試験
III期大腸がんで標準補助療法と併用するセレコキシブとプラセボが無病生存率にもたらす効果の比較:CALGB/SWOG 80702(Alliance)無作為化比較試験
Effect of Celecoxib vs Placebo Added to Standard Adjuvant Therapy on Disease-Free Survival Among Patients With Stage III Colon Cancer: The CALGB/SWOG 80702 (Alliance) Randomized Clinical Trial JAMA. 2021 Apr 6;325(13):1277-1286. doi: 10.1001/jama.2021.2454. 原文をBibgraph(ビブグラフ)で読む 上記論文の日本語要約 【重要性】観察研究や無作為化試験から、アスピリンとシクロオキシゲナーゼ2(COX-2)阻害薬に大腸のポリープおよびがんのリスク低下に関連があることが明らかになっている。転移のない大腸がんの治療に用いるCOX-2阻害薬、セレコキシブの効果は明らかになっていない。 【目的】III期大腸がんで、フルオロウラシル、ロイコボリンおよびオキサリプラチン(FOLFOX)による術後補助化学療法にセレコキシブを追加すると無病生存期率が改善するかを明らかにすること。 【デザイン、設定および参加者】Cancer and Leukemia Group B (Alliance)/Southwest Oncology Group(CALGB/SWOG) 80702試験は、米国およびカナダの市中病院および大学病院654施設で実施された2×2要因デザインの第III相試験である。2010年6月から2015年11月にかけてIII期大腸がん患者計2,526例を組み入れ、2020年8月10日まで追跡した。 【介入】患者をFOLFOX補助化学療法(2週間に1回)を3カ月実施するグループと6カ月実施するグループ、セレコキシブ(400mgを1日1回経口投与)を3年間投与する群(1,263例)とプラセボ群(1,261例)に割り付けた。本稿は、セレコキシブの無作為化の結果を中心に報告する。 【主要評価項目】主要評価項目は、無作為化から再発または全死亡が報告されるまでを評価した無病生存率とした。総生存率、有害事象、心血管系特異的事象を副次的評価項目とした。 【結果】無作為化により割り付けた2,526例(平均値[SD]年齢61.0歳[11歳]、女性1,134例[44.9%])のうち2,524例を主解析の対象とした。プロトコールの治療遵守率(セレコキシブまたはプラセボを2.75年以上投与、再発、死亡または許容できない有害事象発生までの治療継続と定義)は、セレコキシブ投与群70.8%、プラセボ群69.9%だった。セレコキシブ群の337例、プラセボ群の363例に再発または死亡が発生し、追跡期間中央値6年で、3年無病生存率はセレコキシブ群76.3%、プラセボ群73.4%であった(再発または死亡のハザード比[HR]0.89;95%CI、0.76~1.03、P=0.12)。割り付けた術後補助化学療法の継続期間中、無病生存率に対するセレコキシブ治療の効果に有意な変化は見られなかった(交互作用のP=0.61)。5年時の総生存率は、セレコキシブ群84.3%、プラセボ群81.6%であった(死亡のHR 0.86、95%CI、0.72~1.04;P=0.13)。FOLFOX投与期間中、セレコキシブ群の14.6%、プラセボ群の10.9%に高血圧(全グレード)が生じ、FOLFOX終了後、それぞれ1.7%と0.5%にグレード2以上のクレアチニン値上昇が生じた。 【結論および意義】III期大腸がん患者の標準術後補助化学療法に3年間セレコキシブ併用しても、プラセボより無病生存率を有意に改善することができなかった。 第一人者の医師による解説 COX-2阻害薬の上乗せ効果 本当にないのかの確認はさらなる検証が必要 山本 聖一郎 東海大学医学部消化器外科教授 MMJ. October 2021;17(5):151 アスピリンやシクロオキシゲナーゼ2(COX-2)阻害薬を長期服用することで、大腸腺腫や大腸がんの発生のみならず、大腸がん術後の再発率が低下するとの報告がある。結腸がんステージIII術後の予後改善目的の標準補助化学療法は5-フルオロウラシル系薬剤+オキサリプラチン療法(FOLFOX療法)である。本臨床試験 はFOLFOX療法にCOX-2阻害薬セレコキシブの3年間併用で予後が改善するかを検証する優越性試験である。3年無病生存率をプラセボ群(P群)で72%、セレコキシブ群(C群)で77%と想定し、2,500人の患者登録を予定した。2,526人の登録があり、3年無病生存率はP群73.4%、C群76.3%と、C群の優越性を示すことはできなかった(ハザード比[HR],0.89;95%信頼区間[CI],0.76〜1.03;P=0.12)。また5年全生存率もP群81.6%、C群84.3%と、C群の優越性を示せなかった(HR,0.86;95% CI,0.72〜1.04;P=0.13)。有害事象に関しては、C群で高血圧のリスクがFOLFOX療法中(14.6% 対 10.9%;P=0.01)ならびに終了後(13.0%対10.0%;P=0.04)に有意に高かった。また、グレード2以上の血中クレアチニン値の上昇がFOLFOX療法終了後にC群で有意に高率であった(1.7%対0.5%;P=0.01)。これらの結果を踏まえ、本論文の結論は「結腸がんステージIII術後の補助化学療法(FOLFOX療法)にセレコキシブを3年間併用することは3年無病生存率を有意に改善しなかった」と簡潔に記載している。研究者としては、ネガティブな結果は否定できないが、影響を最小限に留めたい想いであろう。 その理由としては無病生存曲線でも全生存曲線でも常にC群の方が生存率が高く推移していること、さらに補遺資料(ウエブ上で公開)の3年無病生存率に影響を与える臨床因子のサブグループ解析では、ほぼすべての因子でC群の方が優位であった。これらの結果より、真にCOX-2阻害薬が予後を改善しないと判断するよりは3年無病生存率をP群で72%、C群で77%と想定したが、結果として2.9%の差しか得られなかったことが大きく影響したと考えられる。もしそれぞれ73%、76%と当初から想定して臨床試験を行い、実現可能性は乏しいものの6,600人以上登録できれば有意な予後改善結果を証明できた可能性が高い。73%対76%の3%の予後改善効果を実感することは困難だが、逆に言えば実感できないほどではあるがわずかな予後改善効果を有する可能性が残った、と判断するのが妥当であろう。
糖尿病診断時の年齢とその後の認知症リスクの関連
糖尿病診断時の年齢とその後の認知症リスクの関連
Association Between Age at Diabetes Onset and Subsequent Risk of Dementia JAMA. 2021 Apr 27;325(16):1640-1649. doi: 10.1001/jama.2021.4001. 原文をBibgraph(ビブグラフ)で読む 上記論文の日本語要約 【重要性】2型糖尿病は、若年齢での発症に伴い有病率が上昇している。早期発症2型糖尿病の血管合併症は知られているが、認知症との関連については明らかになっていない。 【目的】糖尿病発症年齢が低いほど認知症発症との関連が強くなるかを明らかにすること。 【デザイン、設定および参加者】英国の住民対象試験、Whitehall II前向きコホート試験。1985~1988年に設置し、1991~1993年、1997~1999年、2002~2004年、2007~2009年、2012~2013年および2015~2016年に診察を実施、2019年3月までの電子診療録と紐づけた。最終追跡調査日は2019年3月31日であった。 【曝露】2型糖尿病(診察時の空腹時血糖値126mg/dL以上、医師による2型糖尿病の診断、糖尿病薬の使用、1985年から2019年までに病院に糖尿病の記録、のいずれかを満たす場合と定義) 【主要評価項目】電子診療録との紐づけによって確認した認知症の発症。 【結果】参加者10,095例(男性67.3%;1985~1988年に35~55歳)のうち、中央値で31.7年の追跡期間中、糖尿病1,710例、認知症639例が記録された。1,000人年当たりの認知症発症率は、70歳時点で糖尿病がなかった参加者が8.9、5年以内に糖尿病を発症した参加者が10.0、6~10年前に糖尿病を発症した参加者が13.0、10年より前に糖尿病を発症した参加者が18.3であった。多変量補正解析で、70歳時点で糖尿病ではなかった参加者と比べた認知症のハザード比は、10年より前の糖尿病発症が2.12(95%CI、1.50~3.00)、6~10年前の発症が1.49(95%CI、0.95~2.32)、5年以内の発症が1.11(95%CI、0.70~1.76)であった。線形傾向検定(P<0.001)から、2型糖尿病の発症年齢と認知症に段階的な関連が示された。社会人口統計学的因子、健康行動、健康関連測定値で補正した解析で、2型糖尿病発症時の年齢が5歳低下するごとに70歳時点の認知症ハザード比1.24(同1.06~1.46)との有意な関連が認められた。 【結論および意義】中央値で31.7年追跡したこの縦断コホート試験では、糖尿病の発症年齢が低いほど以後の認知症発症リスクが高くなった。 第一人者の医師による解説 認知症予防には中年期からの積極的な介入が望ましい 古和 久朋 神戸大学大学院保健学研究科教授 MMJ. February 2022;18(1):6 糖尿病がアルツハイマー病(AD)をはじめとする認知症の危険因子であることは、すでにさまざまな証拠から支持されている。例えば日本の久山町研究では耐糖能異常を有する人では有さない人よりもADの特徴的病理構造物である老人斑の蓄積量が約2倍多いことが示されている(1)。一方、2型糖尿病の発症年齢が若くなると死亡率や心血管イベント発生率を上昇させることは以前より示されていたものの、発症年齢と認知症発症の関連は検証されてこなかった。 本論文は、英国 Whitehall II前向きコホート研究の参加者を対象に中年期から高齢期にかけてこの関連について検討した研究の報告である。研究参加者10,095人(男性67.3%、ベースライン時点[1985 ~ 88年]年齢 35 ~ 55歳)のうち、中央値31.7年の追跡期間において、合計1,710人に糖尿病、639人に認知症の発症がみられた。解析の結果、35歳から75歳までの糖尿病発症年齢のデータでは、5年ごとに糖尿病の発症が早くなるほど、認知症の発症リスクが高くなることが示された(70歳時点で糖尿病を有する参加者と比較し、糖尿病発症が10年超早い参加者の認知症のハザード比[HR]は2.12[95% CI, 1.50~3.00]、糖尿病発症が6~10年早い参加者では1.49[95% CI, 0.95~2.32])。一方、遅発性の糖尿病はその後の認知症とは有意に関連しなかった。また、糖尿病のある人では、脳卒中の併存が認知症のリスクをさらに高めることが明らかになった。 本論文の結果は、認知症全体の発症リスクを評価したものであるが、その脳内病理学的変化がアミロイド PETなどにより長期の観察が可能となり、発症の20年以上前から老人斑の蓄積が開始されるなど、その病態生理に多くの知見が得られているAD(2)に限定して考えても極めてリーズナブルといえる。遅発性の糖尿病が発症リスクに影響しないことを考慮すると、糖尿病はAD初期からの病理学的変化である老人斑の蓄積に影響を与えうる可能性が高く、久山町研究の観察と合致するものである。 次の疑問は、糖尿病発症後のどの時期までに介入すれば認知症の発症を防げるのか、その際の介入は血糖コントロールのみでよいのか、J-MINDDiabetes研究(3)のように運動や認知機能訓練などの多因子介入を実施すべきか、という点であり、前向き研究の結果が待たれるところである。 1. Ohara T, et al. Neurology. 2011;77(12):1126-1134. 2. Jack CR Jr, et al. Lancet Neurol. 2010;9(1):119-128. 3. Sugimoto T, et al. Front Aging Neurosci. 2021;13:680341.
2017年の米国の老人介護施設にみられる抗菌薬の使用
2017年の米国の老人介護施設にみられる抗菌薬の使用
Antimicrobial Use in a Cohort of US Nursing Homes, 2017 JAMA. 2021 Apr 6;325(13):1286-1295. doi: 10.1001/jama.2021.2900. 原文をBibgraph(ビブグラフ)で読む 上記論文の日本語要約 【重要性】抗菌薬耐性の制御が公衆衛生の優先事項であるが、米国の老人介護施設での抗菌薬使用に関するデータが少ない。 【目的】老人介護施設入居者の抗菌薬使用割合を評価し、抗菌薬の種類と頻度の高い適応症を記載すること。 【デザイン、設定および参加者】2017年4月から2017年10月までに実施した抗菌薬使用に関する横断的な1日点有病率調査。最終調査日が2017年10月31日であった。新興感染症プログラム(EIP)に参加する10州から選定した161の老人介護施設に調査日時点で入居していた入居者15,276例を対象とした。EIPのスタッフが施設の記録を確認し、入居者のデータおよび調査時に投与されていた抗菌薬に関するデータを収集した。施設のスタッフやNursing Home Compareウェブサイトから施設に関するデータを入手した。 【曝露】調査時点で参加施設に居住。 【主要評価項目】調査対象を入居者総数で除した、調査時に抗菌薬を投与していた入居者数と定義した入居者100人当たりの抗菌薬使用割合。抗菌薬使用の多変量ロジスティック回帰モデルおよび各分類内の薬剤の割合。 【結果】対象とした老人介護施設の入居者15,276例(平均[SD]年齢77.6歳[13.7]、女性9,475例[62%])のうち、96.8%から完全なデータを入手した。全体の抗菌薬使用割合は、入居者100人当たり8.2(95%CI、7.8~8.8)であった。調査前30日以内に施設へ入居した入居者の抗菌薬使用率が高く(入居者100人当たり18.8;95%CI、17.4~20.3)、この入居者は中心静脈カテーテル(62.8;95%CI、56.9~68.3)または尿道カテーテル(19.1;95%CI、16.4~22.0)を留置している割合が高かった。抗菌薬は活動性感染症(77%[95%CI、74.8-79.2])に用いられることが最も多く、主に尿路感染症(28.1%[95%CI、15.5-30.7])の治療が理由であった。一方、18.2%(95%CI、16.1~20.1)が予防的投与であり、尿路感染症に対する投与が最も多かった(40.8%[95%CI、34.8~47.1])。フルオロキノロン系抗菌薬(12.9%;95%CI、11.3~14.8)が最も多く使用され、使用された抗菌薬の33.1%(95%CI、30.7~35.6)が広域スペクトラム抗菌薬であった。 【結論および意義】2017年に米国の老人介護施設で実施した横断的調査では、抗菌薬使用割合が入居者100人当たり8.2であった。この試験から、老人介護施設の入居者にみられる抗菌薬の使用パターンに関する情報が得られる。 第一人者の医師による解説 抗菌薬適正使用の指針や評価の検討のため介入前後の調査を繰り返し実施すべき 塩塚 美歌(医員)/岩田 敏(部長) 国立がん研究センター中央病院感染症部 MMJ. February 2022;18(1):22 近年、薬剤耐性菌対策は公衆衛生における重大な優先事項となり、抗菌薬適正使用の必要性が認識されている。一方で米国の療養施設(nursing home)は、長期療養目的だけでなく、リハビリテーション、創傷ケア、デバイス管理などを要する患者が急性期後治療を目的に入所しており、耐性菌の保菌や感染症発症のリスクが高く、耐性菌の温床となりうることが懸念されている。 本論文では、米国内161カ所の療養施設の入所者を対象に、2017年4月~10月のある1日における各施設での抗菌薬使用状況について調査した横断研究(点有病率調査)の結果を報告している。対象施設はカリフォルニア、ニューヨークなど10州で各々選択された地域にある公的医療保険認定施設の中から無作為に選ばれた。主要評価項目は入所者100人ごとの抗菌薬使用率である。対象者15,276人(平均年齢77.6歳、女性62%)のうち、1,258人(プール平均8.2%)に1,454剤の抗菌薬全身投与(経口、経消化管、筋肉内、静脈内、吸入のいずれかの経路)が行われており、そのうち1,082人(86%)で1種類、158人(12.6%)で2種類、18人(1.5%)で3または4種類の抗菌薬が投与されていた。使用率が最も高かったのは、急性期後治療目的の短期入所者、入所から間もない者やデバイス留置者であった。中でも、調査日前2日以内の入所者での使用率は21.0%と高かった。デバイスの中では、中心静脈カテーテル留置中の使用率が最も高く、調整オッズ比は11.1(95% CI, 8.5~14.5)であった。抗菌薬の80.4%は経口または経消化管投与、77.0%は感染症治療の目的で投与され(内科的予防投与18.0%)、適応症は29.0%が 尿路感染症、21.4 %が 皮膚軟部組織感染症、14.9%が呼吸器感染症であった。抗菌薬のクラスはニューキノロンが12.9%と最も多く、33.1%が広域抗菌薬であった。 同様の調査は各国で行われており、最近の欧州の長期療養施設における抗菌薬使用率は4.9%と報告されている1。また日本の介護老人保健施設における2019年の調査2では、調査票に回答した126施設(有効回収率8.4%)における入所者総数10,148人中、抗菌薬使用者は172人(1.7%)であり、肺炎、尿路感染症、蜂窩織炎の治療で第3世代セファロスポリン、キノロンが選択される傾向が認められた。施設の特徴の差や調査条件の違いなどを考慮すると、国際間での単純な比較は難しいが、これらの調査は抗菌薬適正使用推進活動で優先すべき介入事項の検討や、介入後の効果測定に不可欠な情報が得られるものであり、繰り返し実施されるべきである。 1. Ricchizzi E, et al. Euro Surveill. 2018;23(46):1800394. 2. 薬剤耐性(AMR)アクションプランの実行に関する研究班 . http://amr.ncgm.go.jp/pdf/20191125_report.pdf(2021 年 12 月アクセス)
大血管または小血管の病変に起因する脳卒中患者の心房細動検出に用いる長期心臓モニタリングと通常治療の効果の比較:STROKE-AF無作為化試験
大血管または小血管の病変に起因する脳卒中患者の心房細動検出に用いる長期心臓モニタリングと通常治療の効果の比較:STROKE-AF無作為化試験
Effect of Long-term Continuous Cardiac Monitoring vs Usual Care on Detection of Atrial Fibrillation in Patients With Stroke Attributed to Large- or Small-Vessel Disease: The STROKE-AF Randomized Clinical Trial JAMA. 2021 Jun 1;325(21):2169-2177. doi: 10.1001/jama.2021.6470. 原文をBibgraph(ビブグラフ)で読む 上記論文の日本語要約 【重要性】大血管または小血管の病変に起因する虚血性脳卒中患者では、心房細動(AF)のリスクが高いと考えられておらず、この集団でのAF発症率は明らかになっていない。 【目的】12ヵ月間の追跡で検討した大血管または小血管の病変に起因する虚血性脳卒中患者のAF検出で、長期心臓モニタリングが通常治療より有効性が高いかどうかを明らかにすること。 【デザイン、設定および参加者】STROKE-AF試験は無作為化(1対1)多施設共同(米国の33施設)臨床試験であり、2016年4月から2019年7月までに患者496例を登録し、2020年8月まで主要評価項目を追跡した。60歳以上の患者または脳卒中の危険因子が1つ以上ある50~59歳の患者で、植込み型心臓モニタ(ICM)植込み前10日以内に大血管または小血管の病変に起因する指標となる脳卒中を発症した患者を適格とした。 【介入】患者を無作為化により、指標となる脳卒中から10日以内にICMを植え込む介入群(242例)、12誘導心電図やホルター心電図によるモニタリング、遠隔モニタリング、発作時心電図記録計などの施設ごとの標準治療を実施する対照群(250例)に割り付けた。 【主要評価項目】12ヵ月間での30秒以上持続したAFの発症。 【結果】無作為化した492例(平均[SD]年齢、67.1[9.4]歳;女性185例[37.6%])のうち、417例[84.8%]が12ヵ月間の追跡を終了した。CHA2DS2-VASc(うっ血性心不全、75歳以上、糖尿病、脳卒中または一過性脳虚血発作、血管疾患、65~74歳、性別)スコア中央値(四分位範囲)は5(4~6)であった。12ヵ月時点のAFの検出率は、ICM群の方が対照群よりも有意に高かった(27例[12.1%] vs 4例[1.8%];ハザード比、7.4[95%CI、2.6~21.3];P<0.001)。ICMを植え込んだICM群の221のうち、4例(1.8%)にICM処置に起因する有害事象が発現した(植込み部位感染1例、切開部出血2例、挿入部痛1例)。 【結論および意義】大血管または小血管の病変に起因する虚血性脳卒中患者にICMモニタリングを植え込んだ方が通常治療よりも12ヵ月間でAFを多く検出した。 第一人者の医師による解説 臨床的に将来の心原性脳塞栓症の発症予防に寄与するか否かはさらなる研究が必要 秋山 久尚 聖マリアンナ医科大学内科学(脳神経内科)病院教授 MMJ. February 2022;18(1):8 潜因性脳卒中・脳梗塞(cryptogenic stroke)、塞栓源不明脳塞栓症(ESUS)患者における心房細動(AF)の検出に長時間植込み型心臓モニター(ICM)が有用であることはCRYSTAL-AF試験で報告され、現在の治療ガイドラインでも継続的な心臓モニターが推奨されるに至っている。一方、大血管アテローム硬化(以下、アテローム血栓性脳梗塞)または小血管閉塞(以下、ラクナ梗塞)での発症機序は一般に動脈硬化、リポヒアリノーシスとされ、AFの寄与が高くないという考えから、長時間心臓モニターを含めたAFの検索は通常行われず、AFの検出(率)とその臨床的意義は不明である。 今回報告されたSTROKE-AF試験では、米国の包括的脳卒中センター 33施設において、2016年4月~ 2019年7月にかけてTOAST分類により心原性脳塞栓症、cryptogenic stroke、ESUSではなく、アテローム血栓性脳梗塞またはラクナ梗塞と診断された60歳以上、または50 ~ 59歳で、1つ以上の脳卒中危険因子(うっ血性心不全、高血圧、糖尿病、90日以上前の脳梗塞、その他の虚血性血管疾患など)を有する患者496人のうち4人を除外した492人(平均67.1歳、女性37.6%)を対象として登録している。これらを介入群と対照群1:1に無作為化し、介入群(242人)ではアテローム血栓性脳梗塞またはラクナ梗塞の発症後10日以内に長時間 ICM(Reveal LINQ™[Medtronic社])を植込み、対照群(250人)では短時間心電波形モニター(12誘導心電図、ホルタ心電図、テレメトリーまたはイベントレコーダー)を行い、2020年8月まで(平均331.4±90.9日)の観察期間中、どちらがAF(30秒以上持続)を検出する上で有用かを検討した。 その結果、12カ月間の追跡期間中、ICM群では27人(12.1%)、対照群では4人(1.8%)にAFが検出され(ハザード比 , 7.4)、検出時期の中央値はそれぞれ99日目と181日目、AF最長持続時間の中央値はICM群で88分であった。アテローム血栓性脳梗塞、ラクナ梗塞の2病型群でのサブグループ解析では、AF検出率もICM群(それぞれ11.7%、12.6%)、対照群(2.3%、1.0%)と病型にかかわらずICM群の方が有意に高値であったが、ICM群の両病型間では有意差はなかった。また事後解析で、12カ月間における脳梗塞再発率は介入群と対照群との間で有意差はなかった。 本試験では、アテローム血栓性脳梗塞またはラクナ梗塞でのAF検出においても長時間 ICMが有用なことが明らかとなった。長時間心臓モニタリングの汎用はAF検出の重要性に新しい視点を示したが、アテローム血栓性脳梗塞、ラクナ梗塞におけるAF検出が、臨床的に将来の心原性脳塞栓症の発症予防に寄与するか否かはさらなる研究が必要である。
発熱がない尿路感染症男性に用いる7日間と14日間の抗菌薬治療の症状消失に対する効果の比較:無作為化臨床試験
発熱がない尿路感染症男性に用いる7日間と14日間の抗菌薬治療の症状消失に対する効果の比較:無作為化臨床試験
Effect of 7 vs 14 Days of Antibiotic Therapy on Resolution of Symptoms Among Afebrile Men With Urinary Tract Infection: A Randomized Clinical Trial JAMA. 2021 Jul 27;326(4):324-331. doi: 10.1001/jama.2021.9899. 原文をBibgraph(ビブグラフ)で読む 上記論文の日本語要約 【重要性】一般的な感染症の最適な治療期間を明らかにすることが、抗菌薬の効果を維持するための重要な戦略である。 【目的】発熱がない男性の尿路感染症(UTI)の治療にシプロフロキサシンまたはトリメトプリム/スルファメトキサゾールを使用する場合、7日間の治療が14日間の治療に対し非劣性であるかを明らかにする。 【デザイン、設定および参加者】米国退役軍人省の2つの医療センターで、症候性UTIと推定された発熱のない男性にシプロフロキサシンまたはトリメトプリム/スルファメトキサゾールを投与した無作為化二重盲検プラセボ対照非劣性試験(2014年4月から2019年12月にかけて登録;最終追跡日2020年1月28日)。適格男性1058例中272例を無作為化した。 【介入】担当医が処方した抗菌薬を7日間継続したのち、参加者を無作為化により8~14日目に抗菌薬治療を継続するグループ(136例)とプラセボを投与するグループ(136例)に割り付けた。 【主要評価項目】主要評価項目は、実際の抗菌薬治療終了から14日後までのUTI症状消失とした。非劣性マージンを10%とした。主解析にはas-treated集団(28回中26回以上服薬し、連続未服薬が2回以下の参加者)を用いた。治療のアドヒアランスに関係なく無作為化した全患者を副次解析の対象とした。試験薬投与中止後28日以内のUTI症状再発または有害事象を副次評価項目とした。 【結果】無作為化した272例(年齢中央値[四分位範囲]69[62~73]歳)のうち、100%が試験を完了し、254例(93.4%)をas-treated集団として主解析の対象とした。7日群の131例中122例(93.1%)および14日群の123例中111例(90.2%)に症状の消失が認められ(差2.9%[片側97.5%CI -5.2%~∞])、非劣性の基準を満たした。無作為化した患者を対象とした副次解析では、7日群の136例中125例(91.9%)および14日群の136例中123例(90.4%)に症状の消失が認められた(差、1.5%[片側97.5%CI -5.8%~∞])。7日群の131例中13例(9.9%)および14日群の123例中15例(12.9%)にUTI症状再発が認められた(差、-3.0%[95%CI -10.8~6.2%];P=0.70)。7日群の136例中28例(20.6%)および14日群136例中33例(24.3%)に有害事象が発現した。 【結論および意義】発熱はないがUTIが疑われる男性に対するシプロフロキサシンまたはトリメトプリム/スルファメトキサゾールの7日間投与が、抗菌薬治療後14日目までのUTI症状消失で14日間投与に対し非劣性であった。この結果は、発熱がない男性のUTI治療に用いるシプロフロキサシンまたはトリメトプリム/スルファメトキサゾールの14日間投与の代案として7日間投与を支持するものである。 第一人者の医師による解説 短期間でも効果は劣らないが白人の高齢者が多いなどさまざまな前提条件に留意 石倉 健司 北里大学医学部小児科学主任教授 MMJ. February 2022;18(1):21 さまざまな感染症で、抗菌薬投与は従来から行われているより短期間でも有効であることが示されている。しかし男性の無熱性尿路感染症に対する同様の検討は行われておらず、短期間投与の有効性が示されれば、特にグラム陰性菌に対する抗菌薬使用量の減少に寄与することが期待される。そこで本論文の著者らは、これらの患者を対象に米国の2つの退役軍人病院で、抗菌薬の7日間と14日間投与の非劣性検証デザインによる無作為化プラセボ対照試験を計画した。 試験方法はpragmaticであり、対象者は無熱性尿路感染症に対してすでに臨床的診断のもとにシプロフロキサシンまたはトリメトプリル /スルファメトキサゾールによる治療が開始されている男性患者の中から登録された。尿路感染症は症状により診断され、尿培養は推奨されているが必須でなかった。対象者は8日以降の治療に関して、すでに使用されている抗菌薬の継続群もしくはプラセボ投与群に無作為に割り付けられた。主要評価項目は治療遵守群における抗菌薬投与終了14日後の症状改善率とされ、非劣性マージンは効果の差10%以内と設定された。 計画では290人の登録が必要であったが、実際には272人が無作為化された(各群136人)。年齢中央値は各群とも70歳、白人は79%(7日治療群)と78%(14日治療群)、間歇的カテーテル使用は18%と17%、糖尿病の合併率は34%と44%、最も頻度の高い症状はともにdysuriaで68%と65%であった。主要評価項目である抗菌薬終了14日後の症状消失(治療遵守例254人で評価)は、7日治療群で93.1%、14日治療群で90.2%(群間差 ,2.9%;95% CI, -5.2%~∞)であり、事前に定めた非劣性の定義を満たしていた。272人全体での評価、試験終了後14日後での評価などでも結果は変わらなかった。有害事象でも大きな差はなかった。以上から、男性無熱性尿路感染症の症状改善効果に関して、抗菌薬の7日間投与は14日間投与に対して非劣性であることが示された。 本試験の結果に関しては、さまざま前提条件に留意することが必要である。すなわち、米国における白人の男性退役軍人を主な対象にしていること、糖尿病合併が多いこと、抗菌薬が2剤に限られていることなどである。一方、試験の実施には学ぶ点が多い。Pragmaticなことに加え、study personnelの協力の下、データベースから患者候補をリストアップして積極的に患者にアプローチして登録している。さらに電話、メールに加え、状況によっては実際に患者宅に訪問するなど、その後の進捗管理も整備されている。日本での臨床試験の実施においても、大いに参考にしたい。
重度 SGA児の陣痛発来前の介入的分娩 学業成績を悪化させる恐れ
重度 SGA児の陣痛発来前の介入的分娩 学業成績を悪化させる恐れ
Association Between Iatrogenic Delivery for Suspected Fetal Growth Restriction and Childhood School Outcomes JAMA. 2021 Jul 13;326(2):145-153. doi: 10.1001/jama.2021.8608. 上記論文のアブストラクト日本語訳 ※ヒポクラ×マイナビ 論文検索(Bibgraph)による機械翻訳です。 【重要】胎児発育制限(FGR)が疑われる乳児の適時分娩は、死産防止と未熟児の最小化のバランスが重要であり、特にFGRが疑われる乳児の多くは正常に成長しているため。 【目的】FGRの疑いによる異所性出産と小児期の学校でのアウトカムとの関連について検討する。 【デザイン、設定および参加者】オーストラリア・ビクトリア州の2003年1月1日から2013年12月31日までの妊娠32週以上の出生の周産期データと、準備校、学校3・5・7年生での発達・教育テストのスコアを関連付けた後向き全人口コホート研究である。フォローアップは2019年に終了した。 【曝露】FGRの疑い有無、FGRの疑いに対する異所性分娩(分娩前の早期誘発または帝王切開と定義)の有無、妊娠年齢に対する小児の有無(SGA)であった。 【主要評および測定法】副次的アウトカムは,学校入学時に5つの発達領域のうち2つ以上が下位10パーセンタイルであること,3,5,7年生において5つの教育領域のうち2つ以上が国の最低基準以下であることとした。 【結果】出生集団705 937人の乳児において,出生時の平均妊娠期間は39.1週(SD,1.5),平均出生体重は3426(SD,517)gであった。出生集団は、発達の結果が181 902人、教育の結果が425 717人の子どもにつながった。FGRが疑われない重症SGA児(出生時体重3%未満)と比較して、FGRが疑われて出産した重症SGA児は早く生まれていた(平均妊娠週数37.9週 vs 39.4週)。また,就学時の発達不良のリスクも有意に高かった(16.2% vs 12.7%,絶対差 3.5%[95% CI,0.5%-6.5]); 調整オッズ比[aOR],1.5%[0.5%]).36 [95% CI, 1.07-1.74] )および3,5,7年生での教育的アウトカム不良(例えば,7年生では:13.4% vs 10.5%; 絶対差は2.9% [95% CI, 0.4%-5.5%]; aOR, 1.33 [95% CI, 1.04-1.70] )であった.)FGR の疑いで分娩された正常な成長(出生時体重≧10 パーセンタイル)の乳児と FGR の疑いがない乳児の間には,発達の結果において有意差はなかった(8.6% 対 8.1%; 絶対差 0.5% [95% CI, -1.1% ~ 2.0%] ); aOR, 1.17 [95% CI, 0.95 ~ 1.70])。45])または第3学年、第5学年、第7学年の教育的アウトカムが、早生まれ(平均妊娠週数、38.0週 vs 39.1週)であるにもかかわらず、低下した。 【結論と関連性】オーストラリアのビクトリア州で実施したこの探索的研究では、FGRの疑いがある重症SGA児の異所性出産は、FGRの疑いがない重症SGA児と比較してより悪い学校アウトカムと関連していた。FGRが疑われる正常な発育の乳児の異所性分娩は、FGRが疑われない正常な発育の乳児と比較して、より悪い学校での転帰とは関連しなかった。 第一人者の医師による解説 日本でも新生児集中治療後の児の中長期的なデータベースの構築が必要 飛彈 麻里子 慶應義塾大学医学部小児科准教授 MMJ. April 2022;18(2):52 子宮内胎児発育遅延(fetal growth restriction;FGR)と児の出生体重が10パーセンタイル未満(small for gestational age;SGA)は 混同されやすいが、出生時体重で明確に定義されるSGAと異なり、FGRは定義や診断基準、適正な管理方法が確立していない(1)。日本産科婦人科学会の診療ガイドライン(産科編)2020では、FGRを超音波検査で算出される胎児推定体重およびその経時的変化、羊水過少の有無や胎児腹囲計測値などから、総合的かつ臨床的に診断する、としている(2)。 FGRの原因は、胎児因子、母体因子、胎児付属物因子と多様だが、原因に関わらず、出生児がSGAだった場合は、周産期死亡や成長後の精神発達遅滞、生活習慣病罹患との関連が示唆されている。胎児の状態を慎重に評価し、臍帯血流や胎児頭囲成長の異常出現時には医療的介入による分娩とし、胎外での児の治療を適切な時期に始めることが、児の予後改善に寄与するとされている。一方で、在胎期間はすべての病態において、児の予後に最も強い影響を与える因子であり、臨床の場では産科と小児科との総合的な判断で分娩の時期を決定することになる(2)。 本論文は、FGRを指摘されていたSGAでは計画的分娩(陣痛発来前の分娩誘導による経腟分娩や帝王切開分娩)が学業成績不良と関連する可能性を報告した。この結果は、欧州の大規模無作為化比較試験(3)など既存の報告とは反対の結果である。著者らも認めているように、本研究には複数の重要な限界があり、この結果をそのまま臨床に応用することはできない。しかし、1つの州という医療や教育の条件が統一されている環境下での、10年間にわたる母集団70万人以上を背景とする42万人以上の学業評価の結果という膨大なコホート研究であり、今後さらに洗練された報告が期待される。 一方で、本論文は日本の新生児医療の課題を改めて浮き彫りにする。国際的に高く評価され、多くの重症 SGAを救命している我が国の新生児医療だが、新生児集中治療室退院後のフォローアップデータは限られている。特に学業成果については標準化された評価指標がなく、国内の施設間比較ですら困難である。本論文の結論に疑問を持つ日本の小児科医は多いと思うが、反論するための国内のデータは乏しい。すでに関係者間では認識されている課題だが、子どもたちによりよい医療を提供するためにも、日本の中長期的なフォローアップデータベースの構築が必要である。 1. Fetal Growth Restriction: ACOG Practice Bulletin, Number 227. Obstet Gynecol. 2021;137(2):e16-e28. 2. 産婦人科診療ガイドライン産科編(2020).「CQ307-1: 胎児発育不全(FGR)のスクリーニングは?」「CQ307-2:胎児発育不全(FGR)の取り扱いは?」https://www.jsog.or.jp/activity/pdf/gl_sanka_2020.pdf(2022 年 2 月閲覧) 3. Walker DM, et al. Am J Obstet Gynecol. 2011;204(1):34.e1-9.
オピオイド漸減に過剰摂取や精神的危機を引き起こす危険性
オピオイド漸減に過剰摂取や精神的危機を引き起こす危険性
Association of Dose Tapering With Overdose or Mental Health Crisis Among Patients Prescribed Long-term Opioids JAMA. 2021 Aug 3;326(5):411-419. doi: 10.1001/jama.2021.11013. 上記論文のアブストラクト日本語訳 ※ヒポクラ×マイナビ 論文検索(Bibgraph)による機械翻訳です。 【重要性】オピオイド関連の死亡率や国の処方ガイドラインにより、慢性疼痛に対して長期的にオピオイド療法を処方されている患者において、用量の漸減が行われている。過量投与や精神衛生上の危機など、漸減に関連するリスクに関する情報は限られている。 【目的】安定した長期高用量オピオイドを処方された患者において、オピオイドの漸減と過量投与や精神衛生上の危機の割合に関連があるかを評価する。 デザイン・設定・参加者】2008年から2019年のOptumLabs Data Warehouseからの非識別医療・薬局請求と登録データを用いたレトロスペクティブコホートスタディである。12か月のベースライン期間に安定した高用量(平均50モルヒネミリグラム当量/日)のオピオイドを処方され、少なくとも2か月のフォローアップを受けた米国の成人が対象となった。 【曝露】オピオイドテーパリング、7か月のフォローアップ期間内の60日の重なった6週間のいずれかの期間に平均日用量が少なくとも15%相対減少したと定義される。最大月間投与量減少速度は、同期間中に計算された。 【主要評および測定法】最大12か月の追跡期間中に、(1)薬物過剰摂取または離脱、(2)メンタルヘルス危機(うつ、不安、自殺企図)のための救急または病院受診が発生した。離散時間負の二項回帰モデルにより,テーパリング(対テーパリングなし)および減量速度の関数としてアウトカムの調整済み発生率比(aIRR)を推定した。 【結果】ベースライン期間203 920人の安定した期間を経て,113 618人が最終コホートに含まれることになった。漸減を受けた患者の54.3%が女性で(漸減を受けなかった患者では53.2%)、平均年齢は57.7歳(58.3歳)、商業保険に加入していたのは38.8%(41.9%)であった。漸減後の患者期間は、100人年当たり9.3件の過量投与イベントと関連しており、漸減しない期間では100人年当たり5.5件であった(調整後発生率差、100人年当たり3.8件[95%CI、3.0-4.6];aIRR、1.68 [95%CI 、1.53-1.85])。テーパリングは、非テーパリング期間の100人年当たり3.3件と比較して、100人年当たり7.6件のメンタルヘルスクライシスイベントの調整後発生率と関連していた(調整後発生率差、100人年当たり4.3件[95%CI、3.2~5.3];aIRR、2.28[95%CI、1.96~2.65])。月間の最大減量速度を10%増加させると、過剰摂取のaIRRは1.09(95%CI、1.07-1.11)、精神衛生上の危機は1.18(95%CI、1.14-1.21)となった。 【結論と関連性】安定した長期高用量オピオイド療法を処方されている患者では、テーパーリングイベントは過剰摂取および精神衛生上の危機のリスク上昇と有意に関連することが示された。これらの知見はテーパリングの潜在的な害について疑問を投げかけるものであるが,観察研究デザインにより解釈は限定的である。 第一人者の医師による解説 長期間、高用量オピオイドを処方されている患者では拙速な漸減は有害 松本 俊彦 国立精神・神経医療研究センター精神保健研究所薬物依存研究部部長/薬物依存症センター センター長 MMJ. April 2022;18(2):36 北米では、医師による安易なオピオイド鎮痛薬の処方が災いして、過剰摂取による死亡者の急増、深刻な社会問題(「オピオイドクライシス」)を招いている(1)。このため、各種の治療ガイドラインでは、慢性疼痛に対して長期処方されているオピオイドの漸減が推奨されているが(2)、同時に、漸減が、患者の過剰摂取や精神的危機、違法オピオイドへのアクセスを高める可能性も指摘されている。しかし、漸減の有害性リスクに関する検証は十分になされていない。 本論文において著者らは、高用量オピオイドを長期間処方されている患者に対する漸減療法が、過剰摂取や精神的危機の発生と関連するのかどうかを、後ろ向きコホート研究のデザインで検討している。米国の医療関連データベース Optum Labs Data Warehouseを用いて、モルヒネ換算50mg/日以上のオピオイドを1年以上継続投与され、2カ月間以上の追跡を受けた患者113,618人のうち、追跡期間中に漸減(1日の平均投与量にして15%以上の減量と定義)が行われた患者を特定した。この患者データを用いて、追跡期間中における漸減実施期間と非実施期間との間で、薬物の過剰摂取や精神的危機(うつ病、不安、自殺企図)といった有害事象の発生頻度を比較した。 検討の結果、オピオイドの漸減は、過剰摂取で3.8件 /100人・年(95%信頼区間[CI], 3.0〜4.6)、精神的危機で4.3件 /100人・年(95% CI, 3.2〜5.3)の増加と関連していた。精神的危機の種類別にみると、漸減はうつ病2.46件 /100人・年(95%CI, 2.05 ~ 2.96)、不安障害1.79件 /100人・年(95 % CI, 1.48 ~ 2.15)、自殺企図3.30件/100人・年(95% CI, 2.19 ~ 4.98)といった有害事象の増加につながった。さらに、漸減速度と有害事象の関連についての検討では、月あたりの減量速度を10%速めると、過剰摂取は1.09件/100人・年(95% CI, 1.07〜1.11)、精神的危機は1.18件 /100人・年(95% CI, 1.14〜1.21)増えることが明らかにされた。 慢性疼痛に対して高用量オピオイド治療を漫然と長期間続けるのは問題であり、処方医は常に漸減の可能性を探るべきなのは言うまでもない。しかし、拙速な漸減は有害事象を引き起こすリスクもある。現状では、日本は北米のような悲劇的状況とはほど遠いが、数年前よりオピオイドの非がん性疼痛への適応拡大がなされ、痛みの臨床現場にも高用量オピオイド長期使用患者は徐々に増えつつある2。その意味で、臨床医が知っておくべき情報といえよう。 1. 山口重樹ら . 精神科治療学 . 2020;35(7):777-782. 2. 木村嘉之ら . 日本臨牀 . 2019;77(12):2065-2070.
院外心停止患者への体温管理療法 目標体温31℃は34℃と比べ優位性なし
院外心停止患者への体温管理療法 目標体温31℃は34℃と比べ優位性なし
Effect of Moderate vs Mild Therapeutic Hypothermia on Mortality and Neurologic Outcomes in Comatose Survivors of Out-of-Hospital Cardiac Arrest: The CAPITAL CHILL Randomized Clinical Trial JAMA. 2021 Oct 19;326(15):1494-1503. doi: 10.1001/jama.2021.15703. 上記論文のアブストラクト日本語訳 ※ヒポクラ×マイナビ 論文検索(Bibgraph)による機械翻訳です。 【重要性】院外心停止の昏睡生存者は、高い確率で死亡と重度の神経学的損傷を経験する。現在のガイドラインでは、24時間32℃~36℃の標的体温管理を推奨している。しかし、小規模の研究では、より低い体温を目標とすることの潜在的な有益性が示唆されている。 【目的】軽度の低体温(34℃)と比較して中等度の低体温(31℃)が院外心停止の昏睡生存者の臨床転帰を改善するかどうかを判断する。 【デザイン、設定および参加者】カナダ、オンタリオ東部の三次心臓ケアセンターで行われた単施設、二重盲検、無作為、臨床優越試験の実施。2013年8月4日~2020年3月20日に院外心停止患者計389名を登録し、2020年10月15日に最終フォローアップを行った。 【介入】患者を24時間、目標体温を31℃(n=193)または34℃(n=196)に設定して温度管理する方法にランダムに割り付けた。 主要転帰・評価]主要転帰は180日後の全死亡または神経学的転帰不良であった。神経学的転帰は障害評価尺度を用いて評価し,神経学的転帰不良はスコア5以上(範囲:0~29,29は最悪の転帰[植物状態])と定義された。180日後の死亡率や集中治療室滞在期間など19の副次的転帰があった。 【結果】一次解析に含まれた367例(平均年齢61歳,女性69例[19%])のうち,366例(99.7%)が試験を完遂した。主要転帰は31℃群では184例中89例(48.4%),34℃群では183例中83例(45.4%)で発生した(リスク差,3.0%[95%CI, 7.2%-13.2%]; 相対リスク,1.07[95%CI, 0.86-1.33]; P = 0.56)。19の副次的転帰のうち、18は統計的に有意ではなかった。180日後の死亡率は,目標体温を31 ℃と34 ℃に設定して治療した患者では,それぞれ43.5%と41.0%であった(P = 0.63).集中治療室での入院期間の中央値は31 ℃の群で長かった(10日対7日;P = 0.004)。31 ℃群と34 ℃群の有害事象のうち、深部静脈血栓症は11.4%と10.9%に、下大静脈の血栓は3.8%と7.7%に発生した。 【結論と関連性】病院外心停止の昏睡生存者において、目標温度31 ℃は目標温度34℃と比較して180日の死亡率または神経学的転帰不良率を有意には減少させなかった。しかし,臨床的に重要な差を検出するには,この試験は力不足であった可能性がある。 【臨床試験登録】ClinicalTrials. gov Identifier:NCT02011568 第一人者の医師による解説 一律ではなく蘇生後脳障害の重症度に応じた体温管理療法を目指すべき 多村 知剛 Pulmonary and Critical Care Medicine, Brigham and Women’s Hospital/Harvard Medical School(research fellow) MMJ. April 2022;18(2):46 院外心停止は日本で年間12万件以上発生するが、1カ月後の生存率は10%未満で極めて予後不良である。虚血再灌流障害に起因した重篤な脳障害が主な死因である。心停止後昏睡状態の患者に対しては32 ~ 36℃の目標体温を24時間維持する体温管理療法が推奨されている。動物実験や小規模な観察研究では、より低い28 ~ 2℃を目標体温とすることが神経学的転帰改善に有益である可能性が示唆されているが、32℃未満を目標体温とした無作為化試験は行われていない。 本論文は院外心停止患者に対する体温管理療法において、目標体温を31℃(中等度低体温)とすることが、34℃(軽度低体温)に比べ、転帰改善に有用であるか否かを検討した単施設無作為化二重盲検比較試験の報告である。2013年8月4日~20年3月20日に、カナダの3次心臓病治療センターに入院した院外心停止後昏睡状態(入院時のGlasgow Coma Scaleが8点以下)の18歳以上の患者389人を、目標体温31℃群(193人)または34℃群(196人)に無作為に割り付け、血管内に留置したカテーテルを用いた体温冷却装置により24時間の体温管理療法を行った。主要評価項目は180日時点での全死亡または神経学的転帰不良の複合エンドポイント、副次評価項目は180日死亡、集中治療室(ICU)滞在日数など19項目とした。389人のうち、割り付けられた治療を受け主要評価項目の解析対象となったのは367人(平均年齢61歳、男性298人[81%])であった。主要評価項目のイベントは31℃群で48.4%、34℃群で45.4%に発生し、有意差を認めなかった。副次評価項目のうち、ICU滞在日数のみ31℃群で有意に延長した(中央値 10日 対 7日)が、他の18項目に統計学的有意差は認めなかった。低体温療法の有害事象である血栓症については、深部静脈血栓症の発現率は31℃群11.4%、34℃群10.9%、下大静脈血栓症はそれぞれ3.8%と7.7%と、いずれも有意な群間差を認めなかった。 無作為化前から氷嚢によって冷却する努力がされたこと、体温の変動が小さい血管内冷却法を用いて厳密に体温管理が行われた点で本研究は評価に値する。しかし他の体温管理療法の大規模研究(1),(2)と同様に、心原性心停止(急性心筋梗塞や致死性不整脈)かつ約80%が男性と対象に偏りがある点で、本研究結果の一般化は難しい。また蘇生後脳障害の重症度は患者ごとに異なり、重症度の高い患者では中等度低体温が有益である可能性が残る。蘇生後脳障害の改善には重症度評価法の開発とそれに応じたテーラーメイドな体温管理療法を目指す必要がある。 1. Dankiewicz J, et al. N Engl J Med. 2021;384(24):2283-2294. 2. Kirkegaard H, et al. JAMA. 2017;318(4):341-350.
小児集中治療室での呼吸器離脱プロトコル 人工呼吸器装着日数の短縮はわずか
小児集中治療室での呼吸器離脱プロトコル 人工呼吸器装着日数の短縮はわずか
Effect of a Sedation and Ventilator Liberation Protocol vs Usual Care on Duration of Invasive Mechanical Ventilation in Pediatric Intensive Care Units: A Randomized Clinical Trial JAMA. 2021 Aug 3;326(5):401-410. doi: 10.1001/jama.2021.10296. 上記論文のアブストラクト日本語訳 ※ヒポクラ×マイナビ 論文検索(Bibgraph)による機械翻訳です。 【重要性】小児集中治療室で乳幼児や小児を侵襲的な機械的人工呼吸から解放するための最適な戦略に関するエビデンスは限られている。 【目的】鎮静と人工呼吸器解放のプロトコルによる介入が、長期の機械的人工呼吸が必要と予想される乳幼児や小児の侵襲的機械的人工呼吸の期間を短縮するかどうかを明らかにする。 【デザイン、設定および参加者】英国の17の病院施設(18の小児集中治療室)を対象に、通常のケアからプロトコル介入まで順次無作為化した、実用的な多施設、ステップウェッジ、クラスター無作為化臨床試験を実施した。2018年2月から2019年10月にかけて、長期の機械的換気が必要と予想される重症の乳幼児と小児8843人を募集した。最終追跡日は2019年11月11日であった。 【介入】小児集中治療室では、通常のケア(n=4155人の乳幼児と小児)または鎮静と人工呼吸器解放のプロトコル介入(n=4688人の乳幼児と小児)が行われ、鎮静レベルの評価、人工呼吸器解放の可能性をテストするための自発呼吸試験を実施する準備ができているかどうかのスクリーニングを毎日行い、鎮静と準備のスクリーニングを見直し、患者に関連する目標を設定するための毎日のラウンドを行った。 【主要評および測定法】主要アウトカムは、人工呼吸を開始してから最初に抜管に成功するまでの侵襲的機械換気の期間であった。治療効果の主要な推定値は、長期の機械的人工呼吸が必要と予想される乳幼児と小児について、暦時間とクラスター(病院部位)を調整したハザード比(95%CI)であった。 【結果】本試験を完了した乳幼児と小児は8843人(年齢中央値、8カ月[四分位範囲、1~46カ月]、42%が女性)であった。抜管成功までの時間(中央値)は、通常のケアと比較してプロトコルによる介入のほうが有意に短かった(それぞれ64.8時間対66.2時間、調整後中央値の差:-6.1時間[四分位範囲:-8.2~-5.3時間]、調整後ハザード比1.11[95%CI、1.02~1.20]、P = 0.02)。重篤な有害事象である低酸素症は、通常のケアでは11人(0.3%)であったのに対し、プロトコルによる介入では9人(0.2%)で発生し、血管以外のデバイスの脱落はそれぞれ2人(0.04%)と7人(0.1%)で発生した。 【結論と関連性】長期の機械的換気が必要と予想される乳幼児と小児において、鎮静と人工呼吸器の解放のプロトコルによる介入は、通常のケアと比較して、最初の抜管成功までの時間を統計的に有意に短縮する結果となった。しかし、この効果の大きさの臨床的重要性は不明である。 【臨床試験登録】isrctn.org Identifier:ISRCTN16998143。 第一人者の医師による解説 小児では抜管前評価で呼吸器離脱が見送られる特殊な事情も理解を 秋山 類(大学院研究生)/清水 直樹(主任教授) 聖マリアンナ医科大学小児科学講座 MMJ. April 2022;18(2):48 小児集中治療室(PICU)に入室する患者は侵襲的人工呼吸(MV)が必要となる。MV管理は、これに関連した肺炎や肺障害などを併発する可能性があり、早期離脱が必要となる。成人ではABCDEバンドル(※)を導入し、院内死亡率が低下した(1)。小児ではMVからの離脱プロトコル化を検証した研究はエビデンス不足であった。 英国の18PICUが鎮痛鎮静とMV離脱のプロトコル化を目指してSedation AND Weaning In hildren(SANDWICH)に参加している。全施設が集中治療医により管理され、うち8割は年間入室患者数が500を超える。本研究では、まず各施設は従来群として診療を行った。各施設に決められた日程でプロトコル訓練を実施し、以後は介入群として診療した。プロトコル内容は鎮痛鎮静の評価と目標設定、自発呼吸トライアル(SBT)適否の選別、SBT実施、多職種の回診である。期間は2018年2月〜19年10月、24時間以上のMVが必要と想定される8,843人を対象とした。 結果、MV装着時間の中央値は介入群64.8、従来群66.2と1.4時間の短縮だった(各施設の症例数などの調節後では6.1時間)。またPICU入室日数の中央値は両群5日で差がなく、入院期間は有意に延長した(介入群9.6、従来群9.1日)。抜管成功、計画外抜管、再挿管のいずれも有意差はなかった。介入群では抜管後の非侵襲的陽圧換気の装着が有意に多かった。 研究計画時は、MV装着期間が1日短縮すると想定していたが、実際にはわずかだった。原因として、幅広い患者層(心疾患と呼吸器疾患が各3割、神経疾患は1割、内因性疾患による予定外入室が6割前後)、SBT実施が少ない、が挙げられる。プロトコルのうちSBT実施以外の遵守率は40〜90%だが、SBT実施の遵守率は15〜60%であった。SBT非実施の理由は、分泌物や気道浮腫により気道確保が必要、意識の覚醒が不十分、再手術を想定、血管作動薬の投与が多量、筋力低下などであった。 本研究では、抜管の前提を満たさない患者が含まれ、MV装着時間の短縮がわずかだったと考える。今後はSANDWICHプロトコルにSBT非実施となる患者の選別段階を組み込む必要があるかもしれない。本研究は集中治療医の管理する比較的大規模なPICUで実施されており、有害事象が抑制された可能性がある。日本では、集中治療医が24時間管理するのは半数、年間入院患者数が500を超えるのは18%である(2)。本研究結果を日本国内で適用する際には、これらの背景の差異も考慮する必要がある。 ※ ABCDE バンドル;A:毎日の覚醒トライアル、B:毎日の呼吸器離脱トライアル、C:鎮静・鎮痛薬の選択、D:せん妄のモニタリングと管理、E:早期離床をまとめて行う介入。バンドルとは bundle で束の意味。 1. Balas MC, et al. Crit Care Med. 2014;42(5):1024-1036. 2. 日本集中治療医学会小児集中治療委員会 . わが国における小児集中治療室の現状調査 . 日集中医誌 2019;26:217-225.
院内心停止へのアドレナリン・バソプレシン・ステロイド投与で自己心拍再開率向上
院内心停止へのアドレナリン・バソプレシン・ステロイド投与で自己心拍再開率向上
Effect of Vasopressin and Methylprednisolone vs Placebo on Return of Spontaneous Circulation in Patients With In-Hospital Cardiac Arrest: A Randomized Clinical Trial JAMA. 2021 Oct 26;326(16):1586-1594. doi: 10.1001/jama.2021.16628. 上記論文のアブストラクト日本語訳 ※ヒポクラ×マイナビ 論文検索(Bibgraph)による機械翻訳です。 【重要性】これまでの試験で、院内心停止時にバソプレシンとメチルプレドニゾロンを投与することで転帰が改善する可能性が示唆されている。 【目的】院内心停止時にバソプレシンとメチルプレドニゾロンを併用投与すると自然循環の復帰が改善するかどうかを判断する。 【デザイン、設定および参加者】デンマークの10病院で実施した多施設、無作為二重盲検、プラセボ対照臨床試験。2018年10月15日から2021年1月21日の間に、院内心停止の成人患者計512名を対象とした。最終90日フォローアップは2021年4月21日であった。 【介入】患者はバソプレシンとメチルプレドニゾロンの併用投与(n = 245)またはプラセボ投与(n = 267)に無作為に割り付けられた。エピネフリン初回投与後にバソプレシン(20 IU)およびメチルプレドニゾロン(40 mg)、または対応するプラセボを投与した。エピネフリンを追加投与するごとにバソプレシンまたは対応するプラセボを追加投与し、最大4回投与した。 主要評価項目は自然循環の回復であった。副次的アウトカムは30日後の生存率と良好な神経学的転帰(Cerebral Performance Categoryスコア1または2)とした。 【結果】無作為化された512例中、501例がすべての組み入れ基準を満たし、除外基準はなく解析に含まれた(平均[SD]年齢、71[13]歳;男性322[64%])。バソプレシンおよびメチルプレドニゾロン群では 237 例中 100 例(42%),プラセボ群では 264 例中 86 例(33%)で自然循環の回復が得られた(リスク比 1.30 [95% CI, 1.03-1.63]; リスク差 9.6% [95% CI, 1.1%-18.0%]; P = .03).30日時点で,介入群23人(9.7%)とプラセボ群31人(12%)が生存していた(リスク比,0.83 [95% CI,0.50-1.37];Risk difference:-2.0% [95% CI, -7.5% to 3.5%]; P = 0.48)。30日目において,介入群の18人(7.6%)とプラセボ群の20人(7.6%)で良好な神経学的転帰が認められた(リスク比,1.00 [95% CI, 0.55-1.83]; リスク差,0.0% [95% CI, -4.7% to 4.9%]; P > .99).自然循環が回復した患者において,高血糖は介入群で77例(77%),プラセボ群で63例(73%)に発生した.高ナトリウム血症は、介入群で28(28%)、プラセボ群で27(31%)に発生した。 【結論と関連性】院内心停止患者において、バソプレシンおよびメチルプレドニゾロンの投与は、プラセボと比較して、自然循環の復帰の可能性を有意に増加させることが示された。しかし、この治療が長期生存に有益か有害かは不明である。 【臨床試験登録】ClinicalTrials. gov Identifier:NCT03640949 第一人者の医師による解説 治療方針を大きく変更するものではないが 蘇生治療に用いる薬物候補として検証は続く 鈴木 昌 東京歯科大学教授・市川総合病院救急科部長 MMJ. April 2022;18(2):47 本論文は、デンマークの10施設で行われた無作為化二重盲検試験の報告である。18歳以上の病院内発生心停止(院内心停止)に対して、試験薬群(237人)とプラセボ群(264人)の効果比較を行っている。試験薬群では初回のアドレナリン投与後、すみやかにバソプレシン 20 IUとメチルプレドニゾロン 40 mgを投与、その後バソプレシンをアドレナリン投与直後に20 IUずつ、計80 IUまで投与した。その結果、主要評価項目である自己心拍再開(ROSC)率は試験薬群で42%に達し、プラセボ群33%に対して有意に高かったが(リスク比 , 1.30;95%信頼区間 , 1.03〜1.63)、主な副次評価項目である30日後生存と神経学的転帰に有意差はなかった。有害事象(高血糖、高 Na血症)の発現率に差はなかった。以上から、院内心停止においてバソプレシンとメチルプレドニゾロン投与の追加はROSCを改善するが、30日後の予後改善は観察されなかった、と結論している。 バソプレシンは、以前の蘇生ガイドラインに記載のあった薬剤だが、アドレナリンを凌駕する効果がみられず現在は使用しないことになっている。また、ステロイド投与については現状では結論が得られていない。このため投与の推奨はない(1)。しかし、これらの薬剤が俎上に上る背景には、蘇生治療に対する重症敗血症治療の考え方の反映がある。さらに、バソプレシンや内因性ステロイドの血中濃度は心停止後の非生存者で低いことから、その補充への期待が根強い。バソプレシンではV1受容体の直接刺激による血管収縮が期待される。ステロイドでは副腎を含めた全身の虚血と再灌流障害とが惹起する不安定な血行動態や過剰な免疫応答を改善する効果が期待される。 本研究の2つの先行研究(2),(3)は院内心停止に対してROSC、生存退院、ならびに神経学的転帰の改善を報告していた。本研究はこれらの研究の外部検証と位置付けられ、この研究を含めた3つの研究の系統的レビューでは、成人の院内心停止に対するアドレナリン・バソプレシン・ステロイド投与がROSC改善に寄与するとした(4)。なお、先行2研究と系統的レビューの著者は同一グループと目される。 本研究は、院内心停止を対象にしている。院内と院外心停止とで、心肺蘇生術に違いはなく、各種の研究結果も相互に参照されて活用される。一方、この両者が同様の患者群あるいは治療対象であるとは考えられていない。したがって、この結果が直ちに院外心停止を含めた治療方針を大きく変更するものではない。とはいえ、蘇生治療に用いる薬物の候補としてさらに検討と検証が続くものと思われる。 1. Berg KM, et al. Circulation. 2020;142(16_suppl_1):S92-S139. 2. Mentzelopoulos SD, et al. Arch Intern Med. 2009;169(1):15-24. 3. Mentzelopoulos SD, et al. JAMA. 2013;310(3):270-279. 4. Holmberg MJ, et al. Resuscitation. 2022;171:48-56.
小児の市中肺炎へのアモキシシリン投与 高用量、長期である必要なし
小児の市中肺炎へのアモキシシリン投与 高用量、長期である必要なし
Effect of Amoxicillin Dose and Treatment Duration on the Need for Antibiotic Re-treatment in Children With Community-Acquired Pneumonia: The CAP-IT Randomized Clinical Trial JAMA. 2021 Nov 2;326(17):1713-1724. doi: 10.1001/jama.2021.17843. 上記論文のアブストラクト日本語訳 ※ヒポクラ×マイナビ 論文検索(Bibgraph)による機械翻訳です。 【重要】小児市中肺炎(CAP)に対するアモキシシリン経口投与の最適な用量と期間は明らかではない。 【目的】低用量アモキシシリンが高用量に対して非劣性であるか、3日間の治療が7日間に対して非劣性であるかどうかを明らかにする。 【デザイン、設定および参加者】2017年2月から2019年4月の間に英国28病院、アイルランド1病院の救急部および入院病棟から退院時にアモキシシリンで治療した臨床的に診断された6ヶ月以上の小児824人を登録し、最終試験訪問は2019年5月21日とした多施設無作為2×2要因非劣性試験を実施した。 【介入】小児を、低用量(35~50mg/kg/d:n=410)または高用量(70~90mg/kg/d:n=404)で、短期間(3日間:n=413)または長期間(7日間:n=401)のアモキシシリン経口投与に1:1に無作為化した。 【主要評および測定法】主要アウトカムはランダム化後28日以内の呼吸器感染に対する臨床的指示による抗生物質の再処置であった。非劣性マージンは8%であった。副次的アウトカムは、保護者が報告した9つのCAP症状の重症度/期間、3つの抗生物質関連有害事象、および結核菌Streptococcus pneumoniae分離株の表現型抵抗性であった。 【結果】4群のいずれかに無作為化された824名のうち、814名が少なくとも1回の試験薬投与を受け(年齢中央値[IQR]2.5歳[1.6-2.7]、男性421名[52%]、女性393名[48%])、789名(97%)で主要評価項目を確認することができた。)低用量と高用量の比較では,低用量12.6%と高用量12.4%(差:0.2%[1-sided 95% CI -∞~4.0] ),3日投与と7日投与12.5%(差:0.1%[1-sided 95% CI -∞~3.9] )で主要アウトカム発現が認められた。両群とも非劣性が示され、投与量と投与期間の間に有意な相互作用は認められなかった(P = 0.63)。事前に規定した14の副次的エンドポイントのうち、咳嗽期間(中央値12日 vs 10日;ハザード比[HR]、1.2[95%CI、1.0~1.4];P = .04)および咳による睡眠障害(中央値、4日 vs 4日;HR、1.2 [95%CI, 1.0 ~ 1.4];P = .03)については3日 vs 7日治療でのみ有意差がみられた。重症CAPの小児のサブグループでは、主要エンドポイントは、低用量投与者の17.3%対高用量投与者の13.5%(差、3.8%[1サイド95%CI、-∞~10%];相互作用のP値=0.18)、3日間治療者の16.0%対7日間治療者の14.8%で発生した(差、1.2%[1サイド95%CI、-∞~7.結論と意義】救急部または病棟から退院した(48時間以内)CAPの小児において、抗生物質の再処置の必要性に関して、低用量の外来経口アモキシシリンは高用量に対して非劣性、3日間の期間は7日間に対して非劣性であった。しかし、この結果を解釈する際には、疾患の重症度、治療環境、以前に受けた抗生物質、非劣性マージンの許容度について考慮する必要がある。 【臨床試験登録】ISRCTN Identifier:ISRCTN76888927。 第一人者の医師による解説 重症・抗菌薬先行投与例の評価、咳の持続・不眠症状などについて検証必要 中村 敦 名古屋市立大学大学院医学研究科臨床感染制御学教授 MMJ. April 2022;18(2):50 欧州における有病率調査では、小児の救急患者はプライマリケアと比較して死亡率が高く、抗菌薬を必要とする深刻な細菌感染の可能性があり、下気道感染症が2番目に多い。入院を要する5歳未満の小児市中肺炎(CAP)患者の約3分の1は細菌が関与するとされており、抗菌薬が投与され続ける場合が多い。しかし、治癒を達成しつつ薬物曝露を最小限に抑えるために抗菌薬治療を最適化することは重要である。小児 CAPの治療について抗菌薬の異なる投与期間を比較した試験はほとんどなく、用量と期間の両方を同時に比較した試験はない。アモキシシリン(AMPC)は幼児のCAPの第1選択抗菌薬として広く推奨されているが、その最適な投与量は不明である。 本論文は、英国・アイルランド 29施設において小児 CAPに対する経口 AMPC治療を低用量群(35〜50mg/kg/日)と高用量群(70〜90 mg/kg/日)、短期群(3日間)と長期群(7日間)の2×2群にランダム化して非劣性を検証したCAP-IT試験の報告である。治療開始後28日以内の呼吸器感染症に対するAMPC以外の抗菌薬再治療の有無を主要評価項目とし、親から報告されたCAP症状の重症度と期間、AMPC投与と関連する有害事象、28日目の鼻咽頭分離肺炎球菌のペニシリン感受性などを副次評価項目としている。 4群にランダム化され救急部門または病棟から48時間以内に退院した824人のうち、814人(年齢中央値2.5歳)が少なくとも1回のAMPC投与を受けた。主要評価項目の抗菌薬再治療率は低用量群12.6%、高用量群12.4%、短期群12.5%、長期群12.5%と、投与量と投与期間の間に有意な相互作用はなく、非劣性であった(P=0.63)。副次評価項目では、咳の持続時間(P=0.04)、咳による睡眠障害(P=0.03)のみ短期群と長期群の間で有意差がみられたが、AMPCの投与量、期間による咳の重症度、有害事象、分離菌のペニシリン感受性に有意差はなかった。有害事象のうち皮膚発疹は長期群に多くみられ、治療の完遂率は短期群が高かった。 最近 SAFER試験で小児 CAPに対する5日間と10日間の高用量 AMPC治療で同等の治癒率が示され(1)、成人 CAPの3日間のβ -ラクタム療法が8日間の治療に劣らないことも報告された(2)。本研究でも小児 CAPに対するAMPCの低用量、短期投与は高用量、長期投与に対し抗菌薬の再治療の必要性に関して非劣性が示された。ただし重症例の用量比較、抗菌薬前投与例の用量および投与期間の比較では、有意ではないものの非劣性基準を満たしておらず、肺炎の重症度や治療背景、抗菌薬前投与、非劣性マージンの妥当性について検証する必要がある。 1. Pernica JM, et al. JAMA Pediatr. 2021;175(5):475-482. 2. Dinh A, et al. Lancet. 2021;397(10280):1195-1203.
ブドウ球菌菌血症患者に対するアルゴリズムに基づく治療と通常治療の臨床的成功および 重篤な有害事象に与える影響の比較:無作為化臨床試験
ブドウ球菌菌血症患者に対するアルゴリズムに基づく治療と通常治療の臨床的成功および 重篤な有害事象に与える影響の比較:無作為化臨床試験
Effect of Algorithm-Based Therapy vs Usual Care on Clinical Success and Serious Adverse Events in Patients with Staphylococcal Bacteremia: A Randomized Clinical Trial JAMA 2018 Sep 25;320(12):1249-1258. 上記論文のアブストラクト日本語訳 ※ヒポクラ×マイナビ 論文検索(Bibgraph)による機械翻訳です。 【重要】ブドウ球菌性菌血症に対する適切な抗生物質の投与期間は不明である。 【目的】ブドウ球菌性菌血症の治療期間を定めたアルゴリズムと標準治療の比較により、重篤な有害事象を増加させずに非劣性の有効性が得られるかを検証する。 デザイン・設定・参加者】米国(n=15)とスペイン(n=1)の学術医療センター16施設で2011年4月から2017年3月までブドウ球菌性菌血症の成人に関わる無作為抽出試験を実施した。患者は、黄色ブドウ球菌の場合は治療終了後42日間、コアグラーゼ陰性ブドウ球菌菌血症の場合は28日間フォローアップされた。対象は18歳以上で、1回以上の血液培養で黄色ブドウ球菌またはコアグラーゼ陰性ブドウ球菌が陽性となった患者である。無作為化時点で既知または疑いのある複雑な感染症を有する患者は除外した。 【介入】患者はアルゴリズムに基づく治療(n = 255)または通常の診療(n = 254)に無作為に割り付けられた。アルゴリズム群では、診断評価、抗生物質の選択、治療期間が事前に設定されたのに対し、通常診療群では、抗生物質、治療期間、その他の臨床的ケアの側面について、臨床医が自由に選択できた。 【主要評および測定法】主要アウトカムは、(1)盲検判定委員会が決定し15%以内のマージンで非劣性を検証した臨床成功、(2)意図的治療集団の重篤有害事象率、優位性を検証したもの。 【結果】無作為化された509例(平均年齢56.6歳[SD、16.8]、女性226例[44.4%])中、480例(94.3%)が試験を完了した。臨床的成功は、アルゴリズムに基づく治療に割り付けられた255人中209人と、通常の診療に割り付けられた254人中207人で記録された(82.0% vs 81.5%、差、0.5%[1サイド 97.5% CI, -6.2% ~ ∞]])。重篤な有害事象は、アルゴリズムに基づく治療では32.5%、通常の診療では28.3%で報告された(差、4.2%[95%CI、-3.8%~12.2%])。単純または合併症のない菌血症のプロトコールごとの患者において、平均治療期間はアルゴリズムベースの治療で4.4日、通常の診療で6.2日(差、-1.8日[95%CI、-3.1~-0.6])。 結論および関連性]ブドウ球菌菌血症患者では、検査および治療のガイドとしてアルゴリズムを使用すると通常のケアと比べて非劣性の臨床成功率となることが示された。重篤な有害事象の発生率に有意な差はなかったが、信頼区間が広いため解釈は限定的である。アルゴリズムの有用性を評価するために、さらなる研究が必要である。 【臨床試験登録】ClinicalTrials. gov Identifier:NCT01191840。 第一人者の医師による解説 臨床医の経験、勘、努力を補助する AI どのように応用していくか 舘田 一博 東邦大学医学部微生物・感染症学講座教授 MMJ.February 2019;15(1):7 近年、人工知能(artificial intelligence:AI)の応用 がさまざまな分野で検討されている。医療において も例外ではなく、特に画像診断スクリーニング、鑑 別診断の列挙、パニック状態の早期発見などへの応用が検討されている。本論文では、500例を超える実症例を対象にブドウ球菌菌血症患者へのアルゴリズム治療の有効性に関して検討している。黄色ブド ウ球菌による菌血症症例では、uncomplicatedと complicatedに分類し、それぞれ14(±2)日、28~ 42(±2)日での治療を実施し、通常治療群(主治医 の判断による症例ごとの対応)における治療成績と比較している。結果は前述されているように、 intention-to-treat(ITT)群における重篤な有害事象 (副反応)がアルゴリズム群で83/255(32.5%)、通常 治療群で72/254(28.3%)と有意な差は認められていない。死亡率はどうかというと、アルゴリズム群で 16/255(6.3%)、通常治療群で14/254(5.5%)であったと報告されている。これらの結果はブドウ球菌菌 血症、少なくとも黄色ブドウ球菌菌血症患者に対する治療アルゴリズム導入の“非劣性”が示された成績となっている。 本研究ではアルゴリズム導入において注意しなければいけないポイントも示されている。治療中に認められる臓器ごとの重篤な有害事象に関して、アルゴリズム群で腎・尿路系障害が12例(4.7%)、通常治療群では4例(1.6%)となっている。黄色ブドウ球菌の中でもメチシリン耐性黄色ブドウ球菌による菌血症は依然として頻度の高い耐性菌感染症であり、本症に対してはバンコマイシンあるいはダプトマイシンの治療が選択されることになっている。バンコマイシンが選択される場合には当然のことながら本剤の血中濃度測定が原則であり、患者の全身状態や合併症(特に腎機能障害など)に応じて投与量を調整する必要がでてくる。アルゴリズム治療においても、血中濃度測定による投与量の調整が求められるわけであるが、この結果の解釈とその後の対応は治療の成否を決める極めて重要な要因であることは よく知られている。この点に関連しているのかどうかは不明であるが、治療薬による有害事象の発現のための治療中止がアルゴリズム群で 4 例(1.6%)、 通常治療群で1例(0.4%)みられたと報告されている。もちろんこの差異にも有意差は認められていな い。 医療の分野におけるAIの導入は我々が目指さなければいけない方向性である。ただしAIに完全に 置き換わりにくいのも医学であろう。優秀な臨床医の経験と勘、さらには無駄とも思えるような努力が 1人の患者を救うことがあるというのも真実である。その助けとしてどのようにAIを導入していくのか、AIの応用の仕方に関する研究がさらに活発になってくるものと思われる。
エンドトキシン値が高い敗血症性ショック患者における標的化ポリミキシンB血液潅流の 28日死亡率に与える効果:EUPHRATES無作為化臨床試験
エンドトキシン値が高い敗血症性ショック患者における標的化ポリミキシンB血液潅流の 28日死亡率に与える効果:EUPHRATES無作為化臨床試験
Effect of Targeted Polymyxin B Hemoperfusion on 28-Day Mortality in Patients With Septic Shock and Elevated Endotoxin Level: The EUPHRATES Randomized Clinical Trial JAMA 2018 Oct 9;320(14):1455-1463. 上記論文のアブストラクト日本語訳 ※ヒポクラ×マイナビ 論文検索(Bibgraph)による機械翻訳です。 【重要】ポリミキシンBの血液灌流は敗血症の血中エンドトキシン濃度を低下させる.エンドトキシン活性は、迅速なアッセイで血中濃度を測定できる。敗血症性ショックでエンドトキシン活性が高い患者に対して、ポリミキシンBの血液灌流を行うことで臨床転帰が改善する可能性がある。 【目的】敗血症性ショックでエンドトキシン活性が高い患者において、従来の内科治療にポリミキシンB血液灌流を追加することで従来の治療単独と比較して生存率が改善するかどうかを検証することである。 【デザイン、設定および参加者】2010年9月~2016年6月に北米の55の三次病院で登録された敗血症性ショックでエンドトキシン活性測定値が0.60以上の成人重症患者450例を対象とした多施設共同無作為化臨床試験。最終フォローアップは2017年6月。 【介入】登録後24時間以内に完了した2回のポリミキシンB血液灌流治療(90~120分)+標準療法(n=224人)、または偽血液灌流+標準療法(n=226人)。 【主要評および測定法】主要評価項目は、無作為化された患者(全参加者)および多臓器不全スコア(MODS)が9以上の患者における28日後の死亡率とした。 【結果】適格登録患者450例(平均年齢59.8歳;女性177例[39.3%];平均APACHE IIスコア29.4[範囲,0~71,スコアが高ければ重症度が高い])のうち449例(99.8%)が試験に完走した。ポリミキシンBの血液灌流は,全参加者において28日後の死亡率に有意差を認めなかった(治療群223例中84例[37.7%] vs 偽薬群226例中78例[34.5%],リスク差[RD]3.2%,95% CI,-5.7%~12.7%).0.0%;相対リスク[RR],1.09;95%CI,0.85-1.39;P = 0.49) またはMODSが9以上の集団(治療群,146例中65例[44.5%] vs 偽薬,148例中65例[43.9%]; RD,0.6%;95% CI,-10.8% ~ 11.9%; RR,1.01;95% CI,0.78-1.31;P = 0.92 )においてであった。全体で264件の重篤な有害事象が報告された(治療群65.1% vs 偽薬群57.3%)。最も頻度の高い重篤な有害事象は、敗血症の悪化(治療群10.8% vs 偽薬群9.1%)および敗血症性ショックの悪化(治療群6.6% vs 偽薬群7.7%)でした。 【結論と関連性】敗血症性ショックおよび高エンドトキシン活性の患者では、ポリミキシンB血液浄化療法+通常の内科治療と偽治療+通常の内科治療を比較しても28日目の死亡率は低下しませんでした【試験登録】 ClinicalTrials. gov Identifier:NCT01046669. 第一人者の医師による解説 エンドトキシンのみ標的の治療に限界 CARS への対策必要 織田 成人 千葉大学大学院医学研究院救急集中治療医学教授 MMJ.February 2019;15(1):9 ポリミキシンB固定化ファイバーによる直接血液 潅流法(PMX-DHP)は、日本発の治療法であり、日 本では敗血症性ショック、特に消化管穿孔による敗 血症性ショックに対して広く用いられている。しか し、PMX-DHPの有効性に関する報告の多くは日本の観察研究によるものであり、明確なエビデンスに乏しかった。最近になり主に海外で無作為化対照 試験(RCT)が行われ、その効果が検証されてきた。 本論文は3報目のRCT結果である。 最初の RCT は 2009 年に報告された EUPHAS trial(1)である。イタリアの集中治療室(ICU)10施設が参加し、腹膜炎による重症敗血症/敗血症性ショックを対象にPMX群34人、対照群30人が登録された。その結果、28日死亡率が対照群の53%に比べPMX群で32%と有意に改善したことが報告された。しかし、最終的には統計学的な有意差はないことが判明した。 2つ目の RCT は、2015 年に報告された ABDOMIX study(2)である。フランスのICU 16施設 が参加し、腹膜炎による敗血症性ショックを対象に PMX群119人、対照群113人が登録された。その 結果、28日死亡率はPMX群で27.7%、対照群で 19.5%と有意差はなく、2次評価項目である臓器障害の改善でも有意差は認められなかった。 本論文で紹介しているEUPHRATES trialでは過 去最大の450人が登録された。これまでのRCTと異なり、腹膜炎のみでなく他の原因による敗血症性 ショックも対象としており、臓器障害スコアが9以 上の重症患者を対象とし、血中エンドトキシン活性 を米食品医薬品局(FDA)が認めたEAA法で測定し、 EAAが0.6以上の高値例のみを登録した。しかし PMXによる28日死亡率の改善は認められなかった。一方、本文中には記載されていないが、電子サプリメントではPMX群で有意な血圧上昇が認められており、循環動態改善に関しては一定の効果が認められている。 2016年に敗血症の国際定義が見直され、敗血症 は「感染に対する制御不十分な生体反応に起因する 臓器障害」と定義された(Sepsis-3)(3) 。従来、グラム 陰性菌の菌体成分であるエンドトキシンは敗血症の病態で重要な役割を演じていると考えられていたが、最近の研究によりエンドトキシン以外のさまざまな病原体関連分子パターン(PAMPs)や、体内で 産生されるダメージ関連分子パターン(DAMPs)が 自然免疫を活性化し、各種サイトカインの産生を介して臓器障害を発症することが明らかにされている(4) 。 これら3つのRCTの結果は、敗血症という複雑な病態に対して、数多あるPAMPsやDAMPsの中で エンドトキシンのみを標的にした治療の限界を示すものであり、以前行われた抗エンドトキシン抗体や、 エンドトキシン受容体アンタゴニスト(エリトラン) の試験が失敗したのも同じ理由と考えられる。 敗血症という複雑な病態を制御するには、自然免疫の活性化によって生じた高サイトカイン血症の制御や、同時に発生する代償性抗炎症反応症候群 (CARS)への対策が必要であり、これらの治療に関するエビデンスの確立が望まれる。 1. Cruz DN, et al. JAMA. 2009;301(23):2445-2452. 2. Payen DM, et al. Intensive Care Med. 2015;41(6):975-984. 3. Singer M, et al. JAMA. 2016;315(8):801-810. 4. Vincent JL, et al. Lancet. 2013;381(9868):774-775.
APPAC無作為化臨床試験における単純性(合併症のない)急性虫垂炎に対する抗菌化学療 法の5年追跡調査
APPAC無作為化臨床試験における単純性(合併症のない)急性虫垂炎に対する抗菌化学療 法の5年追跡調査
Five-Year Follow-up of Antibiotic Therapy for Uncomplicated Acute Appendicitis in the APPAC Randomized Clinical Trial JAMA 2018 Sep 25;320(12):1259-1265. 上記論文のアブストラクト日本語訳 ※ヒポクラ×マイナビ 論文検索(Bibgraph)による機械翻訳です。 【重要】短期的な結果は、合併症のない急性虫垂炎の治療において手術の代わりに抗生物質を使用することを支持しているが、長期的な結果は知られていない。 【目的】合併症のない急性虫垂炎の治療に抗生物質を使用した後の虫垂炎の後期再発率を明らかにする。 【デザイン、設定および参加者】虫垂切除術と抗生物質療法を比較した多施設共同無作為化臨床試験(Appendicitis Acuta(APPAC))における患者の5年間の観察追跡調査。コンピュータ断層撮影で合併症のない急性虫垂炎が確認された18~60歳の患者530人を,虫垂切除術を受ける群(n=273)と抗生物質療法を受ける群(n=257)に無作為に割り付けた。最初の試験は2009年11月から2012年6月までフィンランドで実施され、最終フォローアップは2017年9月6日だった。今回の解析では、抗生物質のみで治療した患者群の5年間の転帰を評価することに焦点を当てた。 【介入】開腹盲腸手術 vs テルタペネムを3日間静注した後、レボフロキサシンとメトロニダゾールを7日間経口投与する抗生物質療法。本解析では、5年後の追跡調査において、抗生物質治療後の晩期(1年後)の虫垂炎の再発、合併症、入院期間、病欠などの副次的評価項目を事前に規定した。 【結果】本試験に登録された530名の患者(女性201名、男性329名)のうち、273名(年齢中央値、35歳[IQR、27-46])が盲腸手術を受ける群に、257名(年齢中央値、33歳[IQR、26-47])が抗生物質治療を受ける群に無作為に割り付けられた。最初に抗生物質を投与されたものの、最初の1年以内に盲腸手術を受けた70名(27.3%[95%CI、22.0%-33.2%]、70/256名)に加え、さらに30名の抗生物質治療を受けた患者(16.1%[95%CI、11.2%-22.2%]、30/186名)が1年から5年の間に盲腸手術を受けました。虫垂炎の再発の累積発生率は、2年目に34.0%(95%CI、28.2%~40.1%、87/256人)、3年目に35.2%(95%CI、29.3%~41.4%、90/256人)、4年目に37.1%(95%CI、31.2%~43.3%、95/256人)、5年目に39.1%(95%CI、33.1%~45.3%、100/256人)であった。抗生物質投与群で、その後、再発した虫垂炎のために虫垂切除術を受けた85人のうち、76人は合併症のない虫垂炎、2人は合併症のある虫垂炎、7人は虫垂炎ではなかった。5年後の全体の合併症率(手術部位感染、切開ヘルニア、腹痛、閉塞症状)は、盲腸手術群が24.4%(95%CI、19.2%~30.3%)(n=60/246)、抗生物質投与群が6.5%(95%CI、3.8%~10.4%)(n=16/246)であり(P<0.001)、術後に17.9%ポイント(95%CI、11.7~24.1)高くなる計算となった。入院期間は両群間で差がなかったが、病欠には有意な差があった(盲腸手術群が11日多い)。 【結論と関連性】合併症のない急性虫垂炎に抗生物質で初期治療を受けた患者のうち、5年以内の晩期再発の可能性は39.1%であった。この長期追跡調査は、合併症のない急性虫垂炎に対する手術の代替として、抗生物質治療のみを行う可能性を支持するものである。 【臨床試験登録】ClinicalTrials. gov Identifier:NCT01022567。 第一人者の医師による解説 虫垂切除と抗菌化学療法 患者が選択できるよう情報提供が必要 佐々木 淳一 慶應義塾大学医学部救急医学教授 MMJ.February 2019;15(1):11 急性虫垂炎の標準的治療は、過去1世紀以上にわたり、手術(虫垂切除術)であるという概念が確立されていた。抗菌薬の開発以降、急性虫垂炎に対する抗菌化学療法の有効性を示す無作為化対照試験 (RCT)やメタ解析などの結果が発表されたが、いずれも追跡が短期間であった。このため、臨床現場 では穿孔や腹腔内膿瘍形成などの合併症を避けるため、依然として虫垂切除術が標準的治療として君臨 している。ここで、合併症のない急性虫垂炎において抗菌化学療法が手術の代替治療になりうることを長期間の追跡結果に基づき示そうとした研究が、今回報告されたAPPAC(The Appendicitis Acuta)である。 APPACでは、虫垂切除術と抗菌化学療法に割り付けた合併症のない急性虫垂炎患者を10年後まで 追跡する計画を立て、抗菌化学療法群257人と手術 (虫垂切除術)群273人がフィンランド国内6病院から登録された。治療後の評価は外科医が担当し、穿孔、腹膜炎などにより手術が必要になった場合には 抗菌薬化学療法群にも手術を実施することとされ た。主要エンドポイントは、抗菌化学療法群が手術を要さない退院および1年時点の非再発、手術群は 虫垂切除術の成功である。1年間の追跡結果(1)によると、抗菌化学療法成功率は72.7%(186/256)であった。しかし、1年時点でのintention-to-treat(ITT) 解析による両群間の治療効果差は- 27.0%で、虫垂切除術に対する抗菌化学療法の非劣性を示すことはできなかった(非劣性マージン24%)。 本論文ではAPPAC の5年間の追跡結果が報告された。抗菌化学療法群、手術群ともに246人で、抗菌化学療法群の急性虫垂炎の累積再発率は、1年時 点で27.3%、2年時点で34.0%、3年時点で35.2%、 4年時点で37.1%、5年時点で39.1%であった。一方、 5年時点の全合併症(創感染、瘢痕ヘルニア、腹痛、 狭窄症状など)発生率は手術群24.4%、抗菌化学療 法群6.5%、抗菌化学療法群で有意に低かった。これらの結果より、抗菌化学療法は手術の代替治療法として有用なことが示唆されたと結論付けられている。しかし、割り付け後の治療がプロトコールではなく個々の外科医の裁量に委ねられており、必要以上に虫垂切除が行われた可能性があること、手術が現在主流となっている腹腔鏡下ではなく開腹であったこと、抗菌化学療法が最適な処方内容であったか不明であること、抗菌薬に起因した耐性菌問題について検討されていないことなど、いくつかの問題点が指摘されている。単純性(合併症を伴わない)急性虫垂炎の標準的治療は虫垂切除から抗菌化学療法に変わったというには、10年時点での結果も含め、さらなる検討が必要である。しかし、患者に抗菌化学療法と手術に関する情報を提供し、意思決定できるようにすべきであろう。 1. Salminen P, et al. JAMA. 2015;313(23):2340-2348.
2017年米国心臓病学会/米国心臓協会血圧ガイドラインを用いた若年成人における血圧分類とその後の心血管イベントとの関連性
2017年米国心臓病学会/米国心臓協会血圧ガイドラインを用いた若年成人における血圧分類とその後の心血管イベントとの関連性
Association of Blood Pressure Classification in Young Adults Using the 2017 American College of Cardiology/American Heart Association Blood Pressure Guideline With Cardiovascular Events Later in Life JAMA 2018 Nov 6 ;320 (17 ):1774 -1782. 上記論文のアブストラクト日本語訳 ※ヒポクラ×マイナビ 論文検索(Bibgraph)による機械翻訳です。 【重要】若年成人期の血圧(BP)レベルと中年期までの心血管疾患(CVD)イベントとの関連についてはほとんど知られていない。 【目的】2017年米国心臓病学会(ACC)/米国心臓協会(AHA)BPガイドラインで定義された高血圧を40歳前に発症した若年成人は、正常血圧維持者と比較してCVDイベントリスクが高くなるかどうかを評価することである。 【デザイン、設定および参加者】1985年3月に開始された前向きコホート研究Coronary Artery Risk Development in Young Adults(CARDIA)研究において解析を実施した。CARDIAでは,米国の4つのフィールドセンター(アラバマ州バーミンガム,イリノイ州シカゴ,ミネソタ州ミネアポリス,カリフォルニア州オークランド)から18~30歳のアフリカ系米国人と白人5115人が登録された。アウトカムは2015年8月まで入手可能であった。 【曝露】初診から40歳以降に最も近い検査までに測定された最高血圧を用いて,各参加者を正常血圧(未治療収縮期血圧[SBP]<120mmHg,拡張期血圧[DBP]<80mmHg:n=2574)に分類した。)BP上昇(未治療のSBP 120-129 mm HgおよびDBP <80 mm Hg;n = 445)、ステージ1高血圧(未治療のSBP 130-139 mm HgまたはDBP 80-89 mm Hg;n = 1194)、またはステージ2高血圧(SBP ≥140 mm Hg, DBP≥90 mm Hg, または降圧剤を服用;n = 638)であった。 【主要評および測定法】CVDイベント:致死性および非致死性の冠動脈性心疾患(CHD),心不全,脳卒中,一過性脳虚血発作,末梢動脈疾患(PAD)への介入。 【結果】最終コホートには成人4851人(アウトカム追跡開始時の平均年齢,35.7歳[SD,3.6];女性2657人[55%];アフリカ系アメリカ人2441人[50%];降圧剤服用206人[4%])を含める。中央値18.8年の追跡期間中に,228件のCVDイベントが発生した(CHD,109件,脳卒中,63件,心不全,48件,PAD,8件)。正常血圧,血圧上昇,ステージ1高血圧,ステージ2高血圧のCVD発生率は,それぞれ1000人年当たり1.37(95%CI,1.07-1.75),2.74(95%CI,1.78-4.20),3.15(95%CI, 2.47-4.02),8.04(95% CI,6.45-10.03 )であった。多変量調整後,BP上昇,ステージ1高血圧,ステージ2高血圧のCVDイベントのハザード比は,正常BPに対してそれぞれ1.67(95%CI,1.01-2.77),1.75(95%CI,1.22-2.53)および3.49(95%CI,2.42-5.05)であった。 【結論と関連性】若年成人において,2017年米国心臓病学会/米国心臓協会(ACC/AHA)ガイドラインの血圧分類で定義された40歳前の血圧上昇,ステージ1高血圧,ステージ2高血圧の者は,40歳前の血圧が正常の者と比較してその後の心血管疾患イベントリスクが有意に高かった。ACC/AHA血圧分類システムは、心血管疾患イベントのリスクが高い若年成人を特定するのに役立つ可能性があります。 第一人者の医師による解説 若年成人の血圧異常への介入と、さらに若い年齢層での研究が必要 粟津 緑 慶應義塾大学医学部小児科非常勤講師 MMJ.April 2019;15(2) 2017年に発表された米国高血圧ガイドラインは、従来の高血圧前症(prehypertension、収縮期血圧 120 ~ 139 /拡張期血圧 80 ~ 89 mmHg) という分類を廃止し、120~129/80mmHg未満を 血圧上昇(elevated blood pressure)、130 ~ 139 /80 ~ 89 mmHgをstage 1 高血圧、 140/90mmHg以上をstage 2高血圧とした。 高血圧の基準が下がったため若年成人の患者数は 2~3倍に増加した。しかしすぐ薬物療法を行うのではなく、まず生活習慣の改善を指導し、その後も改善しなければ、心血管疾患の既往があるか、10 年間の動脈硬化性心血管疾患(ASCVD)リスクスコアが10%以上である場合に薬物療法を行うよう推奨されている。これにより降圧薬が必要な患者数はわずかな増加にとどまる。一方、ASCVDリスクスコアは40~79歳を対象に設定されたものであるため、若年成人(40歳未満)の多くはリスクが低く薬物療法の適応にならないという問題がある。 本研究は登録時18~30歳の米国人を対象としたCARDIA研究データに2017年基準を適用し検討した。血圧上昇群およびstage 1高血圧群の心 血管疾患発症リスクは 正常血圧群(120mmHg未 満 /80mmHg未満)に比べて有意にそれぞれ1.67、 1.75倍高かった。 本研究と同様の研究が韓国でも行われ、血圧異常者(血圧上昇と高血圧)の正常血圧者に対するハザード比は米国と同様に有意に上昇していた(1)。中年・老年層における同様の研究は多いがこの2研究は若年成人を対象とした初めての研究である。 両研究に共通した点がいくつかある。まず対象の半数以上が血圧異常に分類された。この理由として血圧測定法が適切でなかった可能性が考えられる。信頼性のある24時間血圧測定は行われていない。若年者では高血圧のレッテルを貼る前に真の高血圧であるか否かを確認することがより望ましい。また血圧上昇群とstage 1高血圧群は正常血圧群に比べBMIが大きく、糖・脂質代謝異常合併も多かった。したがってこれらが心血管疾患発症へ関与した可能性もある。若年血圧異常者には生活習慣の改善指導、糖・脂質代謝異常の是正も重要である。 今後、以上の点を考慮した介入研究が必要である。 また本研究の対象は18~30歳であるが、結果を外挿すると小児・思春期の軽度血圧上昇が将来の心血管リスクになる可能性もある。血圧はトラッキングするからである。小児の血圧基準はパーセ ンタイル値で定義されている。心血管疾患との関係が不明であるため便宜的に定義しているのであるが、本研究を発展させ、臓器障害マーカー(左室 肥大など)を盛り込みつつ成人のように心血管リスクとなる血圧値を設定することが望まれる。 1. Son JS1, et al. JAMA. 2018 Nov 6;320(17):1783-1792.
非閉塞性半月板損傷患者における早期手術と理学療法の膝関節機能への影響。ESCAPE Randomized Clinical Trial(無作為化臨床試験)。
非閉塞性半月板損傷患者における早期手術と理学療法の膝関節機能への影響。ESCAPE Randomized Clinical Trial(無作為化臨床試験)。
Effect of Early Surgery vs Physical Therapy on Knee Function Among Patients With Nonobstructive Meniscal Tears: The ESCAPE Randomized Clinical Trial JAMA 2018 Oct 2 ;320 (13 ):1328 -1337 . 上記論文のアブストラクト日本語訳 ※ヒポクラ×マイナビ 論文検索(Bibgraph)による機械翻訳です。 【重要】関節鏡下半月板切除術(APM)は理学療法(PT)よりも有効ではないことを示唆する最近の研究にもかかわらず、この手術は半月板損傷患者に依然として頻繁に行われている。 【目的】半月板損傷患者における患者報告膝機能の改善について、PTがAPMよりも非劣性かどうかを評価する。 【デザイン、設定および参加者】非劣性、多施設、無作為臨床試験をオランダの9病院で実施した。参加者は45~70歳の非閉塞性半月板断裂患者(膝関節のロッキングがない)。膝関節不安定症、重度の変形性関節症、肥満度が35以上の患者さんは除外した。募集は、2013年7月17日から2015年11月4日の間に行われた。参加者は24ヶ月間フォローアップされた(最終参加者フォローアップ、2017年10月11日) 【介入】321名の参加者は、APM(n=159)または事前に定義されたPTプロトコル(n=162)にランダムに割り当てられた。PTプロトコルは、協調運動と閉鎖運動連鎖の強化運動に焦点を当てた8週間16セッションの運動療法で構成された。 主要アウトカムと測定法]主要アウトカムは、国際膝関節文書委員会主観的膝フォーム(範囲、0~100;悪い方から良い方)の患者報告膝機能の24ヶ月フォローアップ期間のベースラインからの変化とした。非劣性マージンは、治療群間の差が8ポイントであると定義され、0.025の片側αで評価された。主要解析はintention-to-treatの原則に従った。 【結果】無作為化された321例(平均[SD]年齢58[6.6]歳,女性161例[50%])中,289例(90%)が試験を完了した(女性161例,男性158例)。PT群では、47人(29%)が24か月の追跡期間中にAPMを発症し、APMに無作為に割り付けられた8人(5%)がAPMを発症しなかった。24ヶ月の追跡期間中、APM群では26.2ポイント(44.8から71.5)、PT群では20.4ポイント(46.5から67.7)膝機能が改善された。全体の群間差は 3.6 ポイント(97.5% CI、-∞~6.5、非劣性の P 値 = 0.001)であった。有害事象は,APM 群で 18 例,PT 群で 12 例に発生した.再手術(APM群3例、PT群1例)および膝痛のための追加外来受診(APM群6例、PT群2例)が最も頻度の高い有害事象だった。 【結論と関連性】非閉塞性半月板断裂の患者において、24か月のフォローアップ期間における患者報告による膝機能の改善に関してPTはAPMに対して非劣位であった。これらの結果から、PTは非閉塞性半月板断裂患者に対する手術の代替療法と考えられる。 【臨床試験登録】ClinicalTrials. gov Identifier:NCT01850719。 第一人者の医師による解説 ロッキング症状を伴わない膝関節半月断裂にはまず理学療法を 芳賀 信彦 東京大学大学院医学系研究科外科学専攻感覚・運動機能医学講座リハビリテーション医学分野教授 MMJ.April 2019;15(2) ロッキング症状を伴わない膝関節半月断裂に対する理学療法の長期成績は、関節鏡視下半月部分切除に劣らないことが、オランダのESCAPE研究で示された。 膝関節半月は、関節の変性プロセスの一部として断裂することがあり、50歳以上では膝関節痛がなくても60%以上に半月断裂を認める。痛みを伴う場合の治療として関節鏡視下半月部分切除術が広く行われるが、一方で理学療法にも短期的な疼痛軽減効果がある。メタ解析では、半月切除術は術後 6カ月までは関節機能と疼痛の面で保存的治療よりも優れる一方、1~2年後までは効果が持続しないと報告されている。2年を超える長期的な効果は不明である。 本研究はオランダの9施設で行われたランダム化比較試験であり、2013~15年に45~70歳の、 ロッキング症状を伴わない膝関節痛を有し、MRIで半月断裂を確認した321人が登録された。約半数が理学療法群に割り付けられ、1回30分の理学療法が8週間にわたり計16回行われた。プログラムは、心肺機能の調整、協調運動・バランス訓練、閉鎖運動連鎖を用いた筋力増強からなる均一なものである。残りは半月切除群に割り付けられた。周術期には自宅で運動療法を行い、回復が不十分な場合は理学療法を受けた。 両群合わせて289人で24カ月間の追跡が可能であった。この間に理学療法群では47人が症状残存のために半月切除術を受けた。半月切除群のうち 8人は手術を受ける選択をせず、また半月切除後 2年以内に2人が人工膝関節置換術を受けた。主要評価項目である膝関節機能(IKDC自己申告スコア) に関するintention-to-treat(ITT)解析において、3 カ月、6カ月および24カ月目までの全体では、理学療法群が半月切除群に劣っていなかった(非劣性が 示された)が、12カ月と24カ月の時点では非劣性が示されなかった。実際に受けた治療による解析 でも同様の結果であった。副次的評価項目である荷 重時の疼痛に関するITT解析では、24カ月までの 全体で半月切除群が理学療法群より優れていたが、 実際に受けた治療による解析では差がなかった。 日本でも過去には変形性膝関節症に対する鏡視 下デブリドマンとして、変性断裂した半月の切除がよく行われていたが、今回の結果のように、短期的には疼痛が軽減し患者は満足するが、数年でむしろ関節症が進むこともあり、近年はあまり積極的には行われていない。膝関節痛の診療にかかわる医師は、本研究の結果も参考にし、ロッキング症状がない状態ではまず理学療法を行い、症状が残存、悪化した場合のみ半月部分切除術を行うスタンスが望ましい。
アメリカ人のための身体活動ガイドライン。
アメリカ人のための身体活動ガイドライン。
The Physical Activity Guidelines for Americans JAMA 2018 Nov 20 ;320 (19 ):2020 -2028 . 上記論文のアブストラクト日本語訳 ※ヒポクラ×マイナビ 論文検索(Bibgraph)による機械翻訳です。 【重要】米国の成人および青年の約80%は、十分な活動をしていない。身体活動は正常な成長と発達を促進し、人々の気分、機能、睡眠を良くし、多くの慢性疾患のリスクを減らすことができる。 【目的】アメリカ人のための身体活動ガイドライン第2版(PAG)の主要ガイドラインを要約する。 プロセスと証拠の統合】2018年身体活動ガイドライン諮問委員会は、身体活動と健康を支える科学について系統的レビューを実施した。委員会は38の質問と104のサブ質問に取り組み、研究の一貫性と質に基づいてエビデンスを評定した。強いまたは中程度と評価されたエビデンスが、主要なガイドラインの基礎となった。保健福祉省(HHS)は、2018年の身体活動ガイドライン諮問委員会科学報告書に基づき、PAGを作成しました。 【推奨事項】PAGは、複数の人口集団のさまざまな健康上の成果を改善するための身体活動の種類と量に関する情報とガイダンスを提供します。就学前の子ども(3歳から5歳)は、成長と発達を高めるために、一日を通して身体活動を行うべきである。6歳から17歳の子供と青年は、毎日60分以上の中等度から強度の身体活動を行う必要があります。成人は、少なくとも週に150分から300分の中強度の有酸素運動、または週に75分から150分の強度の有酸素運動、あるいは中強度と強度の有酸素運動の同等の組み合わせを行う必要があります。また、週に2日以上、筋肉を強化する活動を行う必要があります。高齢者は、有酸素運動や筋力強化の活動だけでなく、バランストレーニングを含む多成分の身体活動を行う必要があります。妊娠中および出産後の女性は、週に少なくとも150分、中強度の有酸素運動を行うべきです。慢性疾患や障害を持つ成人は、可能であれば、成人の主要なガイドラインに従い、有酸素運動と筋力強化の両方の活動を行う必要があります。勧告では、より多く動き、より少なく座ることが、ほぼすべての人に利益をもたらすことを強調しています。身体活動が最も少ない人は、中等度から高度の身体活動を適度に増やすことで最も恩恵を受ける。さらに、身体活動を増やすと、さらなる効果が得られます。 【結論と関連性】『アメリカ人のための身体活動ガイドライン第2版』は、実質的な健康上の利益をもたらす身体活動の種類と量に関する情報とガイダンスを提供している。医療専門家や政策立案者は、ガイドラインの認知を促進し、身体活動の健康上の利点を宣伝し、身体活動の増加を促進し、米国人口の健康を向上させるためのプログラム、実践、政策を実施する努力を支援する必要がある。 第一人者の医師による解説 日本人に対しても推奨される内容 岩田 慎平/野村 政壽(主任教授) 久留米大学医学部内科学講座内分泌代謝内科部門 MMJ.April 2019;15(2) 身体活動は正常な成長と発達を促進し、精神・身体機能、睡眠を改善し、多くの慢性疾患を予防する(1)。 その効果は男女問わず、小児から高齢者まで認められ、さらに周産期の女性や慢性疾患患者にも認められる。多くの米国人が十分な身体活動をしていない現状を踏まえ、今回、米国人に対してエビデンスに基づく推奨度の高い身体活動のガイドライ ン(第2版)がまとめられた。本ガイドラインでは、 以下に示す各人口集団に対して、種々の健康アウト カムを改善するための身体活動の種類と量に関する情報と指針を提示している。 ・未就学児(3~5歳)では、成長発達を促進するため1日を通して身体的に活発であるべきであり、 保護者が支援していくことが必要である。 ・ 6~17歳の就学児・青少年では、運動能力の向上や運動習慣の形成、そして生涯にわたる健康の基盤づくりとして身体活動が重要である。未就学児と同様に保護者の支援が必要であり、骨強 化や筋力増強のために週3日以上の運動が推奨 される。就学児・青少年では慢性疾患の基盤となる肥満やインスリン抵抗性、脂質や血圧の異常が進行する可能性があり、運動習慣はそれらの 危険因子を減らし、将来の慢性疾患の発症抑制につながる。 ・ 成人 では、中等度 の身体活動であれば150~ 300分 /週、高強度であれば75~150分 /週 の有酸素運動が推奨され、さらに週2日以上の筋力トレーニングを加えるべきである。 ・ 高齢者では身体機能の維持を目的に、有酸素運動や筋力トレーニングに加えて転倒予防のための バランストレーニングを含む複数の身体活動を行う。また、身体活動の種類は個々の状態に合わせて設定する必要がある。 ・ 妊娠中および産後の女性は、周産期合併症の予防を目的に少なくとも150分 /週の中等度の有酸素運動を行うべきである。 ・ 慢性疾患や障害のある患者も可能な限り成人の ガイドラインに沿って身体活動を行うことが望ましく、有酸素運動と筋力トレーニングの両方を行うことが推奨される。ただし、個々の病状や身体能力を踏まえ、専門家の指導の下に運動の質や量を設定する。 ガイドラインでは、移動を多くし座位を少なくすることがすべての人に有益であること、身体活 動が少ない人ほど身体活動によるベネフィットが 多く得られることも強調している。このことは我々日本人に対しても推奨されるものと言える。また、 医療専門家や行政に対してこのガイドラインを活用して身体活動による健康改善の取り組みを支援することを訴えている。 1. Lee IM, et al. Lancet. 2012;380(9838):219-229.
再発寛解型多発性硬化症患者における非血小板造血幹細胞移植と疾患修飾療法継続の疾患進行への影響。無作為化臨床試験。
再発寛解型多発性硬化症患者における非血小板造血幹細胞移植と疾患修飾療法継続の疾患進行への影響。無作為化臨床試験。
Effect of Nonmyeloablative Hematopoietic Stem Cell Transplantation vs Continued Disease-Modifying Therapy on Disease Progression in Patients With Relapsing-Remitting Multiple Sclerosis: A Randomized Clinical Trial JAMA 2019 Jan 15 ;321 (2):165 -174. 上記論文のアブストラクト日本語訳 ※ヒポクラ×マイナビ 論文検索(Bibgraph)による機械翻訳です。 【重要性】造血幹細胞移植(HSCT)は、再発型多発性硬化症(MS)における進行性障害を遅らせる、あるいは予防するために有用なアプローチとなりうる。 【目的】非ミエロ切除型HSCTと疾患修飾療法(DMT)の疾患進行への影響を比較する。 【デザイン、設定および参加者】2005年9月20日から2016年7月7日の間に、米国、欧州、南米の4施設で、再発寛解型MSで、前年度にDMTを受けている間に2回以上再発し、拡張障害状態スケール(EDSS:スコア範囲、0~10[10=最悪の神経学的障害])のスコアが2.0~6.0の患者計110名を無作為に割り付けた。最終フォローアップは2018年1月、データベースロックは2018年2月に行われた。 【介入】患者は、シクロホスファミド(200mg/kg)および抗胸腺細胞グロブリン(6mg/kg)と共に造血幹細胞移植を受けるか(n=55)、より有効性の高いDMTまたは前年中に服用したDMTと異なるクラスのDMTを受けるかに無作為に分けられた(n=55) 【主なアウトカムと測定】主要エンドポイントは疾患進行(少なくとも1年後にEDSSスコア上昇が1.0と定義)とし、EDSSスコア上昇は、1.0とした。 【結果】無作為化された110例(女性73例[66%],平均年齢36[SD,8.6]歳)のうち,103例が試験に残り,98例が1年後,23例が5年間毎年評価された(追跡期間中央値2年,平均値2.8年)。疾患の進行は造血幹細胞移植群で3例、DMT群で34例に認められた。進行までの期間の中央値は、HSCT群ではイベントが少なすぎたため算出できなかった。DMT群では24ヵ月(四分位範囲、18~48ヵ月)だった(ハザード比、0.07;95%CI、0.02~0.24;P < 0.001)。最初の1年間で、平均EDSSスコアは、造血幹細胞移植群で3.38から2.36に減少(改善)し、DMT群で3.31から3.98に増加(悪化)した(群間平均差、-1.7;95%CI、-2.03から-1.29;P < 0.001)。死亡例はなく、造血幹細胞移植を受けた患者には非造血グレード4の毒性(心筋梗塞、敗血症、その他の生命を脅かす障害または可能性のある事象など)は認められなかった。 【結論と関連性】再発型MS患者のこの予備研究では、DMTと比較して非血小板造血幹細胞移植は疾患進行までの時間を延長する結果となった。これらの知見を再現し、長期的な転帰と安全性を評価するためにさらなる研究が必要である。 【臨床試験登録】ClinicalTrials. gov Identifier:NCT00273364。 第一人者の医師による解説 長期での疾患活動性の再燃 今後の検証課題 吉良 潤一 九州大学大学院医学研究院神経内科学教授 MMJ.June 2019;15(3) 多発性硬化症(MS)の約90%は、再発寛解型で発症し、その後再発と関係なく徐々に障害が進行する2次進行型に移行する。残りの10%程度は、 最初から再発がなく緩徐に障害が増悪する1次進行型を呈する。再発寛解型 MSに対しては、疾患修飾薬の進歩がめざましい。これらは、再発寛解型の再発を減らし障害の進行を遅らせるが、課題としてnon-responderが一定の割合でどの薬剤でも存在すること、進行型にはほとんど効果がないことが挙げられる。最近、2次進行型にシポニモド、1次進行型にオクレリズマブ(抗 CD20抗体)が部分的に有効であることが報告され注目を集めた。しかし、既存の疾患修飾薬では疾患活動性の高いMSや進 行型 MSの治療効果が限定的である状況は続いている。そこで、骨髄非破壊的造血幹細胞移植(HSCT) が非対照試験で試みられ、MSに対する有用性が報告されている(1)。 本研究は、HSCTの治療効果を、既存の疾患修飾薬と無作為化臨床試験で比較した点が大きな特徴である。先行する1年間に2回以上の臨床再発または1回の臨床再発とそれとは異なる時期に造影病巣を認めた再発寛解型 MS患者110人を、 55人ずつHSCTまたは各種疾患修飾薬の継続に無作為に割り付けた。主要評価項目は、Expanded Disability Status Scale(EDSS)でみた1段階以上(EDSSが6を超える患者では0.5以上)の障害進行を示すまでの期間である。中央値2年(平均2.8 年)の観察期間で障害が進行した患者は、HSCT群で3人、疾患修飾薬群では34人だった。障害進行までの期間は、HSCT群が疾患修飾薬群に比べて有 意に長かった(P<0.001)。1年後の平均 EDSS はHSCT群では3.38から2.36に改善したが、疾患修飾薬群では3.31から3.98に悪化し、この差は有意だった(P<0.001)。副次的評価項目である1年間で再発を起こした患者の割合も、HSCT群 2%に対して疾患修飾薬群69%と有意に低かった (P<0.001)。死亡や有害事象(4度)の重大な副作用は両群ともみられなかった。 本研究はエントリーした患者数が少ないにもかかわらず、疾患活動性の高いMSに対してHSCTが 既存の疾患修飾薬に勝る治療効果を示すことを、無作為化試験により初めて明らかにした点で意義が大きい。しかし、1年後には疾患修飾薬からHSCT への変更が認められていたため長期の比較ができていない点、疾患修飾薬にアレムツズマブやオクレリズマブなどの強力な治療薬が含まれていない点、障害進行の評価では盲検性が保たれていたものの再発の評価は盲検でなかった点などが課題である。特に長期でみた場合に疾患活動性が再燃しないかという点の検証は今後に残されている。 1. Burt RK, et al. JAMA. 2015;313(3):275-284.
ペニシリンアレルギーの評価と管理。総説。
ペニシリンアレルギーの評価と管理。総説。
Evaluation and Management of Penicillin Allergy: A Review JAMA 2019 Jan 15 ;321 (2 ):188 -199 . 上記論文のアブストラクト日本語訳 ※ヒポクラ×マイナビ 論文検索(Bibgraph)による機械翻訳です。 【重要】β-ラクタム系抗生物質は、最も安全で最も有効な抗生物質の一つである。多くの患者がこれらの薬剤に対するアレルギーを報告し、その使用が制限されているため、抗菌薬耐性や有害事象のリスクを高める広域スペクトルの抗生物質が使用されている。 【観察】米国人口の約10%がβ-ラクタム薬ペニシリンに対するアレルギーを報告しており、高齢者や入院患者で報告率が高くなる。多くの患者がペニシリンに対してアレルギーがあると報告していますが、臨床的に重要なIgE介在型またはTリンパ球介在型のペニシリン過敏症はまれです(5%未満)。現在、IgE介在性ペニシリンアレルギーの割合は、非経口ペニシリンの使用が減少していること、およびアモキシシリン経口剤に対する重度のアナフィラキシー反応がまれであることから、減少している可能性があります。IgE介在型ペニシリン・アレルギーは時間の経過とともに減少し、10年後には80%の患者が耐性を獲得しています。ペニシリンとセファロスポリン系薬剤の交差反応が起こるのは約2%で、以前に報告された8%よりも少ない。患者の中には、ペニシリンに対するアレルギー反応を発症するリスクが低いことを示唆する病歴を持つ者もいます。低リスクの病歴には、胃腸症状などの孤立した非アレルギー症状を持つ患者、またはペニシリン・アレルギーの家族歴のみを持つ患者、発疹を伴わないそう痒症の症状、IgE介在反応を示唆する特徴を持たない遠隔(10年超)の未知の反応が含まれます。中等度リスクの既往歴には、蕁麻疹またはその他のそう痒性皮疹、IgE介在性反応の特徴を持つ反応が含まれます。高リスクの既往歴には、アナフィラキシー、ペニシリン皮膚テスト陽性、ペニシリン反応の再発、複数のβ-ラクタム系抗生物質に対する過敏症がある。ペニシリンに対するアレルギーが報告され、メチシリン耐性黄色ブドウ球菌やバンコマイシン耐性腸球菌のリスクを含む抗菌薬耐性リスクを高める広域抗菌薬の使用につながる場合、抗菌薬スチュワードシップの目標は損なわれてしまう。また、広域抗菌薬はクロストリジウム・ディフィシル(別名クロストリジウム・ディフィシル)感染症の発症リスクも増加させます。アモキシシリンの直接投与は、低リスクのアレルギー歴のある患者に適しています。中等度リスクの患者は、ペニシリン皮膚試験で評価することができます。この試験の陰性的中率は95%を超え、アモキシシリン試験と併用することで100%に近づきます。ペニシリンアレルギーの評価を行う臨床医は、利用可能なリソースからどのような方法がサポートされているかを確認する必要がある。 【結論と関連性】多くの患者がペニシリンに対してアレルギーがあると報告しているが、臨床的に重大な反応を示す患者は少ない。ペニシリンや他のβ-ラクタム系抗生物質を使用しないことを決定する前にペニシリンアレルギーを評価することは、抗菌薬スチュワードシップにとって重要な手段である。 第一人者の医師による解説 臨床現場でのアレルギーの存在確認が重要 宮下 修行 関西医科大学内科学第一講座呼吸器感染症・アレルギー科診療教授 MMJ.June 2019;15(3) ペニシリンアレルギーは有名な言葉で、この記載が診療記録にある患者に対してβ-ラクタム薬は禁忌であると解釈している人が多い。そのため、ここ数年ペニシリンアレルギーに関する研究が実施され、誤った考え方を是正するデータが蓄積されている。本論文は、これまでの研究結果をまとめ、 ペニシリンアレルギーを申告する患者への対応を報告したものである。 ペニシリンアレルギーと記載されている患者の90%以上は、ペニシリンに対する即時型の過敏反応を起こさないことが報告されている。その大きな理由の1つとして、子どものウイルス感染症に不要な抗菌薬が投与された場合、ウイルス性発疹をアレルギーと誤認する、いわゆる誤診が挙げられる。 親の申告で「ペニシリンアレルギーの既往がある」 とされた小児を調べた研究結果が報告されている(1)。 小児救急を受診した小児597人(4~18歳)のうち発疹、嘔吐、下痢などの低リスクのペニシリンアレルギー症状のある100人を対象に、①皮膚試験 ②微量のペニシリンを注射する皮内反応試験③厳重な監視下でペニシリンを服用させる経口負荷試験、3種類の検査を実施。結果は、すべての小児で ペニシリンアレルギー反応は認められなかった。 同時期に成人でも同様の研究結果が報告されている。ただし全症例が誤認ではなく、当初は過敏性があったものの後に消失した症例も含まれる(2)。 アレルギー抗体(IgE)による急性ペニシリン反応が認められた患者でも、時間とともに抗体は減少し消失する。したがって、10年後にはほとんどの場合、皮膚試験は陰性になる。ペニシリン皮膚試験が陰性化した患者は、将来ペニシリンや他のβ-ラクタム薬に曝露しても、再びアレルギーを起こすリスクはごくわずかである。 実臨床では、ペニシリンアレルギーの記載のある患者に対して、β-ラクタム薬は使用せず、より広域スペクトルの抗菌薬が選択される場合が多い。 ペニシリンアレルギーという記載が公衆衛生面に及ぼす影響を、メチシリン耐性黄色ブドウ球菌 (MRSA)感染とClostridium difficile(CD)感染のリスクという観点から評価した症例対照研究では、 ペニシリンアレルギー記載群で両感染症の発症率が高かったと報告されている(3)。 以上のようにペニシリンアレルギー記載例は、より広域スペクトルの抗菌薬に不必要に曝露することがあり、耐性菌のリスクを増大させ、医療費を増加させる。ペニシリンアレルギー申告者のほとんどは、ペニシリンに対して忍容性があるため、臨床現場ではアレルギーの存在を確認することが重要であると結論付けている。 1. Vyles D, et al. Pediatrics. 2017;140(2). pii:e20170471. 2. Trubiano JA, et al. JAMA. 2017;318(1):82-83. 3. Blumenthal KG, et al. BMJ. 2018;361:k2400.
経口抗凝固薬およびプロトンポンプ阻害薬併用療法と上部消化管出血による入院の関連性。
経口抗凝固薬およびプロトンポンプ阻害薬併用療法と上部消化管出血による入院の関連性。
Association of Oral Anticoagulants and Proton Pump Inhibitor Cotherapy With Hospitalization for Upper Gastrointestinal Tract Bleeding JAMA 2018 Dec 4 ;320 (21 ):2221 -2230 . 上記論文のアブストラクト日本語訳 ※ヒポクラ×マイナビ 論文検索(Bibgraph)による機械翻訳です。 【重要性】抗凝固薬の選択とプロトンポンプ阻害薬(PPI)併用療法は、経口抗凝固薬治療の頻度が高く重篤な合併症となりうる上部消化管出血のリスクに影響する可能性がある。 【目的】個々の抗凝固薬使用患者における上部消化管出血による入院の発生率をPPIコセラピー有無で比較し、基礎的な消化管出血リスクによる変動を明らかにする。 【デザイン、設定および参加者】2011年1月1日から2015年9月30日までのメディケア受益者における後ろ向きコホート調査。 【曝露】アピキサバン、ダビガトラン、リバーロキサバン、ワルファリンにPPIコセラピーを併用または併用しない。 【主要アウトカムと測定】上部消化管出血による入院:抗凝固剤治療1万人年当たりの調整済み発生率とリスク差(RD)、発生率比(IRR)。 【結果】コホートに含まれる経口抗凝固薬治療の新規エピソードは1,643 123例(平均[SD]年齢76.4[2.4]歳,追跡期間651 427人年[56.1%]は女性,適応は心房細動で870 330人年[74.9%])であった。)PPI共治療を行わない754 389治療人年の間、上部消化管出血による入院の調整後発生率(n = 7119)は、1万人年当たり115人(95%CI、112-118)であった。リバーロキサバン(n=1278)の発生率は1万人年当たり144人(95%CI、136-152)であり、アピキサバン(n=279、1万人年当たり73人、IRR、1.97[95%CI、1.73-2.25]、RD、70.9 [95% CI, 59.1-82.7] )、ダビガトラン(n = 629; 1万人年あたり120人; IRR, 1.19 [95% CI, 1.08-1.32]; RD, 23.4 [95% CI, 10.6-36.2] )、およびワルファリン(n = 4933; 1万人年あたり113人; IRR, 1.27 [95% CI, 1.19-1.35]; RD, 30.4 [95% CI, 20.3-40.6] )であった。アピキサバンの発生率は,ダビガトラン(IRR,0.61 [95% CI,0.52-0.70]; RD,-47.5 [95% CI,-60.6~34.3] )およびワルファリン(IRR,0.64 [95% CI,0.57-0.73]; RD,-40.5 [95% CI,-50.0~31.0] )のそれよりも著しく低率であった。PPIコセラピーを用いた抗凝固療法(264 447人年;1万人年あたり76人)をPPIコセラピーを用いない治療と比較すると,上部消化管出血による入院(n=2245)のリスクは,全体で低かった(IRR,0.66 [95% CI, 0.62-0.69] )、アピキサバン(IRR, 0.66 [95% CI, 0.52-0.85]; RD, -24 [95% CI, -38 to -11])、ダビガトラン(IRR, 0.49 [95% CI, 0.41-0.59]; RD, -61.1 [95% CI, -74.8 to -47.4]),リバーロキサバン(IRR,0.75 [95% CI,0.68-0.84]; RD,-35.5 [95% CI,-48.6 to -22.4]),およびワルファリン(IRR,0.65 [95% CI,0.62-0.69]; RD,-40.3 [95% CI,-44.5 to -34.2])であることがわかった。 【結論と関連性】経口抗凝固薬治療を開始した患者のうち,上部消化管出血による入院の発生率は,リバーロキサバンを処方された患者が最も高く,アピキサバンを処方された患者が最も低いことが示された。また、各抗凝固薬において、上部消化管出血による入院の発生率は、PPIコセラピーを受けている患者さんで低くなっていました。これらの知見は,抗凝固薬を選択する際のリスクとベネフィットの評価に役立つと考えられる。 第一人者の医師による解説 PPI併用は有効 経口抗凝固薬使用の指標になる成果 川邊 隆夫 かわべ内科クリニック院長 MMJ.June 2019;15(3) 直接経口抗凝固薬(DOAC)は出血のリスクを高めるが、薬剤の種類による差異は十分には検討されていない。本研究では、特に上部消化管出血について、3種類のDOACとワルファリンについて、リスクを比較検討し、プロトンポンプ阻害薬(PPI)の 併用の有用性について検討している。 この研究は、メディケアの膨大なデータ(2011 年1月1日~15年9月30日)を用いた、後ろ向きコホート研究である。この期間1,643,123人の 患者が、のべ1,713,183回、新たに経口抗凝固薬の投与を開始しており、平均年齢は76.4歳、女性の割合は56.1%(人・年で計算)、原疾患は心房細動が74.9%(人・年で計算)であった。 PPI非併用754,389人・年で、上部消化管出血 による入院は7,119件、10,000人・年あたりの 調整発生率※は115(95% CI, 112~118)であった。経口抗凝固薬の薬剤別では、リバーロキサバンでは144(/10,000人・年)、アピキサバン 73、 ダビガトラン 120、ワルファリン 113であった。 リバーロキサバンは、アピキサバン(発生率比 1.97、 リスク差 70.9)、ダビガトラン(1.19、23.4)、ワ ルファリン(1.27、30.4)より有意に高かった。アピキサバンの入院発生率は、ダビガトラン(発生率比 0.61、リスク差 -47.5)、ワルファリン(0.64、 -40.5)よりも有意に低かった。 経口抗凝固薬 とPPIを 併用した264,447人・ 年では、上部消化管出血による入院が2,245件、 76/10,000人・年で、PPI非併用より有意に低頻度(発生率比 0.66、リスク差 -39.5)であった。 薬剤別の検討でも、上部消化管出血による入院は PPI併用例で低頻度であった。それぞれの発生率比、 リスク差は、リバーロキサバン(0.75、-35.5)、 アピキサバン(0.66、-24)、ダビガトラン(0.49、 -61.1)、ワルファリン(0.65、-39.3)であった。 また、上部消化管出血のリスクスコアで層別化した検討でも、各階層でPPI併用例の入院発生率は低値であった(リスク最小の層のみ有意差を認めていない)。 本研究で、アピキサバンが最も安全な薬剤であることが示された(これは、これまでの研究に一致する結果である)。また、PPIの併用は、経口抗凝固薬の種類によらず、リスクスコアの高低によらず、 上部消化管出血を減少できることも示された。 しかし、本研究は、米国の高齢者向け保険であるメディケアのデータを解析したコホート研究である。対象の90%が白人であり、アジア人はほとんど含まれていない。この研究の結論を日本でそのまま受け入れてよいかどうかは、もう少し検討する必要があるが、経口抗凝固薬を使用する際の指標となるであろうと思われる。 ※:調整発生率はポアソン回帰から算出されており、件数÷人年とは少し異なる値となっている点に留意
乳がんの治療法。レビュー
乳がんの治療法。レビュー
Breast Cancer Treatment: A Review JAMA 2019 Jan 22 ;321 (3 ):288 -300 . 上記論文のアブストラクト日本語訳 ※ヒポクラ×マイナビ 論文検索(Bibgraph)による機械翻訳です。 【重要】乳癌は、生涯を通じて米国女性の12%に診断され、2017年には米国で25万人以上の乳癌の新規症例が診断された。このレビューでは、乳がんの局所および全身療法に関する現在のアプローチと進化する戦略に焦点を当てる。 【観察】乳がんは、エストロゲンまたはプロゲステロン受容体とヒト上皮成長因子2(ERBB2;旧HER2)の分子マーカーの有無に基づいて、ホルモン受容体陽性/ERBB2陰性(70%の患者)、ERBB2陽性(15~20%)、トリプルネガティブ(3つの標準分子マーカーすべてを欠く腫瘍;15%)に大きく分類される。乳がんの90%以上は、診断時に転移を認めません。転移を伴わない場合、治療目標は腫瘍の消失と再発の防止です。トリプルネガティブ乳がんは、他の2つのサブタイプよりも再発しやすく、I期のトリプルネガティブ腫瘍の5年乳がん特異的生存率は85%で、ホルモン受容体陽性およびERBB2陽性では94%~99%となっています。ホルモン受容体陽性の患者さんには内分泌療法を行い、少数派ですが化学療法も行います。ERBB2陽性の患者さんにはERBB2標的抗体または低分子阻害剤と化学療法の併用、トリプルネガティブの患者さんには化学療法単独を行います。非転移性乳癌の局所療法は外科的切除で、腫瘍摘出術を行った場合は術後放射線療法を考慮する。最近では、術前に全身療法を行うことも増えている。術前治療の効果に応じて術後治療を調整することが検討されています。転移性乳がんは、延命と症状緩和を目標に、サブタイプに応じた治療が行われる。転移性トリプルネガティブ乳癌の全生存期間中央値は約1年、他の2つのサブタイプは約5年。 【結論と関連性】乳癌はエストロゲンまたはプロゲステロン受容体の発現と ERBB2 遺伝子増幅により分類される3つの主要な腫瘍サブタイプからなる。この3つのサブタイプは、それぞれ異なるリスクプロファイルと治療戦略を有している。各患者の最適な治療法は、腫瘍のサブタイプ、解剖学的な癌のステージ、患者の嗜好によって異なる。 第一人者の医師による解説 乳がん治療の変遷から学ぶこれからのゲノムによる個別化治療 山内 英子 聖路加国際病院ブレストセンターセンター長 MMJ.June 2019;15(3) 乳がんは世界中で女性が罹患するがんの1位となっている。米国でも生涯において全女性の12% が乳がんに罹患するといわれており、JAMAに乳がん治療のレビューが掲載された。2013年1月 ~18年11月に報告された文献をレビューしたもので、薬物療法に関するレビューのみならず、臨床所見や画像から、さらには人種による違いまでにも言及しており、最新の乳がん診療を知るうえで、 非常にコンパクトにまとめられている。 著者らも「Limitations」で述べているように、米国での現状に即したものになっており、日本の実情とは異なる点もあることは当然加味しての解釈が必要である。その点から、マンモグラムが普及している米国では、半数以上が検診によるマンモグラムの異常が受診契機であり、腫瘤触知は3分の1 である。 乳がんはホルモン受容体の発現の有無、HER2受容体の発現の有無でタイプが分けられ、それによって治療方針が決まってくることは世界共通である。 早くからホルモン受容体の発現の有無によりホルモン療法を加えるか否かという個別化治療が行われてきている。1970年代から化学療法も導入され、時代変遷を経て変化してきている。その後の HER2遺伝子発現をターゲットとした治療法の開発も目をみはるものがあった。また、手術や放射線療法と組み合わせたいわゆる集学的治療も早くから導入されている。さらには、21-Gene Assayのような化学療法の上乗せ効果をみるためのゲノムによるスコアを利用する試みも多くの臨床試験が行われ、実臨床でも用いられている。 このレビューにて乳がん治療の変遷を振り返りながら、ゲノム医療の時代の今、感じることは、乳がんこそが本当に早くからバイオマーカーのタイプに基づく治療法を確立してきた領域と言えよう。まさに、がんが原発の場所による区別ではなく、さまざまな発現遺伝子によって治療法を選択する時代の中で、これからのがん診療におけるゲノムによる個別化医療を学ぶためにも、乳がん治療の変遷を知っておくべきであり、その意味からも JAMAのレビューとして取り上げられている意義深いと思われる。
初回疾患修飾療法とその後の二次進行性多発性硬化症への転化との関連性。
初回疾患修飾療法とその後の二次進行性多発性硬化症への転化との関連性。
Association of Initial Disease-Modifying Therapy With Later Conversion to Secondary Progressive Multiple Sclerosis JAMA 2019 Jan 15 ;321 (2):175 -187. 上記論文のアブストラクト日本語訳 ※ヒポクラ×マイナビ 論文検索(Bibgraph)による機械翻訳です。 【重要】発症から20年以内に、未治療の再発性多発性硬化症(MS)患者の80%が二次進行性MSと呼ばれる不可逆的な障害発生の段階へと移行する。疾患修飾治療(DMT)とこの転換との関連はほとんど研究されておらず、有効な定義を用いたこともない。 【目的】有効な定義で診断された二次進行性MSへの転換リスクとDMTの使用、種類、時期との関連を明らかにすること。 【デザイン、設定および参加者】1988-2012年にDMT(または臨床モニタリング)を開始し、最低4年間のフォローアップを行った再発寛解型MS患者を対象に、21か国68の神経センターからの前向きデータによるコホート研究。 【曝露】インターフェロンβ、グラチラマー酢酸、フィンゴリモド、ナタリズマブ、アレムツズマブのDMT使用、タイプ、タイミングを評価した。傾向スコアマッチング後、1555例が組み入れられた(最終フォローアップ、2017年2月14日)。 【主要アウトカムおよび測定】客観的に定義された二次進行性MSへの転換。 【結果】1555例中、1123例が女性だった(平均基準年齢、35歳[SD、10])。グラチラマー酢酸塩またはインターフェロンβによる初期治療を受けた患者は、マッチさせた未治療の患者よりも二次進行性MSへの転換のハザードが低かった(HR、0.71;95%CI、0.61-0.81;P < .001;5年間の絶対リスク、12%[407例中49例] vs 27%[213例中58例];フォローアップ中央値、7.6年[IQR, 5.8-9.6]) フィンチモリモドは(HR、0.37;95%CI、0.22-0.62;P < .001; 5年間の絶対リスク)、同様に無治療の患者は、グラチラマー酢酸塩による初期治療を受けた患者と同じように、一次進行を示すMSへの転換のハザードが低いことが示された。001;5年絶対リスク7%[85例中6例]対32%[174例中56例];追跡期間中央値4.5年[IQR、4.3-5.1]);ナタリズマブ(HR、0.61;95%CI、0.43-0.86;P = .005;5年絶対リスク19%[82例中16例]対 38%[164 例];追跡期間中央値4.5年;IQR、4.3-5.1])。9年[IQR, 4.4-5.8]);およびアレムツズマブ(HR, 0.52; 95% CI, 0.32-0.85; P = .009; 5年絶対リスク, 10%[4 of 44] vs 25%[23 of 92];フォローアップ中央値 7.4 年 [IQR, 6.0-8.6])。フィンゴリモド,アレムツズマブ,またはナタリズマブによる初期治療は,グラチラマー酢酸塩またはインターフェロンβによる初期治療よりも転化のリスクが低かった(HR,0.66;95% CI,0.44-0.99;P = .046);5 年絶対リスク,7%[235 例中 16 例]対 12%[380 例中 46 例];フォローアップ中央値 5.8 年[IQR,4.7-8.0])。グラチラマー酢酸塩またはインターフェロンβを発症から5年以内に開始した場合とそれ以降に開始した場合では、転換の確率が低かった(HR、0.77;95%CI、0.61-0.98;P = .03;5年絶対リスク、3%[120例中4例]対6%[38例中2例];追跡期間中央値、11.4年[IQR、18.1年])。グラチラマー酢酸塩またはインターフェロンβを5年以内にフィンゴリモド、アレムツズマブ、またはナタリズマブにエスカレーションした場合とそれ以降の場合では、HRは0.76(95%CI、0.66-0.88;P < 0.001;5年絶対リスク、8%[307例中25例] vs 14%[331例中46例]、フォローアップ中央値、5.3年[IQR]、4.6-6. 【結論と関連性】再発寛解型MS患者において、フィンゴリモド、アレムツズマブ、ナタリズマブの初期治療は、グラチラマー酢酸塩またはインターフェロンβの初期治療と比較して二次進行型MSへの転換リスクが低いことと関連していた。これらの知見は、これらの治療法のリスクと合わせて考慮することで、DMTの選択に関する意思決定に役立つと思われます。 第一人者の医師による解説 病態修飾薬間での2次性進行型多発性硬化症への進展リスクの比較 久冨木原 健二/中原 仁(教授) 慶應義塾大学医学部神経内科 MMJ.August 2019;15(4) 多発性硬化症(multiple sclerosis;MS)は 若年女性に多い疾患であり、多くが再発寛解型 MS (relapsing-remitting MS;RRMS)で 発症する。 自然経過では発症から20年の間に8割の症例が、不可逆的に障害が進行する2次性進行型 MS (secondary progressive MS;SPMS)に 進展する。現在 SPMSに対してエビデンスのある治療はなく、いかにその進展を阻止するかが治療の課題となっていた。本研究では、このRRMSからSPMSへ の進展リスクと病態修飾薬(disease modifying drug;DMD)の選択との関連について、 インターフェロンβ-1a/1b(IFNβ)、グラチラマー 酢酸塩(GA)、フィンゴリモド(FTY)、ナタリズマブ(NTZ)、アレムツズマブ(日本未承認)の計6種 類のDMDで検討した。 本研究は主にMSBase(1)のデータを利用した前向きコホートのリアルワールドスタディーである。 1988~2012年の期間で21カ国68施設を受診したRRMS患者に対して傾向スコアマッチングを行い、適合した1,555人を対象とした。 その結果、① DMD投与群と無治療群で5年間で のSPMS進展リスクを比較し、IFNβ群もしくは GA群(IFN/GA群)におけるハザード比(HR)は 0.71、FTY群では 0.37、NTZ群では0.61、アレ ムツズマブ群では0.52であり、すべてのDMDにおいて無治療群よりもSPMS進展リスクは有意に低かった。②第1選択薬がFTY、NTZまたはアレムツズマブの群とIFN/GA群を比較したHRは0.66 で、前者の方がSPMS進展リスクは有意に低かった。 ③ IFN/GA群の中で、DMD投与開始がRRMS発症 5年以内の群と5年以降の群を比較したHRは0.77 で、早期開始群の方がSPMS進展リスクは有意に低かった。④ IFNβもしくはGAからFTY、NTZ、ア レムツズマブへ切り替える時期による解析では、 発症から5年以内の群の方が5年以降の群よりも SPMS進展リスクは有意に低かった(HR, 0.76)。 本論文中にも記載があるように、FTY、NTZ、アレムツズマブはIFNβおよびGAより再発を抑えることは知られていた。しかしながら、RRMSの再 発を抑制しても長期予後に影響しないという報告以来(2)、DMDの選択基準は安全性が主となり、効果は劣るが安全性の高いIFNβやGAが第1選択として用いられ、効果は勝るが安全性の確立していないFTY、NTZ、アレムツズマブなどが第2選択以降で用いられるescalation therapyが主流であった。そのような背景の中で今回のリアルワールドスタディーでそれぞれのDMDでの長期予後が差別化され、治療導入時から効果の強いDMDを用いる induction therapyの有益性を立証する形となった。 1. IIngram G, et al. Mult Scler. 2010;16(4):472-479. 2. Haider L, et al. Brain. 2011;134(Pt 7):1914-1924.
大腿部脂肪分布と腹部脂肪分布に関連する遺伝子変異と2型糖尿病、冠動脈疾患および心血管危険因子との関連性。
大腿部脂肪分布と腹部脂肪分布に関連する遺伝子変異と2型糖尿病、冠動脈疾患および心血管危険因子との関連性。
Association of Genetic Variants Related to Gluteofemoral vs Abdominal Fat Distribution With Type 2 Diabetes, Coronary Disease, and Cardiovascular Risk Factors JAMA 2018 Dec 25 ;320 (24):2553 -2563. 上記論文のアブストラクト日本語訳 ※ヒポクラ×マイナビ 論文検索(Bibgraph)による機械翻訳です。 【重要性】体脂肪分布は、通常、ウエスト・ヒップ比(WHR)を用いて測定され、肥満度(BMI)とは無関係に、心代謝性疾患に重要な寄与をするものである。)低い臀部(股関節)を介して,あるいは高い腹部(腰部)脂肪分布を介してWHRを増加させるメカニズムが,心代謝リスクに影響するかどうかは不明である。 【目的】低い臀部または高い腹部脂肪分布を介して特に高いWHRと関連する遺伝子変異を特定し,心代謝リスクとの関連を推定する。 デザイン,設定,参加者]WHRに関するゲノム幅関連研究(GWAS)は,英国バイオバンクコホートのデータと過去のGWASからの要約統計(データ収集:2006~2018)を組み合わせたものである。股関節またはウエスト周囲径との特異的な関連を示すWHR関連遺伝子変異を用いて,低臀部大腿部経由または高腹部脂肪分布経由の高WHRに対する特異的な多遺伝子スコアを導出した。3つの人口ベースコホート、ケースコホート研究、6つのGWASの要約統計で、多遺伝子スコアとアウトカムとの関連を推定した(データ収集:1991~2018)。 【曝露】240万以上の共通遺伝バリアント(GWAS)、高いWHRに対する多因子スコア(フォローアップ解析)。 【主要評および測定法】BMI調整WHRと未調整WHR(GWAS);二重エネルギーX線吸収法で測定したコンパートメント脂肪量、収縮期・拡張期血圧、低密度リポタンパク質コレステロール、トリグリセリド、空腹時グルコース、空腹時インスリン、2型糖尿病、冠疾患リスク(フォローアップ分析)。 【結果】ヨーロッパ系祖先を持つUK Biobank参加者452名302名の平均(SD)年齢は57(8)歳、平均(SD)WHRは0.87(0.09)であった。)ゲノムワイド解析では、202の独立した遺伝子変異が、より高いBMI調整WHR(n = 660 648)および未調整WHR(n = 663 598)と関連していた。二重エネルギーX線吸収測定法解析(n = 18 330)では、高いWHRに対する股関節および腰部特異的多因子スコアは、それぞれ低い臀部脂肪および高い腹部脂肪と特異的に関連していた。追跡解析(n = 636 607)では、両方の多遺伝子スコアが、より高い血圧およびトリグリセリド値、ならびにより高い糖尿病リスクと関連していた(ウエスト特異的スコア:オッズ比[OR]、1.57[95%CI、1.34-1.83]、参加1000年当たりの絶対リスク増加[ARI]、4.4[95%CI、2.7-6.5]、P < .001;hip-specific score;ウエスト特異的スコア:オッズ比[OR]、1.57[95%CI]、3.5[95%CI、3.5]、P < 0.001)。OR, 2.54 [95% CI, 2.17-2.96], ARI, 12.0 [95% CI, 9.1-15.3], P < .001) および冠動脈疾患(腰部特異的スコア:OR, 1.60 [95% CI, 1.39-1.84], ARI, 2.3 [95% CI, 1.5-3.3], P < .001; hip-specific score: 【結論と関連性】WHRの算出の基礎となる臀部および腹部脂肪の分布には、異なる遺伝的機序が関連している可能性がある。これらの知見は、糖尿病や冠動脈疾患のリスク評価や治療を改善する可能性がある。 第一人者の医師による解説 ウエスト・ヒップ比の計測に臨床的意義 細江 隼(特任研究員)1)/門脇 孝(特任教授)1,2) 1) 東京大学大学院医学系研究科糖尿病・代謝内科, 2) 東京大学大学院医学系研究科糖尿病・生活習慣病予防講座 MMJ.August 2019;15(4) 体脂肪分布は、通常ウエスト・ヒップ比(WHR)を用いて評価され、WHRの構成要素は腹部と臀大腿部の脂肪分布である。近年、体格指数(BMI)補正 WHRと関連する遺伝子領域に注目して重み付けしたリスクアレルの総和を用いて算出される多因子遺伝リスクスコア(polygenic risk score;PRS) について、2型糖尿病および冠動脈疾患と関連する ことが報告された(1)。このような研究から、WHRは BMIとは無関係に心血管代謝疾患と関連し、腹部脂肪蓄積が同疾患のリスクであることが示唆されていた。 本論文では、腹部脂肪増加を介してWHR上昇と関連する遺伝因子から構成されるPRSに加えて、 臀大腿部脂肪の減少を介してWHR上昇と関連する遺伝因子から構成されるPRSも構築し、合計63万 人以上の参加者について各 PRSと心血管代謝疾患 のリスクの関連を検討した。 ゲノムワイド関連解析のサマリーデータセットとUK Biobankの個別横断データを対象に解析を行ったところ、各 PRS について、血圧高値および中性脂肪高値、2型糖尿 病および冠動脈疾患のリスク上昇との関連を認めた。臀大腿部脂肪の減少を介したWHR上昇に関するPRSがこれらの心血管代謝疾患と関連したことについては、臀大腿部脂肪・皮下脂肪の蓄積能が低いと、肝臓や骨格筋などの異所性脂肪蓄積をきたし、心血管代謝疾患と関連する可能性は考えられる。 このような病態は、より重症の臨床像を示す脂肪 萎縮症の疾患メカニズムとも共通点を有することが示唆される。 これらの結果は、本論文の著者らのグループが以前ゲノム関連解析の統合解析を行い、 末梢脂肪組織における脂肪蓄積能の低下とインスリン抵抗性に伴う心血管代謝疾患との関連性が示唆されていたこととも矛盾しない(2)。 本研究の限界(limitation)として、観察研究のため因果関係を証明することはできないことが 挙げられる。また、本研究は 欧州系(European ancestry)の人種を対象としているが、今後欧州系以外の人種を対象とした解析を行うことも重要と考えられる。本研究で得られた知見から、心血管代謝疾患のリスクを正確に評価するために、腹部脂肪に加えて臀大腿部脂肪の蓄積についても測定することの重要性が考えられた。臨床現場においては、ウエスト・ヒップの計測を行うことの臨床的意義が示唆される。 1. Emdin CA, et al. JAMA. 2017;317(6):626-634. 2. Lotta LA et al. Nat. Genet. 2017;49(1):17-26.
二次性副甲状腺機能亢進症がなく維持血液透析を受けている患者におけるアルファカルシドール経口投与の臨床転帰への影響。J-DAVID Randomized Clinical Trial(J-ダビッド無作為化臨床試験)
二次性副甲状腺機能亢進症がなく維持血液透析を受けている患者におけるアルファカルシドール経口投与の臨床転帰への影響。J-DAVID Randomized Clinical Trial(J-ダビッド無作為化臨床試験)
Effect of Oral Alfacalcidol on Clinical Outcomes in Patients Without Secondary Hyperparathyroidism Receiving Maintenance Hemodialysis: The J-DAVID Randomized Clinical Trial JAMA 2018 Dec 11 ;320 (22):2325 -2334. 上記論文のアブストラクト日本語訳 ※ヒポクラ×マイナビ 論文検索(Bibgraph)による機械翻訳です。 【重要】慢性腎臓病患者では、ビタミンDの活性化が損なわれており、心血管リスクが上昇している。血液透析を受けている患者を対象とした観察研究では、活性型ビタミンDステロールの使用は、副甲状腺ホルモン値にかかわらず、全死亡のリスク低下と関連していた。 【目的】ビタミンD受容体活性化剤が、血液透析を受けている二次性副甲状腺機能亢進症のない患者の心血管イベントおよび死亡率を低下させるかどうかを明らかにする。 【デザイン、設定および参加者】日本の透析施設207施設で患者1289人の無作為化、オープンラベル、エンドポイント盲検多施設試験とした。血清インタクト副甲状腺ホルモン値が180 pg/mL以下で維持血液透析を受けている患者976名が対象となった。最初の参加者は2008年8月18日に、最後の参加者は2011年1月26日に登録された。最終追跡日は2015年4月4日であった。 【介入】アルファカルシドール1日0.5μg経口投与(介入群;n=495) vs ビタミンD受容体活性化剤を用いない治療(対照群;n=481)。 【主要評価項目】主要評価項目は、心筋梗塞、うっ血性心不全による入院、脳卒中、大動脈解離・破裂、虚血による下肢切断、心臓突然死などの致死性および非致死性の心血管イベント、冠動脈再灌流、およびフォローアップ48ヶ月間の下肢動脈再灌流の複合指標であった。副次的アウトカムは全死亡であった。 【結果】108の透析施設から無作為化された976例のうち,intention-to-treat解析に含まれたのは964例(年齢中央値65歳,女性386例[40.0%))で,944例(97.9%)が試験を完遂した。追跡期間中(中央値,4.0年),主要複合転帰である心血管イベントは,介入群では488例中103例(21.1%),対照群では476例中85例(17.9%)で発生した(絶対差,3.25%[95% CI,-1.75% ~ 8.24%]; ハザード比,1.25[95% CI,0.94-1.67]; P = 0.13 ).副次的アウトカムである全死因死亡率には、両群間で有意差は認められなかった(それぞれ18.2% vs 16.8%;ハザード比、1.12[95%CI、0.83-1.52];P = 0.46)。介入群の参加者488人のうち,199人(40.8%)が心血管系に分類される重篤な有害事象を経験し,64人(13.1%)が感染症に分類される有害事象を経験し,22人(4.5%)が悪性腫瘍関連の重篤な有害事象を経験した.対照群476名のうち、191名(40.1%)が心血管関連の重篤な有害事象を、63名(13.2%)が感染症関連の有害事象を、21名(4.4%)が悪性腫瘍関連の有害事象を経験しました。 【結論と関連性】維持血液透析を受けている二次性副甲状腺機能亢進症の患者において、通常ケアと比較してアルファカルシドールを内服しても選択心血管イベントの複合指標のリスクは低下しませんでした。これらの知見は、これらのような患者に対するビタミンD受容体活性化剤の使用を支持しない。 【臨床試験登録】UMIN-CTR Identifier:UMIN000001194. 第一人者の医師による解説 日本の診療ガイドラインの妥当性を支持する有力な根拠 竹内 靖博 虎の門病院内分泌センターセンター長 MMJ.August 2019;15(4) 日本では維持透析患者に対して、活性型ビタミン D(VD)のアルファカルシドールが広く投与されている。腎不全では内因性のVD作用が低下しており、これまで多くの観察研究で、維持透析患者への活性型 VD投与は心血管障害や死亡の減少と関連することが報告されている(1),(2)。 しかし、ランダム化比較試験では有効性が実証されておらず、本研究では、アルファカルシドールにより4年間の心血管イベント発症リスクが20%低下する、という仮説が検証された。活性型 VDを投与しない対照群を設定する倫理的な根拠として、組み入れ基準に血中 intact PTH(iPTH)180pg/mL以下という条件が設定された。結果、仮説は実証されず、フルセット解析でも、プロトコール遵守群に限定した解析でも、アルファカルシドールによる心血管イベント発症抑制は認められなかった。 腎機能低下に伴うリンの排泄不全やVDの活性化 障害による2次性副甲状腺機能亢進症は、骨病変や心血管障害の原因として重要視されている。慢性腎 臓病に伴う骨・ミネラル代謝異常(CKD-MBD)という包括的な概念が提唱されており、日本透析医学会から診療ガイドラインが公表されている。CKDMBDでは2次性副甲状腺機能亢進症の管理は重要な課題であり、日本ではiPTHの血中濃度で60以 上240pg/mL以下に調節することが推奨されている。また、生命予後の点からは180pg/mL未満 が望ましいとされている。 本研究はiPTH 180pg/mL以下の維持透析患者で実施されており、その結果は、維持透析患者に対 する活性型 VDの効果は、PTH分泌亢進を認める患者に限定されることを示唆している。これまで の観察研究からiPTHと生命予後にはゆるやかなJ カーブ現象が指摘されており、iPTHを大きく低下させることは必ずしも望ましくないことも知られている。本研究のアルファカルシドール群におけるiPTH低下効果は一過性ではあるが、開始3カ月 後の平均値は50pg/mLをわずかに下回っている。一方、血清リン値には両群間でほぼ差はない。統計学的な有意差はないが、アルファカルシドール群 でイベント発生率が高い傾向にあることとPTH低 下との関連が懸念される。 本研究の結果は、日本のCKD-MBD診療ガイド ラインにおけるiPTHを60~240pg/mLに管理するという指針の妥当性を支持する有力な根拠となると考えられる。一方、維持透析患者における、 PTH分泌抑制以外の活性型 VDの効果を明らかにするには至らなかった。今後は、どのような維持透 析患者に対して活性型 VDを推奨するべきか、PTH 以外の指標を含めて明らかにしていくことが望まれる。 1. Kovesdy CP, et al. Kidney Int. 2008;73(12):1355-1363. 2. Shoji T, et al. ¬ Ther Apher Dial. 2015;19(3):235-244.
米国心臓病学会/米国心臓協会および欧州心臓病学会のガイドラインを支持するエビデンスレベル、2008-2018年。
米国心臓病学会/米国心臓協会および欧州心臓病学会のガイドラインを支持するエビデンスレベル、2008-2018年。
Levels of Evidence Supporting American College of Cardiology/American Heart Association and European Society of Cardiology Guidelines, 2008-2018 JAMA 2019 Mar 19 ;321 (11):1069 -1080. 上記論文のアブストラクト日本語訳 ※ヒポクラ×マイナビ 論文検索(Bibgraph)による機械翻訳です。 【重要性】臨床上の意思決定は,臨床転帰を評価する複数の無作為化対照試験(RCT)から得られたエビデンスに基づくことが理想的であるが,歴史的に,この種のエビデンスに完全に基づく臨床ガイドラインの推奨はほとんどない。 【目的】現在の主要循環器学会ガイドラインの推奨を支えるクラスと証拠レベル(LOE),およびLOEの経時変化を明らかにすることである。 【データ入手元】心臓血管学会のウェブサイトで確認された現在の米国心臓病学会/米国心臓協会(ACC/AHA)および欧州心臓病学会(ESC)の臨床ガイドライン文書(2008~2018)、および現在のガイドライン文書で参照されたこれらのガイドライン文書の直前の文書(1999~2014)。 研究選択]クラスおよびLOEによって整理された勧告を含む包括的なガイドライン文書。[データの抽出と統合]各ガイドライン文書について、推奨の数とLOEの分布(A[複数のRCTまたは単一の大規模RCTからのデータによって支持されている]、B[観察研究または単一のRCTからのデータによって支持されている]、C[専門家の意見のみによって支持されている])を決定した。 主要な成果と 【測定】複数のRCTからの証拠(LOE A)により支持されたガイドラインの推奨の比率を決定した。 【結果】現行のACC/AHAガイドライン26件(2930の勧告、中央値、ガイドラインあたり121の勧告[25~75%値、76~155])において、248の勧告(8.5%)がLOE A、1465(50.0%)がLOE B、1217(41.5%)がLOE Cとして分類されており、中央値はLOE A勧告の割合が7.9%となった(25-75%値、0.9%~15.2%)。現行のESCガイドライン25文書(3399の勧告,中央値,ガイドラインあたり130の勧告[25th-75thパーセンタイル,111-154])全体では,484の勧告(14.2%)がLOE A,1053(31.0%)がLOE B,1862(54.8%)がLOE Cと分類された. 【結論と関連性】主要な心臓血管学会のガイドラインにおける推奨事項のうち、複数のRCTまたは単一の大規模RCTからのエビデンスによって支持されているものはごくわずかであった。このパターンは、2008年から2018年にかけて有意義に改善されていないようである。 第一人者の医師による解説 RCTと相補的な実臨床データの知見 EBM発展に重要 山下 侑吾/木村 剛(教授) 京都大学大学院医学研究科循環器内科学 MMJ.August 2019;15(4) 医学・医療の進歩は、医学研究により推進されてきた。臨床医学の世界では、従来の経験に基づく医療から、科学的根拠に基づく医療(EBM)が、1980年代後半より提唱され、現在広く普及している。 EBMの目指すところは、最良の定量的データを基に、個々の患者での状況を考慮したうえで、客観的かつ効率的な診療を行うことと言える。これまでに循環器領域では、多数の臨床研究が実施されエビデンスが蓄積されてきた。異なる治療方法を比較 する際には、ランダム化比較試験(RCT)がゴールドスタンダードであった。 本論文では、2008~18年に、米国と欧州の循環器領域の主要学会(ACC/AHA、ESC)より発刊されたガイドラインの推奨事項の中で、そのエビデンスレベルの割合と経年的な推移が調査された。 同ガイドラインでは、個々の推奨事項を高い順にエビデンスレベル AからCに分類し、複数のRCT もしくは大規模な単一のRCTによる結果に基づいた推奨事項を、一番高いエビデンスレベル Aとし ている。 結果として、エビデンスレベル Aの推奨事項の割合は、8.5%(ACC/AHA)および14.2% (ESC)に過ぎず、その割合は経年的に大きな変化を認めなかったと報告している。循環器領域では、毎年数多くのRCTを含めた臨床研究が継続的に報告されている状況を考えると、本結果はやや意外な結果であったが、推奨事項のエビデンスレベル Cが減少し、Bが増加している傾向を見る限りは、 エビデンスが蓄積していることも示唆している。一方、より高いエビデンスレベルに裏打ちされた 推奨事項を増やすためには、RCTを含めた臨床研究がこれからも継続的に必要であることを示唆する報告であると考えられる。 今後も、異なる治療方法の比較においては、RCT がゴールドスタンダードであることに変わりはないと考えられるが、近年はRCTの限界も認識されつつある。EBMの目指すところである「個々の患者での状況を考慮したうえでの最良の診療」を日常臨床で実践するためには、RCTのみならず、実臨床での実態や問題点を明らかにするための「観察研究」 (リアルワールドデータ)からの知見も重要である。 これらの研究は、互いに相補的なものであると考えられ、EBMの発展のために今後もRCTを含めたさまざまな臨床研究が継続されることが期待される。 さらに、ガイドラインの推奨の多くが十分なエビデンスに基づかないことを考えると、ガイドラインは妄信すべきマニュアルではなく、推奨のエビデンスレベルと個々の患者の状況を考慮して患者にとってその時点で最良と考えられる治療法を決定するための重要な参考資料と捉えるべきであろう。
進行性心不全患者における間葉系前駆細胞の心筋内注入と左室アシスト装置サポートからの一時的な離脱の成功。無作為化臨床試験。
進行性心不全患者における間葉系前駆細胞の心筋内注入と左室アシスト装置サポートからの一時的な離脱の成功。無作為化臨床試験。
Intramyocardial Injection of Mesenchymal Precursor Cells and Successful Temporary Weaning From Left Ventricular Assist Device Support in Patients With Advanced Heart Failure: A Randomized Clinical Trial JAMA 2019 Mar 26 ;321 (12):1176 -1186. 上記論文のアブストラクト日本語訳 ※ヒポクラ×マイナビ 論文検索(Bibgraph)による機械翻訳です。 【重要】左室補助循環装置(LVAD)療法は心筋機能を改善するが、摘出できるほど回復する患者は少なく、心臓の回復を増強する幹細胞に注目が集まっている。 【目的】LVAD移植中の間葉系前駆細胞(MPC)の心筋内注射の有効性と副作用を評価する。 デザイン、設定、参加者]北米19施設でLVAD移植中の進行心不全の患者を含む無作為化第2相臨床試験(2015年7月から2017年8月)。1年間のフォローアップは2018年8月に終了。 【介入】同種MPC1億5000万個または偽治療としての凍結保護培地を2:1の割合で心筋内注射(n=106 vs n=53)。 【主要アウトカムおよび評価】主要有効性エンドポイントは、ランダム化後6カ月以内にLVAD支持からの一時的ウィーン(3回の評価予定分)に成功する割合であった。このエンドポイントは、成功を示す事後確率80%という事前に定義された閾値を用いたベイズ分析で評価された。1年間の安全性の主要エンドポイントは、介入関連の有害事象(心筋炎、心筋破裂、新生物、過敏性反応、免疫感作)の発生率であった。副次的評価項目は6ヶ月後の再入院と有害事象、1年生存率とした。 【結果】159例(平均年齢56歳、女性11.3%)のうち155例(97.5%)が1年間のフォローアップを完了した。MPCが離脱成功の可能性を高める事後確率は69%であり、事前に定義された成功の閾値以下であった。6 ヵ月にわたる LVAD 支援からの一時的な離脱が成功した平均割合は,MPC 群で 61%,対照群で 58%であった(rate ratio [RR], 1.08; 95% CI, 0.83-1.41; P = 0.55).安全性の主要エンドポイントを経験した患者はいなかった。事前に指定された10個の副次的エンドポイントが報告されたが、9個は統計的有意差に達しなかった。1年死亡率は、MPC群と対照群の間で有意差は認められなかった(14.2% vs 15.1%;ハザード比[HR]、0.89;95%、CI、0.38-2.11;P = 0.80)。重篤な有害事象の発生率は群間で有意差はなく(100患者月当たり70.9 vs 78.7、差:-7.89、95%CI、-39.95~24.17、P = .63)、再入院率(100患者月当たり0.68 vs 0.75、差:-0.07、95%CI、-0.41~0.27、P = 0.68 )も同様であった。 【結論と関連性】進行した心不全患者において、間葉系前駆細胞の心筋内注射は、偽治療としての凍結保護培地の注射と比較して、6ヵ月後の左室補助装置サポートからの一時離脱の成功率を向上させることはなかった。この結果は、デバイスサポートからの一時的な離脱によって測定される心臓の回復を促進するための心筋内間葉系幹細胞の使用を支持しない。 【臨床試験登録】clinicaltrials. gov Identifier.NCT02362646:NCT02362646. 第一人者の医師による解説 再生医療での可能性持つ間葉系幹細胞 臨床試験の成果を期待 池田 宇一 地方独立行政法人 長野市民病院・病院長 MMJ.August 2019;15(4) 心筋細胞はほとんど増殖能を持たないため、急性心筋梗塞や拡張型心筋症で心筋細胞が壊死をきたすと心臓のポンプ機能は低下し、心不全に至ると生命予後は不良となる。近年、心筋細胞を再生して心不全を治療する細胞療法の研究が注目されている。間葉系幹細胞(mesenchymal stem cells: MSC)や心筋幹細胞などの体性幹細胞を用いた臨床試験はすでに数多く進められており、胚性幹細胞(ES細胞)や人工多能性幹細胞(iPS細胞)を用い た再生医療への期待も高まっている。 体性幹細胞の中で、再生医療の細胞ソースとして最も注目されている細胞はMSCである。MSC は1979年に見いだされ、1999年にヒト骨髄中にその存在が発見されてから、現在では多くの結合組織中に存在することが明らかにされている。 MSCは心筋細胞、内皮細胞、骨芽細胞、軟骨細胞、脂肪細胞、神経細胞や肝細胞などへ分化可能である。 心不全の再生医療の臨床試験において広く使用されている細胞は骨髄由来のMSCである。MSC は冠動脈内、あるいは心筋内に注入される。MSC 治療の作用機序については不明な点も多い。投与部位におけるMSCの心筋細胞への分化はわずかであり、心機能改善にはMSCから分泌される種々の液性因子による血管新生作用、抗アポトーシス作用、 抗炎症作用、免疫制御作用などの関与が大きいと推測されている。 本研究では、北米19施設で左室補助人工心臓 (LVAD)植え込みを実施する重症心不全患者159 例を対象に、植え込み時にMSCを心筋内に注入し、 有効性および安全性を第 II相無作為化試験で評価している。6カ月時点でLVADからの一時的離脱成功率は、MSC群61%、対照群58%で有意差は示されなかった。1年死亡率、再入院率、重篤な有害事 象の発現率も2群間で有意差はなかった。LVAD植え込み時のMSC心筋内注入は支持されない結果となった。 これまでに、虚血性心筋症患者の冠動脈内や心筋内にMSCを注入することにより、左室の収縮能やリモデリング、運動耐容能が改善するという臨床試験の結果がいくつか報告されている。一方、否定的な報告も多く、欧州心臓病学会(ESC)のワーキンググループも、細胞療法への期待はまだ実現していないと結論付けている。 MSCは再生医療における大きな潜在的可能性を有する多能性幹細胞である。今後、さらなる臨床試験によるevidence-based medicine(EBM)の確立が望まれる。
トリグリセリドを低下させるLPLバリアントとLDL-Cを低下させるLDLRバリアントと冠状動脈性心臓病のリスクとの関連性
トリグリセリドを低下させるLPLバリアントとLDL-Cを低下させるLDLRバリアントと冠状動脈性心臓病のリスクとの関連性
Association of Triglyceride-Lowering LPL Variants and LDL-C-Lowering LDLR Variants With Risk of Coronary Heart Disease JAMA 2019 Jan 29 ;321 (4):364 -373. 上記論文のアブストラクト日本語訳 ※ヒポクラ×マイナビ 論文検索(Bibgraph)による機械翻訳です。 【重要性】トリグリセリドとコレステロールは、いずれもアポリポ蛋白B(ApoB)含有リポ蛋白粒子によって血漿中に運ばれている。血漿中のトリグリセリド値を下げることが、低密度リポタンパク質コレステロール(LDL-C)値を下げることと同じ程度に心血管イベントのリスクを下げるかどうかは不明である。 【目的】リポタンパク質リパーゼ(LPL)遺伝子におけるトリグリセリド低下変異体およびLDL受容体遺伝子(LDLR)におけるLDL-C低下変異体とApoB単位変化当たりの心疾患リスクの関連性を比較することである。 【デザイン、設定および参加者】1948年から2017年に北米または欧州で実施された63件のコホート研究または症例対照研究に登録された参加者を対象に、LPL遺伝子のトリグリセリド低下変異体およびLDLR遺伝子のLDL-C低下変異体からなる遺伝子スコアと心血管イベントリスクの関連をそれぞれ評価するメンデルランダム化解析。 【曝露】LPLおよびLDLR遺伝子スコアに関連する血漿トリグリセリド、LDL-C、ApoBレベルの違い 【主要アウトカムおよび測定】冠動脈疾患(CHD)-冠動脈死、心筋梗塞、または冠動脈再灌流と定義-ApoB含有リポタンパク質濃度10mg/dL低下あたりのオッズ比(OR).LL-APO-DLは冠動脈疾患(CHD)リスクと定義される.APO-DBは冠動脈疾患(CHD)リスクと定義される. 【結果】91 129例のCHD症例を含む、合計654 783人の参加者が対象となった(平均年齢62.7歳、女性51.4%)。ApoB含有リポ蛋白のレベルが10mg/dL低くなるごとに、LPLスコアは69.9mg/dL(95%CI、68.1-71.6;P = 7.1×10-1363)低いトリグリセリドレベルと関連し、0.7mg/dL(95%CI、0.03-1.4; P = 0.04)高いLDL-C値を示した。一方、LDLRスコアは、14.2-mg/dL(95% CI, 13.6-14.8; P = 1.4 × 10-465)低いLDL-C値と1.9-mg/dL(95% CI, 0.1-3.9; P = 0.04)低い中性脂肪値と関連があった。関連する脂質レベルにはこれらの違いがあるものの、LPLおよびLDLRスコアは、ApoB含有リポ蛋白レベルが10mg/dL低くなるごとにCHDリスクが同様に低くなることと関連していた(それぞれ、OR, 0.771 [95% CI, 0.741-0.802], P = 3.9 × 10-38およびOR, 0.773 [95% CI, 0.747-0.801], P = 1.1 × 10-46)。多変量メンデリアンランダム化解析では、トリグリセリドおよびLDL-C値とCHDのリスクとの関連は、ApoBの差を調整すると無効となった(トリグリセリド。OR, 1.014 [95% CI, 0.965-1.065], P = 0.19; LDL-C。トリグリセリドを低下させるLPL変異体とLDL-Cを低下させるLDLR変異体は、ApoBの単位差あたりのCHDリスクが同様に低いことと関連していた。したがって、トリグリセリドとLDL-C値を下げることの臨床的利益は、ApoBの絶対的な変化に比例する可能性がある。 第一人者の医師による解説 広がる高 TG血症への治療選択肢 さらなるエビデンス期待 遠藤 康弘/池脇 克則(教授) 防衛医科大学校神経・抗加齢血管内科 MMJ.August 2019;15(4) 本論文は、9万人前後の冠動脈疾患(CHD)患者を含む約65万人の症例を対象に、トリグリセリド (TG)代謝に関わるリポ蛋白リパーゼ(LPL)およびLDL代謝に関わるLDL受容体(LDLR)の一塩基多型(SNP)から遺伝的スコアを計算し、メンデルランダム化解析を用いてCHD(心筋梗塞、血行再建術、心血管死)との関連を解析した報告である。 近年、脂質代謝の臨床研究においてゲノムワイド 関連解析(GWAS)といった遺伝学的手法が用いられるようになった。GWASは、SNPジェノタイピングに基づき、疾患に関わる遺伝的変異を調べる手法で、TGやLDL-C、HDL-C値に関わる遺伝子多型が多く報告されている(1)。 さらにGWASで示された遺伝子多型を従来の疫 学手法に組み合わせたメンデルランダム化解析が 国内外で用いられるようになった。遺伝子多型は、 メンデルの独立の法則によれば、環境の影響を受けずにランダムに配分される。そのため、ランダム化比較試験(RCT)におけるランダム化を遺伝子多型で代用し、CHDとの関連を調べることで、環境因子を考慮せずに対象遺伝子とCHDの関連が評価可能である。 本研究において、LPL遺伝的スコアでは、LDL-C 値は軽度上昇(+0.7mg/dL)を示すもTG値の大幅な低下(-69.9mg/dL)を認め、一方でLDLR 遺伝的スコアでは、TG値の軽度低下(-1.9mg/ dL)およびLDL-C値低下(-14.2mg/dL)を示した。 LPL遺伝的スコアによるTG値低下はLDLR遺伝的 スコアによるLDL-C低下と同程度のCHDのオッズ 比低下(LPL遺伝的スコア:0.771 vs LDLR遺伝 的スコア;0.773)を認め、TG値低下はLDL-C値低下と同様に心血管疾患発症のリスク低下に重要な因子であることが示唆された。 一方、TG低下療法 のRCTで は、エイコサペンタエン酸エチル(EPA)製剤を用いたJELIS試験 (2007年)で心血管疾患(CVD)予防効果を認めたが、それ以降のEPA、フィブラート系薬剤、徐放 ナイアシンを使ったRCTでは明らかな予防効果は 示されなかった中で、REDUCE-IT試験(2018年) で高用量EPA製剤による心血管疾患保護効果(1次、 2次予防)が報告された(2)。メンデルランダム化研究は症例数や交絡因子で結果が左右されるRCTの短所を補完できるメリットを有する。 近年、海外ではTG代謝に関わるAPOC3、 ANGPTL3に対する抗体薬も開発され臨床応用が期待される。日本でも2018年に高 TG血症に対 する新規薬剤として選択的PPARαモジュレーター (SPPARMα)ペマフィブラート(パルモディアⓇ) が処方可能となり、高 TG血症に対する治療選択肢 が広がりつつある。今後、TG低下療法のさらなるエビデンスが期待される。 1. Liu DJ, et al. Nat Genet. 2017;49(12):1758-1766. 2. Bhatt DL, et al. N Engl J Med. 2019;380(1):11-22.
食事性コレステロールまたは卵の摂取と心血管疾患の発症および死亡率との関連性。
食事性コレステロールまたは卵の摂取と心血管疾患の発症および死亡率との関連性。
Associations of Dietary Cholesterol or Egg Consumption With Incident Cardiovascular Disease and Mortality JAMA 2019 Mar 19 ;321 (11):1081 -1095. 上記論文のアブストラクト日本語訳 ※ヒポクラ×マイナビ 論文検索(Bibgraph)による機械翻訳です。 【重要】コレステロールはヒトの食事によく含まれる栄養素であり、卵は食事性コレステロールの主要な供給源である。食事性コレステロールまたは卵の摂取が心血管疾患(CVD)および死亡率と関連するかどうかは依然として議論の余地がある。 【目的】食事性コレステロールまたは卵の摂取とCVDおよび全死亡の発症との関連を明らかにする。 【 デザイン・設定・参加者】1985年3月25日から2016年8月31日までに収集したデータを用い、米国の前向きコホート6施設から個人参加データをプーリングした。自己申告の食事データは、標準化されたプロトコルを用いて調和させた。 【曝露】食事性コレステロール(mg/日)または卵消費量(個/日)。 【主要アウトカムと測定】人口動態、社会経済、行動要因を調整した、CVD発症(致死性および非致死性冠動脈心疾患、脳卒中、心臓不全、その他のCVD死の複合)および全死亡に関するフォローアップ全体にわたるハザード比(HR)と絶対リスク差(ARD)。 【結果】この解析には29 615名の参加者(ベースライン時の平均[SD]年齢,51.6[13.5]歳)が含まれ,うち13 299名(44.9%)が男性,9204名(31.1%)が黒人の参加者であった。追跡期間中央値17.5年(四分位範囲、13.0~21.7、最大31.3)、5400件のCVDイベント発生と6132件の全死因死亡があった。食事性コレステロールまたは卵の摂取とCVD発症および全死亡との関連は単調であった(非線形項のすべてのP値は0.19-0.83)。1日に消費される食事性コレステロールが300mg増えるごとに、CVD発症リスク(調整後HR、1.17[95%CI、1.09-1.26];調整後ARD、3.24%[95%CI、1.39%-5.08%])および全死亡率(調整後HR、1.18[95%CI、1.10-1.26];調整後ARD、4.43%[95%CI、2.51%-6.36%])は高くなると、有意の関連性を示した。1日あたりの卵消費量が半個増えるごとに、CVDの発症リスク(調整後HR、1.06[95%CI、1.03-1.10];調整後ARD、1.11%[95%CI、0.32%-1.89%])および全死亡(調整後HR、1.08[95%CI、1.04-1.11];調整後ARD、1.93%[95%CI、1.10%-2.76%])は高くなると有意に関連していた。卵の消費とCVD発症(調整後HR、0.99[95%CI、0.93-1.05];調整後ARD、-0.47%[95%CI、-1.83%~0.88%])および全死亡(調整後HR、1.03[95%CI、0.97~1.09];調整後ARD、0.71%[95%CI、-0.85%~2.28%])との関連については、いずれも有意差はなくなった。28%])は、食事性コレステロールの消費量を調整すると有意ではなくなった。 【結論と関連性】米国の成人において、食事性コレステロールまたは卵の消費量が多いことは、用量反応的にCVDおよび全死亡の発生リスクの高さと有意に関連することが示された。これらの結果は、食事ガイドラインの策定や更新の際に考慮されるべきものである。 第一人者の医師による解説 因果関係を結論できない観察研究 メンデルランダム化解析は未実施 香川 靖雄 女子栄養大学副学長・医化学教授 MMJ.August 2019;15(4) 従来は心血管疾患(CVD)予防の常識とされてきたコレステロールの摂取量上限値300mg/日を米国心臓病学会(ACC)が廃止し、「日本人の食事摂取基準2010年版」の上限値男性750mg/日、女 性600mg/日も同基準2015年版から廃止された。これは卵の摂取量が37.1g(コレステロール 156mg)と世界で最も多い日本で特に関心の深い課題である。 上限値廃止の理由の1つは日本人の大規模調査で卵の毎日摂取者に比べて週1個以下摂取者は有意に血清コレステロールが高く、冠動脈疾患のハザード比(HR)も有意ではないが28%も高かったからである(1)。しかし、高いコレステロー ル値を知った人が卵の摂取を控えたために起こった「因果の逆転」がありうる。また、食事性コレス テロール量よりも飽和脂肪から体内で合成されるコレステロール量の方がはるかに多く、卵1個には飽和脂肪が1.56mgと乳製品よりも少ない点も 上限量の廃止を支持した(2)。 しかし、今回紹介する研究では、コレステロール 摂取(300mg/日)と卵の摂取が有意にCVDと全死亡の増加と関連することを人口統計学的、社会経 済的、行動的要因で調整した解析で再確認した。参 加者29,615人(平均年齢51.6歳)の追跡期間(中央値17.5年)中に5,400件のCVDイベントおよび6,132件の全死亡が発生した。主要評価項目は CVDおよび全死亡のHRと絶対リスク差(ARD)である。 食事性コレステロール量または卵の消費量 の増加に伴い、CVDおよび全死亡は単調に増加した。コレステロール摂取(300mg/日)はCVD(HR, 1.17;95%CI, 1.09~1.26; ARD, 3.24%;1.39 ~5.08%)および全死亡(HR, 1.18;1.10~1.26; ARD, 4.43%;2.51~6.36%)のリスクと有意に関連していた。卵半個/日毎の摂取増加はCVD (HR, 1.06;95%CI, 1.03~1.10; ARD, 1.11%; 0.32~1.89%)および全死亡(HR, 1.08;1.04 ~1.11; ARD, 1.93%;1.10~2.76%)のリスク上昇と有意に関連していた。 CVDと全死亡のリスクの用量依存的な上昇に基づき、現在の食事ガイドラインの再考を求めている。 しかし、この結果は因果関係を結論できない観察研究による。また、因果関係を求めるためのコレステロールや卵の摂取量の無作為化対照試験はCVD や死亡の確認に長期を要するため実施困難である。 筆者は遺伝子検査を伴った観察研究で冠動脈疾患とLDLコレステロールの因果関係を確立したメンデルランダム化解析の結果(3)を信頼するが、コレステロールや卵の摂取量とCVDのメンデルランダム化解析はまだない。 1. Nakamura Y, et al. Br J Nutr. 2006;96(5):921-928. 2. Soliman GA. Nutrients. 2018;10(6). pii:E780. 3. Jansen H, et al. Eur Heart J. 2014;35(29):1917-1924.
医療従事者のインフルエンザ予防におけるN95レスピレータと医療用マスクの比較.無作為化臨床試験。
医療従事者のインフルエンザ予防におけるN95レスピレータと医療用マスクの比較.無作為化臨床試験。
N95 Respirators vs Medical Masks for Preventing Influenza Among Health Care Personnel: A Randomized Clinical Trial JAMA 2019 Sep 3 ;322 (9):824 -833. 上記論文のアブストラクト日本語訳 ※ヒポクラ×マイナビ 論文検索(Bibgraph)による機械翻訳です。 【重要性】医療従事者(HCP)の職場ウイルス性呼吸器感染症の発症予防におけるN95レスピレータと医療用マスクの効果について、臨床研究では結論が出ていない。 【目的】HCPのインフルエンザおよびその他のウイルス性呼吸器感染症の予防に関するN95レスピレータと医療用マスクの効果を比較する。 【デザイン、設定および参加者】米国の7医療センターの137外来研究施設で2011年9月から2015年5月に実施し、最終フォローアップが2016年6月となるクラスター無作為化実用的効果試験であった。毎年4年間、ウイルス性呼吸器疾患のピークである12週間の期間に、各センター内の外来患者施設のペア(クラスター)をマッチングし、N95レスピレーター群と医療用マスク群に無作為に割り付けた。 【介入】全体として、189クラスタの1993人がN95呼吸器装着群(観察期間2512HCP-seasons)に、191クラスタの2058人が医療用マスク装着群(2668HCP-seasons)に無作為に割り付けられた。 主要アウトカムと測定】主要アウトカムは実験室で確認されたインフルエンザ発症率であった。副次的アウトカムは、急性呼吸器疾患、実験室で検出された呼吸器感染症、実験室で確認された呼吸器疾患、およびインフルエンザ様疾患の発生率であった。 【結果】無作為化された参加者2862名(平均[SD]年齢、43[11.5]歳、2369[82.8%]女性)中、2371名が研究を完了し、5180名のHCP-seasonsを占めた。実験室で確認されたインフルエンザ感染イベントは,N95 レスピレータ群で 207 件(HCP-seasons の 8.2%),医療用マスク群で 193 件(HCP-seasons の 7.2%)( 差は 1.0%,[95% CI,-0.5% ~ 2.5%],P = 0.18)( 調整オッズ比 [OR]:1.18[95% CI,0.95 ~ 1.45] )であった.呼吸器群の急性呼吸器疾患イベント 1556 件対マスク群の 1711 件(差、1000 HCP シーズンあたり -21.9 件 [95% CI、-48.2 ~ 4.4]; P = .10); 呼吸器群の実験室検出呼吸器感染 679 件対マスク群の 745 件(差、1000 HCP シーズンあたり -8.9 件 [95% CI、-33.3 ~ 15.4]; P = .47); 呼吸器感染と診断された実験室検出感染 371 件(差、1 HCP シーズンあたり -4.0 件 [95% CI、0.1 ~ 0.2]、 P = 0.9); 呼吸器感染と診断された実験室検出感染 2 件47)、呼吸器群371件対マスク群417件(差、1000HCP-シーズン当たり-8.6件[95%CI、-28.2~10.9]、P = 0.39)、呼吸器群128件対マスク群166件(差、1000HCP-シーズン当たり-11.3件[95%CI、-23.8~1.3]、P = .08)、検査で確認される呼吸器疾患事象がありました。呼吸器群では、89.4%の参加者が「いつも」または「時々」指定された器具を着用したと報告したのに対し、マスク群では90.2%であった。 【結論と関連性】外来医療従事者において、この試験の参加者が着用したN95呼吸器と医療用マスクは、実験室で確認されたインフルエンザの発生率に有意差をもたらさなかった。 【試験登録】臨床試験番号(CCT):National ClinicalTrials. gov ID。NCT01249625。 第一人者の医師による解説 インフルエンザ流行期の外来医療スタッフ マスク着用が望ましい 関 雅文 東北医科薬科大学医学部感染症学教室教授 MMJ.February 2020;16(1) 呼吸器ウイルス疾患、特にインフルエンザに対するマスク着用の有用性が長年議論されている。 手指衛生のほか、マスク着用による感染予防効果が多数報告されている一方、粒子径の小さい多くの呼吸器ウイルス感染で、一般的なサージカルマスクでの感染予防効果への疑問もあった(1),(2)。 本研究では、結核など空気感染が懸念される際に使用されるN95マスクと、一般に使用されるサージカルマスクを着用した際の、外来医療スタッフにおけるインフルエンザ・その他呼吸器ウイルス 疾患の予防効果を前向きに比較・検討している。米国の7つの大規模医療センターに属する137の外来センターで2011年9月~15年5月に医療スタッフ 2,862人をN95マスク群、サージカルマスク 群にランダムに割り付け、最終的に2016年6月 まで追跡した。 主要評価項目であるインフルエンザ感染の発生は、N95マスク群で207人(8.2%)、サージカルマスク群で193人(7.2%)であった(P=0.18)。 副次評価項目では、急性呼吸器症状の出現がN95 マスク群で1,556イベント、サージカルマスク群で1,711イベント(P=0.10)、無症状時も含めて呼吸器ウイルスが同定されたのはN95マスク群 で679イベント、サージカルマスク群で745イベ ント(P=0.47)、呼吸器症状および呼吸器ウイル ス感染が確認されたのはN95マスク群で371イベント、サージカルマスク群で417イベント(P= 0.39)、インフルエンザ様疾患はN95マスク群で128イベント、サージカルマスク群で166イベント(P=0.08)であった。 これらの結果からは、主に外来で働く医療スタッフにおいて、インフルエンザ・その他呼吸器ウイルス疾患の予防に関して、N95マスクとサージカルマスクで、その効果には有意差はないことが示唆された。一方、N95マスク群とサージカルマスク群で比較した急性呼吸器症状の発生確率(IRR)は intention-to-treat集団で0.99であるのに対し、インフルエンザ様症状のIRRは0.86であり、N95 マスクによる一定のインフルエンザ予防効果が存在する可能性も示されたと言えよう。なお、N95 マスク群では89.4%、サージカルマスク群では90.2%の医療スタッフが「常に」~「時々」と一定以上、それぞれのマスクを装着しており、本研究の質は比較的担保されている。 以上の結果を総合すると、インフルエンザやその他の呼吸器ウイルス感染にさらされる危険性の高い外来医療スタッフは、N95マスクを装着する必要はないが、サージカルマスク装着での勤務は望ましいであろう。 1. MacIntyre CR et al. In¬fluenza Other Respir Viruses. 2017;11(6):511-517. 2. Loeb M et al. JAMA. 2009;302(17):1865-1871.
成人市中肺炎患者における早期臨床効果のための経口レファミュリンとモキシフロキサシンの比較。LEAP 2 Randomized Clinical Trial(無作為化臨床試験)。
成人市中肺炎患者における早期臨床効果のための経口レファミュリンとモキシフロキサシンの比較。LEAP 2 Randomized Clinical Trial(無作為化臨床試験)。
Oral Lefamulin vs Moxifloxacin for Early Clinical Response Among Adults With Community-Acquired Bacterial Pneumonia: The LEAP 2 Randomized Clinical Trial JAMA 2019 ;322 (17):1661 -1671. 上記論文のアブストラクト日本語訳 ※ヒポクラ×マイナビ 論文検索(Bibgraph)による機械翻訳です。 【重要性】市中肺炎(CABP)の治療には、抗菌薬耐性の向上と標準治療の安全性の懸念から、新しい抗菌薬が必要である。 【目的】CABP患者におけるレファミン5日間経口レジメンの有効性と有害事象を評価する。 【デザイン、設定および参加者】Pneumonia Outcomes Research Team(PORT)のリスククラスがII、III、IVである18歳以上の成人、X線写真で肺炎が確認されている、急性疾患、3つ以上のCABP症状、2つ以上のバイタルサイン異常がある、19か国99施設で行われた第3相、非劣性ランダム化臨床試験。最初の患者訪問は2016年8月30日で、患者は30日間フォローアップされ、最終フォローアップ訪問は2018年1月2日であった 【介入】患者は、レファムリン(600 mg 12時間毎 5日間;n = 370)またはモキシフロキサシン(400 mg 24時間毎 7日間;n = 368)の内服を受けるよう1対1にランダムに割り付けられた。 【主要評価項目】米国食品医薬品局(FDA)の主要評価項目は、intent-to-treat(ITT)集団(無作為化された全患者)において、いずれかの試験薬の初回投与後96時間(24時間以内)における早期臨床反応とした。)奏効者は、生存しており、4つのCABP症状のうち2つ以上に改善が見られ、どのCABP症状も悪化しておらず、現在のCABPエピソードに対して試験薬以外の抗菌薬を投与されていない者と定義されました。欧州医薬品庁(EMA)の副次評価項目は、修正ITT集団および臨床評価可能集団における治癒判定(最終投与から5~10日後)の際の治験責任医師による臨床効果の評価としました。非劣性マージンは、早期臨床効果および治験責任医師評価による臨床効果について10%とした。 【結果】無作為化患者738例(平均年齢57.5歳、女性351例[47.6%]、PORTリスククラスがⅢまたはⅣの360例[48.8%])において、707例(95.8%)が試験を完了した。早期臨床効果率は、レファムリンで90.8%、モキシフロキサシンで90.8%でした(差は0.1%[1-sided 97.5% CI, -4.4% ~ ∞])。治験責任医師による臨床的奏功の評価率は、修正ITT集団でレファムリン87.5%、モキシフロキサシン89.1%(差:-1.6%[1-sided 97.5% CI, -6.3~∞])、臨床的評価可能集団では治癒判定でそれぞれ89.7%と93.6%(差:-3.9%[1-sided 97.5% CI, -8.2~∞] )であり、レファムリンはモキシフロキサシンの1/3以下であった。最も頻繁に報告された治療上緊急の有害事象は、消化器系(下痢:レファミュリン群45/368例[12.2%]、モキシフロキサシン群4/368例[1.1%]、吐き気:レファミュリン群19/368例[5.2%]、モキシフロキサシン群5/368例[1.0%])であった。 【結論と関連性】CABP患者において、初回投与後96時間の早期臨床効果に関して、5日間経口レファミュリンは7日間経口モキシフロキサシンに対して非劣性を示した。 【試験登録】ClinicalTrials. gov Identifier:NCT02813694、欧州臨床試験Identifier:2015-004782-92。 第一人者の医師による解説 現行抗菌薬の数倍の価格 使用を制限する要因となる可能性 藤田 次郎 琉球大学大学院医学研究科感染症・呼吸器・消化器内科学(第一内科)教授 MMJ.February 2020;16(1) 本論文は、市中細菌性肺炎を対象としたレファムリンとモキシフロキサシンの無作為化比較対照試 験(LEAP 2試験)に関する報告である。レファムリンは、プレウロムチリン系抗菌薬に分類され、細 菌リボソームの50Sサブユニットのペプチド転移酵素に結合することによって細菌の蛋白合成を阻害する新規抗菌薬である。 今回の第3相臨床試験は、世界19カ 国( 欧州、 南米およびメキシコ、米国、アジア、アフリカ)、 99の 医療施設 で 実施 さ れ た。対象者 は、18歳以 上 でPneumonia Outcomes Research Team (PORT)リスク分類 II、IIIまたはIVで、画像所見にて確認され、臨床症状や身体所見を有する急性細菌 性肺炎患者である。適格患者(738人)のうち、一 群(370人)はレファムリンの経口薬600mgを、 1日2回、5日間服用し、もう一群(368人)はニューキノロン系抗菌薬のモキシフロキサシン 400mg を、1日1回、7日間服用し、両者の市中細菌性肺炎 対する治療効果が比較された。米食品医薬品局 (FDA)の主要エンドポイントは96時間後の早期臨床効果である。 結果として、内服開始96時間以内に、急性細菌性肺炎が一定レベル以上改善する早期臨床効果が得られた割合は、レファムリン群90.8%、モキシ フロキサシン群90.8%で両群に差はなく、治療終了後5~10日時点の治癒率(test of cure)も両群 間で差を認めなかった。ただしレファムリンの有 害事象として、モキシフロキサシンと比較して、消化器症状(下痢12.2%、嘔気5.2%、嘔吐3.3%)の頻度が高い傾向を認めた。  今回の臨床試験により、市中細菌性肺炎に対する有効性においてレファムリンのモキシフロキサシンに対する非劣性が証明された。またモキシフロキサシンと同様に、レファムリンは多剤耐性肺炎球菌による肺炎またはレジオネラ肺炎に対しても有 用性を示した。   本論文に対してEditor’s Note(1)が併載されている。多くの大手製薬企業が抗菌薬開発から撤退する中で、小さな会社が新規作用機序を有する新たな抗菌薬を開発したことは喜ばしいものの、驚くのはモキシフロキサシンやレボフロキサシンよりも数倍高い価格(1日あたり注射剤は205ドル、経口薬は275ドル)であり、使用を制限する要因になりうる、と述べている。 わが国は、これまで多くの抗菌薬を開発してきた。世界で広く使用される抗菌薬として、レボフロ キサシン、クラリスロマイシン、メロベネムなどがある。近年、耐性菌対策で抗菌薬の使用制限が優先され、抗菌薬の開発を中止した製薬企業が多い。レファムリンの登場に際し、日本の製薬企業にも新規抗菌薬の開発を期待したい。 1. Malani PN. JAMA. 2019;322(17):1671-1672.
大気汚染への長期暴露と定量的に評価された肺気腫および肺機能の変化との関連性。
大気汚染への長期暴露と定量的に評価された肺気腫および肺機能の変化との関連性。
Association Between Long-term Exposure to Ambient Air Pollution and Change in Quantitatively Assessed Emphysema and Lung Function JAMA 2019 ;322 (6):546 -556 上記論文のアブストラクト日本語訳 ※ヒポクラ×マイナビ 論文検索(Bibgraph)による機械翻訳です。 【重要性】歴史的レベルの大気汚染物質は心血管疾患や呼吸器疾患と関連しているが、現代の大気汚染物質濃度への曝露が肺気腫の進行と関連しているかどうかは分かっていない。 【目的】周囲のオゾン(O3)、小粒子状物質(PM2.5)、窒素酸化物(NOx)、黒炭曝露とCT画像や肺機能で評価した肺気腫割合の変化について経時的関連を評価することである。 【デザイン、設定および参加者】本コホート研究は、米国の6大都市圏で実施されたMulti-Ethnic Study of Atherosclerosis(MESA)空気と肺の研究からの参加者を含み、2000年7月から2002年8月に募集した45~84歳の成人6814人と2005年2月から2007年5月に追加募集した257人が2018年11月までフォローアップに参加した。 【曝露】居住地特異的大気汚染物質濃度(O3、PM2.5、NOx、黒色炭素)は、1999年からフォローアップ終了までに決定したコホート特異的モニタリングを組み込んだ検証済み時空間モデルにより推定した。 主要アウトカムおよび測定法】肺のピクセルが-950ハウンスフィールド単位未満と定義した気腫率は、心臓CTスキャン(2000~2007)および肺CTスキャンの同等部位(2010~2018)により参加者ごとに最高5回評価された。スパイロメトリーは参加者一人当たり3回まで実施した(2004~2018年)。 【結果】研究参加者7071名(募集時の平均[範囲]年齢60[45~84]歳,3330名[47.1%]が男性)中,5780名がベースライン検査の年とフォローアップ期間に屋外住居大気汚染濃度を割り当てられ,最低1回のフォローアップCTスキャンを受け,2772名が最低1回のフォローアップスパイロメーター評価を,10年間中央値で受けていた。肺気腫の割合の中央値はベースラインで3%、10年ごとに平均0.58%ポイント増加した。PM2.5とNOxの平均環境濃度は、O3ではなく、追跡調査中に大幅に減少した。ベースライン時のO3、PM2.5、NOx、およびブラックカーボンの環境濃度は、10年当たりの肺気腫の割合の大きな増加と有意に関連していた(O3:0.13 per 3 parts per billion [95% CI, 0.03-0.24]; PM2.5: 0.11 per 2 μg/m3 [95% CI, 0.03-0.19]; NOx: 0.06 per 10 parts per billion [95% CI, 0.01-0.12]; Black Carbon: 0.10 per 0.2 μg/m3 [95% CI, 0.01-0.18]) となった。PM2.5濃度ではなく、追跡期間中の周囲のO3およびNOx濃度も、肺気腫の割合がより大きくなることと有意に関連していた。ベースラインおよび追跡調査中の他の汚染物質ではなく、環境中O3濃度は、10年当たりの強制呼気1秒量の大きな減少(ベースライン:10億分の3 13.41 mL [95% CI, 0.7-26.1]; 追跡調査:10億分の3 18.15 mL [95% CI, 1.59-34.71] )に有意に関連した。59-34.71])。 【結論と関連性】米国の6大都市圏で2000年から2018年にかけて実施した本コホート研究において、環境大気汚染物質への長期曝露は、CT画像と肺機能を用いて定量的に評価した肺気腫の増加と有意に関連していた。 第一人者の医師による解説 わが国でも上昇傾向の大気中オゾン 健康影響評価と予防施策が必要 島 正之 兵庫医科大学公衆衛生学主任教授 MMJ.February 2020;16(1) オゾン(O3)や微小粒子状物質(PM2.5)などの大気汚染物質に曝露されることによって慢性閉塞性 肺疾患(COPD)患者の死亡リスクが高くなることは知られている(1)が、肺気腫の進行に与える長期的な影響は明らかではなかった。 本研究は、米国6都市で実施されているアテローム性動脈硬化の多民族コホート研究(Multi-Ethnic Study of Atherosclerosis)の 参加者7,071人 (45 ~ 84歳)を対象に、2000 ~ 18年の大気汚染物質への曝露と胸部 CT撮影によって評価した肺の気腫性病変や肺機能値の変化との関連を定量的に解析したものである。 対象者のうち5,780人は、調査開始時に加えて 追跡期間中に少なくとも1回の胸部 CT検査を受け、その間の大気汚染濃度が推計された。調査開始時の対象者の居住地における大気中 O3、PM2.5、窒素 酸化物(NOX)濃度が高いほど、追跡期間中に肺の気腫性病変の割合が上昇した。O3濃度との関連が最も大きく、3ppb上昇あたり気腫性病変の割合は10年間で0.13%上昇した。 追跡期間中の大気汚染濃度との関連では、O3濃度3ppb上昇あたり気腫性病変は10年間で0.18%(95% CI, 0.08~ 0.28)上昇し、29箱×年(pack-years;1日1箱 〈20本〉×年数)の喫煙による影響と同程度であった。追跡期間中のNOX濃度との関連も有意であったが、PM2.5濃度との関連はみられなかった。肺機能検査での1秒量の低下については、調査開始時および追跡期間中の大気中 O3濃度との関連は有意であったが、他の汚染物質とは関連がなかった。 PM2.5をはじめとする大気汚染は中国、インドなどの新興国において深刻な問題となっているが、 日本や米国などの先進国では年々改善している。 一方、大気中 O3濃度は気候変動などの影響により世界的に上昇傾向にある。本研究は、米国におけるコホート研究で、大気中 O3への長期間曝露は気腫性病変の割合や肺機能値で評価した肺気腫の進行と有意な関連があり、29箱×年の喫煙と同程度の影響であることを示している。PM2.5について、調査開始時の濃度との関連は有意であったが、追跡期間中の濃度との関連が認められなかったのは、この間にPM2.5濃度が低下したためであろう。 日本において環境基準が設定され、大気中濃度が 常時監視されている光化学オキシダント(OX)は、その大部分がO3である。光化学スモッグの原因物質として知られるが、わが国でも濃度は上昇傾向であり、全国ほとんどすべての測定局で環境基準 が達成されておらず、米国とほぼ同じ状況である。今後、日本でもO3への曝露による健康影響を評価し、 それを予防するための施策の確立が望まれる。 1.Hao Y et al. Am J Respir Crit Care Med. 2015;192(3):337-341
移行期のケアと長期的な自己管理支援を組み合わせた病院主導のプログラムが慢性閉塞性肺疾患入院患者の転帰に及ぼす効果。無作為化臨床試験。
移行期のケアと長期的な自己管理支援を組み合わせた病院主導のプログラムが慢性閉塞性肺疾患入院患者の転帰に及ぼす効果。無作為化臨床試験。
Effect of a Hospital-Initiated Program Combining Transitional Care and Long-term Self-management Support on Outcomes of Patients Hospitalized With Chronic Obstructive Pulmonary Disease: A Randomized Clinical Trial JAMA 2019 ;322 (14):1371 -1380. 上記論文のアブストラクト日本語訳 ※ヒポクラ×マイナビ 論文検索(Bibgraph)による機械翻訳です。 【重要性】慢性閉塞性肺疾患(COPD)の増悪で入院した患者は、再入院率が高く、QOLが低下する。 【目的】COPDで入院した患者とその家族介護者に対する移行支援と長期自己管理支援を組み合わせた病院主導のプログラムが、転帰を改善できるかを評価する。 【デザイン、設定および参加者】240名の参加者でメリーランド州ボルチモアにて単一サイトの無作為化臨床試験を実施した。参加者はCOPDにより入院した患者であり,介入または通常ケアに無作為に割り付け,退院後6か月間フォローアップを行った。登録は2015年3月から2016年5月,追跡調査は2016年12月に終了した。 【介入】介入(n=120)は,患者とその家族介護者がCOPDの長期自己管理を行うための包括的な3ヶ月間のプログラムであった。標準化されたツールを用いてCOPD患者を支援するための特別な訓練を受けた看護師が実施した。通常ケア(n = 120)では、退院後の30日間、退院計画の遵守と外来診療への接続を確実にするための移行支援が行われた。 【主要アウトカムおよび測定法】主要アウトカムは、6ヵ月時点での参加者1人あたりのCOPD関連急性期医療イベント(入院および救急部訪問)の数であった。共同主要アウトカムは、退院後6か月におけるSt George’s Respiratory Questionnaire(SGRQ)で測定した参加者の健康関連QOLの変化とした(スコア、0[最高]~100[最悪]、4ポイントの差は臨床的に有意)。 【結果】無作為化された240例(平均[SD]年齢、64.9[9.8]歳;女性61.7%)において203例(85%)が試験を完了した。ベースラインの平均(SD)SGRQスコアは,介入群で62.3(18.8),通常ケア群で63.6(17.4)であった。6ヵ月時の参加者1人当たりのCOPD関連急性期イベントの平均件数は、介入群1.40(95%CI、1.01~1.79)に対して通常ケア群0.72(95%CI、0.45~0.97)だった(差、0.68[95%CI、0.22~1.15];P = 0.004)。6ヵ月時の参加者のSGRQ総スコアの平均変化は、介入群で2.81、通常ケア群で-2.69であった(調整済み差、5.18 [95% CI, -2.15 to 12.51]; P = 0.11)。研究期間中の死亡は15例(介入:8例、通常ケア:7例)、入院は339例(介入:202例、通常ケア:137例)であった。 【結論と関連性】COPDにより入院した患者の単一サイトの無作為臨床試験において、移行と長期自己管理支援を組み合わせた3ヶ月間のプログラムは、COPD関連の入院と救急部訪問が著しく多く、生活の質の改善はみられなかった。この予期せぬ知見の理由を明らかにするため、さらなる研究が必要である。 【臨床試験登録】ClinicalTrials. gov Identifier:NCT02036294。 第一人者の医師による解説 介入群に疾患活動性高の患者多く単施設での試験が影響か 真に効果的な支援について検証必要 小松 茂 神奈川県立循環器呼吸器病センター呼吸器内科部長 MMJ.February 2020;16(1) 慢性閉塞性肺疾患(COPD)は現在世界の死因第 4位で、2020年には3位になると予測されている疾患である(1)。COPD患者の生命予後を悪化させる原因として増悪(症状悪化により安定期治療の変更 が必要となる状態)と呼ばれる病態があり、増悪は患者の生活の質(QOL)や呼吸機能も低下させる2。 重度の増悪になると入院が必要となるが、増悪に よる入院患者は再入院率が高く、入院を繰り返すたびに次の増悪入院への期間が短くなりやすい(2)。 そのため退院後に再増悪させないことが予後改善にはきわめて重要となる。    今回の米国で実施された単施設ランダム化試験 では、COPD退院患者 240 人 をCOPD専門看護 師による3カ月間の統合プログラムが提供される介入群と一般看護師による30日間の移行支援を行う通常ケア群に割り付け、6カ月後のCOPD関連の急性期ケア(入院、救急外来受診)を要したイベント数およびQOLの変化が比較された。結果、 6カ月後の急性期ケアイベント数は介入群1.40、 通常ケア群0.72と介入群で有意に悪く、QOL指標として用いられたSt. George’s Respiratory Questionnaireの 総 ス コ ア の 平均変化 も 介入群 2.81、通常ケア群-2.69と有意差はないが介入 群の方が悪かった。   GLOBAL INITIATIVE FOR CHRONIC OBSTRUCTIVE LUNG DISEASE(GOLD) が 発行した2020 GOLD REPORTでは、系統的レビューなどの結果から、医療専門家との対話を伴う セルフマネジメント介入は健康状態を改善し、入院や救急受診を減少させる(エビデンスレベル B) としている(1)。日本のCOPDガイドラインでも増悪による入院時には、多職種チームによる在宅疾病管理プログラムの導入は再入院を減らし増悪による死亡を減らす可能性があると記載されている(2)。 今回の結果がこれらと正反対になったのは、試験が単施設で行われ、介入群に在宅酸素療法を受けている患者、現喫煙者、心不全合併者など疾患活動性が高い患者が多かったこと、介入群は医療者との連絡が密であり、さらにアクションプランにより早期に増悪の兆候をとらえることができ救急受診(およびその後の入院)が増えたことなどが考えられる。 本論文と同様に、COPDの包括的なケア管理プログ ラムはCOPD関連入院の減少につながらなかったという報告もある(3)。COPD患者にとってどのような介入が真に効果的なのか、今後さらなる検証が望まれる。 1.GOLD 2020 Report. URL:https://bit.ly/302IzTX 2. 日本呼吸器学会 COPD ガイドライン第 5 版作成委員会 . COPD(慢性閉塞性肺疾患)診断と治療のためのガイドライン 2018(第 5 版). 3. Fan VS et al. Ann Intern Med. 2012;156(10):673-683.
膵臓癌のスクリーニング。US Preventive Services Task Force Reaffirmation Recommendation Statement(米国予防医療専門委員会の再確認勧告)。
膵臓癌のスクリーニング。US Preventive Services Task Force Reaffirmation Recommendation Statement(米国予防医療専門委員会の再確認勧告)。
Screening for Pancreatic Cancer: US Preventive Services Task Force Reaffirmation Recommendation Statement JAMA 2019 Aug 6 ;322 (5):438 -444. 上記論文のアブストラクト日本語訳 ※ヒポクラ×マイナビ 論文検索(Bibgraph)による機械翻訳です。 【重要】膵臓がんは、年齢調整した年間発症率が10万人年あたり12.9人という珍しいがんである。しかし、膵臓癌の予後は不良であるため、死亡率は10万人年当たり11.0人である。発症率は低いものの、膵臓がんは米国で3番目に多いがん死亡原因となっています。膵臓癌の発生率は増加しており、他の種類の癌の早期発見と治療の改善とともに、膵臓癌はまもなく米国における癌死亡原因の第2位になると推定される。 【目的】膵臓癌のスクリーニングに関する2004年の米国予防医療作業部会(USPSTF)の勧告を更新することである。 【エビデンスレビュー】USPSTFは、膵癌スクリーニングの有益性と有害性、膵癌スクリーニング検査の診断精度、スクリーニングで発見された膵癌または無症状の膵癌の治療の有益性と有害性に関するエビデンスをレビューした。 【調査結果】USPSTFは、膵癌スクリーニングまたはスクリーニングで発見された膵癌の治療により、疾患特異的な罹病率と死亡率、全死因死亡率が改善するというエビデンスを見いだせなかった。USPSTFは、無症状の成人における膵がんスクリーニングの有益性の大きさは、小さいより大きくないという境界線が引けるという十分な証拠を見いだした。USPSTFは、膵がんスクリーニングおよびスクリーニングで検出された膵がんの治療の有害性の大きさは、少なくとも中程度に抑えることができるという十分な証拠を見いだした。USPSTFは、無症候性成人における膵癌スクリーニングの潜在的利益は潜在的有害性を上回らないという前回の結論を再確認する。 【結論と勧告】USPSTFは、無症候性成人における膵癌のスクリーニングを行わないよう勧告する。(D勧告)。 第一人者の医師による解説 早期膵がん患者の同定へ 新たな検診手段の開発が重要 岡崎 和一 関西医科大学内科学第三講座(消化器肝臓内科)主任教授 MMJ.February 2020;16(1) 米国における膵がんの罹患率は人口10万人あたり年間12.9人で比較的低いものの、年間死亡率 は11.0人、その部位別順位は3位と予後不良である(1)。5年生存率は膵がん全体で9.3%であるが、 局所進行膵がん37.4%、周辺臓器浸潤例12.4%、 遠隔転移例2.9%と診断時の病期に依存し、病初期での手術のみが生存率改善を期待できるため、早期診断が重要である。膵がん患者の85~90%は 遺伝的背景がなく、5~10%が家族性危険因子を有 し、3~5%がPeutz-Jeghers症候群などの遺伝的がん症候群とされている。 今回、米国予防医学特別作業部会(USPSTF)は 2004年の勧告と同じく、検診による不利益(harm)が利益(benefit)を上回るとの理由から、「無症状の一般成人を対象とする膵がん検診は推奨しない」 と結論した。しかしながら、家族性膵がん(第1度 近親者に2人以上の膵がん患者がいる場合)を含み、膵がんの家族性危険因子を考慮することが重要であり、本論文でも、今回の推奨はこれらの高リスク者には適用されないとしている。 実際、CAPS Consortiumは、膵がん患者の第1度近親者、第1 度近親者に罹患者を有するp16 /BRCA2 変異保 有者、Peutz-Jeghers症候群の患者、第1度近親者に膵がん患者を有するリンチ症候群患者を高リスク者と定義し、高リスク者に対して超音波内視鏡検査(EUS)またはMRI/MRCP によるスクリー ニングを推奨している(2)。系統的レビューでは、膵がん家族歴を有する無症状者を対象とした検診のみ、根治的切除および生存期間中央値の延長との 関連が示されている(2)。 Cancer of the Pancreas Screening 2(CAPS2)プロジェクトでは、無症状の膵がん患者の第1度近親者に対するEUSで 10%に浸潤前膵がんが発見され、高リスク者の検 診手段として有望である可能性が示唆されている(2)。 CAPS3 では膵がんリスクの高い無症状者に対して二重盲検下のCT、MRI およびEUS によるスクリーニングにより42%で異常がみられ、その検出率はEUS 42%、MRI 33%、CT 11%であった。さ らに高リスク者を平均4.2 年間追跡した前向きコ ホート研究では、32%の患者で膵臓に異常が認められ、MRI/磁気共鳴膵胆管造影(MRCP)もEUS の 補助的検査法または代替検査法になりうることが 示唆された。 将来的には、早期膵がん患者を同定できる新たな検診手段の開発が重要であると考えられる。現状では、全血中のマイクロ RNA、cell-free DNA、 血清代謝プロファイリングなどが新規バイオマー カー候補とされている。 1.National Cancer Institute (NCI). Cancer Stat Facts: pancreatic cancer. URL:https://bit.ly/2ZNfv2I 2.NCCN Clinical Practice Guidelines in Oncology:Pancreatic Adenocarcinoma(Version 3.2019).
世界貿易センタービル災害後の消防士における長期的な心血管疾患リスク。
世界貿易センタービル災害後の消防士における長期的な心血管疾患リスク。
Long-term Cardiovascular Disease Risk Among Firefighters After the World Trade Center Disaster JAMA Network Open 2019 ;2 (9):e199775 上記論文のアブストラクト日本語訳 ※ヒポクラ×マイナビ 論文検索(Bibgraph)による機械翻訳です。 【重要性】2001年9月11日以降の世界貿易センタービル(WTC)曝露と長期的な心血管疾患(CVD)転帰との関連を検討した既報の研究では、様々な知見が報告されている。 【目的】ニューヨーク市消防局(FDNY)の消防士において、WTC曝露がCVDリスクの上昇と関連しているかどうかを評価すること。 デザイン・設定・参加者]このコホート研究では、FDNY男性消防士において2001年9月11日から2017年12月31日の間にWTC曝露とCVDリスクの関連について評価された。多変量Cox回帰分析を用いて、WTC曝露の2つの指標:WTC現場への到着時間およびWTC現場での勤務期間と関連したCVDリスクを推定した。データ解析は2018年5月1日から2019年3月8日まで実施した。 【主要評および測定法】主要CVDアウトカムは、心筋梗塞、脳卒中、不安定狭心症、冠動脈手術または血管形成術、またはCVD死亡であった。二次アウトカム(全CVD)には,すべての一次アウトカムイベント,または以下のいずれかを含む:一過性脳虚血発作;狭心症治療薬の使用または介入なしの心臓カテーテル治療のいずれかとして定義される安定狭心症;心筋症;およびその他のCVD(大動脈瘤,末梢動脈血管介入,頸動脈外科手術)。 【結果】男性消防士9796人(2001年9月11日の平均[SD]年齢は40.3[7.4]歳、7210人[73.6%]が喫煙歴なし)において489件の主要アウトカムイベントが発生した。)CVDの年齢調整後の発症率は、WTCへの曝露が多い消防士ほど高かった。CVDの主要転帰の多変量調整ハザード比(HR)は、最も早く到着したグループで、遅く到着したグループと比較して1.44(95%CI、1.09-1.90)であった。同様に、WTC敷地内で6ヵ月以上働いた人と、それ以下の期間しか働かなかった人では、CVDイベントを発症する可能性が高かった(HR、1.30;95%CI、1.05-1.60)。高血圧(HR, 1.41; 95% CI, 1.10-1.80), 高コレステロール血症(HR, 1.56; 95% CI, 1.28-1.91), 糖尿病(HR, 1.99; 95% CI, 1.33-2.98), および喫煙(現在: HR, 2.13; 95% CI, 1.68-2.70; 前: HR, 1.55; 95% CI, 1.23-1.95 )などの確立したCVD危険因子が、多変量モデルでCVDとの関連性が顕著であった。結論と妥当性】本研究の結果は,より大きなWTC被曝と長期的なCVDリスクとの間に有意な関連があることを示唆するものであった。この知見は、災害の生存者の健康状態を長期にわたって監視することの重要性を補強するものと思われる。 第一人者の医師による解説 日本でも自衛隊、消防、警察など救援従事者の長期の健康管理を考えるべき 竹石 恭知 福島県立医科大学医学部循環器内科学講座主任教授 MMJ.April 2020;16(2) 本論文は、米国同時多発テロ発生時にニューヨー ク市の世界貿易センタービル(WTC)崩壊現場で活動した消防士を対象に、災害現場での活動が長期の心血管疾患(CVD)発症と関連するかどうかを調査したコホート研究の報告である。対象者は男性消防士9,796人で、2001年9月11日時点の平均年齢は40.3歳、90%以上が非ヒスパニック系の白人、 約74%は非喫煙者であった。評価対象は2017年 12月31日までに発症したCVDであった。主要評価項目は 心筋梗塞、脳卒中、不安定狭心症、冠動脈バイパス術または冠動脈インターベンションの施行、CVD死とされた。本論文ではWTC曝露量を急性期とそれ以降の2種類の指標で解析している。 急性期曝露との関連については、WTCに9月11日 の午前中に到着した群、同日の午後に到着した群、12日以降に到着した群に分け、CVD発症率を比較した。また、WTCは2002年7月24日に閉鎖されたが、急性期以降のWTC曝露を評価するため、 WTC崩壊現場の活動期間が6カ月以上の群と6カ月未満の群で比較がなされた。 その結果、2017年12月までの16年間にわたる追跡期間中に主要評価項目のイベントが489件発生した(心筋梗塞120、脳卒中61、冠動脈バイパス術71、冠動脈インターベンション 236、うっ血性心不全1、CVD死6)。CVD発症リスクは、9月 12日以降の到着群と比較し、11日午前到着群では 1.44倍、11日午後到着群では1.24倍高かった。 また、WTC現場活動期間が6カ月以上の群では、6 カ月未満の群と比較し、CVD発症リスクが1.30倍高かった。一過性脳虚血発作、安定狭心症、大動脈瘤、末梢動脈疾患といったすべてのCVDを含めた解析でも、早期に現場に到着したほど、また現場活動期間が長いほど、CVD発症リスクが高かった。従来から知られているCVD危険因子である高血圧、脂質 異常、糖尿病、喫煙は、この対象群でも、CVD発症と有意に関連していた。 WTC崩壊現場の災害活動により深く関わった消防士では長期のCVD発症リスクが上昇することが明らかになった。この結果から、災害後の長期にわたる健康管理が重要であることが示された。日本でも被災者の支援に加えて、自衛隊、消防、警察など救援に従事された方々の健康管理を今後、考えるべきである。
福島市における東日本大震災および第一原子力発電所事故と出生率の関連性.
福島市における東日本大震災および第一原子力発電所事故と出生率の関連性.
Association of the Great East Japan Earthquake and the Daiichi Nuclear Disaster in Fukushima City, Japan, With Birth Rates JAMA Network Open 2019 ;2 (1):e187455 . 上記論文のアブストラクト日本語訳 ※ヒポクラ×マイナビ 論文検索(Bibgraph)による機械翻訳です。 【重要】2011年3月11日、12日に福島で発生した東日本大震災とその後の福島第一原発事故と出生率の関連は、既存の文献では適切に検討されていない。 【目的】東日本大震災と福島第一原発事故の出生率に対する中期および長期の関連性を評価することである。 【デザイン、設定および参加者】コホート研究において、中断時系列分析を用いて、2011年3月1日から2017年12月31日までの福島市の住民の出生率の月次変化を、震災前の傾向に基づく震災なしの出生率の予測と比較して評価した。2007年1月1日から2017年12月31日までの福島市の出生率は、福島市役所の情報を用いて求めた。 【Exposure】東日本大震災と福島第一原子力発電所事故、出生率との関連性の5つの潜在モデルを通じて表現した:水準変化、水準と傾斜変化、時間水準変化、1または2の傾斜変化(複数)を伴う時間水準変化。 【主な結果・指標】出生率、出生数と総人口の月次データから算出 【結果】東日本大震災および福島第一原発事故前の平均出生率は10万人あたり69.8人/月、事故後の平均出生率は10万人あたり61.9人/月であった。東日本大震災・福島第一原発事故前の出生率と比較すると、福島市では震災後2年間は月別出生率が10%減少したと推定される(率比、0.90;95%CI、0.86-0.93)。その後、出生率のトレンドは震災前のトレンドとほぼ同じであった。震災前のトレンドは出生率の継続的な減少を示唆した(1年間の出生率比,0.98;95%CI,0.98-0.99).このギャップモデルは,他のモデルと比較して最適かつ簡略化されたものであった. 【結論と関連性】東日本大震災と福島第一原子力発電所事故の後、2年間出生率が有意に低下した。震災後3~7年の動向が震災前の動向と異なることを示す根拠は十分ではなかった。出生率の低下からの回復は、復興に向けた努力を示しているのかもしれない。東日本大震災や福島第一原子力発電所事故以前に見られた長期的な出生率の低下が継続していることから、行政レベルでの出生計画支援策の継続が検討されるべきと考えられる。 第一人者の医師による解説 2013年以降は事故前の将来推計値まで回復するも長期的な下落傾向は継続 小坂理子(助教)/梅﨑昌裕(教授) 東京大学大学院医学系研究科国際保健学専攻人類生態学分野 MMJ.April 2020;16(2) 本論文は、2011年3月に発生した東日本大震災とそれに続く福島第1原発事故以降の福島県福島市の出生率を、福島市の公表する人口動態統計を用いて解析したコホート研究の報告であり、出生率は 11年3月から2年間は平均で約10%低下したものの、2013年には震災・事故前に算出された将来 推計の水準まで回復していたことが明らかになった。 震災・事故によって、出生率が低下することはこれまでにも指摘されている。繰り返しメディアで流される衝撃的な映像をみることでカップルの性行動が抑制される可能性、震災・事故に伴う心理的ストレスが受精卵の着床確率または胎児死亡率に影響する可能性などが報告されている。 本研究では2007~17年の11年間を対象に出生率を解析した結果、07年1月から震災・事故までの平均出生率は人口10万人あたり毎月69.8人であったのに対し、震災・事故後から2013年2月までの2年間は59.5人まで低下したこと、2013 年3月には震災前の傾向から推計される水準まで上昇し、2013年3月~17年12月 の平均出生率は62.9人であった。解析では、複数の仮定(震災・ 事故後ベースライン値も傾向も変化する、一時的にベースライン値のみ変化する、ベースライン値の変化は一時的だが傾向の変化は長期的であるなど) がモデル化され比較検討された。加えて、季節性の調整の効果も検討された。最も当てはまりの良いモデルは「季節性を調整した上で、一時的にベースライン値のみが変化し、長期的傾向は変化しない」 というものであった。 著者も述べているように、この解析の潜在的な問題は、福島市の出生登録数が同市の実際の出生数とどのくらい一致しているかがわからないことである。よく知られているように、震災・事故後、放射線の被曝を恐れた人々、特に小さな子どもをもつ世帯やこれから子どもを育てようとする世帯が 福島県から県外へ居住地を移した。そのような世帯の中には住民票を福島市に置いたまま、他県で出産したものもあったはずで、そのような出生も、本研究で計算された出生率には含まれている。一方、 出生率を計算するための分母となる人口について も、震災後の人口移動の影響を大きく受けている。 今後、個別の調査によって市外移住者の人口学的属 性の解析、市外移住者と非移住者の出産行動の比較 などが可能になれば、震災・事故と出生率の関係に ついての理解がさらに進むと考えられる。
ハリケーン・マリア後のプエルトリコの青少年における災害への暴露と精神的健康。
ハリケーン・マリア後のプエルトリコの青少年における災害への暴露と精神的健康。
Disaster Exposure and Mental Health Among Puerto Rican Youths After Hurricane Maria JAMA Network Open 2019 ;2 (4):e192619 . 上記論文のアブストラクト日本語訳 ※ヒポクラ×マイナビ 論文検索(Bibgraph)による機械翻訳です。 【重要性】青少年の災害曝露とトラウマ関連症状の大きさを定量化することは,資金不足の環境における心理サービスの展開に不可欠である。2017年9月20日にプエルトリコに上陸したハリケーン・マリアは,大規模な破壊と前例のない死亡率をもたらした。 【目的】ハリケーン・マリア後のプエルトリコの若者の災害曝露の大きさとメンタルヘルスアウトカムを明らかにする。 【デザイン・設定・参加者】2018年2月1日から6月29日(ハリケーン・マリアの5~9ヶ月後)にプエルトリコのすべての学校の公立学校の生徒一人ひとりに学校を中心とした調査を実施した調査研究であった。参加資格のある生徒226 808名のうち,96 108名が調査を完了した。 【主要アウトカム・測定法】参加者は,スペイン語で実施された標準化自己報告尺度を用いて,ハリケーン関連ストレス因子への曝露,心的外傷後ストレス障害(PTSD),うつ症状について評価された。すべての結果変数について記述統計が作成され、PTSDまたはうつ病の臨床的に高い症状を報告する個人の頻度も作成された。これらの統計量の男女間の差もt検定で検討した。 【結果】プエルトリコの7つの教育地域すべてにおいて、3年生から12年生までの代表的な9618名の生徒が研究に参加した(回答率42.4%、女性50.3%)。ハリケーンの結果、83.9%の青少年が家屋の被害を受け、57.8%が友人や家族が島を離れ、45.7%が自分の家に被害を受け、32.3%が食料や水の不足を経験し、29.9%が生命の危険を感じ、16.7%がハリケーンの5~9ヶ月後も電気がない状態であることが分かりました。全体として,青少年(n = 6900)の 7.2%が臨床的に重大な PTSD の症状を報告した.臨床的に上昇した PTSD の症状を報告する頻度を性別に比較すると,有意差があり(t = 12.77; 差の 95% CI, 0.018-0.025; P < .001),女子(8.2%)が臨床カットオフスコアを超える頻度は,男子(6.1%)に比べて高かった.最後に,うつ病の男女間の差についても同様の解析を行ったところ,有意であった(t = 17.56;差の95%CI,0.31-0.39;P < 0.001)。女子では男子よりも高い平均(SD)スコア(2.72[3.14])を示し(2.37[2.93]),男子では女子よりも高い平均(SD)スコアを示していた。人口統計学的変数とリスク変数は,PTSDの症状の分散の約20%を占めた(r2 = 0.195; 95% CI, 0.190-0.200)。 【結論と関連性】調査結果は,ハリケーン・マリアがプエルトリコの若者を高いレベルの災害関連ストレス因子に曝し,若者が高いレベルのPTSDとうつ症状を報告したことを示すものである。結果は現在,プエルトリコ教育省によって,プエルトリコの青少年の精神衛生上の成果を改善することを目的とした,的を絞った持続可能なエビデンスに基づく実践に活用されている。 第一人者の医師による解説 日本での災害時の多国籍児童メンタルヘルス支援体制の構築検討に寄与 島津 恵子 国立精神・神経医療研究センター精神保健研究所行動医学研究部 MMJ.April 2020;16(2) 2017年9月にプ エ ルトリコ 南東部に上陸したハリケーンマリアの被害は死者推定2,975 ~ 4,645人を数え、米国史上未曽有の規模となった。家族や学校など子どもにとって重要な社会的支援 システムの長期的な崩壊・混乱は、社会的弱者である子どもの一層の脆弱化を招く(1)。先行研究では自然災害から1年以内に集中的な介入なしに子どもの約半数は適応し回復する一方で、3分の1にのぼる子どもでは心的外傷後ストレス(PTSD)、うつ病、 不安、薬物使用、自殺念慮、攻撃的な行動などが認められ(2)、米国本土の青少年サンプルではこれらの 症状はマイノリティー間でより顕著であることが示された(3)。 プエルトリコ青少年の被災後のメンタルヘルス ニーズに対処するためのエビデンスに基づくシステムの開発・実装の促進を目的とし、本研究はプエルトリコ教育省が主導するスクリーニングの一環として実施された。ハリケーンカトリーナ後に作成されたNational Child Traumatic Stress Network Hurricane Assessment and Referral Tool(NCTSN-HART)をもとにスペイン語に翻訳された自記式質問票を用い、被災5~9カ月後にプエルトリコの全公立学校生徒のうち3~12年生を対象に1,086校(対象226,808人)で調査が実施された。 その結果、83.9%が家屋損壊の目撃、57.8%が友人や家族の離島、45.7%が自宅の被害を報告、32.3%が食料・水の不足を経験し、 29.9%が自らの生命の危険を認識し、16.7%は被災5~9カ月後も停電の継続を経験していた。全体の7.2%が臨床的に重要なPTSD症状を報告し、その頻度は女子の方が男子よりも有意に高かった (8.2% 対 6.1%)。うつ病スコアの平均も女子の方が男子よりも有意に高かった(2.72 対 2.37)。 人口統計およびリスク変数はPTSD症状の分散の約20%を占めた。 本研究の限界として、スペイン語版質問票の妥当性の欠如、PTSD・うつ症状のみへのフォーカス、 被災前データの欠如が挙げられているが、倫理的に災害時の研究実施は非常に困難であること、青少年、その中でもマイノリティーのメンタルヘルスの研究データは希少であること、また模索的政策研究としての性質から質問票の内容を参加者の言語・ 文化的・社会的背景に適切に配慮したうえで調整していること、災害時メンタルヘルス介入でエビデンスの豊富なトラウマ・うつに焦点をあてたことは妥当であり評価に値する。災害大国の日本で増加する外国語を話す多国籍・多文化児童を対象とした体系的災害時メンタルヘルス支援体制の構築の検討は必須であり、今後実施が予定されているさらなる解析結果の発表が期待される。 1. Bonanno GA et al. Psychol Sci Public Interest. 2010;11(1):1-49. 2. La Greca AM et al. Child Youth Care Forum. 2013;42(4):351-369. 他 3. Perilla JL et al. J Soc Clin Psychol. 2002; 21(1):20-45.
中年期から晩年期の血圧パターンと認知症発症との関連性。
中年期から晩年期の血圧パターンと認知症発症との関連性。
Association of Midlife to Late-Life Blood Pressure Patterns With Incident Dementia JAMA 2019 ;322 (6):535 -545. 上記論文のアブストラクト日本語訳 ※ヒポクラ×マイナビ 論文検索(Bibgraph)による機械翻訳です。 【重要】後期血圧と認知の関連は、過去の高血圧の有無と慢性度に依存する可能性がある。長期間の高血圧に続く後期の血圧低下は,認知機能の低下と関連する可能性がある。 【目的】中年期から後期の血圧パターンとその後の認知症,軽度認知障害,認知機能の低下との関連を検討する。 【デザイン、設定および参加者】Atherosclerosis Risk in Communities前向き集団ベースコホート研究では,中年期に4761人が登録され(訪問1,1987~1989),2016~2017年に6回の訪問でフォローアップした(訪問6,)。血圧は、訪問1~5回目(2011~2013年)の間に5回の対面訪問で24年間にわたり調査された。訪問5と6では、参加者は詳細な神経認知評価を受けた。舞台は米国の4つの地域である。メリーランド州ワシントン郡、ノースカロライナ州フォーサイス郡、ミシシッピ州ジャクソン、ミネソタ州ミネアポリス。フォローアップは2017年12月31日に終了。 【曝露】訪問1~5回目の正常血圧、高血圧(140/90mmHg以上)、低血圧(90/60mmHg未満)の縦断パターンに基づく5群。 主要アウトカムと 【測定】主要アウトカムは、Ascertain Dementia-8の情報提供者アンケート、6項目スクリーナーの電話評価、病院退院と死亡診断書コード、訪問6回の神経認知評価に基づいて、訪問5日後の認知症発症となった。副次的アウトカムは、神経認知評価に基づく訪問6日目の軽度認知障害であった。 【結果】参加者4761名(女性2821名[59%]、黒人979名[21%]、訪問5の平均[SD]年齢、75[5]歳、訪問1の平均年齢範囲、44~66歳、訪問5の平均年齢範囲、66~90歳)において、訪問5から6までの間に516(11%)が入認識症例であった。中年期の正常血圧(n=833)と晩年期の参加者の認知症発生率は、100人年当たり1.31(95%CI、1.00-1.72)、中年期の正常血圧と晩年期の高血圧(n=1559)は、100人年当たり1.99(95%CI、1.69-2.32)、中年期の晩年期の高血圧(n=1030)については、2.83(95%CI、2.69-2.72)であった。83(95%CI、100人年当たり2.40-3.35);中年正常血圧および後年低血圧(n = 927)、2.07(95%CI、100人年当たり1.68-2.54);および中年高血圧および後年低血圧(n = 389)、4.26(95%CI、100人年当たり3.40-5.32)であった。中年期および後期高血圧群(ハザード比[HR]、1.49[95%CI、1.06-2.08])および中年期高血圧および後期低血圧群(HR、1.62[95%CI、1.11-2.37])では正常血圧を維持した人々と比較してその後の認知症のリスクが有意に増加した。後期血圧に関係なく、中年期の持続性高血圧は認知症リスクと関連していた(HR、1.41[95%CI、1.17-1.71])。中年期と晩年期に正常血圧であった人と比較して,中年期の高血圧と晩年期の低血圧を有する参加者のみが,軽度認知障害(37人の罹患者)のリスクが高かった(オッズ比,1.65[95%CI,1.01-2.69])。BPパターンと晩年の認知機能変化との有意な関連は認められなかった。 【結論と関連性】長期追跡を行ったこの地域ベースのコホートでは,中年から晩年にかけての持続的高血圧と,中年および晩年の正常血圧と比較して中年高血圧および晩年低血圧のパターンは,その後の認知症のリスク上昇と関連していた。 第一人者の医師による解説 中年期から高齢期の血圧変動管理 認知症発症促進因子として重要 松村 美由起 東京女子医科大学附属成人医学センター脳神経内科講師 MMJ.April 2020;16(2) 認知症危険因子の1つとして高血圧が指摘されている。中年期における高血圧は、高齢期の認知症や認知機能低下の危険因子とされ、積極的治療が推進されているが、高齢期の血圧が認知機能に及ぼす影響および認知症発症との関連について十分な エビデンスは確立されていない。本論文は、中年期から高齢期にかけての血圧値の経年的変化と高齢期の認知機能低下および認知症発症との関連を検討した米国の地域住民対象コホート研究の報告で ある。 平均年齢44~66歳(平均54歳)の住民15,792 人に対して24年間にわたり血圧測定と認知機能検査を実施した。高血圧は140/90 mmHg超、低血圧は90/60 mmHg未満と定義し、認知機能は包括的心理バッテリーにより記憶、処理速度、実行機能、言語機能を加えて情報提供者へのインタビュー により評価した。1987~89年を初回調査とし、以降3年ごとに4回の血圧を中年期、その15年後とさらにその3年後の2回の調査を高齢期の血圧 とした。 中年期と高齢期の血圧を①中年期から高齢期まで正常血圧②中年期正常血圧、高齢期高血圧③ 中年期から高齢期まで高血圧④中年期正常血圧、高齢期低血圧⑤中年期高血圧、高齢期低血圧の5つのパターンに分類した。最終的に516人(11%)が 認知症を発症した。100人・年あたりの認知症発症率は、①は1.31、②は1.99、③は2.83、④は 2.07、⑤は4.26であった。③のハザード比は1.49、 ⑤は1.62であり、中年期から高齢期まで正常血圧を維持した群に比べ、中年期・高齢期とも高血圧の群と中年期高血圧・高齢期低血圧の群において認知症発症リスクが有意に高まっていた。 慢性的な血圧高値は脳循環自動調節能の異常や脳の小血管障害などを生じることが知られており、本研究の結果から中年期高血圧に伴うこうした脳の異常が認知症リスクを高めた可能性が推察される。一方、高齢期の血圧低下は、心循環機能障害や自律神経シグナル伝達の異常、動脈硬化によると推測されるが、高齢者は降圧薬を服用している割合も高く、晩年期低血圧は降圧薬の過剰投与によりもたらされた可能性も考えられる。脳循環自動調節能の障害がある場合、全身の血圧低下が脳血流低下を来しやすく、結果として認知機能低下に至った可能性もある。 本研究では、中年期高血圧で認知症発症例の脱落数が多く、病型診断との関連性が評価されていないなどの課題が残り、今後さらなる検証が必要である。
化学療法誘発性心筋症患者における心臓再同期療法と左室駆出率の変化との関連性。
化学療法誘発性心筋症患者における心臓再同期療法と左室駆出率の変化との関連性。
Association of Cardiac Resynchronization Therapy With Change in Left Ventricular Ejection Fraction in Patients With Chemotherapy-Induced Cardiomyopathy JAMA 2019 ;322 (18):1799 -1805. 上記論文のアブストラクト日本語訳 ※ヒポクラ×マイナビ 論文検索(Bibgraph)による機械翻訳です。 【重要】化学療法誘発性心筋症の発生率は増加しており、臨床転帰不良と関連している。 【目的】化学療法誘発性心筋症患者における心臓再同期療法(CRT)と心機能改善、および臨床改善との関連性を評価すること。 【デザイン、設定および参加者】Multicenter Automatic Defibrillator Implantation Trial-Chemotherapy-Induced Cardiomyopathyは、米国内の心臓腫瘍学プログラムを有する12の三次センターで2014年11月21日から2018年6月21日の間に実施した非対照・前向き・コホートスタディである。左室駆出率低下(LVEF≦35%)、New York Heart AssociationクラスII-IV心不全症状、広QRS複合体のため、化学療法による心筋症が確立した患者30名をCRT植え込み、CRT植え込み後6ヶ月間フォローアップを行った。最終フォローアップ日は2019年2月6日。 【曝露】標準治療によるCRT植え込み。 【主要アウトカムと測定】主要エンドポイントはCRT開始後のベースラインから6ヶ月後までのLVEFの変化とした。副次的評価項目は全死亡,左室収縮末期容積と拡張末期容積の変化とした。 【結果】登録された30例(平均[SD]年齢,64[11]歳,女性26例[87%],73%に乳癌歴,20%にリンパ腫または白血病歴)において,26例で一次エンドポイントのデータが,23例で二次エンドポイントのデータが利用可能であった。患者は左脚ブロックのある非虚血性心筋症で、LVEF中央値は29%、平均QRS時間は152msであった。CRTを行った患者では、6ヵ月後の平均LVEFが28%から39%に統計的に有意に改善した(差、10.6% [95% CI, 8.0%-13.3%]; P < .001)。これには,LV 収縮末期容積の 122.7 から 89.0 mL への減少(差 37.0 mL [95% CI,28.2-45.8]),LV 拡張末期容積の 171.0 から 143.2 mL への減少(差 31.9 mL [95% CI,22.1-41.6]) が伴った(いずれも P<.001 ).有害事象は、処置に関連した気胸(1例)、装置ポケットの感染(1例)、およびフォローアップ中に入院を必要とした心不全(1例)であった。 【結論と関連性】化学療法による心筋症の患者を対象としたこの予備的研究では、CRTは6ヵ月後のLVEFの改善と関連していた。この知見は、サンプルサイズが小さいこと、フォローアップ期間が短いこと、対照群を設定していないことにより制限される。 【臨床試験登録】ClinicalTrials. gov Identifier:NCT02164721 第一人者の医師による解説 がん患者へのCRT導入 有望だが原疾患の予後も勘案する必要 諏訪 惠信(助教)/塩島 一朗(教授) 関西医科大学第二内科学講座 MMJ.April 2020;16(2) 抗がん剤の進歩によって、がん患者の長期生存が可能となった。しかし、その中にはアントラサイクリン系薬剤に代表される心毒性を有する抗がん剤によって心不全に至った化学療法関連心筋症の患者が含まれる。アントラサイクリン系薬剤は用量依存的に心毒性を発現させることが知られているが、その有効性から現在も頻用されている。抗がん剤の影響で左脚ブロックを合併した化学療法関連心筋症患者に対する治療を検討した報告は少な い。特に心臓再同期療法(CRT)を用いた多施設前向きコホート研究の報告は本論文が初めてとなる。 組み入れ基準は、2014年11月~18年6月に 米国の腫瘍循環器内科を有する12施設で化学療法を受けた18~80歳の患者で、化学療法によって CRTの適応クラス 1または2に至った患者である。 患者は、化学療法前に心機能障害がないこと、かつがん治療終了後少なくとも6カ月間に収縮機能障 害を伴う臨床的心不全の発症がないことが確認されている。CRT導入から6カ月後に心臓超音波検査が行われ、心尖部2腔像と4腔像のSimpson法 で左室容積と左室駆出率が計測された。主要評価項目は6カ月後の左室駆出率の変化、副次評価項目は 全死亡および左室容積の変化とされた。NYHA心機能分類の変化や左房サイズの変化も検討された。 登録患者は30人( 平均年齢61歳、女性87 %) であった。原疾患は乳がん73%、リンパ腫または白血病20%、肉腫7%であった。83%の患者 にアントラサイクリン系薬剤が投与された(平 均投与量307 mg/m2)。心不全重症度は、NYHA II 57%、NYHA III 43%であった。登録時の薬物 療法は、β遮断薬93%、アンジオテンシン 変換酵素(ACE)阻害薬77%、ループ 利尿薬93%であった。CRT導入から6カ月後に左室駆出率の平均は28から39%に有意に改善した。左室収縮末期容積(122.7→89.0mL)および拡張末期容積 (171.0→143.2mL)も有意に改善した。死亡例はなく、6カ月後に左房容積は60.3から47.9ml に改善し、NYHA II 患者の19%、III患者の69%で改善が得られた。小規模な研究であるが、化学療法関連心筋症に対するCRTの有望性を示しており、さらなる研究が期待される。 日本循環器学会の「不整脈非薬物治療ガイドライ ン」(1)によると、1年以上の余命が期待できない患者へのCRTは推奨クラス 3の適応となっており、化学療法関連心筋症患者に導入する場合は心不全のみならず原疾患の予後も勘案して治療を提案する必要がある。 1. 日本循環器学会 / 日本不整脈心電学会合同ガイドライン:不整脈非薬物治 療ガイドライン (2018 年改訂版 ) URL:https://bit.ly/3bXESEE
慢性腎臓病患者におけるBaclofenと脳症の関連性。
慢性腎臓病患者におけるBaclofenと脳症の関連性。
Association of Baclofen With Encephalopathy in Patients With Chronic Kidney Disease JAMA 2019 ;322 (20):1987 -1995. 上記論文のアブストラクト日本語訳 ※ヒポクラ×マイナビ 論文検索(Bibgraph)による機械翻訳です。 【重要】少なくとも30の症例報告が、慢性腎臓病(CKD)患者における筋弛緩薬バクロフェンと脳症の関連性を示している。 【目的】CKD患者で、バクロフェンを1日20mg以上と1日20mg未満で新規処方した場合の30日間の脳症リスクを比較することである。副次的目的は,バクロフェン使用者と非使用者の脳症リスクを比較することであった。 デザイン・設定・参加者】カナダ・オンタリオ州(2007~2018年)における,リンクした医療データを用いたレトロスペクティブな人口ベースコホート研究であった。参加者は,CKD(推定糸球体濾過量[eGFR]<60 mL/min/1.73 m2で透析を受けていないと定義)を有する高齢者(66歳以上)15 942名であった。一次コホートは、バクロフェンを新たに処方された患者に限定し、二次コホートの参加者は新規使用者と非使用者とした。 【 暴露】経口バクロフェン20mg/日以上 vs 20mg/日未満の処方。 【主要アウトカムと測定】バクロフェン開始後30日以内にせん妄、意識障害、一過性意識変化、一過性脳虚血発作、特定できない認知症と主病名を定義した脳症の入退院。ベースラインの健康状態の指標で比較群のバランスをとるために、傾向性スコアに治療の逆確率加重を用いた。加重リスク比(RR)は修正ポアソン回帰で、加重リスク差(RD)は二項回帰で求めた。事前に規定したサブグループ解析をeGFRカテゴリー別に実施した。 【結果】主要コホートはCKD患者15 942例(女性9699例[61%],年齢中央値77歳[四分位範囲71~82],バクロフェン開始量20 mg/日以上9707例[61%],<20 mg/日6235例[39%])より構成された。主要転帰である脳症による入院は,バクロフェンを 1 日 20 mg 以上で開始した患者 108/9707 例(1.11%),バクロフェンを 1 日 20 mg 未満で開始した患者 26/6235 例(0.42%)で発生した;重み付け RR,3.54(95% CI,2.24~5.59),重み付け RD,0.80%(95% CI,0.55%~1.04%).サブグループ解析では,絶対リスクはeGFRが低いほど徐々に増加した(重み付けRD eGFR 45~59,0.42%[95% CI,0.19%-0.64%];eGFR 30~44,1.23%[95% CI,0.62%-1.84%];eGFR <30,2.90%[95% CI,1.30%-4.49%],P for interaction,<.001]).非使用者284 263人との二次比較では、バクロフェン使用者の両群で脳症のリスクが高かった(<20 mg/d加重RR, 5.90 [95% CI, 3.59 to 9.70] および≥20 mg/d加重RR, 19.8 [95% CI, 14.0 to 28.0] )。 【結論と関連性】バクロフェンを新規処方されたCKD高齢者において、30日の脳症発生率は低用量と比べ高用量処方者において増加した。検証された場合、これらのリスクはバクロフェン使用の利益と釣り合うものでなければならない。 第一人者の医師による解説 腎排泄型のバクロフェン 脳症発症機序は不明だが臨床上重要な問題を提起 中嶋 秀人 日本大学医学部内科学系神経内科学分野教授 MMJ.April 2020;16(2) バクロフェンは中枢作用型γ -アミノ酪酸受容体 アゴニストであり、筋弛緩薬として脳血管障害、変性疾患、脊椎疾患などによる痙縮の治療に用いられるほか、三叉神経痛や胃食道逆流症にも使用されることがある。またアルコール依存症においては腹側線条体で上昇しているドパミンをバクロフェンが抑制的に調節すると考えられ、その治療効果を示唆する報告もある。 他の多くの筋弛緩薬が肝代謝型であるのに対して、バクロフェンは腎排泄 型のため腎機能低下に伴い排泄半減期が延長する。これまでバクロフェンの使用により脳症を発症した慢性腎臓病(CKD)症例が報告されていることより、腎機能の低下する高齢者においてバクロフェン関連脳症のリスクが上昇することが危惧される。 本論文は、カナダ・オンタリオ州の患者情報を登録したデータベースを利用し、新規にバクロフェンを 処方した66歳以上 のCKD(推算糸球体濾過 量[eGFR] 60 mL/分 /1.73 m2未満であるが透析を受けていないものと定義)患者15,942人を対象に、バクロフェン投与量20mg/日以上群と 20mg/日未満群に分け、バクロフェン投与開始から30日以内の脳症入院リスクを比較した後ろ向きコホート研究の報告である。なお、脳症は、せん妄、見当識障害、一過性の意識の変化、一過性脳虚 血発作、分類不能な認知症の診断として規定された。 その結果、主要評価項目であるバクロフェン開始後 30日以内の脳症による入院は20mg/日以上群で 1.11%、20mg/日未満群では0.42%に発生し、高用量群で脳症入院リスクが上昇した(重み付けリスク比[RR], 3.54[95% CI, 2.24~5.59];重み付けリスク差[RD], 0.80%[0.55~1.04%])。 また、バクロフェン開始から入院までの期間の中央値 は、20mg/日以上群では3日間( 四分位範囲 [IQR], 2~5)、20mg/日未満群 では8日間(3 ~12)であった。さらに、バクロフェン非使用者 284,263人の脳症発症率は0.06%であり、バクロフェン 20mg/日未満群、20mg/日以上群とも脳症入院リスクが高かった(重み付けRRはそれぞれ5.90[95 % CI, 3.59~9.70]、19.8[14.0 ~28.0])。 バクロフェンには眠気、めまい、ふらつきなどの副作用があるが、過剰な血中バクロフェンが脳症を起こす機序については不明な点も多い。しかし、新規にバクロフェンが開始された高齢 CKD患者において、低用量群に比べて高用量群では、より高頻度かつより早期に脳症が発症することから、臨床上重要な問題を提起しており、興味深い研究結果と考えられる。
敗血症および重症急性呼吸不全患者におけるビタミンC輸液の臓器不全および炎症・血管傷害のバイオマーカーに対する効果。CITRIS-ALI Randomized Clinical Trial(CITRIS-ALI無作為化臨床試験).
敗血症および重症急性呼吸不全患者におけるビタミンC輸液の臓器不全および炎症・血管傷害のバイオマーカーに対する効果。CITRIS-ALI Randomized Clinical Trial(CITRIS-ALI無作為化臨床試験).
Effect of Vitamin C Infusion on Organ Failure and Biomarkers of Inflammation and Vascular Injury in Patients With Sepsis and Severe Acute Respiratory Failure: The CITRIS-ALI Randomized Clinical Trial JAMA 2019 Oct 1;322(13):1261-1270. 上記論文のアブストラクト日本語訳 ※ヒポクラ×マイナビ 論文検索(Bibgraph)による機械翻訳です。 【重要性】ビタミンCの静脈内投与は、敗血症や急性呼吸窮迫症候群(ARDS)に伴う炎症と血管障害を抑制することが実験的に示唆されている。 【目的】敗血症およびARDS患者におけるビタミンC静脈内投与の臓器不全スコアおよび炎症と血管障害の生体マーカーに対する効果を明らかにすることである。 【デザイン・設定・参加者】CITRIS-ALI試験は、米国内の医療集中治療室7施設で実施した無作為化二重盲検プラセボ対照多施設試験で、24時間以内に発症した敗血症およびARDSの患者(N=167)が登録された。試験実施期間は2014年9月から2017年11月、最終フォローアップは2018年1月。 【介入】患者をビタミンCの点滴静注(ブドウ糖5%水煮、N=84)またはプラセボ(ブドウ糖5%水煮のみ、N=83)に6時間おきに96時間無作為に割り付けました。 【主要評価項目】ベースラインから96時間後までのmodified Sequential Organ Failure Assessment score(範囲:0~20、スコアが高いほど機能障害が強い)により評価した臓器障害の変化、および0、48、96、168時間で測定した炎症(CRP値)と血管損傷(トロンボモデュリン値)の血漿バイオマーカーであった。 【結果】無作為化された167例(平均[SD]年齢:54.8歳[16.7],男性90例[54%])中,103例(62%)が60日目まで試験を完了した。主要評価項目であるベースラインから96時間後までの平均modified Sequential Organ Failure Assessment scoreの変化(ビタミンC群9.8から6.8[3点]、プラセボ群10.3から6.8[3.5点];差、-0.10;95%CI、-1.23から1.03;P = .86 )またはCRP値(54.1対46.1μg/ml;差、7.94。 【結論と関連性】敗血症とARDSの患者を対象としたこの予備的研究では、プラセボと比較したビタミンCの96時間点滴は、臓器機能障害のスコアまたは炎症と血管損傷のマーカーに有意な改善をもたらさなかった。敗血症とARDSの他の転帰に対するビタミンCの潜在的な役割を評価するために、さらなる研究が必要である。 【臨床試験登録】ClinicalTrials. gov Identifier.NCT02106975:NCT02106975。 第一人者の医師による解説 全死亡率には有意な改善効果 今後の研究継続を期待 射場 敏明 順天堂大学大学院医学研究科救急・災害医学教授 MMJ.April 2020;16(2) セプシスでは以前からビタミン C欠乏がみられることが知られており、またビタミン Cは抗炎 症作用や血管内皮保護作用を有することが報告されている(1)。そこで著者らは、急性呼吸窮迫症候群 (ARDS)合併セプシス患者に対するビタミン C静 脈投与の有用性をCITRIS-ALI試験で検討し、その結果を報告した。同試験は米国の7つの集中治療室で実施された無作為化二重盲検プラセボ対照試験である。167人のARDS合併セプシス患者が登録され、介入群と対照群でそれぞれ6時間ごとに96 時間ビタミン C(50 mg/kg;n=84)もしくはプラセボ(n= 83)が静脈内投与された。主要評価項目として、治療開始から96時間までのSequential Organ Failure Assessment(SOFA)スコアの改善、168時間までの炎症マーカー(CRP)の変動、 および血管障害マーカー(可溶性トロンボモジュリン)の変化が設定された。 その結果、SOFAスコアの改善に関して両群間に有意差はみられなかった(ビタミンC群:9.8→6.8; プラセボ群:10.3→6.8;変化の差 , -0.10)。 さらにCRP値(54.1 対 46.1μ g/mL;差 , 7.94 μ g/mL)、可溶性トロンボモジュリン値(14.5 対 13.8 ng/mL;差 , 0.69 ng/mL)についても有意差はなかった。したがって、ARDS合併セプシス患者においてビタミン Cの96時間注入による臓器障害、炎症反応、血管障害の改善効果は確認できなかった、と結論された。しかし、副次評価項目の全死亡率について統計学的に有意な改善効果が認められていることから(ハザード比 , 0.55;P= 0.01)、セプシスに対するビタミン Cの効果についてはさらなる検討が必要と考えられる。 CITRIS-ALI試験のベースは単施設後ろ向き研究におけるビタミン Cの劇的な予後改善効果である (オッズ比 , 0.13)(2)。この先行研究では院内死亡の改善以外にも昇圧薬使用期間の短縮やSOFAの改善が示されており、期待値が高まっていた。 本試験でやや奇異に感じられたのは、主要評価項目をセプシスの研究で伝統的に設定されてきた全死亡率の改善ではなく、SOFAスコアの改善、 CRPや可溶性トロンボモジュリンの低下としたことである。これは全死亡率の改善に関わる要因は多岐にわたるため、その達成が困難であると予想されることから、より直接的な指標を選択した結果であろうと考えられる。しかし結果としてそのことが裏目に出てしまったことについては、ただただ臨床試験デザインの難しさを感じる。今回の研究では目標は達成されなかったが、ビタミン C は安価で重篤な副作用もないと予想されることから、今後も研究が継続されることが期待される。 1. Wilson JXe et al. Subcell Biochem. 2012;56:67-83. 2. Marik PE et al. Chest. 2017;151(6):1229-1238. ビタミンC静注は急性呼吸窮迫症候群  合併セプシスの臓器障害を改善せず
急性骨髄性白血病の小児および若年成人患者における侵襲性真菌症に対するカスポファンギンとフルコナゾールの予防投与効果。無作為化臨床試験。
急性骨髄性白血病の小児および若年成人患者における侵襲性真菌症に対するカスポファンギンとフルコナゾールの予防投与効果。無作為化臨床試験。
Effect of Caspofungin vs Fluconazole Prophylaxis on Invasive Fungal Disease Among Children and Young Adults With Acute Myeloid Leukemia: A Randomized Clinical Trial JAMA 2019 Nov 5;322(17):1673-1681. 上記論文のアブストラクト日本語訳 ※ヒポクラ×マイナビ 論文検索(Bibgraph)による機械翻訳です。 【重要性】急性骨髄性白血病の小児、青年、若年成人は、酵母とカビの両方による生命を脅かす侵襲性真菌症のリスクが高い。 【目的】急性骨髄性白血病化学療法後の好中球減少時の侵襲性真菌症および侵襲性アスペルギルス症に対してカスフォンギンとフルコナゾールの予防の有効性を比較することである。 【デザイン、設定および参加者】この多施設共同無作為化オープンラベル臨床試験は、米国およびカナダの115施設で治療を受けている新規診断のde novo、再発、二次性急性骨髄性白血病の3カ月から30歳の患者を登録した(2011年4月から2016年11月、最終フォローアップ2018年6月30日) 【介入】参加者を最初の化学療法サイクルでカスフォンギンを用いた予防(n=257)またはフルコナゾール(n=260)にランダムに割り付けた。 【主要アウトカムおよび測定法】主要アウトカムは、盲検中央判定による侵襲性真菌症の証明または可能性の判定であった。副次的アウトカムは、侵襲性アスペルギルス症、経験的抗真菌療法、および全生存とした。 【結果】2回目の中間有効性解析と394人の患者に基づく予定外の無益性解析で無益性が示唆されたため、試験は登録が締め切られた。無作為化された517名(年齢中央値9歳[範囲:0~26歳],女性44%)のうち,508名(98%)が試験を完遂した。23 件の証明または可能性のある侵襲性真菌症イベント(カスポファンギン 6 件対フルコナゾール 17 件)には、カビ 14 件、酵母 7 件、さらに分類されない真菌 2 件が含まれていた。5ヵ月間の侵襲性真菌症の累積発生率は,カスポファンギン群3.1%(95% CI, 1.3%-7.0%) vs フルコナゾール群7.2%(95% CI, 4.4%-11.8%)(overall P = .03)であった(logank検定).また、証明または可能性のある侵襲性アスペルギルス症の累積発生率は、カスポファンギン群で0.5%(95%CI、0.1%-3.5%)、フルコナゾール群で3.1%(95%CI、1.4%-6.9%)だった(ログランクテストによる全P = 0.046)。経験的抗真菌療法(カスポファンギン 71.9% vs フルコナゾール 69.5%、全体 P = 0.78、log-rank 検定)および 2 年全生存率(カスポファンギン 68.8% vs フルコナゾール 70.8%、全体 P = 0.66 log-rank 検定)は統計的に有意差を認めないこととなりました。最も一般的な毒性は、低カリウム血症(カスポファンギン22 vs フルコナゾール13)、呼吸不全(カスポファンギン6 vs フルコナゾール9)、アラニントランスアミナーゼ上昇(カスポファンギン4 vs フルコナゾール8)だった。 【結論と関連性】急性骨髄性白血病の小児、青年、若年成人において、フルコナゾールと比較してカスポファンギンで予防を行うと侵入真菌症発生率は著しく低下する結果となった。本結果は、カスポファンギンが侵襲性真菌症に対する予防薬として考慮される可能性を示唆しているが、無益性を示唆すると思われる予定外の中間解析による早期終了により、研究の解釈には限界がある。 【試験登録】 ClinicalTrials. gov Identifier:NCT01307579. 第一人者の医師による解説 至適予防法検証には 抗糸状菌活性のあるアゾール系とキャンディン系の比較必要 宮入 烈 国立成育医療研究センター感染症科診療部長 MMJ.April 2020;16(2) 深在性真菌症は小児の急性骨髄性白血病(AML) 患者の予後を左右する重大な合併症である。成人AMLでは、ポサコナゾールの予防的投与が欧米のガイドラインで推奨されている。一方、小児 AML ではいまだに抗糸状菌活性の乏しいフルコナゾールが推奨されている。本論文は、糸状菌および酵母に活性が期待できるカスポファンギンの予防効果をフルコナゾールと比較するため、米国を中心に実施された多施設共同ランダム化比較試験の報告 である。最終的に508人が主解析の対象となり、カスポファンギン群における深在性真菌症の5カ月累積発生率は3.1%でフルコナゾール群の7.2% と有意差が示され、アスペルギルス感染症確定例の発生も有意に少なかった。より有効な選択肢のエビデンス構築にかかわる重要な知見と言えるが、考察すべきポイントが2点挙げられる。 従来、フルコナゾールはカンジダなど酵母による感染症の治療や予防には有効であるが、予後を左右するアスペルギルス症など糸状菌には無効であり問題視されていた。抗糸状菌活性があるカスポファンギンがより有効であるという今回の結果は想定内であったが、真菌症予防としてこの2剤の比較が最適であったかというと疑念が残る。内服可能で抗アスペルギルス活性のあるボリコナゾールなどのアゾール系薬剤との比較が今後の課題と思われる。 著者らは、カスポファンギン投与下の深在性真菌症発生率は成人におけるポサコナゾール 経口投与と同等であり、ポサコナゾールによる有害事象発生率の高さに言及しているが、カスポファンギンの点滴静注投与による不利益については触れていない。 本試験は進行中に独立データモニタリング委員会の指摘により臨時の解析が行われた。そこで、無益性の判定により早期中止となったものの、最終解析で有意差が認められたことは特筆すべきである。大規模なランダム化比較試験は、被験者への負担を伴い、多大な労力と資金がつぎ込まれることから、早期中止により効率化が期待される。 その一方で、今回のように有益な検討が早期に中止される可能性もあることは従来から指摘されている。今回、中間解析と最終解析で結果が一致しなかった理由として、中間解析では侵襲性真菌感染症のない患者の解析が早い段階で行われたため、発生率が低くみつもられたこと、中間解析が行われている間も患者登録が続けられ最終解析に114人が追加されたことが挙げられている。今後の中間解析の在り方について示唆を含む検討といえる。
重症小児患者における多臓器不全症候群に対する新鮮赤血球輸血と標準発行赤血球輸血の効果。無作為化臨床試験
重症小児患者における多臓器不全症候群に対する新鮮赤血球輸血と標準発行赤血球輸血の効果。無作為化臨床試験
Effect of Fresh vs Standard-issue Red Blood Cell Transfusions on Multiple Organ Dysfunction Syndrome in Critically Ill Pediatric Patients: A Randomized Clinical Trial JAMA 2019 Dec 10;322(22):2179-2190 上記論文のアブストラクト日本語訳 ※ヒポクラ×マイナビ 論文検索(Bibgraph)による機械翻訳です。 【重要性】重症小児患者に対する赤血球保存年齢の臨床的影響は、大規模な無作為化臨床試験で検討されていない。 【目的】重症小児において、新鮮赤血球(7日以下保存)の輸血が、標準発行赤血球の使用と比較して新規または進行性の多臓器不全症候群を減らすかどうかを判断することである。 【デザイン・設定・参加者】The Age of Transfused Blood in Critically-Ill Children試験は、2014年2月から2018年11月にかけて50の三次医療施設で行われた国際多施設共同盲検化無作為化臨床試験であった。集中治療室入室後7日以内に最初の赤血球輸血が行われた、生後3日~16歳の小児患者を対象とした。合計15568人の患者をスクリーニングし、13308人を除外した。 【介入】患者を新鮮赤血球または標準発行赤血球のいずれかを投与するよう無作為に割り付けた。合計1538人の患者が無作為化され、新鮮赤血球群768人、標準発行群770人となった。 法]主要アウトカム指標は、新規または進行性の多臓器不全症候群で、28日間または退院または死亡まで測定された。 【結果】無作為化された1538名のうち、1461名(95%)が一次解析に含まれ(年齢中央値1.8歳、女子47.3%)、そのうち新鮮赤血球群に728名、標準発行群に733名が無作為に割り付けられた。保存期間の中央値は,新鮮群 5 日(四分位範囲 [IQR], 4~6 日)に対して標準発行群 18 日(IQR, 12~25 日)であった(P < 0.001).新規または進行性の多臓器不全症候群については,新鮮赤血球群(728 例中 147 例[20.2%])と標準発行赤血球群(732 例中 133 例[18.2%])に有意差はなく,未調整絶対リスク差は 2.0%(95% CI,-2.0%~6.1%;P = .33)であった.敗血症の有病率は,新鮮群では 25.8%(619 例中 160 例),標準発行群では 25.3%(608 例中 154 例)であった.急性呼吸窮迫症候群の有病率は,新鮮群では 6.6%(619 例中 41 例),標準発行群では 4.8%(608 例中 29 例)であった.集中治療室での死亡率は新鮮群4.5%(728例中33例)に対して標準発行群3.5%(732例中26例)だった(P = .34)。 【結論と関連性】重症小児患者において、新鮮赤血球の使用は標準発行赤血球と比較して新規または進行性の多臓器不全症候群(死亡率を含む)の発生率を低減しなかった。 【試験登録】 ClinicalTrials. gov Identifier:NCT01977547。 第一人者の医師による解説 血液製剤の保存期間 どの年齢でも予後悪化の因子でない可能性 寺田 類/岡崎 仁(教授) 東京大学医学部附属病院輸血部 MMJ.April 2020;16(2) 赤血球製剤の保存期間が長くなると、製剤中の赤血球のviabilityは低下し、赤血球の重要な役割である酸素運搬能が低下してくる。保存期間の長い赤血球製剤を重症患者に輸血すると臓器不全の発生率や死亡率が上昇するのではないかと危惧され、今までに多くの観察研究や無作為化試験が行われてきた。しかし、赤血球の保存期間と死亡率や多臓器 機能障害スコアの変化に有意な関係性は見いだされていない。 しかし、こうした研究のほとんどは成人を対象としたもので、新生児や心臓外科手術患者など重症小児患者では保存期間による違いが予後に影響するかどうかは不明である。また、重症小児患者に対してどの程度の保存期間の赤血球製剤を使用するかは、病院や血液製剤センターの方針などにより一律ではないのが現状である(1)。 そこで本論文で報告された多施設共同無作為化試験では、重症小児患者において、血液製剤の保存期間が死亡率や患者予後と強く相関する多臓器不全の新規発症・増悪に与える影響を調べている。対象は、米国、カナダ、フランスなど50施設の小児集中治療室(PICU)で輸血を受けた生後3日から16歳までの患者で、保存期間7日以内(中央値 , 5日)の赤血球を輸血した群728人と、標準的な 保存期間(中央値 , 18日)の赤血球を輸血した群 733人の転帰が比較された。 その結果、主要評価項目である多臓器不全の増悪に加え、副次評価項目の28、90日死亡率、敗血症や急性呼吸促迫症候群(ARDS)、院内感染の発生率に関して保存期間の違いによる有意差は認められなかった。また、年齢、施設、国、性別、合併症、疾患の重症度で調整した解析でも結果は変わらなかった。 日本における赤血球製剤の使用期限は採血後21 日で、世界的な使用期限42日の半分であり、元来保存期間の短い製剤が供給されている。しかし、献血者の減少や少子高齢化による血液製剤の不足が懸念されている昨今においては、いつでも重症小児患者に優先的に保存期間の短い新鮮な赤血球製剤を供給できるかは必ずしも定かではない。 今回の結果が日本人にも同様に当てはめられるのか、また死亡などの重要な因子以外に与える影響についての検討も今後必要ではあるが、血液製剤を運用する病院や血液センター、また治療を受ける側としても意味のある1つの結果と言えるだろう。 どの年齢においても、現状での保存期間による製剤の変化は臨床上、患者の予後に影響を与えるまでの変化ではないのかもしれない。 1. Spinella PC et al. Transfusion. 2010;50(11):2328-2335.
片頭痛の急性期治療における痛みと最も煩わしい関連症状に対するウブロゲパントとプラセボの効果。ACHIEVE II Randomized Clinical Trial(無作為化臨床試験)。
片頭痛の急性期治療における痛みと最も煩わしい関連症状に対するウブロゲパントとプラセボの効果。ACHIEVE II Randomized Clinical Trial(無作為化臨床試験)。
Effect of Ubrogepant vs Placebo on Pain and the Most Bothersome Associated Symptom in the Acute Treatment of Migraine: The ACHIEVE II Randomized Clinical Trial JAMA 2019 Nov 19;322(19):1887-1898. 上記論文のアブストラクト日本語訳 ※ヒポクラ×マイナビ 論文検索(Bibgraph)による機械翻訳です。 【重要性】Ubrogepantは、片頭痛の急性期治療薬として検討されている経口カルシトニン遺伝子関連ペプチド受容体拮抗薬である。 【目的】片頭痛発作1回の急性期治療におけるubrogepantの有効性と忍容性をプラセボと比較して評価することである。 【デザイン・設定・参加者】米国で実施した第3相多施設共同無作為化二重盲検プラセボ対照単発臨床試験(ACHIEVE II)(プライマリーケアおよび研究クリニック99施設、2016年8月26日~2018年2月26日)。参加者は、月に2~8回の片頭痛発作を経験している前兆のある片頭痛または前兆のない片頭痛の成人である。 【介入】中等度または重度の疼痛強度の片頭痛発作に対してウブロゲパント50mg(n=562)、ウブロゲパント25mg(n=561)またはプラセボ(n=563)である。【主要評価項目及び測定 【方法】有効性の主要評価項目は、服用後2時間における疼痛緩和及び参加者が指定した最も煩わしい片頭痛関連症状(羞明、幻覚、吐き気のうち)の消失とした。 【結果】無作為化参加者1686名のうち1465名が試験治療を受け(安全集団、平均年齢41.5歳、女性90%)、1465名のうち1355名が有効性として評価可能であった(92.5%)。2時間後の痛みの消失は、ウブロゲパント50mg群では464人中101人(21.8%)、ウブロゲパント25mg群では435人中90人(20.7%)、プラセボ群では456人中65人(14.3%)に認められました(50mg vs プラセボの絶対差、7.5%、95%CI, 2.6%-12.5%; P = .01; 25mg vs プラセボ, 6.4%; 95%CI、 1.5%-11.5%; P = .03)。2時間後に最も煩わしい関連症状がなかったと報告されたのは、ウブロゲパント50mg群463人中180人(38.9%)、ウブロゲパント25mg群434人中148人(34.1%)、そしてプラセボ群456人中125人(27.4%)であった。4%)であった(50mg対プラセボの絶対差、11.5%;95%CI、5.4-17.5%;P = .01;25mg対プラセボ、6.7%;95%CI、0.6-12.7%;P = .07)。いずれかの投与後48時間以内に最も多く見られた有害事象は、吐き気(50 mg、488人中10人[2.0%];25 mg、478人中12人[2.5%];およびプラセボ、499人中10人[2.0%])およびめまい(50 mg、488人中7人[1.4%];25 mg、478人中10人[2.1%];プラセボ、499人中8人[1.6%])であった。 【結論と妥当性】成人の片頭痛患者において、ウブロゲパントの急性期治療では、プラセボと比較して、50mgと25mgの用量で2時間後の痛みの解放率が有意に高く、50mgの用量でのみ2時間後の最も煩わしい片頭痛関連症状の欠如がみられた。ウブロゲパントの他の片頭痛急性期治療に対する有効性を評価し、非選択的患者集団におけるウブロゲパントの長期安全性を評価するために、さらなる研究が必要です。 【試験登録】ClinicalTrials. gov Identifier:NCT02867709。 第一人者の医師による解説 トリプタン無効例や禁忌例の第1選択候補 反復投与での安全性検証が必要 今井 昇 静岡赤十字病院脳神経内科部長 MMJ.June 2020;16(3) 片頭痛は急性期に頭痛だけではなく随伴症状により日常生活が著しく障害される神経疾患である。 片頭痛の急性期治療にトリプタン系薬剤や非ステ ロイド系抗炎症薬(NSAID)が使用されているが、効果が不十分である、あるいは副作用や禁忌項目のために使用できない患者は多い。 多くの研究によりカルシトニン遺伝子関連ペプチド(CGRP)は片頭痛の病態に重要な役割を演じていることが示され、CGRPを標的とした治療薬が開発されている。すでに抗 CGRPモノクロー ナル抗体は2019年に欧米で片頭痛予防薬として上市されている。ユブロゲパントは小分子の経口 CGRP受容体拮抗薬で、片頭痛急性期治療薬として開発された。多施設ランダム化二重盲検プラセボ 対照第 IIb相試験において、ユブロゲパントの有効性が5用量(1、10、25、50、100mg)で検討され、 25、50、100mgはプラセボに比べ服薬2時間後 の頭痛消失率が有意に高かった。 本論文は、ユブロゲパント 25、50mgの有効性 と安全性を評価するために、米国99施設で実施された多施設ランダム化二重盲検プラセボ対照第 III 相(ACHIEVE II)試験の報告である。18 ~ 65歳(平 均41.5歳)の前兆のない片頭痛または前兆のある 片頭痛の病歴を1年以上有し、発作に中等度以上の光過敏、音過敏、悪心のいずれかの随伴症状が認められる1,686人(女性90%)を対象に、中等度・ 重度の発作に対してユブロゲパント 25、50mg、 またはプラセボが投与された。主要評価項目は投与2時間後の頭痛消失と最も負担となる随伴症状(最 大負担随伴症状)の消失。最大負担随伴症状の内訳 は光過敏57%、音過敏26%、悪心17%。 解析の 結果、2時間後 の 頭痛消失率 は50mg群21.8%、 25mg群20.7%、プラセボ群14.3%で、50mg群、 25mg群ともにプラセボ群と比較し有意な頭痛改 善効果が示された(それぞれP=0.01、P=0.03)。 最大負担随伴症状 の 消失率 は50mg群38.9 %、 25mg群34.1%、プラセボ群27.4%で、50mg 群のみプラセボ群に対し有意な効果が認められた(P =0.01)。投与48時間以内の重篤な有害事象はなく、主な事象は悪心、めまいで各群に有意差はなかった。 本試験におけるユブロゲパント 50mgの有効性と安全性は、トリプタン系薬剤で報告されているものとおおむね同程度であることが示唆される。現在他の経口 CGRP受容体拮抗薬の開発も進んでいる。経口 CGRP受容体拮抗薬はトリプタン系薬 剤やNSAIDでみられる心血管・消化器への影響が少ないと考えられており、新しい片頭痛急性期治 療薬として期待される。ただ、以前開発された経口 CGRP受容体拮抗薬は肝機能障害のため開発中止に至った経緯を考慮すると、今後反復投与での安全性の確認が必要と思われる。
発作性心房細動と高血圧を有する患者の心房細動の再発に対する腎除神経とカテーテルアブレーションの併用とカテーテルアブレーションのみの併用の効果。The ERADICATE-AF Randomized Clinical Trial.
発作性心房細動と高血圧を有する患者の心房細動の再発に対する腎除神経とカテーテルアブレーションの併用とカテーテルアブレーションのみの併用の効果。The ERADICATE-AF Randomized Clinical Trial.
Effect of Renal Denervation and Catheter Ablation vs Catheter Ablation Alone on Atrial Fibrillation Recurrence Among Patients With Paroxysmal Atrial Fibrillation and Hypertension: The ERADICATE-AF Randomized Clinical Trial JAMA 2020 Jan 21;323(3):248-255. 上記論文のアブストラクト日本語訳 ※ヒポクラ×マイナビ 論文検索(Bibgraph)による機械翻訳です。 【重要】腎除神経は心臓の交感神経活動を低下させ、心房細動に対する抗不整脈効果をもたらす可能性がある。 【目的】肺静脈隔離術に腎除神経を追加することで、長期的な抗不整脈効果が高まるかどうかを検討する。 【デザイン、設定、参加者】Evaluate Renal Denervation in Addition to Catheter Ablation to Eliminate Atrial Fibrillation(ERADICATE-AF)試験は、ロシア連邦、ポーランド、ドイツの心房細動のカテーテルアブレーションを行う5つの紹介センターで行われた、研究者主導の多施設、単盲検、無作為化臨床試験である。2013年4月から2018年3月までに、少なくとも1種類の降圧薬を服用しているにもかかわらず高血圧で、発作性心房細動があり、アブレーションの予定がある患者302人が登録された。フォローアップは2019年3月に終了した。 【介入】患者を肺静脈隔離単独(n=148)または肺静脈隔離+腎除神経(n=154)に無作為に割り付けた。肺静脈の電位をすべて除去することをエンドポイントとした完全な肺静脈隔離と、灌流チップ付きアブレーションカテーテルを用いて、両腎動脈の遠位から近位までスパイラルパターンで個別の部位に高周波エネルギーを供給する腎除神経。 【主なアウトカムと評価】主要エンドポイントは、12ヵ月後の心房細動、心房粗動、または心房頻拍からの解放であった。 【結果】無作為化された302名の患者(年齢中央値60歳[四分位範囲55~65歳]、男性182名[60.3%])のうち、283名(93.7%)が試験を完了した。全員が指定された手術を無事に受けた。12ヵ月後の心房細動,粗動,頻脈の消失は,肺静脈隔離術のみを受けた148例中84例(56.5%)と,肺静脈隔離術と腎除神経術を受けた154例中111例(72.1%)に認められた(ハザード比,0.57;95%CI,0.38~0.85;P=0.006).事前に規定された5つの副次的エンドポイントのうち、4つが報告され、3つはグループ間で差がありました。ベースラインから12ヵ月後の平均収縮期血圧は,隔離のみの群では151 mm Hgから147 mm Hgに,腎除神経群では150 mm Hgから135 mm Hgに低下した(群間差:-13 mm Hg,95% CI,-15~-11 mm Hg,P < 0.001).手続き上の合併症は、隔離のみのグループで7人(4.7%)、腎除神経グループで7人(4.5%)に発生した。 【結論と関連性】発作性心房細動と高血圧を有する患者において、カテーテルアブレーションに腎除神経を追加した場合、カテーテルアブレーションのみの場合と比較して、12ヵ月後に心房細動が起こらない可能性が有意に増加した。本試験の結果を解釈する際には、正式なシャムコントロールの腎除神経術が行われていないことを考慮する必要がある。 【臨床試験登録】ClinicalTrials. gov Identifier:NCT01873352。 第一人者の医師による解説 交感神経活性の抑制 降圧とともに臨床的に有用と示した点に意義 藤田英雄 自治医科大学附属さいたま医療センター副センター長・総合医学第1講座主任教授 MMJ.June 2020;16(3) 発作性心房細動に対するカテーテルアブレーションによる肺静脈隔離の有効性は確立されているが、 10~30%の再発率があり改善の余地がある。交感神経活性を抑制する腎デナベーション治療(腎交 感神経除神経術)を加えることで有効性が向上するか否かを検証するため、多施設単盲検無作為化試験 「ERADICATE-AF」が研究者主導で実施され、その 成績が本論文に報告された。 本試験では、高血圧合併発作性心房細動患者302人をカテーテルアブレーション単独群(148人)と 併用群(154人)に無作為に割り付け、12カ月後の心房細動+心房粗動+心房頻拍 の 無再発 を 主要評価項目とした。 主要エンドポイントは単独群84人 (56.5%)、併用群111人(72.1%)で得られ(ハザー ド 比[HR], 0.57;95 % 信頼区間[CI], 0.38~ 0.85;P=0.006)、併用群で有意に無再発例が多く、さらに副次評価項目の1つである治療後の平 均収縮期血圧値の低下は、単独群では151mmHg (ベースライン)から147mmHgであったのに対し、 併用群 で は150mmHgか ら135mmHgへ と より大幅な低下が示された(群間差:-13mmHg; 95% CI,-15 ~-11mmHg;P<0.001)。手技関連合併症は、単独群7人(4.7%)、併用群7人 (4.5%)ですべてがアブレーションによるものであった。 これらの結果から著者らは、単独群にシャム手術を施行していない限界を考慮しつつも高血圧合併発作性心房細動患者に対し、カテーテルアブレーションは腎デナベーションの併用によって心房性不整脈の再発を有意に抑制できたと結論づけた。 今回の結果は、腎デナベーション併用治療の有効性を示唆し、その機序として交感神経活性の抑制 が降圧とともに臨床的に有用であることを示した点に意義がある。腎デナベーション治療はかつて 薬物治療抵抗性高血圧への非薬物療法としての期待を集めたが、2014年のSIMPLICITY-HTN3試験において有効性を示すことができなかった。 しかしながら、その後カテーテル開発競争の中で改良が進み、SPYRAL HTN-OFF MED Pivotal試験(1) では薬物投与なく有意な降圧効果が確認されるなど新たな治療法として復活しつつあり、現状ではその広い適応は医療経済的に困難であるとしても、 心房細動治療の今後の方向性を示す貴重な試験結 果といえよう。 1.Böhm M et al. Lancet. 2020 May 2;395(10234):1444-1451.
2型糖尿病患者の腎機能に対するビタミンDおよびオメガ3脂肪酸の補給の効果。無作為化臨床試験。
2型糖尿病患者の腎機能に対するビタミンDおよびオメガ3脂肪酸の補給の効果。無作為化臨床試験。
Effect of Vitamin D and Omega-3 Fatty Acid Supplementation on Kidney Function in Patients With Type 2 Diabetes: A Randomized Clinical Trial JAMA 2019 Nov 19;322(19):1899-1909. 上記論文のアブストラクト日本語訳 ※ヒポクラ×マイナビ 論文検索(Bibgraph)による機械翻訳です。 【重要】慢性腎臓病は2型糖尿病の一般的な合併症であり、末期腎不全に至る可能性があり、高い心血管リスクと関連している。 【目的】ビタミンD3またはオメガ3脂肪酸の補給が2型糖尿病のCKDの発症または進行を予防するかどうかを検証する。 【デザイン・設定・参加者】マサチューセッツ州の単一施設がコーディネートしたVitamin D and Omega-3 Trial (VITAL)の補助研究として、2011年11月から2014年3月に米国全50州から募集した2型糖尿病の成人1312名を対象に、2×2階調デザインの無作為化臨床試験を実施した。フォローアップは2017年12月に終了した。 【介入】参加者は、ビタミンD3(2000 IU/d)とオメガ3脂肪酸(エイコサペンタエン酸とドコサヘキサエン酸;1 g/d)(n = 370)、ビタミンD3とプラセボ(n = 333)、プラセボとオメガ3脂肪酸(n = 289)、またはプラセボ2種(n = 320)を5年間投与する群に無作為に割り付けられた。 【結果】無作為化された1312名(平均年齢67.6歳、女性46%、人種的または民族的少数派31%)のうち、934名(71%)が試験を完了した。ベースラインの平均eGFRは85.8(SD、22.1)mL/min/1.73 m2だった。ベースラインから5年目までのeGFRの平均変化量は、ビタミンD3投与群で-12.3(95%CI、-13.4~-11.2)mL/min/1.73m2、プラセボ投与群で-13.1(95%CI、-14.2~-11.9)mL/min/1.73m2(差、0.9[95%CI、-0.7~2.5]mL/min/1.73m2)であった。eGFRの平均変化量は、オメガ3脂肪酸群で-12.2(95%CI,-13.3~-11.1)mL/min/1.73m2、プラセボ群で-13.1(95%CI,-14.2~-12.0)mL/min/1.73m2(差,0.9[95%CI,-0.7~2.6]mL/min/1.73m2)。2つの治療法の間に有意な相互作用はなかった。腎結石は58名(ビタミンD3投与群32名,プラセボ投与群26名),消化管出血は45名(オメガ3脂肪酸投与群28名,プラセボ投与群17名)に発生した。 【結論と関連性】成人の2型糖尿病患者において,ビタミンD3またはオメガ3脂肪酸を補給しても,プラセボと比較して,5年後のeGFRの変化に有意な差はなかった。今回の結果は、2型糖尿病患者の腎機能維持のためのビタミンDまたはオメガ3脂肪酸の補給の使用を支持するものではない。 【臨床試験登録】ClinicalTrials. gov Identifier:NCT01684722。 第一人者の医師による解説 ビタミン D、オメガ -3脂肪酸補充は2型糖尿病患者の腎機能改善効果なし 寺内 康夫 横浜市立大学医学群長(大学院医学研究科分子内分泌・糖尿病内科学教授) MMJ.June 2020;16(3) 日本では糖尿病の発症予防、合併症の発症・進展・重症化予防のためにさまざまな対策が講じられており、中でも糖尿病腎症による透析導入を減らす対策は国家的プロジェクトとして位置づけられている。糖尿病腎症に対する基本方針としては、生活 習慣改善、血糖・血圧・脂質・貧血の管理などとともに、腎不全を悪化させる脱水、薬剤、感染症を避けることが重要である。 腎機能が低下すると、血中のCa濃度およびP濃度が変化する。腎臓は徐々に血中のPを除去する能力を失い、Ca濃度を正常に保つのに十分な量のビタミン Dを活性化できなくなる。副甲状腺はこれらの変化を感知して副甲状腺ホルモン(PTH)の産生・放出を増大させ、Ca濃度を上昇させる。これらの代謝変化は骨代謝を変化させ、骨形成異常などが 認められる。こうした骨ミネラル代謝異常は血管 合併症を含む生命予後に影響する。 Kidney Disease /Impr o ving Global Outcomes(KDIGO)ガイド 2009(1)では、慢性腎臓病(CKD)G3~5期では、ビタミンD欠乏の有無チェックのため血中25(OH)D測定が望ましいとされ、25(OH)Dが20 ng/mL以下ではビタミンD欠乏症と判定し、天然型ビタミン D製剤や活性型ビタミン D製剤の投与が推奨された。しかし、「エビデンスに基づくCKD診療ガイドライン2018」(2) では、保存期 CKD患者において、活性型ビタミン D製剤はPTH値を低下させ、尿蛋白を抑える効果が期待されるため、投与を考慮してもよいが、腎機能予後、骨折、心血管イベント、生命予後への効果は明らかでなく、高 Ca血症の原因となることから、 適応は患者ごとに検討し、少量から慎重に開始することが望ましいと基本姿勢が変わってきた。 このように、糖尿病患者における2次性副甲状腺機能 亢進症に対し、活性型ビタミンD製剤の使用は生命予後を改善する可能性があるが、エビデンスは 十分とは言えない。 また、オメガ -3脂肪酸は心血管イベントのリスク軽減には重要かもしれないが、アルブミン尿増加、 糸球体濾過量(GFR)低下のリスクが報告されている。 こうした状況を踏まえ、本論文の著者らは、ビタミン D製剤かつまたはオメガ -3脂肪酸を2型糖尿病患者1,312人に5年間投与した際の腎機能への影響を検討するVITAL試験を実施した。結論としては、ビタミン D製剤、オメガ -3脂肪酸のいずれも腎機能を改善しなかった。ビタミンD欠乏状態にある患者は15%程度と少数であったことが本研究の結論に影響している可能性があり、骨折、心血管イベント、生命予後を検証するランダム化対照試験も待たれる。 1. KDIGO CKD-MBD Work Group. Kidney Int Suppl. 2009 Aug;(113):S1-130. 2. 「エビデンスに基づく CKD 診療ガイドライン 2018」(日本腎臓学会)
早すぎる自然閉経および手術による閉経と心血管疾患の発症との関連性
早すぎる自然閉経および手術による閉経と心血管疾患の発症との関連性
Association of Premature Natural and Surgical Menopause With Incident Cardiovascular Disease JAMA 2019 Dec 24;322(24):2411-2421. 上記論文のアブストラクト日本語訳 ※ヒポクラ×マイナビ 論文検索(Bibgraph)による機械翻訳です。 【重要性】最近のガイドラインでは、中年女性の動脈硬化性心血管疾患リスク評価の精緻化のために、40歳以前の閉経歴を用いることが推奨されている。この集団における心血管疾患リスクに関する確固たるデータは不足している。 【目的】40歳前の自然閉経および外科的閉経を有する女性における心血管疾患の発症と心血管リスク因子を検討する。 【デザイン,設定,参加者】2006年から2010年に募集した英国の成人居住者によるコホート研究(UK Biobank)である。試験登録時に40~69歳で閉経後の女性のうち,144 260人が組み入れ対象となった。フォローアップは2016年8月まで行われた。 【曝露】自然早発閉経(卵巣摘出術を行わない40歳前の閉経)および外科的早発閉経(40歳前の両側卵巣摘出術)。早発閉経のない閉経後女性を参照群とした。 【主要評および測定法】主要アウトカムは、冠動脈疾患、心不全、大動脈弁狭窄、僧帽弁閉鎖不全症、心房細動、虚血性脳卒中、末梢動脈疾患、静脈血栓塞栓症の発症を複合したものであった。副次的アウトカムには,主要アウトカムの個々の要素,高血圧,高脂血症,2型糖尿病の発症が含まれた。 【結果】対象となった閉経後女性144 260名(登録時の平均[SD]年齢,59.9[5.4]歳)のうち,自然早発閉経は4904(3.4%),外科的早発閉経は644(0.4%)であった。参加者は中央値で 7 年間追跡された(四分位範囲,6.3~7.7).主要転帰は、早発閉経を認めなかった女性5415人(3.9%)(発生率、5.70/1000人年)、自然早発閉経の女性292人(6.0%)(発生率、8.78/1000人年)(早発閉経なしとの差、+3.08/1000人年[95])に発生した。08/1000女性年[95% CI, 2.06-4.10];P < 0.001)、および外科的早発閉経(発生率、11.27/1000女性年)49女性(7.6%)(早発閉経なしとの差、+5.57/1000女性年[95% CI, 2.41-8.73];P < 0.001 )であった。主要アウトカムについては、従来の心血管疾患リスク因子および更年期ホルモン療法の使用で調整した後、自然および外科的早発閉経は、それぞれ1.36(95%CI、1.19-1.56;P < .001)および 1.87(95%CI, 1.36-2.58;P < .001)のハザード比と関連していた。【結論と関連性】自然および外科的早発閉経(40歳前)は、閉経後女性における心血管疾患の複合リスクとわずかではあるが統計的に有意な増加と関連していた。これらの関連性の根底にあるメカニズムを理解するために、さらなる研究が必要である。 第一人者の医師による解説 エストロゲン低下が心血管疾患の病態に直接関与することを示唆 武谷 雄二 東京大学名誉教授・医療法人レニア会理事長(アルテミスウイメンズホスピタル産婦人科) MMJ.June 2020;16(3) 先進諸国における中高年女性の主要な死因は心血管疾患である。日本でも高齢女性の死因の第2位、 3位はそれぞれ心疾患、脳卒中であり、合わせると悪性腫瘍に匹敵する。 エストロゲン(E)の血中濃度は閉経数年前から徐々に低下傾向がみられ、閉経後4~5年程度で検出限界に近づき、その後は低値を維持する。閉経年齢は50~52歳が平均的である。10%程度の女性は45歳未満に閉経を迎える。40歳未満の閉経は約1%であり早発閉経(premature menopause; PM)と呼ばれる。 閉経年齢が若いほどE欠落症状 (骨粗鬆症、動脈硬化など)が早期に現れる。さらに Eの低下は心血管疾患の危険因子としても注目されている。 本研究は、PMはい か な る 心血管疾患 のリスク を高めるかを調査した。対象は英国在住の閉経女性144,260人(40~69歳、平均年齢59.9歳、 約95%は白人)で約7年間前向きに心血管疾患の発症を調査した。3.8%はPMで、そのうち外科的 PM(卵巣切除)は10%強であった。 そ の 結果、何 ら か の 心血管疾患 が 非 PM群 の 3.9%、自然 PM群の6.0%、外科的 PM群の7.6% に発生した。いずれのPM群も非 PMと比較し、心血管疾患の発生率が有意に高く(P<0.001)、喫煙、Eの補充療法、高血圧、2型糖尿病、高脂血症などの有無で補正しても、ハザード比はそれぞれ1.36、 1.87と有意に高かった(P<0.001)。PM群で高 頻度にみられた心血管疾患は、冠動脈疾患、心不全、 大動脈弁狭窄症、心房細動、心虚血性発作、静脈血栓 塞栓症,虚血性脳卒中(特に自然 PM群)などであった。なお、末梢動脈疾患は閉経年齢との関連は乏しかった。全体として、閉経年齢は心血管疾患、メタボリックシンドローム関連疾患(高血圧、2型糖尿病、 高脂血症)などの発生率と逆相関を示し、特にPM 群ではいずれも高率であった。 早期のEの低下が、虚血性心疾患のみならず、脳卒中を含む多様な心血管疾患およびメタボリックシンドローム関連疾患のリスクを高めることは新たな知見である。しかし、PM群ではメタボリックシンドローム関連疾患の有無で補正しても心血管疾患のリスクは高く、Eの低下が心血管疾患の病態に直接関与することが示唆される。 外科的 PMの方が自然 PMと比べ、心血管疾患やメタボリックシンドローム関連疾患の発生率が高かった。この説明として、自然 PMの卵巣では、閉経後も微量のEとある程度のテストステロン(T) の分泌は保たれており、Tは脂肪組織などでEに転換される。一方、外科的PMではEやTの分泌はなく、 そのため血中E濃度はより低値であったことが考えられる。
男性への葉酸と亜鉛の補給が不妊治療を受けているカップルの精液の質と生児に及ぼす影響。A Randomized Clinical Trial.
男性への葉酸と亜鉛の補給が不妊治療を受けているカップルの精液の質と生児に及ぼす影響。A Randomized Clinical Trial.
Effect of Folic Acid and Zinc Supplementation in Men on Semen Quality and Live Birth Among Couples Undergoing Infertility Treatment: A Randomized Clinical Trial JAMA 2020 Jan 7;323(1):35-48. 上記論文のアブストラクト日本語訳 ※ヒポクラ×マイナビ 論文検索(Bibgraph)による機械翻訳です。 【重要】男性不妊治療のために販売されている栄養補助食品は、精液の質を改善するという限られた先行エビデンスに基づいて、葉酸と亜鉛を含むのが一般的である。 【目的】毎日の葉酸と亜鉛の補給が、精液の質と出生に及ぼす影響を明らかにする。不妊治療を計画しているカップル(n=2370,男性は18歳以上,女性は18~45歳)を,2013年6月~2017年12月に米国の生殖内分泌学・不妊治療研究センター4施設に登録した。精液採取のための最後の6カ月間の研究訪問は2018年8月中に行われ、ライブバースおよび妊娠情報のチャート抽出は2019年4月中に完了した。 【介入】男性は、研究センターと計画している不妊治療(体外受精、研究サイトでのその他の治療、外部クリニックでのその他の治療)によってブロック無作為化され、葉酸5mgおよび元素亜鉛30mg(n=1185)またはプラセボ(n=1185)のいずれかを6カ月間毎日投与された。 【結果】無作為化された2370人の男性(平均年齢33歳)のうち、1773人(75%)が6カ月後の最終診察に参加した。すべてのカップルの出生成績が得られ、1629人(69%)の男性が無作為化後6カ月の時点で分析用の精液を入手していた。生児出生数は治療群間で有意な差はなかった(葉酸・亜鉛群404[34%],プラセボ群416[35%],リスク差-0.9%[95%CI,-4.7%~2.8%])。無作為化後6カ月の時点で,ほとんどの精液品質パラメータ(精子濃度,運動性,形態,体積,総運動精子数)は治療群間で有意な差はなかった。葉酸と亜鉛の補給により,統計的に有意なDNA断片化の増加が認められた(DNA断片化の割合の平均は,葉酸と亜鉛群で29.7%,プラセボ群で27.2%,平均差は2.4%[95%CI,0.5~4.4%])。消化器症状は、葉酸および亜鉛の補給により、プラセボと比較してより多く見られた(腹部の不快感または痛み:それぞれ66[6%]対40[3%]、吐き気:50[4%]対24[2%]。50[4%]対24[2%]、および嘔吐。 【結論と関連性】不妊治療を受けようとしている一般的なカップルにおいて、男性パートナーが葉酸と亜鉛のサプリメントを使用しても、プラセボと比較して、精液の質やカップルの生児率を有意に改善することはできませんでした。これらの知見は、不妊治療における男性パートナーによる葉酸と亜鉛の補給の使用を支持するものではありません。 【臨床試験登録】ClinicalTrials. gov Identifier:NCT01857310。 第一人者の医師による解説 結果の解釈は慎重に 適応を絞れば効果がある可能性も 岩月 正一郎(助教)/安井 孝周(教授) 名古屋市立大学大学院医学研究科腎・泌尿器科学分野 MMJ.June 2020;16(3) 近年、不妊症に対するサプリメントへの関心が高まっており、男性不妊を対象としたサプリメントの多くは葉酸と亜鉛が含まれている。最近のメタアナリシスにおいて、亜鉛と葉酸は男性不妊症患者の精子濃度や精子正常形態率を改善することが示された(1)。しかしこの報告で参照された論文は、結果のばらつきが大きく、大規模な研究が望まれてきた。 本論文では、米国内の不妊治療中カップル 2,370 組を対象とし、男性に1日に葉酸 5 mgと亜鉛 30 mgもしくはプラセボを6カ月間服用する群に無作為に割り付け、6カ月後の精液所見の変化およびその間の不妊治療の成績を比較した。 その結果、精液検査所見に変化はないばかりか、葉酸と亜鉛を投与すると、精子のDNA断片化率がプラセボ群 の27.2%に対して29.7%に上昇していた(精子 DNA断片化は精子への障害を表す指標で、30%以 下が正常範囲内である)。さらに、出産率にも変化はなく、むしろ葉酸と亜鉛を投与すると、プラセボ群に比べ、早産率が1.49倍に上昇していたという。 副作用についても、葉酸・亜鉛群で主に悪心・嘔吐 といった消化器症状が増加していた。 しかし本論文にはいくつかの制限がある。特に今回の知見が男性不妊症患者一般に当てはまるか どうかは慎重に吟味する必要があり、その理由として大きな問題点が3つ挙げられる。1つ目は対象の偏りである。確かに本研究は多数の男性を対象としたランダム化比較試験である。しかし参加した夫婦には男性不妊、女性不妊が混在しており、対象集団の8割近くの男性の精液検査所見は正常であった。2つ目は葉酸・亜鉛群でDNA断片化率と早期産の割合が有意に上昇したとあるが、プラセボ 群との差はわずかで、いずれも正常範囲内であることから臨床的意義は不明である。3つ目は、対象男性の投薬開始前(ベースライン)の葉酸と亜鉛の 血中濃度に関する情報がないことである。 2019 年の1年間に筆者らの施設を受診した男性不妊症 患者114人の血中亜鉛濃度を測定したところ、潜在性亜鉛欠乏(60μg/dL以上80μg/dL未満)は 36人(31.6%)、亜鉛欠乏(60μg/dL未満)は4 人(3.5%)であり、予想していた以上に亜鉛欠乏の患者の存在が明らかになった。亜鉛に限って言えば、対象を限定した亜鉛補充の有効性はさらに検証されるべきで、葉酸についても同様のことが予想される。 本研究は、エビデンスの乏しい不妊症に対する補助療法についての大規模なランダム比較試験として意義のある報告である。しかし、その結果の解釈は慎重に行うべきであると考えられる。 1.Irani M et al. Urol J. 2017;14(5):4069-4078.
慢性膵炎患者の疼痛に対する早期手術と内視鏡ファーストアプローチの効果。The ESCAPE Randomized Clinical Trial.
慢性膵炎患者の疼痛に対する早期手術と内視鏡ファーストアプローチの効果。The ESCAPE Randomized Clinical Trial.
Effect of Early Surgery vs Endoscopy-First Approach on Pain in Patients With Chronic Pancreatitis: The ESCAPE Randomized Clinical Trial JAMA 2020 Jan 21;323(3):237-247. 上記論文のアブストラクト日本語訳 ※ヒポクラ×マイナビ 論文検索(Bibgraph)による機械翻訳です。 【重要】疼痛を伴う慢性膵炎の患者に対しては、内科的治療や内視鏡的治療が奏功しない限り、外科的治療は延期される。観察研究では、早期の手術が疾患の進行を緩和し、より良い疼痛コントロールを提供し、膵臓の機能を維持する可能性が示唆されている。 【目的】早期の手術が内視鏡優先のアプローチよりも臨床転帰の点で有効であるかどうかを明らかにする。 【デザイン・設定・参加者】ESCAPE試験は、Dutch Pancreatitis Study Groupに参加しているオランダの30の病院が参加した非盲検の多施設無作為化臨床優越試験である。2011年4月から2016年9月まで、慢性膵炎で主膵管が拡張しており、激しい痛みのために処方されたオピオイドを最近になって使用し始めた患者(強オピオイドを2カ月以下、弱オピオイドを6カ月以下)計88人を対象とした。18カ月の追跡期間は2018年3月に終了した。 【介入】無作為化後6週間以内に膵臓ドレナージ手術を受けた早期手術群に無作為化された44人と、内科的治療、必要に応じて砕石を含む内視鏡検査、必要に応じて手術を受けた内視鏡検査優先アプローチ群に無作為化された44人がいた。[法]主要アウトカムは痛みで、Izbicki疼痛スコアで測定し、18カ月間で積分した(範囲、0~100[スコアが上がると痛みの重症度が増す])。副次的評価項目は、追跡調査終了時の疼痛緩和、介入回数、合併症、入院回数、膵臓機能、QOL(36項目からなるショートフォーム健康調査[SF-36]で測定)、死亡率であった。 【結果】無作為に割り付けられた88名の患者(平均年齢52歳、女性21名(24%))のうち、85名(97%)が試験を完了した。18ヵ月間の追跡調査では、早期手術群の患者は、内視鏡による初回アプローチ群に無作為に割り付けられた群の患者よりもIzbicki疼痛スコアが低かった(37対49、群間差:-12ポイント[95%CI, -22~-2]、P =0.02)。フォローアップ終了時に完全または部分的な疼痛緩和が得られたのは、早期手術群では40例中23例(58%)であったのに対し、内視鏡的アプローチ優先群では41例中16例(39%)であった(P = 0.10)。介入の総数は早期手術群で少なかった(中央値、1対3、P < 0.001)。治療の合併症(27%対25%)、死亡率(0%対0%)、入院、膵臓の機能、およびQOLは、早期手術と内視鏡検査優先アプローチとの間に有意な差はなかった。 【結論と関連性】慢性膵炎患者において、早期手術と内視鏡検査優先アプローチとを比較した場合、18カ月間の統合では、痛みのスコアが低くなった。しかし、経時的な差の持続性を評価し、研究結果を再現するにはさらなる研究が必要である。 【臨床試験登録】ISRCTN Identifier:ISRCTN識別子:STRECTN45877994。 第一人者の医師による解説 内視鏡的治療が選択される患者群も想定 引き続き長期の検討評価を 宅間 健介(助教)/五十嵐 良典(主任教授) 東邦大学医療センター大森病院消化器内科 MMJ.August 2020;16(4) 慢性膵炎は持続する炎症と線維化が進行し、最終的に膵が荒廃する疾患であり、主要徴候の約80%は疼痛である。主膵管狭窄や膵石症を伴うことが多く、膵液うっ滞による膵管内や膵間質内圧の上昇などにより持続疼痛や急性膵炎をきたし、それがさらなる病態進行の原因となる。 疼痛は生活の質(QOL)を低下させ、特に欧米では多用される麻薬鎮痛薬の長期使用による依存などの副作用が懸念されており、疼痛コントロールは極めて重要である。膵切除術・膵管ドレナージ術などの手術療法や膵管ステントを用いた内視鏡的ドレナージ術は膵管内の減圧が得られ、疼痛緩和や外分泌機能改善などに有用な治療とされる。 本研究では主膵管拡張および疼痛を伴う慢性膵炎患者88人を対象に、薬物療法から内視鏡的治療を第1選択として行う群(44 人)と早期手術療法を第1選択とした群(44人)の疼痛コントロールが比較された。主要評価項目である疼痛はIzbicki pain scoreで評価された。 観察期間18カ月における疼痛スコアは早期手術療法群が内視鏡的治療群よりも低く、疼痛コントロールに優れていることを示した。観察終了時での完全・部分的疼痛緩和について統計学的有意差はなく、観察中の合併症発生率、死亡率、入院回数、膵機能変化、QOLも群間差はなかった。内視鏡的治療群では膵石や膵管狭窄の程度により体外式衝撃波結石破砕療法(ESWL= Extracorporeal Shock Wave Lithotripsy)、膵管ステントが用いられ治療介入回数が多かった。また疼痛の持続する治療困難例が24例(62%)に認められ、19人は手術療法に移行・待機となった。 一方、早期手術療法群は単一の介入で明瞭な結果となり、鎮痛に対する早期手術療法の優位性を示した。過去の報告(1),(2)でも外科手術は、より早期の介入ほど鎮痛効果を示し、治療初期のオピオイド使用と内視鏡的治療は早期手術療法に比べ疼痛の軽減が低いことが示されており、今回の結果に一致している。 日本ではESWLによる膵石破砕術や膵管ステント留置術が保険診療として認可されたことにより、広く認知・普及している。患者も心情的に内科的治療を第1選択とする傾向にある。本研究において主膵管内膵石の完全除去例に関しては早期手術療法に近い鎮静効果を示しており、内視鏡的治療を第1と考慮する患者群も想定される。臨床症状や病態を含めた的確な選択と今後の膵管鏡やレーザー、ESWLなどの技術革新に期待しつつ、機能温存や悪性疾患合併などの長期にわたる治療効果評価が必要であろう。 1. Cahen DL et al. N Engl J Med. 2007;356(7):676-684. 2. Díte P et al. Endoscopy. 2003;35(7):553-558.
2017年の集中治療室患者における感染症の有病率および転帰。
2017年の集中治療室患者における感染症の有病率および転帰。
Prevalence and Outcomes of Infection Among Patients in Intensive Care Units in 2017 JAMA 2020 Apr 21;323(15):1478-1487. 上記論文のアブストラクト日本語訳 ※ヒポクラ×マイナビ 論文検索(Bibgraph)による機械翻訳です。 【重要性】集中治療室(ICU)の患者では、感染が頻繁に発生している。)感染の種類、原因病原体、転帰に関する最新の情報は、予防、診断、治療、資源配分の政策立案に役立ち、介入研究のデザインに役立つ可能性がある。 【目的】世界のICUにおける感染の流行と転帰、利用可能な資源に関する情報を提供する。 【デザイン、設定、参加者】88か国1150施設での縦断追跡調査付き24時間点有病率調査。2017年9月13日08:00から始まる24時間の間に参加ICUで治療を受けたすべての成人患者(18歳以上)を対象とした。最終フォローアップ日は2017年11月13日。 【曝露】感染症診断および抗生物質の受領。 【主要アウトカムおよび測定】感染症と抗生物質曝露の有病率(横断的デザイン)および全原因院内死亡(経時的デザイン)。 【結果】対象患者15202例(平均年齢61.1歳[SD、17.3歳]、男性9181例[60.4%])において、感染症データが得られたのは15165例(99.8%)で、8135例(54%)が感染症の疑いまたは証明、そのうちICU感染症の1760例(22%)であった。合計10 640人(70%)の患者が少なくとも1種類の抗生物質を投与された。感染が疑われるまたは証明された患者の割合は,オーストラレーシアの43%(141/328)からアジアおよび中東の60%(1892/3150)であった.感染が疑われたまたは証明された8135人の患者のうち,5259人(65%)が少なくとも1つの微生物学的培養が陽性であった。これらの患者の67%(n=3540)でグラム陰性微生物が,37%(n=1946)でグラム陽性微生物が,16%(n=864)で真菌性微生物が同定された。感染が疑われる,あるいは感染が証明された患者の院内死亡率は30%(2404/7936)であった.マルチレベル解析では,ICU 内感染は市中感染と比較して高い死亡リスクと独立して関連していた(オッズ比 [OR], 1.32 [95% CI, 1.10-1.60]; P = 0.003).抗生物質耐性微生物のうち,バンコマイシン耐性腸球菌(OR,2.41 [95% CI,1.43-4.06]; P = .001),第3世代セファロスポリンおよびカルバペネム系抗生物質を含むβラクタム系抗生物質に耐性を示すクレブシエラ(OR,1.29 [95% CI,1.02-1.63]; P = .03)またはカルバペネム耐性アシネトバクター種への感染(OR,1.40 [95% CI, 1.08-1.81]; P = .01)は、他の微生物による感染と比較して、死亡リスクの高さと独立して関連していた。 【結論と関連性】2017年9月にICUに入院した世界中の患者のサンプルにおいて、感染の疑いまたは証明がある割合は高く、院内死亡のかなりのリスクを伴うものであった。 第一人者の医師による解説 88カ国、1 ,150施設での自発調査 患者背景などに差 解釈には考慮必要 萬 知子 杏林大学医学部麻酔科学教室主任教授 MMJ.August 2020;16(4) 本論文は、世界の集中治療室(ICU)における感染率観察調査研究の報告である。2017年9月13日午前8時から24時間の調査を、88カ国、1,150施設で行った。総患者数15,202人の感染率は54%であった。地域別では、最も低いオーストラリアの43%から、最も高いアジア・中東の60%までと幅があった。国民総所得別のICU患者感染率は、低~下位中所得国58%、上位中所得国59%、高所得国50%であった。 感染のうち市中感染は44 %、病院関連感染は35%、ICU関連感染は22%であった。感染部位は気道60%、腹腔18%、血液(血流感染)15%であった。抗菌薬投与の実施率は70%(予防的28%、治療目的51%)であった。 検体培養陽性率は65%で、検出菌はグラム陰性菌67%、グラム陽性菌37%、真菌16%であった。市中感染の57 %、病院関連感染の71%、ICU関連感染の78%からグラム陰性菌が分離され、その内訳はクレブジエラ属27 %、大腸菌 25 %、緑膿菌属24%、アシネトバクター属17%であった。グラム陽性菌陽性患者の割合は、市中感染42%、病院関連感染37%、ICU関連感染31%であった。感染の危険因子は、男性、合併症(慢性閉塞性肺疾患、がん、糖尿病、慢性腎不全、HIV、免疫抑制)、調査日前のICU長期滞在であった。感染率は各国内の病院間でバラツキが有意に大きかった。 感染者の院内死亡率は30 %であった。院内死亡の危険因子は、ICU関連感染(対市中)、高齢、Simplified Acute Physiological Score II高値、転移がん、心不全(NYHA III ~ IV)、HIV感染、肝硬変、人工呼吸、腎代替療法、院内急変(対術後)であった。薬剤耐性菌のみに限ると、バンコマイシン耐性腸球菌、広域(第3世代セフェム、カルバペネムを含む)βラクタマーゼ産生クレブジエラ属、カルバペネム耐性アシネトバクター属が独立した院内死亡の危険因子であった。したがって、抗菌薬の適正使用監視が重要である。 本研究の限界は、自発参加のため世界のICUを網羅していないことである。大多数の参加施設は欧州、中国、南米であり、低~下位中所得国の施設は全体の6%のみである。地域により、患者背景、疾患、医療体制、ICU入室基準、医療資源、看護師数、感染防御対策、抗菌薬適正使用監視体制などに差があった。感染に対するこれらの影響は本研究では明らかでないが、調査結果の解釈にはこれらの要素を考慮する必要はあろう。
米国における人種・民族別の糖尿病有病率(2011年~2016年)。
米国における人種・民族別の糖尿病有病率(2011年~2016年)。
Prevalence of Diabetes by Race and Ethnicity in the United States, 2011-2016 JAMA 2019 Dec 24;322(24):2389-2398. 上記論文のアブストラクト日本語訳 ※ヒポクラ×マイナビ 論文検索(Bibgraph)による機械翻訳です。 【重要】米国におけるヒスパニック系及びアジア系アメリカ人亜集団の糖尿病有病率は不明である。 【目的】米国の20歳以上の成人における糖尿病有病率の人種・民族差を主要な人種・民族グループ別及び選択したヒスパニック及び非ヒスパニック系アジア人亜集団別に推定する。 【デザイン・設定・参加者】National Health and Nutrition Examination Surveys,2011~2016年、非施設化民間、米国人集団を代表する横断的サンプルである。サンプルは、面接時に自己申告で糖尿病と診断された者、またはヘモグロビンA1c(HbA1c)、空腹時血糖値(FPG)、2時間血糖値(2hPG)を測定した20歳以上の成人であった。 【曝露】人種/民族群:非ヒスパニック系白人、非ヒスパニック系黒人、ヒスパニック系およびヒスパニック系サブグループ(メキシコ、プエルトリコ、キューバ/ドミニカ、中央アメリカ、南アメリカ)、非ヒスパニック系アジア人および非ヒスパニック系アジア人サブグループ(東、南、東南アジア)、非ヒスパニック系その他、診断済み糖尿病は自己報告の事前診断によるものであった。未診断の糖尿病は、糖尿病と診断されていない参加者のHbA1c 6.5%以上、FPG 126mg/dL以上、または2hPG 200mg/dL以上と定義された。 【結果】調査対象は米国の成人7575人(平均年齢47.5歳,女性52%,非ヒスパニック系白人2866人[65%],非ヒスパニック系黒人1636人[11%],ヒスパニック1952人[15%],非ヒスパニック系アジア人909人[6%],非ヒスパニック系その他212人[3%])であった。合計2266人が糖尿病と診断され、377人が糖尿病と診断されていなかった。年齢と性別で調整した糖尿病有病率は,非ヒスパニック系白人で 12.1%(95% CI,11.0%-13.4%), 非ヒスパニック系黒人で 20.4%(95% CI,18.8%-22.1%), ヒスパニック系で 22.1%(95% CI,19.6%-24.7%), 非ヒスパニック系アジア人で 19.1%(95% CI,16.0%-22.1%) となった(全体 P < 0.001) .ヒスパニック系成人では,糖尿病全体の有病率は,メキシコ人で 24.6%(95% CI,21.6%-27.6%), プエルトリコ人で 21.7%(95% CI,14.6%-28.8%), キューバ/ドミニカ人で 20.5%(95% CI,13.7%-27.3%), 中米人で 19.3%(95% CI,12.4%-26.1%), 南米人で 12.3%(95% CI, 8.5%-16.2%) となっていた(全体での P < .001).非ヒスパニック系アジア人の成人では,糖尿病全体の有病率は,東アジア人で 14.0%(95% CI,9.5%-18.4%), 南アジア人で 23.3%(95% CI,15.6%-30.9%), 東南アジアのサブグループで 22.4%(95% CI,15.9%-28.9%) であった(全体の P = .02).診断されていない糖尿病の有病率は、非ヒスパニック系白人で3.9%(95%CI、3.0%-4.8%)、非ヒスパニック系黒人で5.2%(95%CI、3.9%-6.4%)、ヒスパニックで7.5%(95%CI、5.9%-9.1%)、非ヒスパニック系アジアの成人で7.5%(95%CI、4.9%-10.0%)だった(全体でのP < .001). 【結論と関連性】2011年から2016年の米国の成人を対象としたこの全国代表的な調査において,糖尿病および診断されていない糖尿病の有病率は,人種/民族によって,またヒスパニックおよび非ヒスパニック・アジア人集団内で特定されるサブグループによって異なっていた。 第一人者の医師による解説 至近データでの推算 糖尿病予防対策に有意義 原井 望、森 保道(部長)虎の門病院内分泌代謝科 MMJ.August 2020;16(4) 米国における成人の糖尿病患者数は2018年時点で約3400万人(成人の13%)にのぼる(1) 。現在、米国ではヒスパニック(H)系とアジア(A)系の人口が23%を占め、増加傾向にある。世界的にH系、A系の糖尿病有病率はヨーロッパ系・アフリカ系よりも高いとされ、同有病率の違いが生じる要因として、遺伝的・後成的因子、生活因子、環境因子などが指摘されている。 本論文は、米国における人種間での糖尿病有病率を比較するために、H系とA系の調査が重点化された2011~16年の米国民健康栄養調査(NHANES)をもとに、20歳以上の米国成人7 ,575人(平均年齢47 .5 歳、女性51.9%)を対象に実施された横断的研究の報告である。 人種構成は、①非H系白人2 ,866人、②非H系黒人1,636人、③ H系(メキシコ、プエルトリコ、キューバ/ドミニカ、中央アメリカ、南アメリカ)1.952人、④ A系(東アジア、南アジア、東南アジア)909人、⑤その他212人であった。NHANES統計解析ガイドライン(2)に基づき母集団を推算した米国成人の糖尿病有病率は14 .6%であった。さらに年齢と性別で調整した人種別の糖尿病有病率はそれぞれ①12.1%、②20.4%、③22.1%、④19.1%、⑤18.5%であり、米国成人の糖尿病有病率は人種間で有意差が認められた。各人種内でも出身由来地による差があり、H系の中ではメキシコ系の有病率が24 .6%と最も高率で、一方A系では中国、日本、韓国の東アジア由来が14.0%と最も低率という結果になった。 既報では、糖尿病有病率上昇の関連因子として肥満や低~中所得者が挙げられている(3) 。本論文では、人種間における教育や体格指数(BMI)の違いについても比較検討している。高校以上の教育を受けた割合は、A系が73 .9%で最も高く、H系は40 .3%で最も低かった。一方BMIに関しては、A系はBMI 24 .4と他の人種(BMI, 29.1 ~ 30.6)と比較し低値であった。BMIによる調整を追加した人種別の糖尿病有病率はそれぞれ①11.9%、②18.4%、③20.3%、④27.0%、⑤17.7%となり、A系が最も高かった。アジア人は欧米人と比較し、インスリン分泌能が低く、軽度のBMI上昇でも糖尿病になりやすいといわれており、本論文でもそれを裏付ける結果となった。 今後の米国での糖尿病予防対策を考慮するうえで、至近のデータをもとに人種ごとの糖尿病有病率が推算されたことは意義深い。なお、本研究の限界として、横断的研究であり因果関係の推測が困難であること、糖尿病の病型が不明であること、人種区分が自己申告に基づくことなどが挙げられる。 1. National Diabetes Statistics Report 2020 (https://bit.ly/2WImXwq) 2. NHANES Survey Methods and Analytic Guidelines, 2011-2014 and 2015- 2016. CDC website. (https://bit.ly/2LyLWMe) 3. World Health Organization: Global report on diabetes 2016 (https://bit. ly/2TcQnQP)
人工膝関節置換術を受ける患者の静脈血栓塞栓症予防におけるオソシマブの効果。FOXTROT Randomized Clinical Trial(無作為化臨床試験)。
人工膝関節置換術を受ける患者の静脈血栓塞栓症予防におけるオソシマブの効果。FOXTROT Randomized Clinical Trial(無作為化臨床試験)。
Effect of Osocimab in Preventing Venous Thromboembolism Among Patients Undergoing Knee Arthroplasty: The FOXTROT Randomized Clinical Trial JAMA 2020 Jan 14;323(2):130-139 上記論文のアブストラクト日本語訳 ※ヒポクラ×マイナビ 論文検索(Bibgraph)による機械翻訳です。 【重要性】血栓予防に対する第XIa因子阻害の有効性は不明です。オソシマブは、第XIa因子を阻害する長時間作用型の完全ヒトモノクローナル抗体です。 【目的】膝関節形成術を受けた患者の血栓予防のために、エノキサパリンおよびアピキサバンと異なる用量のオソシマブを比較する。 【設計、設定、および参加者】無作為化、非盲検-13か国の54の病院で実施された、オソシマブ投与量を盲検化するオブザーバーによるラベル、裁定者盲検、第2相非劣性試験。片側膝関節形成術を受けている成人患者は、2017年10月から2018年8月まで無作為化され、2019年1月まで追跡調査されました。 1.2 mg / kg(n = 108)、または1.8 mg / kg(n = 106); 0.3 mg / kg(n = 109)または1.8 mg / kg(n = 108)の術前投与量。または、40 mgの皮下エノキサパリンを1日1回(n = 105)または2.5 mgの経口アピキサバンを1日2回(n = 105)、少なくとも10日間または静脈造影まで。および術後13日(手術後10-13日で実施された強制的な両側静脈造影または症候性深部静脈血栓症または肺塞栓症の確認により評価)。エノキサパリンと比較して5%の非劣性マージンが選択されました。主要または臨床的に関連する非主要出血の主要な安全性の結果は、術後10-13日まで評価されました。および74.2%の女性)、600人が一次分析に使用されたプロトコルごとの母集団に含まれていました。主な転帰は、0.3 mg / kgを投与された18人の患者(23.7%)、0.6 mg / kgを投与された8人(15.7%)、1.2 mg / kgを投与された13人(16.5%)、および1.8 mg / kgを投与された14人(17.9%)で発生しました。術後のオソシマブの; 23(29.9%)は0.3 mg / kgを投与され、9(11.3%)は術前に1.8 mg/kgのオソシマブを投与されました。エノキサパリンを投与された20人(26.3%)。そして12人(14.5%)がアピキサバンを投与されました。術後に投与されたオソシマブは、0.6 mg / kgの用量で10.6%(95%CI、-1.2%から∞)のリスク差(片側95%CI)で、エノキサパリンと比較して非劣性の基準を満たしました。 1.2 mg / kgの用量で9.9%(95%CI、-0.9%から∞)、1.8 mg / kgの用量で8.4%(95%CI、-2.6から∞)。 1.8mg / kgのオソシマブの術前投与量は、15.1%のリスク差でエノキサパリンと比較して優越性の基準を満たしていました。両面90%CI、4.9%から25.2%)。 0.3 mg / kgのオソシマブの術後および術前用量は、2.6%(95%CI、-8.9%から∞)および-3.6%(95%CI)のリスク差(片側95%CI)で、非劣性の事前に指定された基準を満たしていませんでした。それぞれ95%CI、-15.5%から∞)。主要または臨床的に関連する非主要な出血は、オソシマブを投与された患者の最大4.7%、エノキサパリンを投与された患者の5.9%、アピキサバンを投与された患者の2%で観察されました。 kg、および1.8 mg / kgは、エノキサパリンと比較して非劣性の基準を満たし、術前1.8 mg / kg用量のオソシマブは、術後10-13日での静脈血栓塞栓症の発生の主要転帰についてエノキサパリンと比較して優位性の基準を満たしました。標準的な血栓予防と比較したオソシマブの有効性と安全性を確立するには、さらなる研究が必要です。 【臨床試験登録】 ClinicalTrials. gov識別子:NCT03276143。 第一人者の医師による解説 出血の際のリバース困難な可能性 実用化には研究必要 桂川 陽三 国立国際医療研究センター病院整形外科診療科長 MMJ.August 2020;16(4) 人工膝関節置換術後の静脈血栓塞栓症(VTE)は比較的起こりやすい合併症である。日本整形外科学会のVTE予防ガイドラインでは高リスク合併症とされ、予防のために弾性ストッキングや間欠的空気圧迫法、抗凝固薬が推奨される。ワルファリン、アスピリン、ヘパリン、直接作用型経口抗凝固薬(DOAC)など従来の抗凝固薬は、効きすぎるとコントロール困難な出血を来すこともあるため、血栓予防効果と安全性、価格のバランスが重要となる。 第XI因子は内因系経路の要であり、その活性化を阻害することで、止血に影響を与えずに血栓形成を抑制できる可能性がある。オソシマブ(osocimab)は、初めての第XIa因子を阻害する長時間作用型ヒトモノクローナル抗体である。静注後1~ 4時間で最高血中濃度に達し、半減期は30~44日である。今回報告された国際多施設共同無作為化第2 相FOXTROT試験では、片側人工膝関節置換術を受ける患者813人をオソシマブ群、エノキサパリン群、またはアピキサバン群に割り付け、VTE予防効果と安全性を検討した。 オソシマブは術後に単回(0.3、0.6、1.2、1.8mg/kg)、または術前に単回(0.3、1.8mg/kg)静注された。一方、エノキサパリンは周術期から40mgを1日1回、10日間以上皮下注、アピキサバンは術後から2.5mgを1日2回、10日間以上経口投与された。主要評価項目は術後10~13日間におけるVTE発生(評価対象600人)、安全性評価項目は同時期の臨床的に問題となる出血とした。 VTE発生に関して、術後投与のオソシマブ0.6、1.2、1.8mg/kg群および術前投与1.8 mg/kg群はエノキサパリンに対して非劣性であり、術前投与のオソシマブ1.8mg/kgはエノキサパリンに対して優越性を示した(リスク差, 15.1%)。臨床的に問題となる出血の発生率は、エノキサパリン群5.9%、アピキサバン群2%に対して、オソシマブ群では最大4.7%(術前1.8 mg/kg群)であった。効果、安全性ともオソシマブは0.6mg / kg以上の用量でエノキサパリンと同等またはやや優れていたが、アピキサバンには及ばなかった。 日本の約10倍の手術件数が施行されている米国では、薬価が重視されることもあり、アスピリン服用が標準となっている。日本の臨床の現場では、エノキサパリンは1日2回、毎日の皮下注であるのに対し、単回静注のオソシマブは医療者と患者の負担軽減という面ではプラスであるが、出血事象が生じた際のリバースが困難となる可能性があり、薬価も含めて、実用化にはさらなる研究が必要であろう。
HMG-CoA還元酵素の遺伝的代替阻害と上皮性卵巣がんの関連
HMG-CoA還元酵素の遺伝的代替阻害と上皮性卵巣がんの関連
Association Between Genetically Proxied Inhibition of HMG-CoA Reductase and Epithelial Ovarian Cancer JAMA. 2020 Feb 18;323(7):646-655. doi: 10.1001/jama.2020.0150. 原文をBibgraph(ビブグラフ)で読む 上記論文の日本語要約 【重要性】前臨床および疫学的試験から、スタチンによる上皮性卵巣がんリスクの化学的予防作用の可能性が示唆されている。 【目的】一般住民とBRCA1/2遺伝子変異保持者の間で、3-ヒドロキシ-3-メチルグルタリル補酵素A(HMG-CoA)還元酵素(スタチンの標的となるHMG-CoA還元酵素の機能低下による遺伝子変異など)と上皮性卵巣がんの関連を評価すること。 【デザイン、設定および参加者】ゲノムワイド関連研究(GWAS)のメタ解析(19万6475例)でLDLコレステロールとの関連が示されているHMGCR、Niemann-Pick C1-Like 1(NPC1L1)およびプロタンパク転換酵素サブチリシン/ケキシン9型(PCSK9)の一塩基多型(SNP)を用いて、HMG-CoA還元酵素、NPC1L1およびPCSK9それぞれの治療による代替阻害を実施した。Ovarian Cancer Association Consortium(OCAC、6万3347例)に登録された浸潤性上皮性卵巣がんの症例対象解析のGWASメタ解析およびConsortium of Investigators of Modifiers of BRCA1/2(CIMBA、3万1448例)に登録されたBRCA1/2変異陽性上皮性卵巣がんの後ろ向きコホート解析のGWASメタ解析から要約統計量を取得。2つのコンソーシウムで、1973年から2014年にかけて参加者を登録し、2015年まで追跡した。OCACは14カ国、CIMBAは25カ国から参加者を登録した。SNPを複数対立遺伝子モデル(multi-allelic model)に統合し、逆分散法ランダム効果モデルを用いて標的の終生阻害(lifelong inhibition)を表すメンデル無作為化推定値を求めた。 【曝露】HMG-CoAの遺伝的代替阻害を主要曝露、NPC1L1およびPCSK9の遺伝的代替阻害および遺伝的代替循環LDLコレステロール値を副次曝露とした。 【主要転帰および評価項目】全体および組織型別の浸潤性上皮性卵巣がん(一般集団)および上皮性卵巣がん(BRCA1/2遺伝子変異保持者)とし、卵巣がんオッズ(一般集団)とハザード比(BRCA1/2遺伝子変異保持者)で測定した。 【結果】OCAC標本には浸潤性上皮性卵巣がん女性2万2406例および対照4万941例、CIMBA標本には上皮性卵巣がん女性3887例および対照2万7561例を対象とした。2件のコホートの年齢中央値は41.5歳から59.0歳までと幅があり、全参加者が欧州系であった。主要解析では、LDLコレステロール値1mmol/L(38.7mg/dL)に相当するHMG-CoA還元酵素の遺伝的代替阻害で上皮性卵巣がんリスクが低下した(オッズ比[OR]0.60、95%CI 0.43-0.83、P=0.002)。BRCA1/2遺伝子変異保持者では、HMG-CoA還元酵素の遺伝的代替阻害で卵巣がんリスクが低下した(ハザード比0.69、95%CI 0.51-0.93、P=0.91)。副次解析で、NPC1L1(OR 0.97、95%CI 0.53-1.75、P=0.91)、PCSK9(OR 0.98、95%CI 0.85-1.13、P=0.80)および循環LDLコレステロール(OR 0.98、95%CI 0.91-1.05、P=0.55)の遺伝的代替阻害と上皮性卵巣がんに有意な関連はなかった。 【結論および意義】HMG-CoA還元酵素の遺伝的代替阻害があると上皮性卵巣がんのオッズが低下した。しかし、この結果は、HMG-CoA還元酵素を阻害する薬剤によってリスクが低下することを示唆するものではなかった。このような薬剤に同等の関連があるかを理解するために、詳細な研究が必要である。 第一人者の医師による解説 スタチンの卵巣がん治療への応用につながる可能性 青木大輔 慶應義塾大学医学部産婦人科教授 MMJ. October 2020; 16 (5):142 スタチンはメバロン酸合成経路の上流に位置するヒドロキシメチルグルタリル補酵素 A(HMG-CoA)還元酵素を阻害することによって、血液中のコレステロール値を低下させるため、脂質異常症治療薬として世界中で汎用されている薬物である。加えて、抗炎症作用や血管拡張、凝固・線溶などの血管リモデリング抑制作用など多面的作用を発揮し、冠動脈疾患や心不全、不整脈などへの予防効果も明らかとなりつつある。さらに、デンマークの疫学研究により、がんと診断される前にスタチンを使用したがん患者は、がんによる死亡のリスクが15%低いという解析結果が報告されたことにより(1)、スタチンの抗腫瘍効果に注目が集まることとなった。この報告を皮切りに、大腸がん、前立腺がん、乳がんなどにおいて同様の疫学研究や基礎実験で抗腫瘍効果が報告されてきている。卵巣がんに対しても、その有用性が基礎的実験レベルで明らかとなりつつあり(2)、臨床への応用も期待されてきている。一方で、スタチンとがん発症率・死亡率との間に因果関係を認めないとの報告もあり議論が続いていた。  本研究は、上皮性卵巣がん患者22,406人と対照者 40,941人、BRCA1/2 変異陽性上皮性卵巣がん患者3,887人と対照者27,561人を対象に、HMG-CoA還元酵素の機能低下に関連する遺伝子の一塩基多型(SNP)により同酵素が遺伝的代替阻害を受けている人の上皮性卵巣がん発症リスクについて検討した。その結果、LDLコレステロール1mmol/L(38.7mg/dL)低下に相当するHMG-CoA還元酵素の遺伝的代替阻害により上皮性卵巣がん発症リスクが40%低くなること(オッズ比 ,0 .60;95 % CI, 0 .43 ~ 0 .83;P= 0 .002)、BRCA1/2変異保持者でも上皮性卵巣がん発症リスクが31%低くなること(ハザード比 , 0.69;95% CI, 0.51~0.93;P=0.01)が示された。  本研究はこれまで議論の続いていたスタチンと卵巣がん抑制の関連性について遺伝子レベルから検討している点が非常に興味深く、その研究成果からHMG-CoA還元酵素を薬理的に阻害しているスタチンでも同様の効果が得られる可能性が想起される。一方で、スタチン投与により卵巣がん発症リスクを抑制したとの臨床試験データは依然として得られていない。今後はスタチンが奏効する卵巣がんの臨床病理・分子生物学的因子の探索が必要である。 1. Nielsen SF, et al. N Engl J Med. 2012;367(19):1792-1802. 2. Kobayashi Y, et al. Clin Cancer Res. 2015;21(20):4652-4662.
限局性前立腺癌のアンドロゲン抑制療法実施の有無別にみた積極的監視、手術、小線源療法、放射線外部照射療法の5年にわたる患者報告転帰
限局性前立腺癌のアンドロゲン抑制療法実施の有無別にみた積極的監視、手術、小線源療法、放射線外部照射療法の5年にわたる患者報告転帰
Patient-Reported Outcomes Through 5 Years for Active Surveillance, Surgery, Brachytherapy, or External Beam Radiation With or Without Androgen Deprivation Therapy for Localized Prostate Cancer JAMA. 2020 Jan 14;323(2):149-163. doi: 10.1001/jama.2019.20675. 原文をBibgraph(ビブグラフ)で読む 上記論文の日本語要約 【重要性】低リスクおよび高リスク限局性前立腺癌に用いる現在の治療戦略の有害作用を理解することで、治療選択に有用な情報を得ることができると思われる。 【目的】前立腺がん治療後5年間の転帰を機能的転帰を比較すること。 【デザイン、設定および参加者】2011年から2012年の間に診断を受けた低リスク(臨床分類cT1~cT2bN0M0、PSA 20ng/mL以下、グレードグループ1~2を満たす)前立腺癌患者1385例と高リスク(臨床分類cT2cN0M0、PSA 20~50ng/mL、グレードグループ3-5のいずれかに該当)前立腺癌患者619例を対象とした前向き住民対象コホート研究。Surveillance, Epidemiology and End Results(SEER)プログラム5施設および米前立腺がん登録から開始し、2017年9月まで追跡した。 【曝露】低リスク患者への積極的監視(363例)、神経温存前立腺全摘除(765例)、放射線外部照射療法(EBRT、261例)または低線量率小線源療法(87例)による治療、高リスク患者への前立腺全摘除(402例)またはアンドロゲン抑制療法併用EBRT(217例)。 【主要転帰および評価項目】26項目からExpanded Prostate Index Composite(0~100点)を基に判定した治療5年後の患者報告機能。調査開始時の機能、患者背景および腫瘍の特徴を回帰モデルで調整した。性機能10~12点、尿失禁6~9点、排尿刺激症状5~7点、排便およびホルモン機能4~6点を臨床的に意義がある最重要差とした。 【結果】計2005例が適格基準を満たし、追跡開始時と開始後1回以上の調査を完了した(年齢中央値64歳、77%が非ヒスパニック系白人)。低リスク前立腺癌患者では、神経温存前立腺摘除が積極的監視療法と比べて5年時の尿失禁(調整平均差-10.9、95%CI -14.2~-7.6)および3年時の性機能(同-15.2、-18.8~-11.5)が不良だった。低線量率小源線療法は、積極的監視療法と比べると1年時の排尿刺激症状(同-7.0、-10.1~-3.9)、性機能(同-10.1 [95% CI, -14.6 to -5.7)および排便機能(同-5.0、-7.6~-2.4)が不良だった。EBRTは、5年間のいずれの時点でも、排尿機能、性機能および排便機能の変化に監視療法との臨床的に重要な差はなかった。高リスク患者では、前立腺全摘除と比べると、アンドロゲン抑制療法併用EBRTで6カ月時のホルモン機能(同-5.3、-8.2~-2.4)および1年時の排便機能(同-4.1、-6.3~-1.9)が低下したが、5年時の性機能(同12.5、6.2~18.7)および5年間の尿失禁(同23.2、17.7~28.7)が良好だった。 【結論および意義】限局性前立腺癌患者のコホートでは、現在の治療選択肢による機能的な差のほとんどが5年間で縮まった。しかし、前立腺全摘除を施行した患者はその他の選択肢と比べて5年間で臨床的に重要な差を認める尿失禁を報告し、前立腺全摘除を施行した高リスク患者はアンドロゲン抑制療法併用EBRTを実施した患者よりも5年時の性機能の悪化を報告した。 第一人者の医師による解説 各治療オプションにおける機能的アウトカム 患者説明において有用なデータ 神鳥 周也(講師)/西山 博之(教授) 筑波大学医学医療系腎泌尿器外科学 MMJ. October 2020; 16 (5):141 限局性前立腺がんの治療は、監視療法、手術や放射線治療など治療の選択肢が多く、10年がん特異的生存率はほぼ100%であり(1)、治療に伴う合併症による生活の質(QOL)低下が治療法を決定するうえで重要な因子の1つである。今回報告された前向きコホート研究では、米国のSurveillance,Epidemiology and End Results(SEER)プログラムおよび米国前立腺がん登録において2011~12年に限局性前立腺がんと診断された男性2,005人を対象とし、各治療から5年間の機能的アウトカム(排尿、排便、性、ホルモン機能)を限局性前立腺がん患者の健康関連 QOLの調査票であるExpanded Prostate Cancer Index Composite(EPIC)を用いて検証している。  低リスクの患者(cT1 ~ cT2bN0M0、前立腺特異抗原[PSA]20 ng/mL以下およびグレード分類1 ~ 2)では、監視療法(363人)、神経温存前立腺全摘術(675人)、外照射(261人)、低線量率小線源治療(87人)が行われていた。また、高リスクの患者(ステージ cT2cN0M0、PSA 20 ~50ng/mLまたはグレード分類3~5)では、前立腺全摘術(402人)、アンドロゲン除去療法(ADT)併用外照射(217人)が行われていた。  低リスクの患者では、神経温存前立腺全摘術は監視療法と比較して5年時の尿失禁(補正平均差 ,-10.9)、3年時の性機能(-15.2)の悪化を認めた。低線量率小線源治療は監視療法と比較して1年時の排尿刺激症状(-7.0)、性機能(-10.1)、排便機能(-5.0)の悪化を認めた。一方、外照射は監視療法と比較して5年間のいずれの時点でも排尿機能、性機能、排便機能に臨床的に重要な差は認められなかった。高リスクの患者では、ADT併用外照射は前立腺全摘術と比較して6カ月時点のホルモン機能(-5.3)や1年時の排便機能(-4.1)の悪化を認めたが、5年時の性機能(12.5)および5年 間の尿失禁(23.2)は良好であった。  本研究ではこれまでの報告(2),(3)とは異なり、多くの患者がロボット手術や強度変調放射線治療(IMRT)を受けており、新しい治療モダリティによる機能的アウトカムを示している。日本における限局性前立腺がん診療の現状に即したデータであり、臨床医が患者への治療オプションを説明する際に有用であると思われる。一方、これらの治療は期待余命が10年以上見込まれる患者に対して選択されるため、長期的なQOL調査の実施が期待される。 1. Hamdy FC, et al. N Engl J Med. 2016;375:1415-1424. 2. Sanda MG, et al. N Engl J Med. 2008;358:1250-1261. 3. Donovan JL, et al. N Engl J Med. 2016;375:1425-1437.
2017 ACC/AHA血圧ガイドラインで定義した孤立性拡張期高血圧症と心血管転帰発症との関連
2017 ACC/AHA血圧ガイドラインで定義した孤立性拡張期高血圧症と心血管転帰発症との関連
Association of Isolated Diastolic Hypertension as Defined by the 2017 ACC/AHA Blood Pressure Guideline With Incident Cardiovascular Outcomes JAMA. 2020 Jan 28;323(4):329-338. doi: 10.1001/jama.2019.21402. 原文をBibgraph(ビブグラフ)で読む 上記論文の日本語要約 【重要性】2017年米国心臓病学会・米国心臓協会(ACC/AHA)ガイドラインでは、高血圧症の定義を血圧140/90mmHg以上から130/80mmHg以上に引き下げた。新たな拡張期血圧閾値80mmHgは、専門家の意見と孤立性拡張期高血圧症(IDH)の定義変更を基に推奨された。 【目的】米国のIDH有病率を2017 ACC/AHAと2003年米国合同委員会(JNC7)による定義で比較し、IDHと転帰の横断的および縦断的な関連を明らかにすること。 【デザイン、設定および参加者】米国国民健康栄養調査(NHANES 2013~2016年)の横断的解析および動脈硬化症リスク(ARIC)試験(1990~1992年に調査開始、2017年12月31日まで追跡)の縦断的解析。縦断的結果を2つの外部コホート――(1)NHANES III(1988~1994年)とNHANES 1999~2014、(2)Give Us a Clue to Cancer and Heart Disease(CLUE)IIコホート(1989年に調査開始)――を用いて検証した。 【曝露】2017 ACC/AHA(収縮期血圧130mmHg未満、拡張期血圧80mmHg以上)とJNC7(収縮期血圧140mmHg未満、拡張期血圧90mmHg以上)で定義したIDH。 【主要転帰および評価項目】米国成人のIDH有病率および2017 ACC/AHAガイドラインによりIDHに対する薬物治療を推奨された米国成人の割合。ARIC試験では、アテローム動脈硬化性心血管疾患(ASCVD)、心不全、慢性腎臓病(CKD)の発症リスク。 【結果】被験者集団はNHANESから9590例(調査開始時の平均年齢49.6歳、52.3%が女性)とARIC試験から8703例(調査開始時の平均年齢56.0歳、57.2%が女性)を対象とした。NHANESのIDH推定有病率は2017 ACC/AHAガイドライン定義で6.5%、JNC7定義で1.3%だった(絶対差5.2%、95%CI 4.7~5.7%)。新たにIDHに分類された被験者のうち推定0.6%(95%CI 0.5-0.6%)がガイドラインの降圧治療の基準を満たした。正常血圧のARIC試験参加者と比べると、2017 ACC/AHA定義によるIDHにASCVD(1386件、追跡期間中央値25.2年、HR 1.06、95%CI 0.89~1.26)、心不全(1396件、HR 0.91、95%CI 0.76~1.09)、CKD(2433件、HR 0.98、95%CI 0.65~1.11)の発症リスクとの有意な関連が認められなかった。2件の外部コホートでもまた、心血管死との関連が否定的であった[例:2017 ACC/AHA定義によるIDHのHRがNHANES(1012件)で1.17、95%CI 0.87~1.56、CLUE II(1497件)で1.02、95%CI 0.92~1.14]。 【結論および意義】米国成人を対象とした本解析では、IDHの推定有病率は2017 ACC/AHA血圧ガイドラインの定義の方がJNC7ガイドラインよりも高かった。しかし、IDHによる心血管転帰のリスクの有意な上昇は見られなかった。 第一人者の医師による解説 拡張期血圧がIDH基準の80mmHg以上なら経過観察を 下澤 達雄 国際医療福祉大学医学部臨床検査医学主任教授 MMJ. October 2020; 16 (5):130 数年に1度、高血圧診療ガイドラインは改訂されており、日本でも2019年に新しい版が発行された(1)。日本と米国のガイドラインで大きく異なる点は高血圧の定義となる血圧の臨床判断値であろう。日本は従来どおりの140/90mmHgを高血圧の臨床判断値としているが、米国は2017年のACC/AHAガイドラインで130/80mmHgへと変更した。これに伴い、2003年 のJoint National Committee(JNC7)において拡張期血圧90mmHg以上かつ収縮期血圧140mmHg未満としていた孤立型拡張期高血圧症(isolated diastolic hypertension; IDH)の定義が拡張期血圧80mmHg以上かつ収縮期血圧130mmHg未満に変更された。その結果、米国のデータベースを用いた今回の横断的調査によると、IDHの頻度 は1.3%(2003 JNC7)から6.5%(2017ACC/AHA)へと上昇し、治療介入対象者が増える結果となった。しかし、Atherosclerosis Risk in Communities(ARIC)試験参加者のデータを再解析したところ、新しいIDHの定義は心血管イベントや慢性腎臓病(CKD)発症リスク、死亡リスクとの関連において従来の定義より優れていることは示されなかった。  疫学調査では拡張期血圧が75mmHgを超えると徐々に心血管イベントが増加することから、ACC/AHAガイドラインでは高血圧の臨床判断値を80mmHgに引き下げた。しかし、IDHの収縮期血圧は130mmHg未満であり、平均血圧にすると拡張期、収縮期とも130/80mmHgを超える例より低くなる。また、介入試験においてHOT試験(2)のように拡張期血圧を90mmHgまたは80mmHgまで下げても心血管イベント抑制効果に差は認められないといった報告もある。つまり、観察研究と介入研究で差異がある。  さらに、今回の検討ではIDHの定義を満たす成人の年齢構成は55歳未満が多くなっている。また従来の定義で診断されるIDHに比べ、低比重リポ蛋白(LDL)-コレステロール値、トリグリセリド値が低く、脂質異常症に対する介入割合が高く、推算糸球体濾過量(eGFR)も高くなっている。ウィンドケッセルモデルからもわかるように若年高血圧患者では血管の弾力性が保たれているため拡張期血圧は高くなりやすい。そのため心血管リスクとしてのIDHの意義が薄れていた可能性も考えられる。  この結果から拡張期血圧は放置してよいということにはならず、米国のIDHの基準であれば経過観察を行い、収縮期血圧が上がってくるようであれば生活習慣の改善から介入を進めるべきであろう。 1. 高血圧診療ガイドライン 2019 年版(日本高血圧学会編) 2. Hansson L et al. Lancet. 1998;351(9118):1755‐1762.
米国成人の1日の歩数および歩行強度の死亡率との関連
米国成人の1日の歩数および歩行強度の死亡率との関連
Association of Daily Step Count and Step Intensity With Mortality Among US Adults JAMA. 2020 Mar 24;323(12):1151-1160. doi: 10.1001/jama.2020.1382. 原文をBibgraph(ビブグラフ)で読む 上記論文の日本語要約 【重要性】1日当たりの歩数と歩行強度が死亡率低下と関連があるか明らかになっていない。 【目的】歩数および歩行強度と死亡率の用量依存関係を明らかにすること。 【デザイン、設定および参加者】全米健康栄養調査(National Health and Nutrition Examination Survey)に参加し最長7日間、加速度計を装着した40歳以上の米国成人の代表標本(2003~2006年)。2015年12月まで死亡を確認した。 【曝露】加速度計で測定した1日当たりの歩数および3段階の強度測定(歩調の平均速度、30分間最大値、1分間最大値)。加速度計のデータは、調査開始から7日間に取得した測定値を基にした。 【主要転帰および評価項目】主要転帰は、全死因死亡とした。心血管疾患(CVD)および癌による死亡を副次転帰とした。3次スプラインと四分位分類を用いて年齢、性別、人種・民族、教育、食習慣、喫煙状況、BMI、自己申告の健康状態、運動制限および糖尿病、脳卒中、心疾患、心不全、癌、慢性気管支炎、肺気腫の診断で調整し、ハザード比(HR)、死亡率および95%信頼区間を推算した。 【結果】計4840例(平均年齢56.8歳、54%が女性、36%が肥満)が加速度計を平均5.7日間、1日平均14.4時間装着した。1日当たりの平均歩数は9124歩であった。追跡期間平均10.1年間で1165例が死亡し、そのうち406例がCVD死、283例が癌死であった。調整前の全死因死亡発生密度は、1日4000歩未満の655例で1000人年当たり76.7(死亡419例)、1日4000~7999歩の1727例で1000人年当たり21.4(死亡488例)、1日8000~11999歩の1539例で1000人年当たり6.9(死亡176例)、1日12000歩以上の919例で1000人年当たり4.8(死亡82例)だった。1日当たりの歩数4000歩と比べると1日当たりの歩数8000歩(HR 0.49、95%CI ~0.55)、12000歩(HR 0.35、95%CI 0.28~0.45)で全死因死亡が有意に低下した。歩調の30分間最大値別の調整前の全死因死亡発生密度は、18.5~56.0歩/分の1080例で1000人年当たり32.9(死亡406例)、56.1~69.2歩/分の1153例で1000人年当たり12.6(死亡207例)、69.3~82.8歩/分の1074例で1000人年当たり6.8(死亡124例)、82.9~149.5歩/分の1037例で1000人年当たり5.3(死亡108例)だった。1日当たりの総歩数で調整すると、歩行強度が上がっても死亡率の有意な低下は認められなかった(例:歩調の30分最大値最低四分位に対する最高四分位のHR 0.90、95%CI 0.65~1.27、傾向のP=0.34)。 【結論】米国成人の代表標本を基にすると、1日当たりの歩数が多いほど全死因死亡が有意に低下した。1日の総歩数で調整すると、歩行強度と死亡率に有意な関連はなかった。 第一人者の医師による解説 複雑な歩行強度より単純な歩数が有用 患者の運動指導に有用なエビデンス 石橋 由基/岡村 智教(教授) 慶應義塾大学医学部衛生学公衆衛生学教室 MMJ. October 2020; 16 (5):147 身体活動がさまざまな疾患を予防することはよく知られており、中でも歩行は広く取り入れられている。また単なる歩数ではなく、歩行強度と呼ばれる歩行速度に焦点を当てた研究も近年注目されており、歩行速度が死亡リスクの低下と関連しているとの報告もある(1)。しかしながら、どのくらいの歩行速度が良いのか、また歩数と歩行速度のどちらがより健康に資するのかは明らかになっていない。  本研究は、全米健康栄養調査(National Health and Nutrition Examination Survey)に参加し、2003~06年に加速度計(ActiGraph model 7164)を着用した40歳以上の成人を対象とし、1日あたりの歩数と歩行強度(歩行強度の評価には以下の3つの指標を用いた:歩調の平均速度[60歩 /分で2分以上の歩行での速度]、30分間最大値[peak30]、1分間最大値[peak1])と死亡率との間の用量反応関係を記述するために実施された。主要アウトカムは全死亡、副次アウトカムは、心血管疾患死とがん死とされた。補正前全死亡率は、1日の歩数が4,000歩未満の群で76.7/1,000人年、同4,000~7,999歩群で21.4/1,000人年、同8,000~11,999歩群で6.9/1,000人年、同12,000歩以上群で4.8/1,000人年であり、調整後も4,000歩 /日と比べて、8,000歩 /日と12,000歩 /日は全死亡率の有意な低下と関連していた。一方、peak30別にみた補正前全死亡率は歩行強度に従って低下傾向にあったが、総歩数/日で調整すると、歩行強度が増大しても死亡率の低下はみられなかった。これは他の歩行強度の指標でも同様の傾向であった。  本研究は加速度計の測定を加味した歩行強度に比べて、より単純に計測できる1日の歩数がより低い死亡率と関連していることを明らかにした点で新規性がある。日本でも「健康日本21」で、男女別に歩数の目標値が設定されている(2)。本研究は歩行強度という複雑な指標よりも日々の歩数という単純な指標が予後予測に有用であったことを示唆した点で、患者の運動指導にも有用なエビデンスになりうるだろう。 1. Studenski S et al. JAMA. 2011;305(1):50-58. 2. 厚生労働省 健康日本 21 目標値一覧 (https://www.mhlw.go.jp/www1/topics/kenko21_11/t2a.html)
酸素投与不要の細気管支炎入院患児に対するパルスオキシメータによる持続的監視実施率
酸素投与不要の細気管支炎入院患児に対するパルスオキシメータによる持続的監視実施率
Prevalence of Continuous Pulse Oximetry Monitoring in Hospitalized Children With Bronchiolitis Not Requiring Supplemental Oxygen JAMA. 2020 Apr 21;323(15):1467-1477. doi: 10.1001/jama.2020.2998. 原文をBibgraph(ビブグラフ)で読む 上記論文の日本語要約 【重要性】酸素投与の必要がない入院中の細気管支炎患児へのパルスオキシメータ使用は、米国のガイドラインで推奨されていない。 【目的】細気管支炎患児へのパルスオキシメータ持続的使用率の評価。 【デザイン、設定および参加者】2018年12月1日から2019年3月31日にかけて、Pediatric Research in Inpatient Settings (PRIS) Networkに参加する米国およびカナダの56施設の小児病棟で多施設共同横断研究を実施した。酸素投与中でない生後8週~23カ月の細気管支炎患児の恣意的標本を対象とした。超早産、チアノーゼ性先天性心疾患、肺高血圧症、在宅呼吸サポート、神経筋疾患、免疫不全、癌がある患児を除外した。 【曝露】酸素投与中でない細気管支炎による入院。 【主要転帰および評価項目】主要転帰はパルスオキシメータの持続的使用とし、直接観察して測定した。パルスオキシメータの持続的使用の割合を以下の変数を用いてリスク標準化した――夜間(午後11時~午前7時)、早産と合わせた月齢、酸素補充療法または高流量酸素療法離脱後の時間、原疾患発症後の無呼吸またはチアノーゼの有無、神経学的障害および経管栄養の有無。 【結果】標本は、独立小児病院33施設、病院内小児病院14施設および地域病院9施設に入院した3612例を対象とした。標本内の59%が男児、56%が白人、15%が黒人で、48%が生後8週間~5カ月、28%が6~11カ月、16%が12~17カ月、9%が18~23カ月であった。全体のパルスオキシメータの持続的使用率は46%(95%CI 40~53%)で、酸素補充療法や高流量鼻カニューレ酸素療法を実施している患者がいなかった。病院別の調製前パルスオキシメータの持続的使用率は、2~92%と幅があった。リスクを標準化すると、資料率が6~82%になった。級内相関係数から観察された変数の27%(同19~36%)が評価対象外の病院別の因子によるものであった。 【結論および意義】酸素投与中でない入院中の細気管支炎患児の恣意的標本で、パルスオキシメータの持続的使用が頻繁に実施されており、病院によって大きな差があった。この患者集団ではガイドラインや科学的根拠に基づいた持続的監視の適応がないため、過度に使用されているものと思われる。 第一人者の医師による解説 酸素投与中止後はパルスオキシメータを外すべきなのか? 高瀬 真人 日本医科大学多摩永山病院小児科部長 MMJ. October 2020; 16 (5):139 急性細気管支炎は乳幼児期(2歳未満)にRSウイルス(RSV)に初感染した際などに多く経験される急性の閉塞性換気障害で、特に月齢の若い乳児では突然の呼吸停止で死亡する例もあり、恐れられている。喘息急性増悪とは異なり気管支拡張薬に対する反応性が乏しいことが特徴である。日本でも米国、欧州と同様に乳児期最大の入院原因である。エビデンスの確立した特異的治療は存在せず、入院例では経口摂取不良に対する点滴補液や低酸素血症に対する酸素投与などの支持療法が行われ、重症例では人工呼吸管理も行われる。米国小児科学会(AAP)のガイドラインでは、本症で入院中のパルスオキシメータによる持続的 SpO2モニタリングについて、予後に影響なく入院期間を長引かせる疑いからスポット測定での代替が推奨され、Society of Hospital Medicineは“Choosing Wisely Recommendations”において過剰な診療の例に挙げている(1)。  本論文では、米国とカナダの小児医療機関56施設に2018年12月~19年3月に急性期病棟(集中治療室以外)に入院した2歳未満の急性細気管支炎症例(合計3,612人)を対象に酸素投与中でない入院期間における持続的 SpO2モニタリングの実施率を調査した結果、病院によって2%から92%(平均46%)と大差を認め、過剰診療がいまだに多く行われているとの評価に至っている。  ところでAAPのガイドラインでは酸素投与中止後の持続的 SpO2モニタリングを行わない方向性を示してはいるが、これは低いエビデンスレベルに基づく弱い推奨である。その後発表されたランダム化対照試験でも、入院期間の短縮効果は示されていない(2)。今回の研究では持続的 SpO2モニタリング実施率に大差が認められたが、それと平均在院日数との相関が検討されていないのは残念である。  ちなみに、米国では5歳未満のRSV感染症死亡数が年間100~500人と推計されているが(3)、日本では5歳未満のRSV感染症死亡数は2018年には3人、急性細気管支炎による死亡数はゼロで、死亡率は極めて低い(厚生労働省:人口動態統計)。米国では医療費の高騰に対してエビデンスに基づかない治療をやめて費用対効果を最大化することが喫緊の課題となっており、“Choosing Wisely Campaign”の隆盛に結びついているが、わが国では安全第一の意識が強く、呼吸器疾患で入院中の乳幼児からパルスオキシメータを一刻も早く外そうという機運はあまり高まりそうにない。 1. Quinonez RA, et al. J Hosp Med. 2013;8:479-485. 2. McCulloh R, et al. JAMA Pediatr. 2015;169:898-904. 3. CDC ウエブサイト〈https://bit.ly/3kka9Wy〉
急性心不全患者の死亡率と心不全による再入院に対する包括的な血管拡張戦略と通常ケアの比較:GALACTICランダム化臨床試験
急性心不全患者の死亡率と心不全による再入院に対する包括的な血管拡張戦略と通常ケアの比較:GALACTICランダム化臨床試験
Effect of a Strategy of Comprehensive Vasodilation vs Usual Care on Mortality and Heart Failure Rehospitalization Among Patients With Acute Heart Failure: The GALACTIC Randomized Clinical Trial JAMA. 2019 Dec 17;322(23):2292-2302. doi: 10.1001/jama.2019.18598. 原文をBibgraph(ビブグラフ)で読む 上記論文の日本語要約 重要性: 単一の血管拡張薬の短期間投与は通常は固定用量が用いられますが、急性心不全(AHF)の患者の転帰を改善していません。 目的: AHF患者に対して、血管拡張薬を個別設定の漸増用量で使用する、早期の集中的かつ持続的な血管拡張戦略の効果を評価することを目的としました。 試験デザイン、設定、および対象: 呼吸困難、ナトリウム利尿ペプチドの血漿濃度の上昇、少なくとも100 mmHgの収縮期血圧があり、スイス、ブルガリア、ドイツ、ブラジル、スペインの10の三次および二次病院の一般病棟で治療を受ける計画のある、AHFで入院した788人の患者を登録したランダム化非盲検盲検エンドポイント試験。登録は2007年12月に開始され、フォローアップは2019年2月に完了しました。 介入: 患者は、入院期間を通して早期の集中的かつ持続的な血管拡張戦略を行う群(n=386)と通常ケア群(n=402)に1:1でランダムに割り付けられました。早期の集中的かつ持続的な血管拡張は、最大かつ持続的な血管拡張の包括的/実用的なアプローチで、個別設定の漸増用量の舌下/経皮硝酸塩、48時間の低用量経口ヒドララジン、および急速に漸増するアンジオテンシン変換酵素阻害薬、アンジオテンシン受容体遮断薬、またはサクビトリル-バルサルタンを組み合わせたものです。 主な結果: 主要エンドポイントは、180日におけるAHFのすべての原因による死亡または再入院の複合としました。 結果: ランダム化された788例の患者のうち、781例(99.1%;年齢中央値78歳; 36.9%女性)が試験を完了し、一次エンドポイント分析に適格でした。180日のフォローアップは779例の患者(99.7%)が完遂しました。 180日でのAHFの全ての死因での死亡率または再入院の複合である主要エンドポイントは、介入群で117例(30.6%)(55人の死亡[14.4%]を含む)、通常ケア群で111例(27.8%)(61人の死亡[15.3%]を含む)でした:(主要エンドポイントの絶対差、2.8%[95%CI、-3.7%-9.3%];調整済みハザード比、1.07 [95%CI、0.83-1.39 ]; P =0.59)。介入群、通常ケア群で最も多くみられた臨床的に意味のある有害事象は、低カリウム血症(23% vs. 25%)、腎機能の悪化(21% vs. 20%)、頭痛(26% vs. 10%)、めまい(15% vs. 10%)、および低血圧(8% vs. 2%)でした。 結論と関連性: AHF患者に対する早期の集中的かつ持続的な血管拡張戦略は、通常ケアと比較して、180日間の全ての原因による死亡とAHF再入院の複合結果を改善しませんでした。 臨床試験登録:ClinicalTrials.gov:NCT00512759 利益相反: Goudev博士は、ファイザー、ノバルティス、アストラゼネカ、アムジェンから個人的な謝金(アドバイザリーボードの謝金と講演料)を受け取っています。Walter博士は、スイス心臓財団とスイス医科学アカデミーから助成金を受け取っています。 Gualandro博士は、サービエ社から謝金を受け取っています。Wenzel博士は、ノバルティス、バイエル、ドイツ連邦教育研究省、アボットから謝金を受け取り、サービエ社から非財務支援を受けました。Pfister博士は、ノバルティス、ビフォー ファーマ、メルク シャープ&ドームから謝礼を受け取り、サノフィとベーリンガーインゲルハイムから助成金を受け取っています。Conen博士は、サービエ・カナダから謝金を受け取りました。Kobsa博士は、バイオセンス ウェブスター、バイオトロニック、メドトロニック、アボット、SI–S メディカル、およびBoston Scientificから助成金を受け取っっています。Munzel博士は、DZHK(ドイツ心臓血管研究センター)パートナーサイトラインマインの主任研究員であることを報告しています。Mueller博士は、スイス国立科学財団、スイス心臓財団、心臓血管研究バーゼル財団、およびスタンリージョンソン財団から助成金、シングレックス、およびブラームスからの助成金、謝金および非財務サポート、ノバルティス、カーディオレンティス、ベーリンガーインゲルハイムからの謝金、アボットからの助成金と非財政的支援を受けています。以上です。 第一人者の医師による解説 急性心不全への積極的・包括的血管拡張薬投与 180日後の予後を改善せず 小林 さゆき1), 石川 哲也2), 中原 志朗3), 田口 功4) 獨協医科大学埼玉医療センター循環器内科 1,2,3;准教授、4;主任教授 MMJ. October 2020; 16 (5):128 日本では超高齢化に伴い2035年をピークに医療体制が疲弊する「心不全パンデミック」が危惧され、米国でも“シルバーツナミ”と称して対策が講じられている。この30年間、急性心不全(AHF)に対する利尿薬と血管拡張薬の併用療法が評価されてきたが、いずれも初期治療における早期集中・継続的な血管拡張薬投与のみでは予後の改善は困難であるという結果であった。  本論文の無作為化GALACTIC試験では、欧州5カ国10施設にAHFで入院した収縮期血圧100mmHg以上の患者788人を対象に、血管拡張薬の早期集中・継続的な投与群(介入群;386人)もしくは通常治療(対照群;402人)に無作為に割り付け、180日時点の複合アウトカム(全死亡およびAHFによる再入院)が比較・評価された。介入群では収縮期血圧90~110mmHgを目標に個別化された用量の舌下・経皮硝酸塩、経口ヒドララジンを投与後、アンジオテンシン変換酵素(ACE)阻害薬、アンジオテンシン受容体拮抗薬(ARB)、サクビトリル-バルサルタンの迅速漸増投与を組み合わせた包括的な管理が行われた。対照群では低用量硝酸塩投与後、他の薬剤の漸増は緩徐に行われた。その結果、主要エンドポイント発生率は介入群30.6%、対照群27.8% (ハザード比[HR], 1.07;P=0.59)で有意差はなかった。  GALACTIC試験でも、先行研究と同様、早期集中・継続的な血管拡張薬の投与は、通常治療に比べ予後を改善しないことが示された。介入群の心不全改善速度をみると、より用量の高いループ利尿薬が使用された対照群と同程度であった。高用量ループ利尿薬は腎機能増悪により心不全予後を悪化させる可能性はあるものの、初期治療では適切なループ利尿薬の効果は高用量の血管拡張薬と同等といえる。しかし、本試験の患者では「急性慢性心不全診療ガイドライン(2017)」のクリニカルシナリオ(CS)分類1と2が混在していることから、同ガイドラインで血管拡張薬をクラス IとしているCS1に限定した解析が期待される。  心不全の長期予後改善を目指した治療は、基礎疾患、左室収縮能、合併症、血行動態などの評価に基づいた個別化治療の検討も必要である。そして、新規薬剤(SGLT2阻害薬(1)、ベルイシグアト(2)、イバブラジン、サクビトリル-バルサルタン、無機亜硝酸塩)も視野に入れ、ガイドラインに沿った適切な薬剤選択と管理に加え、生活習慣改善やリハビリテーションなども含めた包括的管理が慢性期までシームレスに行われることが重要である。 1. McMurray JJV, et al. N Engl J Med 2019; 381:1995-2008. 2. Armstrong PW et al. N Engl J Med 2020 Mar 28.DOI: 10.1056/NEJMoa1915928
ダパグリフロジンが糖尿病がない心不全患者の心不全悪化と心血管死にもたらす効果
ダパグリフロジンが糖尿病がない心不全患者の心不全悪化と心血管死にもたらす効果
Effect of Dapagliflozin on Worsening Heart Failure and Cardiovascular Death in Patients With Heart Failure With and Without Diabetes JAMA. 2020 Mar 27;323(14):1353-1368. 原文をBibgraph(ビブグラフ)で読む 上記論文の日本語要約 【重要性】駆出率が低下した心不全(HFrEF)患者に追加治療が必要である。ナトリウム・グルコース輸送体2(SGLT2)阻害薬は、糖尿病がないHFrEF患者にも有効な治療であると考えられる。 【目的】糖尿病の有無を問わずHFrEF患者に用いるダパグリフロジンの効果を評価すること。 【デザイン、設定および参加者】20カ国410施設で実施された第III相無作為化試験の探索的解析。2017年2月15日から2018年8月17日にかけて、NYHA分類II~IV、左室駆出率が40%以下で、血漿NT-proBNP値が上昇した患者を組み入れ、2019年6月19日まで追跡した。 【介入】推奨治療に追加したダパグリフロジン1日1回10mgまたはプラセボ。 【主要転帰または評価項目】主要転帰は心不全の悪化または心血管死の複合とした。この転帰は、試験開始時の糖尿病の有無で、さらに糖尿病がない患者では糖化ヘモグロビン値5.7%以上と5.7%未満に分けて解析した。 【結果】無作為化した4744例(平均年齢66歳、23%が女性、55%が非糖尿病)のうち4742例が試験を完遂した。糖尿病がない患者で、主要転帰はダパグリフロジン群1298例中171例(13.2%)、プラセボ群1307例中231例(17.7%)に発生した(HR 0.73、95%CI 0.60~0.88)。糖尿病患者では、主要転帰はダパグリフロジン群1075例中215例(20.0%)、プラセボ群1064例中271例(25.5%)に発生した(同0.75、0.63~0.90、交互作用のP=0.80)。糖尿病がなく糖化ヘモグロビン値5.7%未満の患者で、主要転帰はダパグリフロジン群438例中53例(12.1%)、プラセボ群419例中71例(16.9%)に発生した(同0.67、0.47~0.96)。糖化ヘモグロビン値5.7%以上の患者で、主要転帰はダパグリフロジン群860例中118例(13.7%)、プラセボ群888例中160例(18.0%)に発生した(同0.74、0.59~0.94)、交互作用のP=0.72)。糖尿病がない患者でダパグリフロジン群の7.3%、プラセボ群の6.1%、糖尿病患者のそれぞれ7.8%、7.8%に有害事象として体液減少が報告された。糖尿病がない患者でダパグリフロジン群の4.8%、プラセボ群の6.0%、糖尿病患者でそれぞれ8.5%、8.7%に腎有害事象が報告された。 【結論および意義】このHFrEF患者を対象とした無作為化試験の探索的解析では、ダパグリフロジンを推奨治療に追加すると、プラセボと比べて、糖尿病の有無に関係なく心不全の悪化または心血管死を有意に抑制した。 第一人者の医師による解説 細胞外液量減少を介した心負荷減少が心不全改善に寄与の可能性 長田太助 自治医科大学医学部内科学講座腎臓内科学部門教授・附属病院副病院長 MMJ. October 2020; 16 (5):131 EMPA-REG outcome試験(1)を皮切りに、この数年でNa+ /グルコース共役輸送担体2(SGLT2)阻害薬による心血管予後の改善効果を示した大規模臨床試験が次々と発表された(2)。ジペプチジルペプチダーゼ(DPP)-4阻害薬を使ったSAVORTIMI53試験では心不全が増加したのに対し、同じ糖尿病薬でもSGLT2阻害薬は心不全の悪化を抑制することが強く印象づけられた。「心不全患者だけを集めてSGLT2阻害薬を投与した場合、心不全悪化は抑制されるのか?」と誰しも疑問に思うことだろう。DAPA-HF試験(3)がその答えを提示した。同試験ではNYHAクラス II ~ IVで駆出率(EF)40%未満の心不全患者4,744人をダパグリフロジン(DAPA)10mg/日群とプラセボ群に無作為に割り付け、約2年間観察した。主要評価項目(心不全悪化と心血管死亡の複合アウトカム)の発生率はDAPA群16.3%、プラセボ群21.2%、ハザード比(HR)は0.74で統計学的に有意であった。  本研究では、これと同じ母集団を使い、糖尿病群(2,139人)と非糖尿病群(2,605人)に分けたpost hoc解析が実施された。2019年の報告(3)では、複合アウトカム(心不全悪化と心血管死亡)発生率は糖尿病の有無で差がなかったと報告され、糖尿病薬が非糖尿病患者でも心不全を抑制した結果は衝撃だったが、今回はさらに詳しく糖尿病の有無で差が検証された。主要評価項目は本試験と同じで、その発生率は糖尿病群においてDAPA群20.0%、プラセボ群25.5%(HR, 0.75)、非糖尿病群ではDAPA群13.2%、プラセボ群17.7%(HR, 0.73)であり、それぞれの群内で有意差はあったが、糖尿病の有無間で有意な交互作用はなかった。腎機能悪化以外の2次評価項目を含めて結果はすべて同様で、DAPAの心不全悪化抑制効果は糖尿病の有無ならびにベースライン HbA1c値の高低によらずほぼ一定であることも示された。  血糖値改善作用によらないのであれば、DAPAによる心不全抑制機序は何であろうか。本論文でDAPAには糖尿病の有無によらずヘマトクリット値を上げる作用があることが示されており、細胞外液量減少を介した心負荷減少が心不全の改善に寄与している可能性がある。そのほかにもhypoxia-inducible factor (HIF)1活性化とエリスロポエチン産生増加、心筋の線維化抑制作用、ケトン体産生を介した心筋のエネルギー効率改善、交感神経抑制作用など諸説あるが、SGLT2阻害薬の心不全改善作用の正確な機序はまだ不明である。機序が明らかになれば、心不全の薬だが血糖降下作用もある薬とSGLT2阻害薬の立ち位置が変わるのかもしれない。 1. Zinman B et al. N Engl J Med. 2015;373:2117-2128. 2. Zelniker TA et al. Lancet. 2019;393:31-39. 3. McMurray et al. N Engl J Med. 2019;381:1995-2008.
禁煙とその後の心血管疾患リスクの関連
禁煙とその後の心血管疾患リスクの関連
Association of Smoking Cessation With Subsequent Risk of Cardiovascular Disease JAMA. 2019 Aug 20;322(7):642-650. doi: 10.1001/jama.2019.10298. 原文をBibgraph(ビブグラフ)で読む 上記論文の日本語要約 【重要性】禁煙後の心血管疾患(CVD)の経時変化は明らかになっていない。リスク算出に喫煙経験者がわずか5年のみリスクがあることを考慮している。 【目的】禁煙後の年数とCVD発症の関連を明らかにすること。 【デザイン、設定および参加者】調査開始時にCVDがなく2015年12月まで追跡したフラミンガム心臓研究参加者から前向きに収集したデータの後ろ向き解析(第1世代コホート:1954~1958年の4回目の調査の参加者、第2世代コホート:1971~1975年の初回調査の参加者)。 【曝露】更新した自己申告の喫煙状況、禁煙後の年数、累積pack-years。 【主要評価項目】CVD(心筋梗塞、脳卒中、心不全、心血管死)の発症。両コホート(統合)を主要解析の対象とし、大量喫煙経験者(20pack-years以上)に限定した。 【結果】試験集団には8770例(第1世代コホート3805例、第2世代コホート4965例)を対象とし、平均年齢42.2歳、45%が男性だった。5308例が調査開始時に17.2pack-years(中央値)の喫煙経験者で、そのうち2371例が大量喫煙経験者[406例(17%)が元喫煙者、1965例が現喫煙者(83%)]であった。追跡期間中央値26.4年間で、初回CVDイベントが2435件発生した[第1世代コホート1612例(大量喫煙者665例)、第2世代コホート823例(同430例)]。統合コホートでは、現喫煙と比べると、5年以内に禁煙するとCVD発症率(1000人年当たりの発症率:現喫煙11.56(95%CI 10.30~12.98)、5年以内の禁煙6.94(5.61~8.59)、差-4.51(-5.90--2.77)およびCVD発症リスク(HR 0.61、95%CI 0.49~0.76)が有意に低下した。喫煙未経験者と比べると、統合コホートでは、禁煙10~15年後にCVDリスクの有意な上昇が止まった[1000人年当たりの発症率:喫煙未経験5.09(95%CI 4.52~5.74)、10~15年以内の禁煙6.31(4.93~8.09)、差1.27(-0.10-3.05)、HR 1.25(95%CI 0.98~1.60)]。 【結論および意義】大量喫煙者が禁煙すると、現喫煙者より5年以内のCVDリスクが有意に低下した。しかし、喫煙未経験者と比べた元喫煙者のCVDリスクは、禁煙後5年を超えると有意に高くなった。 第一人者の医師による解説 禁煙はすべての人に健康改善をもたらす 日常診療で禁煙を勧め希望者への禁煙治療提供が重要 中村 正和 地域医療振興協会 ヘルスプロモーション研究センター センター長 MMJ. October 2020; 16 (5):135 喫煙の健康影響は、循環器疾患にとどまらず、がん、呼吸器疾患、糖尿病など多岐にわたる。禁煙はこれらのリスクを改善するが、禁煙後の経時的なリスク低下については研究間で結果が異なる。例えば虚血性心疾患のリスクは禁煙10~15年で非喫煙者と同じレベルまで低下するという報告と、禁煙10~20年経過しても非喫煙者より10~20%高いという報告がある(1)。結果が異なる理由として、ベースライン喫煙量の把握が不十分、追跡後の喫煙状況の変化を把握せずにベースライン情報だけで解析しているなど、方法論上の問題が指摘されている。喫煙以外の危険因子の変化についても同様の問題がある場合、交絡要因の調整が不十分となり、結果に影響を与える可能性がある。  本研究では、フラミンガム心臓研究(FHS)の第1世代(original)、第2世代(offspring)コホートの2015年までの追跡データを後ろ向きに解析し、禁煙による循環器疾患リスクの低下を厳密に評価している。FHSは、上述した問題をクリアする質の高い追跡研究であり、循環器疾患発症の把握についても精度が高い。対象者は8,770人、追跡期間の中央値は26.4年に及ぶ。アウトカムの循環器疾患(心筋梗塞、脳卒中、心不全、心血管死)のリスクは、禁煙すると、喫煙を続けた場合に比べ5年以内に約40%低下し、禁煙による顕著なリスク改善がみられた。しかし、非喫煙者に比べ、禁煙後少なくとも10年以内はリスクが40%ほど高い状態が続くことが明らかになった。  本研究により、禁煙しても非喫煙者と同程度のリスクとなるには時間がかかることが確認された。リスク改善のスピードは喫煙本数や年数、禁煙の時期によって異なる。本研究の解析はヘビースモーカー(20パック年以上)に限定しており、その影響も考えられる。  しかし、本研究結果の中で一番重要な点は、喫煙を継続した場合に比べ、禁煙によってリスクが大きく低下することである。循環器疾患を有する場合でも同様の効果が確認されている(1)。糖尿病患者では禁煙によってHbA1cが一時的に上昇するものの、全死亡や循環器疾患のリスクは禁煙とともに確実に低下することが89のコホート研究のメタ解析で明らかになっている(2)。禁煙は性、年齢、疾患の有無を問わず、すべての人に健康改善をもたらす(3)。日常診療の場ですべての喫煙者に禁煙を勧め、禁煙希望者に禁煙治療を提供することが重要である。 1. IARC (2007). IARC Handbooks of Cancer Prevention, Tobacco Control. Volume 11: Reversal of Risk aer Quitting Smoking (p227-268). 2. Pan A, et al. Circulation. 2015;132(19):1795-1804. 3. U.S. Department of Health and Human Services. The Health Benefits of Smoking Cessation: A Report of the Surgeon General. 1990.
MRSA菌血症に用いるバンコマイシンまたはダプトマイシンへの抗ブドウ球菌βラクタム系薬が死亡率、菌血症、効果および治療失敗にもたらす効果 無作為化比較試験
MRSA菌血症に用いるバンコマイシンまたはダプトマイシンへの抗ブドウ球菌βラクタム系薬が死亡率、菌血症、効果および治療失敗にもたらす効果 無作為化比較試験
Effect of Vancomycin or Daptomycin With vs Without an Antistaphylococcal β-Lactam on Mortality, Bacteremia, Relapse, or Treatment Failure in Patients With MRSA Bacteremia: A Randomized Clinical Trial JAMA. 2020 Feb 11;323(6):527-537. doi: 10.1001/jama.2020.0103. 原文をBibgraph(ビブグラフ)で読む 上記論文の日本語要約 【重要性】メチシリン耐性黄色ブドウ球菌(MRSA)菌血症は20%以上の死亡率と関連がある。標準治療にβラクタム系抗菌薬の併用によって死亡率が低下することが報告されているが、この介入を検討した十分な検出力がある無作為化臨床試験は実施されていない。 【目的】MRSA菌血症に用いる抗ブドウ球菌βラクタム系抗菌薬の標準治療との併用が標準治療単独よりも有効性が高いかどうかを明らかにすること。 【デザイン、設定および参加者】2015年8月から2018年7月にかけて、4カ国27施設で、MRSA菌血症成人患者352例を対象に非盲検無作為化臨床試験を実施した。2018年10月23日に経過観察を終了した。 【介入】被験者を標準治療(バンコマイシンまたはダプトマイシン静注)と抗ブドウ球菌βラクタム系抗菌薬(flucloxacillin、クロキサシリンまたはセファゾリン)の併用(174例)と標準治療単独(178例)に割り付けた。治療に当たる医師が総治療期間を決め、βラクタム系抗菌薬は7日間投与した。 【主要転帰および評価項目】主要評価項目は90日時の死亡、5日目の菌血症持続、微生物学的再発および微生物学的治療失敗の複合とした。14、42、90日時の死亡率、2、5日時の菌血症持続、急性腎障害(AKI)、微生物学的再発、微生物学的治療失敗および抗菌薬静注の期間を副次評価項目とした。 【結果】データ安全性評価委員会は、安全性の観点から、440例を登録する前に試験の早期中止を推奨した。無作為化した352例(平均年齢62.2歳、121例[34.4%]が女性)のうち345例(98%)が試験を完遂した。主要評価項目は、併用療法群の59例(35%)と標準治療群の68例(39%)に発生した(絶対差-4.2%、95%CI -14.3-6.0%)。事前に規定した副次評価項目9項目のうち7項目に有意差が認められなかった。併用療法群と標準治療群を比較すると、35例(21%)と28例(16%)に90日総死亡率が発生し(差4.5%、95%CI -3.7-12.7%)、166例中19例(11%)と172例中35例(20%)に5日目に菌血症の持続(同-8.9%、-16.6--1.2%)が見られ、ベースラインで透析を実施していた患者を除くと、145例中34例(23%)と145例中9例(6%)がAKIを来した(同17.2%、9.3-25.2%)。 【結論および意義】MRSA菌血症、バンコマイシンまたはダプトマイシンを用いた標準的抗菌薬療法に抗ブドウ球菌βラクタム系抗菌薬を追加しても、死亡率、菌血症持続、再発または治療失敗の主要複合評価項目の有意な改善にはつながらなかった。この結果を解釈する際は、安全性の懸念による試験の早期中止および臨床的に重要なさを検出する力が試験に不足していた可能性を考慮しなければならない。 第一人者の医師による解説 腎毒性の低いβラクタム薬の併用は 検討の余地あり 塩塚 美歌 国立がん研究センター中央病院感染症部/岩田 敏 国立がん研究センター中央病院感染症部部長 MMJ. December 2020;16(6):175 成人のメチシリン耐性黄色ブドウ球菌(MRSA)菌血症に対する現在の標準治療薬は、バンコマイシン(VCM)とダプトマイシン(DAP)である。近年、MRSA菌血症の標準治療薬とβラクタム薬の併用による患者のアウトカム改善を示唆する研究報告が増えている中、本論文はオーストラリア、シンガポールなどの27病院で実施されたランダム化非盲検比較試験の結果を報告している。血液培養からMRSAが検出された18歳以上の入院患者を対象とし、血液培養採取から72時間以内に次の2群に無作為に割りつけた。標準治療群では、VCMまたはDAPを、臨床経過に従い14~42日間投与した。併用療法群では、標準治療に加えて抗黄色ブドウ球菌βラクタム薬(フルクロキサシリン、クロキサシリン、またはセファゾリン)を治療開始から7日間併用した。主要評価項目は、90日間の全死亡、5日後の持続的菌血症、血液培養陰性化後72時間以降の菌血症再燃、14日目以降における細菌学的な治療失敗を併せた複合エンドポイントとされた。副次評価項目として急性腎障害(AKI)や投薬治療期間なども解析した。当初440人の登録が予定されたが、中間解析で併用療法群のAKI発症率が有意に高いことが示され、以降の参加登録は中止された。  結果として、無作為化された患者352人の年齢中央値は64歳(47~79歳)、349人(99%)はVCM、13人(4%)は最低1回DAPを投与されていた。主要評価項目の発生率に関して併用療法群と標準治療群に有意差は認められなかった(35% 対 39%:差 , -4.2%;95%信頼区間[CI], -14.3~6.0%;P=0.42)。また、併用療法群は標準治療群と比較し、5日後の持続的菌血症の頻度が有意に低い(11% 対 20%:差 , -8.9%;95% CI, -16.6~ -1.2%;P=0.02)一方、AKIの頻度は有意に高かった(23% 対 6%:差 , 17.2%;95% CI, 9.3~25.2%;P<0.001)。  MRSA菌血症に対するβラクタム薬の併用療法は、本研究では腎毒性の弊害を上回る有効性は示されなかった。抗MRSA薬とβラクタム薬の併用療法は、日本感染症学会・日本化学療法学会による「JAID/JSC感染症治療ガイド2019」でも推奨されておらず、これに矛盾のない結果となった。過去にも、黄色ブドウ球菌による心内膜炎に対するアミノグリコシド併用療法(1)や、黄色ブドウ球菌菌血症に対するリファンピシン併用療法(2)の効果が期待されたが、死亡率の改善はなく、有害事象(前者では腎毒性)が多く認められた。今回、腎毒性がより低いセファゾリンを用いた患者数は併用療法群の約20%と少なく、今後改めて比較試験を実施してもよいかもしれない。 1. Korzeniowski O, et al. Ann Intern Med. 1982;97(4):496-503. 2. waites GE, et al. Lancet. 2018;391(10121):668-678.
急性低酸素血症性呼吸不全成人患者に用いる非侵襲的換気療法と全死亡率の関連 系統的レビューとメタ解析
急性低酸素血症性呼吸不全成人患者に用いる非侵襲的換気療法と全死亡率の関連 系統的レビューとメタ解析
Association of Noninvasive Oxygenation Strategies With All-Cause Mortality in Adults With Acute Hypoxemic Respiratory Failure: A Systematic Review and Meta-analysis JAMA . 2020 Jul 7;324(1):57-67. doi: 10.1001/jama.2020.9524. 原文をBibgraph(ビブグラフ)で読む 上記論文の日本語要約 【重要性】急性低酸素血症性呼吸不全成人患者に用いる非侵襲的換気や高流量経鼻酸素などの非侵襲的酸素療法が、標準酸素療法より有効性が高いと思われる。 【目的】急性低酸素血症性呼吸不全成人患者で、各種非侵襲的換気療法の死亡率と気管内切開との関連を比較すること。 【データ入手元】以下の文献データベースの開始から2020年4月までを検索した――MEDLINE、Embase、PubMed、Cochrane Central Register of Controlled Trials、CINAHL、Web of ScienceおよびLILACS。言語、出版年、性別、人種に制限は設けないこととした。 【試験選択】急性低酸素血症性呼吸不全成人患者を組み入れ、高流量経鼻酸素、フェイスマスク型非侵襲換気、ヘルメット型非侵襲的換気および標準的酸素療法を比較した無作為化臨床試験。 【データ抽出および合成】レビュアー2人が別々に個別試験データを抽出し、コクランバイアスリスクツールを用いてバイアスリスクを評価した。リスク比(RR)およびリスク差を95%信用区間(CrI)と共に得るため、ベイジアンフレームワークを用いたネットワークメタ解析を実施した。根拠の確実性の等級付けには、GRADEの方法論を用いた。 【主要転帰および評価項目】主要転帰は、最大90日間の全死因死亡率に規定した。最大30日間の気管挿管を副次評価項目とした。 【結果】無作為化試験25件(参加者3804例)を対象とした。標準酸素療法と比べると、ヘルメット型非侵襲的換気(RR 0.40、95%CrI 0.24-0.63、絶対リスク差-0.19、95%CrI -0.37--0.09、低度の確実性)とフェイスマスク型非侵襲的換気(RR 0.83、95%CrI 0.68-0.99、絶対リスク差-0.06、95%CrI -0.15--0.01、中等度の確実性)の死亡リスクが低かった(21試験、3370例)。ヘルメット型非侵襲的換気(RR 0.26、95%CrI 0.14-0.46、絶対リスク差-0.32、95%CrI -0.60--0.16、低度の確実性)、フェイスマスク型非侵襲的換気(RR 0.76、95%CrI 0.62-0.90、絶対リスク差-0.12、95%CrI -0.25--0.05、中等度の確実性)および高流量経鼻酸素(RR 0.76、95%CrI 0.55-0.99、絶対リスク差-0.11、95%CrI -0.27--0.01、中等度の確実性)の気管切開率が低かった(25試験、3804例)。挿管時の盲検下の欠如によるバイアスリスクは、高いと考えられた。 【結論および意義】この急性低酸素血症性呼吸不全成人患者を検討した試験のネットワークメタ解析では、非侵襲的換気による治療が標準酸素療法よりも死亡リスクが低かった。各戦略の相対的便益に対する理解を深めるために、さらに詳細な研究を要する。 第一人者の医師による解説 重症度と病態に合わせたデバイスの選択が重要 出井 真史 東京女子医科大学病院 集中治療科 助教 MMJ. December 2020;16(6):163 集中治療室に入室する急性低酸素性呼吸不全(AHRF)患者の管理では、原疾患の治療と同時に呼吸療法の良否が予後に大きく影響する。呼吸療法は標準酸素療法、非侵襲的酸素療法、侵襲的酸素療法(気管挿管を伴う人工呼吸)の3段階が存在し、重症ほど侵襲性の高い手段が必要となるが、合併症のリスクや求められる医療資源も増大する。本論文では、成人AHRF患者において3種類の非侵襲的酸素療法(フェイスマスク非侵襲的換気[NIV]、ヘルメットNIV、経鼻高流量酸素療法[HFNC])と従来の酸素療法を比較した無作為化対照試験(RCT)のネットワークメタアナリシスを行い、死亡率・気管挿管率を評価した。  すでにNIVの有用性が示されている(1)慢性閉塞性肺疾患(COPD)急性増悪、心不全、心臓外科周術期、抜管直後の患者を主な対象とした研究は除外した。2020年4月までの25のRCT(患者数3,804人)が収集され、原因疾患は64%のRCTで市中肺炎が最多であった。標準酸素療法と比較し、ヘルメットNIV(相対リスク[RR], 0.40)、フェイスマスクNIV(RR, 0.83)は死亡リスク低値と関連していた。気管挿管率もヘルメット NIV(RR, 0.26)、フェイスマスク NIV(RR, 0.76)、HFNC(RR, 0.76)でいずれも低値であった。  標準酸素療法と比べて、3つの非侵襲的酸素療法とも有用性が示された結果である。ヘルメットNIVはフェイスマスク NIVほど一般的ではないが、リークが少なく安定した呼気終末陽圧(PEEP)をかけることができ、肺胞のリクルートメントに寄与する可能性がある(2)。患者の忍容性も良く、顔面の皮膚損傷もないため比較的長期に使用できるという利点もある。フェイスマスクNIVはすでに多くのAHRFの病態で予後改善のエビデンスが蓄積されている(1)。HFNCは挿管率の低下のみ示されたが、これはNIVに比べ圧のサポートが不十分なことが影響していると思われる。  一方、非侵襲的酸素療法で粘りすぎ、いたずらに気管挿管のタイミングを遅らせることは危険である。一般的にデバイスを装着して30分から1時間で酸素化の改善、呼吸数の減少、呼吸苦の軽減などがみられなければ次の段階(HFNCならNIV、NIVなら気管挿管)を考慮すべきである。また呼吸努力の強い患者では、非侵襲的酸素療法に頼り自発呼吸で管理すると肺障害を増悪させる可能性があるので慎重な選択が求められる。PEEPを要さずPaCO2の貯留がない軽症のAHRFはHFNC、PEEPや吸気圧のサポートを要する中等症以上のAHRFはNIVを検討し、ヘルメット NIVが使用できる施設では積極的に選択する。そして反応が乏しい場合は機を逸せず気管挿管に移行する、というのが現段階での最良の介入であろう。 1. Rochwerg B, et al. Eur Respir J. 2017 Aug 31;50(2):1602426. 2. Patel BK, et al. JAMA. 2016 Jun 14;315(22):2435-2441.
高齢者のdysanapsisと慢性閉塞性呼吸器疾患の関連
高齢者のdysanapsisと慢性閉塞性呼吸器疾患の関連
Association of Dysanapsis With Chronic Obstructive Pulmonary Disease Among Older Adults JAMA . 2020 Jun 9;323(22):2268-2280. doi: 10.1001/jama.2020.6918. 原文をBibgraph(ビブグラフ)で読む 上記論文の日本語要約 【重要性】喫煙が慢性閉塞性呼吸器疾患(COPD)の大きな危険因子であるが、COPDのリスクの多くはいまだに説明できていない。 【目的】CT画像で評価したdysanapsis(気道内径と肺の大きさが不釣り合いな状態)が、高齢者のCOPD発症およびCOPDの肺機能低下と関連があるかを明らかにすること。 【デザイン、設定および参加者】2531例を検討したMulti-Ethnic Study of Atherosclerosis(MESA)Lung Study(米6都市、2010-2018年)と1272例を検討したCanadian Cohort of Obstructive Lung Disease(CanCOLD、カナダ9都市、2010-2018年)の地域住民標本2件の後向きコホート研究およびCOPDの症例対照研究Subpopulations and Intermediate Outcome Measures in COPD研究(SPIROMICS、米12都市、2011-2016年)。 【曝露】肺気量の立方根で分割した標準的な解剖学的部位19箇所で測定した気道内径の幾何平均として、CT画像上でdysanapsisを定量化した(気道の肺に対する比率)。 【主要評価項目】主要評価項目は、気管支拡張薬吸入後の1秒率(FEV1:FVC)0.70未満と定義したCOPDとした。長期肺機能を副次評価項目とした。全解析を患者背景とCOPD危険因子(喫煙および受動喫煙、職業的および環境的汚染、喘息)。 【結果】MESA Lung標本(平均[SD]年齢69[9]歳、1334例[52.7%]が女性)では、参加者2531例中237例(9.4%)にCOPDがあり、気道の肺に対する比率の平均(SD)は0.033(0.004)、平均(SD)FEV1低下量は-33mL/y(31 mL/y)であった。COPDがないMESA Lungの参加者2294例のうち、98例(4.3%)が中央値6.2年時にCOPDを発症した。気道の肺に対する比率の最高四分位範囲の参加者と比べると、最低四分位範囲の参加者はCOPD発症率が有意に高かった(1000人年当たり9.8 vs 1.2例、率比[RR]8.121、95%CI 3.81-17.27、1000人年当たりの率差8.6例、95%CI 7.1-9.2、P<0.001)が、FEV1低下量に有意差はなかった(-31 vs. -33mL/y、差2mL/y、95%CI -2-5、P=0.30)。CanCOLD参加者(平均[SD]年齢67[10]歳、564例[44.3%]が女性)、752例中113例(15.0%)が中央値3.1年時にCOPDを発症し、平均(SD)FEV1低下量は-36mL/y(75 mL/y)であった。気道の肺に対する比率最低四分位範囲の参加者のCOPD発症率は、最低四分位範囲の参加者よりも有意に高かった(1000人年当たり80.6 vs. 24.2例、RR 3.33、95%CI 1.89-5.85、1000人年当たりの率差 56.4例、95%CI 38.0-66.8、P<0.001)が、FEV1低下量に有意差はなかった(-34 vs. -36mL/y、差 1 mL/y、95%CI -15-16、P=0.97)。COPDがあり、中央値2.1年追跡したSPIROMICS参加者1206例(平均[SD]年齢65[8]歳、542例[44.9%]が女性)のうち気道の肺に対する比率が最低四分位範囲の参加者は平均FEV1低下量が-37 mL/y(15mL/y)で、MESA Lung参加者の低下量を有意差がなかった(P=0.98)が、最高四分位範囲の参加者ではMESA Lung参加者よりも有意に速く低下した(-55mL/y[16 mL/y]、差-17mL/y、95%CI -32--3、P=0.004)。 【結論および意義】高齢者で、dysanapsisにCOPDと有意な関連があり、気道内径が肺の大きさよりも狭いとCOPDリスクが高くなった。DysanapsisはCOPDの危険因子であると思われる。 第一人者の医師による解説 気道の発育障害は 喫煙とは独立したCOPD発症要因 永井 厚志 新百合ヶ丘総合病院呼吸器疾患研究所所長 MMJ. December 2020;16(6):162 慢性閉塞性肺疾患(COPD)の発症要因としては喫煙が主因とみなされているが、非喫煙者でもCOPDの病態となる高齢者が少なくなく、今日なおCOPD発症リスクの多くは未解決である。本研究では、2つの前向きコホート研究(MESA、CanCOPD)および症例対照研究(SPIROMICS)の参加者を対象に、CT画像(19画像/肺)により計測された気道径と肺胞容積の比率を算出することにより、肺胞に対する気道発育の不均衡(dysanapsis)を評価し、COPD病態の発症との関連性について後ろ向きの検討が行われた。なお、これらの検討では喫煙や環境汚染への曝露、喘息など既知のCOPD発症要因について補正がなされた。結果は、肺胞に比べ気道径の発育が低値を示す群では、COPD(気管支拡張薬吸入後の1秒率70%未満と定義)の発症頻度がいずれのコホート研究でも高値を示した。呼吸機能(FEV1)の経年的低下に関して、それぞれの研究観察期間内では気道肺胞容積の不均衡とは関連性がみられなかった。以上の結果から、高齢者においては、肺胞に比べ気道の発育が低下を示すdysanapsisはCOPDの重要な危険因子であることが示唆される、と結論づけられている。また、重喫煙者でありながらCOPDとしての気流閉塞がみられない一因として、気道の発育がより高度である可能性にも触れている。  従来から、成人期の気流閉塞には幼小児期における肺の発育過程で生じた気道と肺の発達の不均衡が関与しており、20~30歳代に成熟する呼気流量が健常者に比べ低値にとどまることで、その後、加齢とともに低下する高齢者においてCOPD病態に至る可能性が指摘されていた(1),(2)。事実、これまでの研究では、中等度以上の気流閉塞を示すCOPD患者の約半数に肺の発育障害がみられると報告されている(3)。本研究では、COPD患者の30%近くが非喫煙者であることへの疑問に対して肺の発育障害が関与している可能性を示している。しかしながら、対象者は肺の発育期ではない成人であり、CTでの気道計測がCOPD病態の主病変である末梢気道ではなく比較的中枢気道であること、COPDは多因子により形成されるが本研究では質の異なった3研究が統合解析されていることなど、多くの指摘されるべき点がみられる。本研究に限界はあるにせよ、肺の発育と高齢に至り発症するCOPDに密接な関連があるという指摘は注目すべきであり、今後の課題として気道と肺の不均衡な発育(dysanaptic lung growth)の詳細な原因を解明することが求められる。 1. Stern DA, et al. Lancet. 2007;370(9589):758-764. 2. Svanes C, et al. Thorax. 2010;65(1):14-20. 3. Lange P, et al. N Engl J Med. 2015;373(2):111-122.
変形性膝関節症患者の疼痛に生体力学を応用した靴がもたらす効果 BIOTOK無作為化臨床試験
変形性膝関節症患者の疼痛に生体力学を応用した靴がもたらす効果 BIOTOK無作為化臨床試験
Effect of Biomechanical Footwear on Knee Pain in People With Knee Osteoarthritis: The BIOTOK Randomized Clinical Trial JAMA. 2020 May 12;323(18):1802-1812. doi: 10.1001/jama.2020.3565. 原文をBibgraph(ビブグラフ)で読む 上記論文の日本語要約 【重要性】症候性変形性膝関節症患者の疼痛や身体機能が、生体力学を用いて個別に調整した靴で改善すると思われるが、この治療の便益が明らかになっていない。 【目的】生体力学を応用した靴の効果を追跡期間24週間にわたって対照の靴と比較すること。 【デザイン、設定および参加者】スイスの大学病院で実施した無作為化臨床試験。2015年4月20日から2017年1月10日の間に症状があり、レントゲンで確認された変形性膝関節症患者220例を組み入れた。最後の被験者の受診は2017年8月15日であった。 【介入】被験者を、靴底に個別調整が可能な凸面ポッドを付けた生体力学を応用した靴を履くグループ(111例)と、靴底にポッドは付いているが、調整不可能で靴裏の表面が凸面にならない対照の靴(109例)を履くグループに無作為に割り付けた。 【評価項目】主要評価項目は、WOMAC標準化疼痛スコア(0点[症状なし]から10点[きわめて重い症状]の範囲)で判定した24週時の膝痛とした。副次評価項目は、24週時のWOMAC身体機能とこわばりのスコアおよびWOMAC全般スコア(いずれも0点[症状なし]から10点[きわめて症状が重い]の範囲)および重度有害事象とした。 【結果】無作為化した参加者220例(平均年齢65.2歳[SD 9.3歳]、104例[47.3%]が女性)のうち、219例が割り付けた治療を受け、213例(96.8%)が追跡を完了した。24週時、平均標準化WOMAC疼痛スコアは生体力学応用靴群で4.3点から1.3点、対照靴群で4.0点から2.6点に改善した(24週時のスコア群間差-1.3、95%CI -1.8--0.9、P<0.001)。この結果は、24週時のWOMAC身体機能スコア(群間差-1.1、95%CI -1.5--0.7)、WOMACこわばりスコア(同-1.4、95%CI -1.9--0.9)およびWOMAC全般スコア(同-1.2、95%CI -1.6--0.8)でも一致していた。有害事象が、生体力学応用靴群で3件、対照靴群で9件発生した(2.7% vs. 8.3%)が、いずれも治療によるものではなかった。 【結論および意義】変形性関節症の膝痛がある患者で、生体力学を応用した靴を使用すると、対照の靴を比較して24週時の疼痛が統計的有意差をもって改善したが、その臨床的意義は明らかになっていない。この機器の臨床的価値について結論に達する前に、長期的な有効性と安全性に加えて、複製についても、詳細な研究で評価する必要がある。 第一人者の医師による解説 不安定な靴底で神経筋コントロール改善 長期的な有効性、安全性の検証が必要 武冨 修治 東京大学大学院医学系研究科整形外科学講師 MMJ. December 2020;16(6):173 膝が痛い高齢者は非常に多く、その原因の大部分は変形性膝関節症(膝OA)である。日本における画像的な膝OA患者は約2590万人、有症状者に限っても約800万人と推計されている(1)。膝OAの治療は体重減量、運動療法、生活様式の変更などの非薬物療法、非ステロイド系抗炎症薬(NSAID)の内服・外用やヒアルロン酸製剤の関節内注射などの薬物療法、これらの治療に抵抗性の場合は膝関節周囲骨切り術や人工膝関節置換術などの外科的治療が行われる。靴に関しては靴の中敷きである足底板の使用によって疼痛の緩和、歩行能力の改善が得られることが知られ、非薬物療法の1つとして広く行われている。  本論文で取り上げられている生体力学を応用した靴によるAPOS療法はイスラエルで開発され、膝痛、股関節痛や腰痛に対し、靴底に2つの凸型ポッドのついた不安定な靴で歩行することにより神経筋コントロールを回復させることで、症状を改善させる治療法である(https://www.apostherapy.co.uk)。  本研究では平均65.2歳の有症状の膝OA患者220人を対象に、生体力学を応用した靴を用いたAPOS療法による膝痛軽減効果をランダム化(BIOTOK)試験で検討した。靴底に個別調整可能な凸型ポッドのついた靴を使用した患者群を、凸型の底面を持たない靴を使用した対照群と比較した。主要アウトカムはOAの評価指標Western Ontario and McMaster Universities Osteoarthritis Index(WOMAC)の標準化疼痛スコアで評価した24週時の膝痛とした。その結果、疼痛スコア平均値は生体力学応用靴(介入)群で4.3から1.3へ(改善度3.0)、対照群で4.0から2.6へ(改善度1.4)改善し、改善度は介入群で有意に大きかった。身体機能、こわばりに関しても介入群で対照群に比べ有意に大きい改善が得られた。介入群の2.7%と対照群の8.3%に重篤な有害事象がみられたが、治療関連のものはなかった。著者らは生体力学応用靴の使用は統計学的に有意な膝痛の改善をもたらしたが、この差に臨床的重要性があるかは不明であり、長期的な有効性と安全性を評価するためにはさらなる研究が必要であると結論づけている。  近年では、靴底を不安定な形状にすることで筋力強化効果や運動効果を期待したダイエットシューズやフィットネスシューズが市販され、注目されている。膝 OAにおいても、生体力学を応用した靴の使用によって神経筋コントロールが回復し、膝痛や歩行能力が改善する効果が得られることで薬物療法や外科的治療を要する患者が減少すれば、医療経済的にも大きな効果が期待できる。ただし、より高齢の患者に安全に使用できるかなどの疑問もあり、長期的な有効性、安全性の検証結果が待たれる。 1. Yoshimura N, et al. J Bone Miner Metab. 2009;27(5):620-628.
サイトメガロウィルス血清陽性ドナーから肝移植を受ける血清陰性レシピエントに用いる先制治療と予防投与の効果 無作為化臨床試験
サイトメガロウィルス血清陽性ドナーから肝移植を受ける血清陰性レシピエントに用いる先制治療と予防投与の効果 無作為化臨床試験
Effect of Preemptive Therapy vs Antiviral Prophylaxis on Cytomegalovirus Disease in Seronegative Liver Transplant Recipients With Seropositive Donors: A Randomized Clinical Trial JAMA . 2020 Apr 14;323(14):1378-1387. doi: 10.1001/jama.2020.3138. 原文をBibgraph(ビブグラフ)で読む 上記論文の日本語要約 【重要性】サイトメガロウィルス(CMV)血清陽性ドナーから肝移植を受ける高リスク血清陰性レシピエントに用いる抗ウイルス薬を予防投与しても、高い確率で予防投与後にCMV感染症が発生する。代替アプローチに先制治療(検査で検出した早期無症候性CMV血症に抗ウイルス薬投与を開始)があるが、この状態の患者で抗ウイルス薬予防投与と直接比較したことがない。 【目的】CMV血清陽性ドナーから肝移植を受ける血清陰性レシピエントのCMV感染症予防に用いる先制治療と抗ウイルス薬予防投与を比較すること。 【デザイン、設定および参加者】CMV血清陽性ドナーから肝移植を受ける18歳以上の血清陰性レシピエント205例を対象に、先制治療と抗ウイルス薬予防投与を比較する無作為化臨床試験。試験は、米国の大学病院移植センター6施設で、2012年10月から2017年6月にかけて実施し、2018年6月に最終追跡を完了した。 【介入】被験者を、100日間の間に週1回の血清CMV PCR検査で検出したウイルス血症に対して先制治療(バルガンシクロビル900mg、1日2回、1週間空けた検査で2回連続陰性が出るまで投与)を実施するグループと、予防投与としてバルガンシクロビル900mgを1日1回100日間投与するグループ(105例)に1体1の割合で無作為に割り付けた。 【主要評価項目】主要評価項目は、12カ月以内のCMV症候群(CMV血症および臨床的または検査的所見)と定義したCMV感染症の発症率または末端器官疾患とした。急性移植片拒絶反応、日和見感染、移植片および患者生存、好中球減少症を副次評価項目とした。 【結果】無作為化した205例(平均年齢55歳、62例[30%]が女性)のうち全205例(100%)が試験を完遂した。CMV感染症の発症率は、先制治療群の方が抗ウイルス薬予防投与群よりも有意に低かった(9% vs. 19%、差10%、95%CI 0.5-19.6%、P=0.04)。先制治療と予防投与の移植片拒絶反応の発生率(28% vs 25%、差3%、95%CI -9-15%)、日和見感染(25%vs. 27%、同2%、-14-10%)、移植片機能喪失(2% vs. 2%、同1%未満、-4-4%)および好中球減少症(13% vs. 10%、同3%、-5-12%)に有意差はなかった。最終追跡時の全死因死亡率は先制治療群15%、抗ウイルス薬予防投与群19%であった(同4%、95%CI -14-6%、P=0.46)。 【結論および意義】CMV血清陽性ドナーから肝移植を受ける血清陰性レシピエントで、先制治療は抗ウイルス薬予防投与群と比べて、12カ月間のCMV感染症発症率が低かった。この知見を再現し、長期的転帰を評価する詳細な研究が必要とされる。 第一人者の医師による解説 先制治療の優位性はCMV特異的免疫の誘導効率に依存するかもしれない 宮木 陽輔 東海大学医学部医学科/野田 敏司 東海大学医学部生体防御学領域講師 MMJ. December 2020;16(6):174 ヒトサイトメガロウイルス(CMV)は免疫不全患者や免疫抑制状態にある人において日和見感染症を起こすことが知られている。特にCMV陽性ドナーからCMV陰性レシピエントへの臓器移植では、その発症が患者の予後に多大なる影響を及ぼすため、CMV感染症のコントロールが大きな課題となっている。  現在、肝移植後のCMV感染症の管理には、先制治療(preemptive therapy)と予防投与(antiviral prophylaxis)が用いられている。前者では、介入期間100日において、1週間ごとにCMV PCR検査を行い、CMVが検出された場合、経口抗ウイルス薬のバルガンシクロビル 900mgを1日2回投与する。その後の検査で2回連続のCMV陰性結果が得られた場合、投薬を終了し、通常のモニタリングに戻る。後者では、同100日において、定期的なPCR検査を実施することなく、バルガンシクロビル 900mgを1日1回投与することを骨子とする。ところが、肝移植後のCMV感染症発生抑制にいずれが優位であるのか、従前まで、十分な症例数を用いたランダム化比較試験は行われてこなかった。  本研究では、米国の6つの大学における移植センターの患者(計205人)を対象にランダム化比較試験を実施し、これら両戦略の優位性の判定、加えて当該優位性獲得における免疫学的考察がなされた。結果、主要アウトカムである移植後12カ月までのCMV感染症は、先制治療群(100人)の方が予防投与群(105人)と比較し、その発生率を有意に抑制した(9% 対 19%;P=0.04)。移植後100日目以降の副次アウトカム(遅発 CMV感染症の発生率)でも、先制治療群が予防投与群よりも良好な成績を示した(6% 対 17%;P=0.01)。  前出のアウトカムが得られたメカニズムを、免疫病理学的に考察するにあたり、著者らはCMV特異的 T細胞と中和抗体に注目した。前者の細胞数では、先制治療群が予防投与群を有意に上回った(CD8+, P<0.001)。後者に関しては、予防投与群に比べ、抗体保有者が先制治療群に多く認められた(P=0.04)。これらの結果から、両群間の成績相違をもたらした機序の1つに、CMV特異的免疫の効率的動員が挙げられるのではないか、と著者らは示唆している。  以上、肝移植におけるCMV感染症のリスク低減には、先制治療が予防投与に比べ優位であるとする初めての知見が得られた。今後、本研究の再現性と長期アウトカムをより詳細に評価することが強く期待される。
メタボリック症候群のエネルギーを制限した地中海食遵守に栄養および行動への介入がもたらす効果 PREDIMED-Plus無作為化試験の中間解析
メタボリック症候群のエネルギーを制限した地中海食遵守に栄養および行動への介入がもたらす効果 PREDIMED-Plus無作為化試験の中間解析
Effect of a Nutritional and Behavioral Intervention on Energy-Reduced Mediterranean Diet Adherence Among Patients With Metabolic Syndrome: Interim Analysis of the PREDIMED-Plus Randomized Clinical Trial JAMA . 2019 Oct 15;322(15):1486-1499. doi: 10.1001/jama.2019.14630. 原文をBibgraph(ビブグラフ)で読む 上記論文の日本語要約 【重要性】質の高い食習慣によって慢性疾患を予防することができるが、栄養および行動への介入が食習慣にもたらす効果を検討した無作為化試験のデータはほとんどない。 【目的】食事の質に関する栄養と運動の教育プログラムの効果を評価すること。 【デザイン、設定および参加者】進行中の無作為化試験の予備的探索的中間解析。スペインの研究センター23施設で、2013年3月から2016年21月にかけて、メタボリック症候群があり心血管疾患がない55-75歳の男女6874例を組み入れ、2019年3月に最終データを収集した。 【介入】参加者をエネルギーを制限した地中海食を推奨し、運動を促進し、行動支援を提供する介入群(3406例)とエネルギーを制限しない地中海食を推奨する対照群(3468例)に無作為に割り付けた。全例にエクストラバージンオリーブオイル(1カ月当たり1L)とナッツ(1カ月当たり125g)を無料で至急した。 【主要評価項目】主要評価項目は、エネルギーを制限した地中海食(er-MedDiet)スコアの12カ月間の変化(範囲0-17点、スコアが高いほど遵守度が高いことを示す、最小重要差1ポイント)。 【結果】無作為化した参加者6874例(平均年齢[SD]65.0[4.9]歳、3406例[52%]が男性)、6583例(96%)が12カ月間の追跡を完遂し、主要解析の対象とした。平均(SD)er-MedDietスコアは、介入群で追跡開始時8.5(2.6)点、12カ月時13.2(2.7)点(4.7点増加、95%CI 4.6-4.8)、対照群で8.6(2.7)点と11.1(2.8)点(2.5点増加、95%CI 2.3-2.6)であった(群間差2.2点、95%CI 2.1-2.4点、P<0.001) 【結論および意義】この進行中の試験の予備的解析では、エネルギーを制限した地中海食と運動を推奨する介入によって、エネルギーを制限しない地中海食に従う助言のみの対照と比べて、12カ月後の食事法遵守率が優位に上昇した。長期的な心血管にもたらす作用を詳細に評価する必要がある。 第一人者の医師による解説 エネルギー制限か、丁寧な個別介入による効果かは判断し難い 武見 ゆかり 女子栄養大学栄養学部食生態学研究室教授 MMJ. December 2020;16(6):178 地中海食は、循環器疾患、がん、糖尿病などの生活習慣病との関連で、“健康食”として世界中で高い評価を得ている。地中海食の遵守度は、もともとは9項目のMediterranean Diet Score(MDS)により判定され、さらに調理法などの観点を加えた14項目のMediterranean Diet Adherence Screener(MEDAS)も使われてきた。本研究ではこれらに加え、エネルギー制限を伴う17項目の17 -item energy-reduced Mediterranean diet(er-MedDiet)スコアを用いて、スペインのメタボリックシンドロームに該当する55~75歳の男女6,874人を対象とした無作為化対照試験の1年後の中間評価が行われた。  介入群には約600kcal/日減のエネルギー制限を伴う地中海食と身体活動の支援が行われ、対照群には伝統的な地中海食が推奨された。介入群には、赤身肉・加工肉、バター・マーガリン・クリーム、甘い炭酸飲料の厳しい制限、飲み物に砂糖を加えないこと、精製された穀類ではなく全粒穀類の摂取が推奨された。er-MedDietの項目は、MEDASの14項目(摂取推奨の項目:主たる調理油としてオリーブオイルの使用、野菜、果物、豆類、魚、ナッツ類の摂取、sofrito[トマト、にんにく、玉ねぎなどをオリーブオイルで炒めたスペイン料理の基本ソース]の使用、赤身肉より鶏肉・七面鳥肉を好むか、ワイン。摂取を控えるべき項目:赤身肉・加工肉、バター・マーガリン・クリーム、甘い炭酸飲料、市販の菓子パンや菓子類)に加え、全粒穀類の摂取、飲み物への砂糖添加の制限、精製された白パン・パスタ・米の摂取を控えることが追加されている。  1年後の評価では、介入群は対照群に比べ、これらすべての食事評価スコアが有意に高くなり介入効果がみられた。また、体重、腹囲、BMI、総コレステロール、HDLコレステロール、血圧などの循環器系危険因子も有意に改善した。  著者らは、本試験の目的を、エネルギー制限を伴う地中海食と伴わない伝統的な地中海食の介入効果の比較としているが、実際には、介入の支援方法もかなり異なる。介入群は、具体的な減量目標を設定し、食事内容だけでなく身体活動の促進も推奨された。一方、対照群は、エネルギー制限のない伝統的な地中海食にそった食事を推奨された。さらに、介入群は1年間、集団学習、動機づけ面接による支援、毎月1回の電話支援を受けたのに対し、対照群は年に2回だけ個人指導、集団指導、電話を受けた。  エネルギー制限を伴う地中海食の推奨だから効果があるのか、具体的な減量目標を設定した上で丁寧な介入を行ったことによる効果なのかは判断し難い。本報告は大規模介入試験の中間解析とのことなので、最終結果を注視していく必要がある。
小児心臓手術時の人工心肺中のNO添加 術後人工呼吸期間への効果なし
小児心臓手術時の人工心肺中のNO添加 術後人工呼吸期間への効果なし
Effect of Nitric Oxide via Cardiopulmonary Bypass on Ventilator-Free Days in Young Children Undergoing Congenital Heart Disease Surgery: The NITRIC Randomized Clinical Trial JAMA. 2022 Jul 5;328(1):38-47. doi: 10.1001/jama.2022.9376. 上記論文のアブストラクト日本語訳 ※ヒポクラ×マイナビ 論文検索(Bibgraph)による機械翻訳です。 [重要] 心臓手術を受ける小児では、心肺バイパス人工肺のガス流に一酸化窒素を投与すると、術後の低心拍出量症候群が軽減され、回復の改善と呼吸補助の持続時間の短縮につながる可能性があります。心肺バイパス人工肺に一酸化窒素を投与することで、人工呼吸器を使用しない日 (人工呼吸器を使用しない生存日数) が改善されるかどうかは不明のままです。 [設計、設定、および参加者]オーストラリア、ニュージーランド、およびオランダの6つの小児心臓外科センターでの二重盲検、多施設、無作為化臨床試験。 2017 年 7 月から 2021 年 4 月の間に、先天性心臓手術を受けている 2 歳未満の合計 1,371 人の子供が無作為に割り付けられ、最後の参加者の 28 日間の追跡調査が 2021 年 5 月 24 日に完了しました。 [介入] 患者は一酸化窒素を受けるように割り当てられました。心肺バイパス人工肺 (n = 679) または一酸化窒素なしの標準ケア心肺バイパス (n = 685) に送達された 20 ppm で。 28日目まで。低心拍出量症候群、体外生命維持、または死亡の複合を含む4つの二次エンドポイントがありました。集中治療室での滞在期間;入院期間; [結果] 無作為に割り付けられた 1371 人の患者 (平均 [SD] 年齢、21.2 [23.5] 週、女児 587 人 [42.8%]) のうち、1364 人 (99.5%) が試験を完了した。人工呼吸器を使用しない日数は、一酸化窒素群と標準治療群の間で有意差はなく、絶対差の中央値はそれぞれ 26.6 日 (IQR、24.4 ~ 27.4) 対 26.4 日 (IQR、24.0 ~ 27.2) でした。 -0.01 日 (95% CI、-0.25 ~ 0.22; P = .92)。一酸化窒素グループの合計 22.5% と標準治療グループの 20.9% が、48 時間以内に低心拍出量症候群を発症し、48 時間以内に体外サポートが必要になるか、28 日目までに死亡しました。調整後のオッズ比は 1.12 (95% CI、0.85 ~ 1.47)。その他の副次的転帰は、群間で有意差はなかった. [結論と関連性] 先天性心疾患のために心肺バイパス手術を受けている2歳未満の小児では、心肺バイパスによる一酸化窒素の使用は、人工呼吸器を使用しない日数に有意な影響を与えなかった. .これらの調査結果は、心臓手術中に心肺バイパス人工肺に送達された一酸化窒素の使用を支持していません。 第一人者の医師による解説 急性腎不全や血小板機能へのNO添加効果 進行中の研究結果に期待 清水 達彦 岡山大学病院周術期管理センター助教/岩﨑 達雄 岡山大学病院小児麻酔科教授 MMJ.February 2023;19(1):14 一酸化窒素(NO)はアポトーシスの制御を介した急性炎症の制御において複雑な役割を担っており、小児心臓手術時の人工心肺(CPB)中のNO添加は、低心拍出量症候群を減少させることが単施設ランダム化対照試験(1)で示されており、Follow四徴症において術後トロポニン値の低下、人工呼吸期間と集中治療室(ICU)滞在日数の短縮をもたらすことがパイロット研究(2)で報告されている。そこで本論文の大規模多施設共同ランダム化対照試験(NITRIC試験)では、オーストラリア、ニュージーランド、オランダ で登録した2歳未満の先天性心疾患患者1,317人を対象に術後人工呼吸期間への影響が検討された。 患者はCPB酸素供給装置 に20ppmのNOを添加するNO群とNOを添加しない標準的 CPB(対照)群に割り付けられた。その結果、主要評価項目であるCPB開始後28日目までの人工呼吸離脱日数の中央値は、NO群26.6日(四分位範囲[IQR], 24.4~ 27.4)、対照群26.4日(IQR, 24.0~27.2)で、有意差はなかった(P=0.92)。また、副次評価項目であるCPB開始後48時間以内の低心拍出量症候群・体外式心肺補助、28日以内の死亡を合わせた複合アウトカムの発生率はNO群22.5%、対照群20.9%、補正オッズ比は1.12(95%信頼区間 , 0.85 ~ 1.47)となり、ICU滞在期間、入院期間、術後トロポニン値についても群間で有意差はなかった。 CPB酸素供給装置 へ のNO添加は、NO吸入投与と異なり、肺自体にはほぼ到達しないと考えられるため、先行研究での人工呼吸期間への効果は心臓および他臓器の虚血再灌流障害軽減による間接的な影響と推測される。しかし、NITRIC試験ではその心保護作用も認められず、Jamesらの報告1と異なる結果になっているが、著者らはその理由について特に述べていない。近年、“del Nido”という新しい心筋保護液が広まりつつあるが、本論文およびJamesらの報告1には心筋保護に関する記載がなく、心筋の虚血再灌流障害に影響した可能性は気になるところである。 今回の結果からは、2歳未満児の先天性心疾患手術において、低心拍出量症候群の軽減と人工呼吸期間の短縮を目的としたCPB酸素供給装置へのNO添加は推奨されないということになる。しかしながら、完全大血管転位症に限定した研究や、急性腎不全および血小板機能への効果を検討した研究は進行中であり、それらの結果が待たれるところである。 1. James C, et al. Intensive Care Med. 2016;42(11):1744-1752. 2. Checchia PA, et al. J Thorac Cardiovasc Surg. 2013;146(3):530-536.
呼吸補助が必要な急性期小児患者で高流量鼻カニューレ療法はCPAPに非劣性
呼吸補助が必要な急性期小児患者で高流量鼻カニューレ療法はCPAPに非劣性
Effect of High-Flow Nasal Cannula Therapy vs Continuous Positive Airway Pressure Therapy on Liberation From Respiratory Support in Acutely Ill Children Admitted to Pediatric Critical Care Units: A Randomized Clinical Trial JAMA. 2022 Jul 12;328(2):162-172. doi: 10.1001/jama.2022.9615. 上記論文のアブストラクト日本語訳 ※ヒポクラ×マイナビ 論文検索(Bibgraph)による機械翻訳です。 [重要] 急性期の小児に対する非侵襲的呼吸補助の最適な一次モードは不明です。 [デザイン、設定、および参加者] 英国の 24 の小児救命救急ユニットで実施された実用的、多施設、無作為化非劣性の臨床試験2019 年 8 月から 2021 年 11 月の間に募集され、2022 年 3 月に最後のフォローアップが完了した、非侵襲的呼吸補助が必要であると臨床的に評価された 0 歳から 15 歳までの 600 人の急性疾患の子供の中で。患者の体重 (n = 301) に基づく流量での HFNC または 7 ~ 8 cm H2O の CPAP (n = 299)。これは、参加者があらゆる形態の呼吸補助 (侵襲的または非侵襲的) から解放された 48 時間の期間の開始として定義され、0.75 の調整済みハザード比の非劣性マージンに対して評価されました。救命救急室退院時の死亡率、48 時間以内の挿管、鎮静の使用など、7 つの副次評価項目が評価されました。サポートは 22 で開始されませんでした (HFNC: 5; CPAP: 17)。 573 人の子供 (HFNC: 295; CPAP: 278) が一次分析に含まれました (年齢の中央値、9 か月; 226 人の女の子 [39%])。 HFNC グループの解放までの時間の中央値は 52.9 時間 (95% CI、46.0 ~ 60.9 時間) であったのに対し、CPAP グループでは 47.9 時間 (95% CI、40.5 ~ 55.7 時間) (絶対差、5.0 時間 [95% CI - 10.1 ~ 17.4 時間]; 調整ハザード比 1.03 [片側 97.5% CI、0.86-∞])。これは非劣性の基準を満たしていました。事前に指定された 7 つの副次的転帰のうち、3 つが HFNC グループで有意に低かった。救命救急入院の平均期間 (5 日対 7.4 日、調整平均差、-3 日 [95% CI、-5.1 ~ -1 日]);および平均急性入院期間 (13.8 日 vs 19.5 日; 調整平均差、-7.6 日 [95% CI、-13.2 ~ -1.9 日])。最も一般的な有害事象は鼻の外傷でした (HFNC: 6/295 [2.0%]; CPAP: 18/278 [6.5%])。ケアユニットでは、CPAP と比較した HFNC は、呼吸補助から解放されるまでの時間について非劣性の基準を満たしていました。[試験登録]ISRCTN.org 識別子: ISRCTN60048867. 第一人者の医師による解説 高流量鼻カニューレ療法の使いどころはステップアップ 不快感を軽減し鎮静薬使用の頻度が低い 志馬 伸朗 広島大学大学院医系科学研究科救急集中治療医学教授 MMJ.February 2023;19(1):9 呼吸サポートは、集中治療を要する小児患者において、最も頻繁に適用される臓器補助である。近年、気管挿管による侵襲的換気の弊害の認識が高まり、非侵襲的換気の適用により挿管を回避する試みが重要視されている。従来、小児患者に対する非侵襲的換気サポートは経鼻プロングあるいはマスクを用いた持続気道陽圧(CPAP)であったが、患者認容度が高く使いやすい高流量鼻カニューレ療法(HFNC)が普及してきた(1)。 非侵襲的換気の適用は、「ステップアップ」=気管挿管前の使用と、「ステップダウン」=抜管後の使用に分けられる。FIRST-ABC研究ではこの2つの状態を対象に、独立したランダム化比較試験を行った。既報 の ス テップ ダ ウン 研究 で はHFNCはCPAPに対する非劣性を示せなかった(2)。本研究では「ステップアップ」時のHFNCがCPAPに非劣性であるか検討した。24ユニットから登録された小児患者573人のデータが解析された。原疾患の約半数は急性細気管支炎で、残りは他の呼吸器疾患、喘息、心疾患、敗血症など多彩であった。HFNC群の開始流量は体重12kg未満では2L/kg/分、12kg以上ではプロトコールにより設定され、離脱に際して流量を50%減らした。CPAP群は、7 ~ 8cmH2Oの空気圧で開始され、離脱時には5cmH2Oまで下げられた。HFNC群の33.1%、CPAP群の53.3%において、中央値でそれぞれ6.1、4.5時間後に治療失敗が生じた。CPAPからHFNCへの切り替えは、HFNCからCPAPへの切り替えよりも多かった(30.9% 対 20.0%)。切り替えの理由は、HFNC群では主に臨床的悪化、CPAP群では患者の不快感に関連していた。鎮静薬の使用頻度はHFNC群(27.7%)がCPAP群(37.0%)よりも低かった。 本研究は、呼吸補助を必要とする小児呼吸不全患者において、HFNCはCPAPと同様に気管挿管を回避する一手段となりうることを示している。より重要なことに、HFNCは不快感に関連する治療失敗率が低いことに加え、不快感を軽減しデバイス装着を維持するために鎮静薬を要する頻度が低い。“こどもに優しい”は、小児医療における不変のキーワードであり、洗練されたデバイスを用いたより非侵襲的な介入により高い効果が得られることは、小児患者にとって何よりも福音である。ステップアップにおける効果発現にも、この認容性の良さが関係しているだろう。HFNCは日本の臨床現場に急速に普及したが、まだ標準化の余地がある(3)。幸いにもクリニカルエビデンスの集積は盛んであり、より良い使用法、適応、あるいは限界が明確化されることを望む。 1. Kwon JW. Clin Exp Pediatr. 2020;63(1):3-7. 2. Ramnarayan P, et al. JAMA. 2022;327(16):1555-1565. 3. Kawaguchi A, et al. Pediatr Crit Care Med. 2020;21(5):e228-e235.
トラネキサム酸の高用量持続点滴投与は心臓手術における輸血率を低減
トラネキサム酸の高用量持続点滴投与は心臓手術における輸血率を低減
Effect of High- vs Low-Dose Tranexamic Acid Infusion on Need for Red Blood Cell Transfusion and Adverse Events in Patients Undergoing Cardiac Surgery: The OPTIMAL Randomized Clinical Trial JAMA. 2022 Jul 26;328(4):336-347. doi: 10.1001/jama.2022.10725. 上記論文のアブストラクト日本語訳 ※ヒポクラ×マイナビ 論文検索(Bibgraph)による機械翻訳です。 【重要】トラネキサム酸は、心臓手術における失血や輸血の軽減に推奨されています。しかし、高用量のトラネキサム酸が、心臓手術における血栓性合併症や発作のリスクを増加させることなく、低用量よりも優れた血液節約効果をもたらすかどうかは不明のままです。心肺バイパスを伴う心臓手術を受ける患者における低用量トラネキサム酸。この研究には、2018 年 12 月 26 日から 2021 年 4 月 21 日まで、中国の 4 つの病院で 3079 人の患者が登録されました。最終フォローアップは 2021 年 5 月 21 日でした。 kg プライム (n = 1525) または 10 mg/kg ボーラス、2 mg/kg/h 維持用量、および 1 mg/kg プライムを含む低用量レジメン (n = 1506)。[主な結果主要な有効性エンドポイントは、手術開始後の同種赤血球輸血率であり(優越性仮説)、主要な安全性エンドポイントは、術後30日間の死亡率、発作、腎機能障害の複合値でした(ステージ 2 または 3 の腎臓病: Improving Global Outcomes [KDIGO] 基準)、および血栓性イベント (心筋梗塞、虚血性脳卒中、深部静脈血栓症、および肺塞栓症) (マージン 5% の非劣性仮説)。主要な安全性エンドポイントの個々の構成要素を含む、15 の副次的エンドポイントがありました。 [結果] 治療群に無作為に割り付けられた 3079 人の患者 (平均年齢 52.8 歳、女性 38.1%) のうち、3031 人 (98.4%) が試験を完了しました。 .同種赤血球輸血は、高用量群の患者 1525 人中 333 人 (21.8%)、低用量群の患者 1506 人中 391 人 (26.0%) で発生した (リスク差 [RD]、-4.1% [片側97.55% CI、-∞ ~ -1.1%]; 相対リスク、0.84 [片側 97.55% CI、-∞ ~ 0.96; P = .004])。術後発作、血栓性イベント、腎機能障害、および死亡の複合は、高用量群の 265 人の患者 (17.6%) および低用量群の 249 人の患者 (16.8%) で発生した (RD、0.8%; 片側97.55% CI、-∞ ~ 3.9%; 非劣性の場合は P = .003)。事前に指定された 15 の二次エンドポイントのうち 14 は、高用量群の 15 人の患者 (1.0%) と低用量群の 6 人の患者 (0.4%) で発生した発作を含め、群間で有意差はありませんでした (RD、0.6 %; 95% CI, -0.0% to 1.2%; P = .05). [結論と関連性] 心肺バイパスを伴う心臓手術を受けた患者では、高用量は低用量のトラネキサム酸注入と比較して統計的に中程度の結果でした.同種赤血球輸血を受け、30 日死亡率、発作、腎機能障害、および血栓性イベントからなる複合主要安全性エンドポイントに関して非劣性の基準を満たした患者の割合が大幅に減少しました。[試験登録]臨床試験。政府識別子: NCT03782350。 第一人者の医師による解説 RCTによる低用量群との比較で 有害事象を増加させず輸血率低下効果を示した貴重な研究 島村 和男 大阪大学大学院医学系研究科外科学講座心臓血管外科学准教授 MMJ.February 2023;19(1):12 心臓手術に用いられる人工心肺(CPB)は線溶系を亢進させ,術中・術後の出血傾向を促進することが知られている。このため凝固障害を予防する目的で体外循環の施行中に線溶阻害薬が投与され、近年ではトラネキサム酸が主に使用されてきた。しかしながら、高用量のトラネキサム酸による有害事象として、てんかんと血栓塞栓症が報告されており、2017年に報告された多施設共同無作為化試験(1)では、冠動脈バイパス手術におけるトラネキサム酸1回投与の輸血量低減効果が示されたものの、てんかん発生率の上昇が認められた。一方、トラネキサム酸の持続点滴投与は1回投与に比べ安定した血中濃度が維持され、かつピーク濃度を低下させることから、理論的には有害事象を減少させ安定した止血効果が得られるとの報告(2)もあり、さまざまなトラネキサム酸投与プロトコールがこれまで使用されてきた。 本論文で報告されているOPTIMAL試験は、CPBを使用する心臓手術例に対するトラネキサム酸持続点滴投与の有効性を検証する二重盲検無作為化試験であり、2018年12月~ 21年4月に中国4施設で実施された。対象患者は、高用量群(ボーラ ス 静注30mg/kg+術中維持量16mg/kg/時、CPB充填2mg/kg)または低用量群(ボーラス静注10mg/kg+術中維持量2mg/kg/時、CPB充填1mg/kg)に、1対1の比で割り付けられた。3,079人(平均年齢52.8歳、女性38.1%)が無作為化の対象となり、このうち3,031人(98.4%)が試験を完了し、高用量群に1,525人、低用量群に1,506人が割り付けられた。結果では、少なくとも1回の同種赤血球輸血を受けた患者の割合は、高用量群は21.8%と低用量群の26.0%に比べ有意に低かった(群間リスク差[RD], -4.1%、リスク比 , 0.84;P=0.004)。また、安全性の主要複合エンドポイント発現率は、高用量群17.6%、低用量群16.8%と、高用量群の低用量群に対する非劣性が確認された(群間 RD, 0.8%;非劣性検定 P=0.003)。てんかん発作は、高用量群15例(1.0%)、低用量群6例(0.4%)に発現したが、両群間に有意差はなかった(群間 RD, 0.6%;P=0.05)。 本論文ではトラネキサム酸持続投与の有効性が高いエビデンスレベルにて示されており、心臓血管外科手術の安全性向上に寄与する投与プロトコールが提示された点で貴重なものと考えられる。 1. Myles PS, et al. N Engl J Med. 2017;376(2):136-148. 2. Dowd NP, et al. Anesthesiology. 2002;97(2):390-399.
症候性頭蓋内動脈狭窄症に対するステント治療は内科的治療への優位性を示せず
症候性頭蓋内動脈狭窄症に対するステント治療は内科的治療への優位性を示せず
Effect of Stenting Plus Medical Therapy vs Medical Therapy Alone on Risk of Stroke and Death in Patients With Symptomatic Intracranial Stenosis: The CASSISS Randomized Clinical Trial JAMA. 2022 Aug 9;328(6):534-542. doi: 10.1001/jama.2022.12000. 上記論文のアブストラクト日本語訳 ※ヒポクラ×マイナビ 論文検索(Bibgraph)による機械翻訳です。 [重要] 以前の無作為化試験では、一般的に、症候性の重度の頭蓋内アテローム性動脈硬化性狭窄を有する患者の薬物療法にステント留置術を追加しても害があるか、または利益がないことが示されていますが、洗練された患者選択とより経験豊富な外科医が結果を改善する可能性があるかどうかは不明のままです. [目的]症候性の重度の頭蓋内アテローム性動脈硬化性狭窄症患者におけるステント留置と内科療法と内科療法単独を比較すること。重度の頭蓋内狭窄 (70%-99%) に起因する一過性脳虚血発作または非障害性、非穿通枝 (非脳幹または非大脳基底核末端動脈として定義) 領域虚血性脳卒中の合計 380 人の患者、および最新の発作から 3 週間以上の期間虚血症状の発症は、2014 年 3 月 5 日から 2016 年 11 月 10 日の間に募集され、3 年間追跡調査された (最終追跡調査: 2019 年 11 月 10 日)。単独で(n = 182)。薬物療法には、90 日間の 2 剤併用抗血小板療法(その後は 1 回の抗血小板療法)と脳卒中危険因子のコントロールが含まれていました。日から1年。 2 年目と 3 年目の適格な動脈領域での脳卒中、3 年目の死亡率を含む 5 つの二次的転帰がありました。 [73.5%]) と 343 (95.8%) が試験を完了しました。ステント留置と薬物療法のグループと薬物療法のみのグループでは、脳卒中または死亡のリスクの主要アウトカムに有意差は見られなかった (8.0% [14/176] vs 7.2% [13/181]; 差、0.4% [95] % CI、-5.0% ~ 5.9%]; ハザード比、1.10 [95% CI、0.52-2.35]; P = .82)。事前に指定された 5 つの副次的エンドポイントのうち、2 年で適格な動脈領域の脳卒中を含む有意差を示したものはありませんでした (9.9% [17/171] 対 9.0% [16/178]; 差、0.7% [95% CI、- 5.4% から 6.7%]; ハザード比、1.10 [95% CI、0.56-2.16]; P = .80) および 3 年 (11.3% [19/168] vs 11.2% [19/170]; 差、-0.2 % [95% CI、-7.0% ~ 6.5%]; ハザード比、1.00 [95% CI、0.53-1.90]; P > .99)。 3 年時点での死亡率は、ステント留置術と薬物療法を併用したグループでは 4.4% (7/160) であったのに対し、薬物療法のみのグループでは 1.3% (2/159) でした (差、3.2% [95% CI、-0.5% ~ 6.9%] ; ハザード比 3.75 [95% CI 0.77-18.13]; P = .08). [結論と関連性] 症候性の重度の頭蓋内動脈硬化性狭窄による一過性脳虚血発作または虚血性脳卒中の患者では、経皮経管血管形成術の追加と内科的治療単独と比較して、内科的治療へのステント留置術は、30日以内の脳卒中または死亡のリスク、または30日を超えて1年間の対象となる動脈領域での脳卒中のリスクに有意差はありませんでした.この調査結果は、症候性の重度の頭蓋内アテローム性動脈硬化性狭窄を有する患者の治療のための内科療法への経皮経管血管形成術およびステント留置術の追加を支持していません.[試験登録]ClinicalTrials.gov識別子: NCT01763320. 第一人者の医師による解説 頭蓋内動脈狭窄症に対する血管内治療は当面の間 限定的な患者への実施にとどまるだろう 秋山 武紀 慶應義塾大学医学部脳神経外科専任講師 MMJ.February 2023;19(1):8 頭蓋内動脈狭窄症はアジアで頻度が高く、薬物療法での脳梗塞再発率が高い疾患である。このため血管内治療(経皮的脳血管形成術および頭蓋内ステント留置術)による予後の改善が期待され、さまざまな研究が行われてきたが、SAMMPRIS試験(1)などでは血管内治療の周術期合併症リスクが非常に高い(15%前後)ために内科的治療に対する優位性を示すことができずにいた。今回、厳密な患者選択と術者限定を行うことで周術期合併症リスクが低下し、頭蓋内動脈狭窄症に対するステント治療の有益性が証明されるという仮説のもと、CASSISS(China Angioplasty and Stenting for Symptomatic Intracranial Severe Stenosis)試験が行われた。本試験は2014年3月~16年11月に患者組み入れを行い、1年以内に一過性脳虚血発作または軽症脳梗塞を発症、頭蓋内主幹動脈(内頚動脈、中大脳動脈 M1、椎骨・脳底動脈)に70 ~99%の動脈硬化性狭窄を有し、虚血発症後3週間を経過した患者358人を対象とした。穿通枝梗塞・大梗塞や進行性に増悪する神経症状を有する患者などは除外された。3年以内に頭蓋内狭窄に対する血管内治療30例以上を経験、30日後の死亡・脳梗塞発生率が15%未満である術者に限定された。治療プロトコールは抗血小板薬2剤と最善の内科的治療(LDL-C 100mg/dL、血圧140/90mmHg、HbA1c 6.5%未満、生活習慣の改善)は共通、血管内治療はバルーンおよび自己拡張型頭蓋内ステント(WingspanR)の使用という一般的な手技で行われた。 その結果、主要評価項目(30日以内の脳卒中・死亡、30日以降1年以内の支配領域における脳卒中)の発生率は血管内治療群8.0%、内科的治療群7.2%であった。副次評価項目の1つである3年以内の支配領域の脳卒中は血管内治療群11.3%、内科的治療群11.2%であり、主要、副次評価項目ともに2群間に有意差を認めなかった。すなわち、患者選択、術者限定を行っても症候性頭蓋内動脈狭窄症における血管内治療の内科的治療に対する優位性は示されなかった。 CASSISS試験は先行試験の批判に応え、血管内治療の優位性を示しやすい条件のもと行われたが、やはり内科的治療の成績を上回ることはなかった。本試験における血管内治療の周術期合併症発生率は5%であり、決して高いものではない。今後も薬物療法の進歩が続くことを考えると、それを上回る血管内治療の成績を示すことは現状のデバイスでは困難かもしれない。頭蓋内動脈狭窄症に対する血管内治療は当面の間、限定的な患者に行われることになるとともに、革新的なデバイスの開発が求められるであろう。 1. Derdeyn CP, et al. Lancet. 2014;383(9914):333-341.
冠動脈造影における急性腎障害 臨床意思決定支援ツールで発生率低下
冠動脈造影における急性腎障害 臨床意思決定支援ツールで発生率低下
Effect of Clinical Decision Support With Audit and Feedback on Prevention of Acute Kidney Injury in Patients Undergoing Coronary Angiography: A Randomized Clinical Trial JAMA. 2022 Sep 6;328(9):839-849. doi: 10.1001/jama.2022.13382. 上記論文のアブストラクト日本語訳 ※ヒポクラ×マイナビ 論文検索(Bibgraph)による機械翻訳です。 [重要] 造影剤関連急性腎障害 (AKI) は、冠動脈造影および経皮的冠動脈インターベンション (PCI) の一般的な合併症であり、高額な費用と有害な長期転帰に関連しています。冠動脈造影または PCI 後の AKI の予防に効果的です。 [デザイン、設定、および参加者] カナダのアルバータ州で、3 つの心臓カテーテル研究所のすべての侵襲性心臓専門医を対象に無作為化されたステップウェッジ クラスター無作為化臨床試験が実施されました。 2018 年 1 月から 2019 年 9 月までの介入のさまざまな開始日。適格な患者は、非緊急冠動脈造影、PCI、またはその両方を受けた 18 歳以上でした。透析を受けていない人; 5%を超えるAKIリスクが予測された人。 34 人の医師が、選択基準を満たした 7,106 人の患者の中で 7,820 の処置を行いました。参加者のフォローアップは 2020 年 11 月に終了しました。 [介入] 介入期間中、心臓専門医は教育的アウトリーチ、造影剤量と血行動態に基づく輸液目標に関するコンピュータ化された臨床的意思決定支援、および監査とフィードバックを受けました。コントロール(介入前)期間中、心臓専門医は通常のケアを提供し、介入を受けませんでした。 [主な結果と測定値]主要な結果はAKIでした。造影剤量、静脈内輸液投与、主要な有害心血管イベントおよび腎臓イベントを含む 12 の二次的転帰がありました。分析は、時間調整モデルを使用して実施されました。 [結果] 診療グループとセンターによって 8 つのクラスターに分割された 34 人の参加心臓専門医のうち、介入グループには、4032 人の患者 (平均年齢、70.3 [SD、 10.7] 年; 1384 人は女性 [32.0%]) であり、対照群には 3251 人の患者のうち 3493 回の手術を行った 34 人が含まれていた (平均年齢、70.2 [SD, 10.8] 歳; 1151 人は女性 [33.0%])。 AKI の発生率は、介入期間中 7.2% (4327 処置後の 310 イベント)、対照期間中の 8.6% (3493 処置後の 299 イベント) でした (グループ間差、-2.3% [95% CI、-0.6% to -4.1%]; オッズ比 [OR], 0.72 [95% CI, 0.56 ~ 0.93]; P = .01)。事前に指定された 12 の副次的結果のうち、8 つは有意差を示しませんでした。過剰な造影剤を使用した手技の割合は、対照期間中の 51.7% から介入期間中に 38.1% に減少しました (グループ間差、-12.0% [95% CI、-14.4% から -9.4%];または、0.77 [95% CI、0.65 ~ 0.90]; P = .002)。輸液が不十分な適格患者の処置の割合は、対照期間中の 75.1% から介入期間中に 60.8% に減少しました (グループ間差、-15.8% [95% CI、-19.7% ~ -12.0] %]; OR、0.68 [95% CI、0.53 ~ 0.87]; P = .002)。主要な有害心血管イベントまたは主要な有害腎イベントにおいて、グループ間で有意差はありませんでした。対照期間中に治療を受けた患者と比較して AKI を発症し、時間調整後の絶対リスクは 2.3% 減少しました。この介入がこの研究設定の外で有効性を示すかどうかは、さらなる調査が必要です.[試験登録]ClinicalTrials.gov 識別子: NCT03453996. 第一人者の医師による解説 LVEDPに代わる別な因子の利用で より応用範囲が拡大する可能性 小泉 淳 千葉大学大学院医学研究院画像診断・放射線腫瘍学(放射線科)特任教授 MMJ.February 2023;19(1):15 造影剤関連の急性腎障害(AKI)は、冠動脈造影および経皮的冠動脈インターベンション(PCI)の一般的な合併症であり、高額な費用と患者に透析など長期的な悪影響を及ぼしうる。本論文は、カナダ・アルバータ州で実施された、34人の心臓専門医による7,820の処置を対象とした、侵襲性心臓専門医に対する臨床意思決定支援システムのAKI軽減に関するステップウェッジクラスター無作為化臨床試験の報告である。透析患者、ST上昇型心筋梗塞への緊急 PCI、およびAKIのリスクが5%以下の患者は除外された。参加医師は、まずAKIおよびその予防に関する1時間の教育セッションを受講した。心臓専門医は、手技前にスタッフからePRISMツール(Health Outcomes Sciences)を元に計算された安全な造影剤の量の目標を受け、心臓カテーテル検査中に得られた左心室拡張末期圧測定値(LVEDP)を元に計算された静脈内輸液量目標値を告知された。実際に使用された造影剤量と輸液量が記録され、以後3カ月ごとにAKI発生率について報告された。その結果、ePRISMにより計算された造影剤量に比べて過剰な造影剤量を使用した手技の割合は、対照期間中の51.7%から介入期間中に38.1%に低下した(群間差 , -12.0%;95%信頼区間[CI],-14.4 ~-9.4%;時間調整オッズ 比[OR], 0.77;95 % CI, 0.65 ~ 0.91;P=0.002)。LVEDPを元に計算された静脈内輸液量目標値に比べて不十分な静脈内輸液が与えられた割合は、対照期間中の75.1%から介入期間中に60.8%に低下したが(群間差 , -15.8%;95%CI, -19.7~ -12.0%;OR, 0.68;95%CI, 0.53~ 0.87;P=0.002)、主要な心血管・腎イベントに関して有意な群間差はなかった。しかし、介入期間中のAKI発生率は7.2%と対照期間の8.6%に対して有意差がみられ、時間調整されたAKI絶対リスク低下は2.3%であった。なお、ePRISMの入力項目に“Black or African American”はあるが、日本人を含む黄色人種の選択肢はなく、糖尿病に関しても有無のみで程度を入力する項目はない。今回の介入が本研究環境以外の日本でも有効であるかどうかは国内でさらなる検討が必要であろう。一方、左室造影に代わって超音波検査が行われている時代で,LVEDPを測らなければいけないのは現実的ではないようにも思われる。CTの普及・実施率が世界一である日本では、CT時に使用される造影剤によるAKI予防も重要であり、LVEDPに代わる別な因子が輸液量決定に使えるなら、より応用範囲が拡大するかもしれない。今後の研究が待たれる。 1. James MT, et al. Can J Cardiol.2019;35(9):1124-1133. 2. Malik AO, et al. Am Heart J.2021;234:51-59. 3. Brar SS, et al. Lancet 2014;383(9931):1814-1823.
痛風患者における発作は発作後の近い将来の心血管イベントと関連する
痛風患者における発作は発作後の近い将来の心血管イベントと関連する
Association Between Gout Flare and Subsequent Cardiovascular Events Among Patients With Gout JAMA. 2022 Aug 2;328(5):440-450. doi: 10.1001/jama.2022.11390. 上記論文のアブストラクト日本語訳 ※ヒポクラ×マイナビ 論文検索(Bibgraph)による機械翻訳です。 【重要】痛風は心血管疾患と関連があります。痛風の再燃と心血管イベントとの時間的関連性は調査されていません。 [目的]最近の痛風の再燃後に心血管イベントのリスクが一時的に増加するかどうかを調査すること。 [デザイン、設定、および参加者]レトロスペクティブな観察研究が実施されました。 1997 年 1 月 1 日から 2020 年 12 月 31 日までの英国の Clinical Practice Research Datalink からの電子カルテを使用。季節と年齢、痛風の再燃と心血管イベントを有する1421人の患者の間で実施されました。急性心筋梗塞または脳卒中。最近の以前の痛風再燃との関連性は、ネストされたケースコントロール研究で 95% CI の調整済みオッズ比 (OR) を使用し、自己管理型ケース シリーズで 95% CI の調整済み発生率比 (IRR) を使用して測定されました。痛風の新たな診断を受けた患者 (平均年齢、76.5 歳、男性 69.3%、女性 30.7%) では、心血管イベントが発生した 10,475 人の患者が、心血管イベントのない 52,099 人の患者と一致しました。心血管イベントが発生した患者は、心血管イベントが発生しなかった患者と比較して、0~60 日以内に痛風が再燃する確率が有意に高かった (204/10 475 [2.0%] vs 743/52 099 [1.4%]; 調整済み OR , 1.93 [95% CI, 1.57-2.38]) および過去 61 日から 120 日以内 (170/10 475 [1.6%] vs 628/52 099 [1.2%]; 調整済み OR, 1.57 [95% CI, 1.26- 1.96])。過去 121 日から 180 日以内に痛風再発のオッズに有意差はありませんでした (148/10 475 [1.4%] vs 662/52 099 [1.3%]; 調整済み OR、1.06 [95% CI、0.84-1.34] )。自己管理された症例シリーズ (N = 1421) では、0 日から 60 日までの 1000 人日あたりの心血管イベント発生率は 2.49 (95% CI 2.16-2.82) でした。 2.16 (95% CI、1.85-2.47) 61 日から 120 日以内。痛風の再燃後 121 日から 180 日以内に 1.70 (95% CI 1.42-1.98)、150 日または 181 日以内の 1000 人日あたりの心血管イベント発生率は 1.32 (95% CI 1.23-1.41)痛風の再燃から540日後まで。痛風再燃の 150 日前または 181 ~ 540 日後と比較すると、心血管イベントの発生率の差は 1,000 人・日あたり 1.17 (95% CI、0.83 ~ 1.52) であり、調整済み IRR は 1.89 (95% CI、1.54) でした。 -2.30) 0 日から 60 日以内。 1000 人日あたり 0.84 (95% CI、0.52-1.17)、61 ~ 120 日以内に 1.64 (95% CI、1.45-1.86)。 [結論と関連性] 痛風の患者のうち、痛風を経験した人心血管イベントは、そのようなイベントを経験しなかった人と比較して、前の日に最近痛風が再発する可能性が有意に高かった.これらの調査結果は、痛風の再燃が、再燃後の心血管イベントの一時的な増加と関連していることを示唆しています。 第一人者の医師による解説 最も肝心なのは生活習慣改善などによる痛風発作の予防 脇野 修 徳島大学大学院医歯薬学研究部腎臓内科学分野教授 MMJ.February 2023;19(1):20 心血管イベントの基盤病態には炎症が存在する。痛風は特に高齢者に多い全身性の炎症疾患で(1)、痛風患者は心血管イベントの発症率が高い(2)。日本のガイドラインによれば、痛風は高尿酸血症を必須条件とし、尿酸 -ナトリウム結晶による急性関節炎(痛風発作)をすでに生じた状態である(3)。痛風関節炎を伴わない無症候性高尿酸血症とは区別される。痛風自体は微小炎症を特徴とし、炎症性サイトカイン、血中活性酸素濃度の上昇、neutrophil extracellular trapの形成、内皮障害、血小板活性化などの病態を認め、動脈硬化性塞栓症発症の基盤となる。さらに、痛風発作はNAPL3インフラマソームの活性化により生じ、NAPL3インフラマソーム阻害薬は心血管イベントの発症を予防することが示されている。以上のことより痛風発作はその後の心血管イベント発症と関連があることが推測されるが、時間的因果関係の証明はなされていない。 そこで本研究では、英国の電子医療記録を用いて、痛風診断後に心血管イベント(急性心筋梗塞または脳卒中)を起こした患者(症例)と、年齢、性、罹病期間を一致させた心血管イベントを起こしていない患者(対照)を比較することで、痛風発作後に一時的に心血管イベント発症が増加するかどうかを検討した。その結果、症例群では対照群に比べ0~60日前および61~120日前の痛風発作の出現が有意に高かった(それぞれオッズ比1.93および1.57)。一方、121~180日前の痛風発作の出現には群間差がなかった。痛風発作と心血管イベントを発症した症例のみで行ったself-controlled case series解析によると、1,000人・年あたりの心血管イベント発症数は痛風発作後0~60日は2.49件、61~120日は2.16件、121~180日は1.70件、一方、150日前か181~540日後は1.32件であった。150日前か181~540日後をベースラインとして算出した心血管イベントの調整発症率比は、痛風発作0~60日後1.89、61~120日後1.64、121~180日後1.29であった。以上より、痛風患者において心血管イベントを発症した患者はその前の近いうちに痛風発作を起こしている可能性が高い。すなわち、痛風発作を起こした場合はその後の心血管イベント発症に関連することが明らかとなった。 本研究の感度分析では、標準的治療であるコルヒチン、非ステロイド系抗炎症薬、ステロイドの各使用群でも痛風発作に伴う心血管イベント増加が認められていることから、追加治療の潜在的な役割も期待される。ただし、リスクの絶対差は痛風発作後0~60日で0.6%(2.0%[症例群] 対 1.4%[対照群])であり、最も肝心なのは生活習慣改善などによる痛風発作の予防である。 1. Roth GA, et al. J Am Coll Cardiol. 2020;76(25):2982-3021. 2. Choi HK, et al. Circulation. 2007;116(8):894-900. 3. 高尿酸血症・痛風の治療ガイドライン第 3 版(2019 年改訂)
心房細動に対するアブレーションは 精神的ストレスの改善にも有効
心房細動に対するアブレーションは 精神的ストレスの改善にも有効
Atrial Fibrillation Catheter Ablation vs Medical Therapy and Psychological Distress: A Randomized Clinical Trial JAMA. 2023 Sep 12;330(10):925-933. doi: 10.1001/jama.2023.14685. 上記論文のアブストラクト日本語訳 ※ヒポクラ 論文検索(Bibgraph)による機械翻訳です。 【重要性】精神衛生の結果に対する心房細動(AF)カテーテルアブレーションの影響はよく理解されていません。 【目的】AFカテーテルアブレーションが、医学療法のみと比較して、心理的苦痛のマーカーのより大きな改善に関連しているかどうかを判断する。 【設計、設定、および参加者】心房細動(修復)研究における心理的苦痛に対するカテーテルアブレーションの影響のランダム化された評価は、2018年6月から2021年3月の間にオーストラリアの2つのAFセンターで実施された症状のある参加者のランダム化試験でした。 【介入】参加者は無作為化され、AFカテーテルアブレーション(n = 52)または医学療法(n = 48)を投与されました。 【曝露】主な結果は、12か月での病院不安とうつ病スケール(HADS)スコアでした。二次的な結果には、重度の心理的苦痛の有病率(HADSスコア> 15)、不安HADSスコア、うつ病HADSスコア、およびベックうつ病インベントリII(BDI-II)スコアの追跡評価が含まれていました。不整脈の再発とAF負荷データも分析されました。 【結果】合計100人の参加者が無作為化されました(平均年齢、59 [12]年; 31 [32%]女性、54%が発作性AF)。成功した肺静脈分離は、アブレーショングループのすべての参加者で達成されました。結合されたHADSスコアは、6か月(8.2 [5.4]対11.9 [7.2]; p = .006)で、アブレーショングループ対医療グループでは低かった(7.6 [5.3] vs 11.8 [8.6];グループの違い、-4.17 [95%CI、-7.04〜 -1.31]; p = .005)。同様に、重度の心理的苦痛の有病率は、アブレーション群と6ヶ月(14.2%対34%、P = .02)で、12か月(10.2%対31.9%; P = .01)での医学療法群では低かった、6か月で不安HADSスコア(4.7 [3.2]対6.4 [3.9]; p = .02)および12か月(4.5 [3.3]対6.6 [4.8]; p = .02);うつ病は3か月で得点(3.7 [2.6]対5.2 [4.0]; p = .047)、6か月(3.4 [2.7]対5.5 [3.9]; p = .004)、および12か月(3.1 [2.6)] vs 5.2 [3.9]; p = .004);6か月でのBDI-IIスコア(7.2 [6.1]対11.5 [9.0]; p = .01)および12か月(6.6 [7.2]対10.9 [8.2]; p = .01)。アブレーション群の中央値(IQR)AF負担は、医学療法グループよりも低かった(0%[0%-3.22%]対15.5%[1.0%-45.9%]; p <.001)。 【結論と関連性】症候性AFの参加者のこの試験では、不安とうつ病の心理的症状の改善がカテーテルアブレーションで観察されましたが、医学療法は観察されませんでした。 【試行登録】ANZCTR識別子:ACTRN12618000062224 第一人者の医師による解説 不安や抑うつ症状軽減に果たす役割は大きく 治療方針決定でも考慮すべき重要な要因 五十嵐 都 筑波大学医学医療系循環器内科准教授 MMJ.April 2024;20(1):8 本論文に報告されたREMEDIAL試験では症候性の心房細動(AF)患者をアブレーション群と薬物療法群に割り付け、精神的ストレス状態の評価指標HADSスコア、重症な精神的ストレス状態(HADSスコア 15超)の患者の割合などさまざまな指標について登録時と治療後の複数時点において両群間で比較した。AF再発の有無および累積時間率(burden)についても解析した。 登録時には評価項目に関して両群間に有意差はなかった。重症の精神的ストレス状態は32%の患者に認められ、不安症や抑うつ的な性格(タイプD)およびAF症状の重症度を示す指標 AFSSSとも関連があった。12カ月の経過でアブレーション群では薬物療法群に比べAFの再発が有意に少なく(47% 対 96%)、burden中央値も少なかった(0%対 15.5%)。その結果アブレーション群では抗不整脈薬を中止する傾向にあった(登録時90%、12カ月時点30%)。12カ月時点でアブレーション群では薬物療法群よりHADSスコア中央値が有意に低く(7.6 対 11.8)、重症の精神的ストレス状態の患者も少なかった(10.2% 対 31.9%)。 本試験ではまずAFがメンタルヘルスに悪影響を及ぼすことが示された。過去には逆に緊張や不安といった精神的ストレスがAF発症に関わるといった報告があり(1)、AFと精神的ストレスは両方向性に作用し悪循環を招く。その悪循環を断ち切るためにもAFの適切な治療は重要である。本試験では、精神的ストレス状態が薬物療法よりもアブレーションにより経時的に改善されることが明らかとなった。このことはAFの再発がないこと、抗不整脈薬やβ遮断薬の中止と関連があった。AFの症状を解消することが精神的ストレスを改善した可能性はあるが、抗不整脈薬やβ遮断薬の使用はうつや不眠、倦怠感などさまざまな神経精神症状に影響を与えることが知られているため(2)、これらの中止による直接の影響もあるかもしれない。 カテーテルアブレーションは侵襲的な治療ではあるが、3Dマッピングシステムやコンタクトフォースセンシング付きカテーテルなどテクノロジーの進歩に伴い有効性と安全性が年々向上している。そのため日本のガイドラインでも以前は「薬剤抵抗性症候性心房細動患者」に対してclass IIa適応であったが、現在は「薬剤抵抗性」という文言は削除された(3)。一方で有症候性 AFに対する薬物療法は国際的なガイドラインでもclass Iaのままである。本試験の結果から、カテーテルアブレーションがAF患者の不安や抑うつ症状を軽減するために果たす役割は大きいことが示され、このことは今後治療方針を決定する際に考慮すべき重要な要因であるといえる。 1. Eaker ED, et al. Psychosom Med. 2005;67(5):692-696. 2. von Eisenhart Rothe A, et al. Europace. 2015;17(9):1354-1362. 3. Nogami A, et al. Circ J. 2021;85(7):1104-1244.
急性期脳梗塞での血栓回収療法後の積極降圧は機能的転帰を悪化させる
急性期脳梗塞での血栓回収療法後の積極降圧は機能的転帰を悪化させる
Intensive vs Conventional Blood Pressure Lowering After Endovascular Thrombectomy in Acute Ischemic Stroke: The OPTIMAL-BP Randomized Clinical Trial JAMA. 2023 Sep 5;330(9):832-842. doi: 10.1001/jama.2023.14590. 上記論文のアブストラクト日本語訳 ※ヒポクラ 論文検索(Bibgraph)による機械翻訳です。 【重要性】急性虚血性脳卒中患者の血管内血栓切除術(EVT)による成功した再灌流後の最適血圧(BP)制御は不明です。 【目的】再灌流が成功した最初の24時間後の集中的なBP管理が、EVTを受けた患者の従来のBP管理よりも優れた臨床結果につながるかどうかを判断する。 【設計、設定、および参加者】2022年6月から2022年11月まで韓国の19のストロークセンターで行われた盲検化エンドポイント評価を伴う多施設、無作為化、非盲検試験(2023年3月8日)。EVTで治療された大血管閉塞急性虚血性脳卒中を有する306人の患者が含まれ、2B以上の脳梗塞スコアで修正された血栓溶解(部分的または完全な再灌流)が含まれていました。 【介入】参加者は、集中的なBP管理(収縮期BPターゲット<140 mm Hg; n = 155)または従来の管理(収縮期BPターゲット140-180 mm Hg; n = 150)を登録後24時間受け取るようにランダムに割り当てられました。 【曝露】主な結果は、3か月での機能的独立性でした(0-2のランキンスケールスコアを変更しました)。主な安全性の結果は、36時間以内の症候性脳内出血と、3か月以内のインデックス脳卒中に関連する死亡でした。 【結果】安全性の懸念に注目したデータと安全監視委員会の推奨に基づいて、試験は早期に終了しました。306人の無作為化患者のうち、305人が適格であることが確認され、302人(99.0%)が試験を完了しました(平均年齢、73.0歳、122人の女性[40.4%])。集中的な管理グループは、従来の管理グループ(54.4%)よりも機能的独立性(39.4%)を達成する割合が低く、有意なリスク差(-15.1%[95%CI、-26.2%から-3.9%))および調整オッズ比(0.56 [95%CI、0.33-0.96]; p = .03)。症候性脳内出血率は、集中群で9.0%、従来のグループで8.1%でした(リスク差、1.0%[95%CI、-5.3%〜7.3%];調整済みオッズ比、1.10 [95%CI、0.48–2.53]; p = .82)。3か月以内のインデックスストロークに関連する死亡は、集中群の7.7%と従来のグループの5.4%で発生しました(リスク差、2.3%[95%CI、-3.3%から7.9%];調整済みオッズ比、1.73 [95%CI、0.61-4.92]; p = .31)。 【結論と関連性】大きな血管閉塞を伴う急性虚血性脳卒中のEVTとの成功した再灌流を達成した患者の間で、24時間の集中的なBP管理により、従来のBP管理と比較して3か月での機能的独立の可能性が低くなりました。これらの結果は、急性虚血性脳卒中でEVTが成功した後、集中的なBP管理を避けるべきであることを示唆しています。 【試験登録】ClinicalTrials.gov識別子:NCT04205305。 第一人者の医師による解説 急性期脳梗塞では有効な再灌流が得られても 血圧を下げ過ぎない方が良い 山上 宏 筑波大学医学医療系脳卒中予防・治療学教授 MMJ.April 2024;20(1):9 急性期脳梗塞では血圧を下げると脳血流量が減少し梗塞巣の拡大を来すため、降圧療法は勧められていない。一方、大血管閉塞による急性期脳梗塞に対する血栓回収療法後では、血圧高値の持続が頭蓋内出血や脳浮腫の発生を介して機能的転帰の悪化と関係することが報告されている(1)。血栓回収療法により有効な再灌流が得られれば、降圧しても脳血流量の減少は軽度である可能性もあり、積極降圧療法が患者の転帰を改善させることが期待されていた。 OPTIMAL-BP(Outcome in Patients Treated With Intra-Arterial Thrombectomy-Optimal Blood Pressure Control)試験は、大血管閉塞による急性期脳梗塞に対して血栓回収療法により有効再開通が得られ、治療後2時間以内に収縮期血圧(SBP)140mmHg以上である患者を、積極降圧群(目標 SBP 140mmHg未満)と標準降圧群(目標SBP 140~180mmHg)に割り付けたランダム化比較試験である。降圧療法は主にニカルジピンが用いられ、割り付けから1時間以内に目標血圧を達成し24時間維持することとされた。主要有効性評価項目は、3カ月後に日常生活が自立した患者の割合、主要安全性評価項目は36時間以内の症候性頭蓋内出血および3カ月以内の脳梗塞関連死の発生とした。本試験は、安全性の懸念などから早期に中止され、ランダム化された306人のうち302人が解析対象となった。積極降圧群では標準降圧群に比べ、日常生活自立を達成した患者が有意に少なく(39.4% 対 54.4%)、症候性頭蓋内出血の発生に差はなかった(9.0% 対 8.1%)。また、悪性脳浮腫の発生は積極降圧群で有意に多かった(7.7% 対1.3%)。 先行研究であるENCHANTED2/MT試験(2)においても、血栓回収療法後の積極降圧(目標 SBP 120mmHg未満)は標準降圧(目標 SBP 140~180mmHg)に比べ90日後の機能的転帰を悪化させており、本試験の結果と一致していた。有効な再灌流が得られても、すでに虚血が進行している領域では降圧療法によって脳血流量が減少して細胞障害が悪化する可能性や、脳微小循環への悪影響によって脳浮腫が進行する可能性がある。また、積極降圧が症候性頭蓋内出血の発生を抑制できなかったことも共通した結果であり、この点でも積極降圧のメリットはないと言えるであろう。急性期脳梗塞では、再灌流が得られてもSBPを140~180mmHgに維持すべきと考えられる。 1. Goyal N, et al. Neurology. 2017;89(6):540-547. 2. Yang P, et al. Lancet. 2022;400(10363):1585-1596